You are on page 1of 362

BaJaJi

for
JEE Main & Advanced

by:
Er. Anurag Mishra
B.Tech (Mech. Engg.)
HBTI Kanpur

5HRI BALAJI PUBLICATIONS


(EDUCATIONAL PUBLISHERS & DISTRIBUTORS)

[AN ISO 9001-2008 CERTIFIED OFtGANIZATIQNl


Muzaffarnagar (UP.) - 251001
IJ Published by: t
5HRI BAUUI PUBLICATIONS
(EDUCATiaNAL PUBLISHERS & DISTRIBUTORS)
[AN ISO 9001-2008 CERTIFIED ORGANIZATION]
6, Gulshan Vihar, Gall No. 1,
0pp. Mahalaxmi Enclave.
Jansath Road. Muzaffarnagar (U.P.) :2L;

Phone : 0131-2660440 (O). 2600503 (R)


website; www.shribalajibooks.com
email: sbjpub@gmail.com

M First edition : 2011


U Third edition : 2013
11 Reprint : 2017

U ©Ali rights reserved with author

11 Price : ^ 250.00

S] Typeset by :
Sun Creation
Muzaffarnagar

9! Printed at:
Nav Prabhat Printing Press
Meerut (U.P.)

All the rights reserved. No part of this publication


may be reproduced, stored In a retrieval system
or transmitted, in any form or by any means,
electronic, mechanical, photocopying, recording
or otherwise, without the prior permission of the
author and publisher. Any violation/breach shall
be taken into legal action.
Preface
Myinteraction with students, sincerely preparing for IIT-JEE motivated
me to write this book on Optics.

Optics is also an important part of physics Likemy other books. "Optics


for IIT-JEE" is designed to clearthe conceptsthrough numerical approach.
This^book will help the students in building analytical and quantitative
skills, addressingkeymisconceptions and developing confidence insolving
the problems.

I sincerely wish that this book will fulfill all the aspirations of-the,
readers. Although utmost care has been taken to make the book free'from
errors but some errors inadvertently may creep in. Author and Publisher
shall be highly obliged if suggestions regarding improvement and errors
are pointed "outlby readers. I am indebted Neeraj Ji for providing me an
opportunity to write a book of this magnitude.
I am indebted to my father Sh. Bhavesh Mishra, my mother Smt.
Priyamvada Mishra, my wife Manjari, my sister Parul, my little kids
Vrishank and Ira for giving their valuable time which I utilized during the
writing of this book and people of Moradabad, who supported me
throughout my career.

I am also thankful to Mr. T. Kondala Rao, Mr. Abhishek Sinha


(Kanchi), Mr. Sunil Manohar, Mr. S.P. Sharma, Mr. Sudhir Sharma and
Mi^P. Narendra Reddy for their valuable suggestions in improving the
book.

In the last, I also pay my sincere thanks to all the esteemed members of
Shri Balaji Publications in bringing out this book in the present form.

Anurag Mishra
How to face the challenge ?
Following are some doubts which arise in the mind of almost all the students
but may face them by taking some care.

1. / can not solve numerical because my concepts are not clear. In fact numerical
solving itselfis an exercise to learn concepts.

2. I can not study because I am in depression, I fell into it because I was not
studying] Depression is escape mechanism of people afraid of facing failures.
Failure is integral part oflearning.

3. I understand every thing in class but can notsolve on my own.WRlTlNGworkis


vital. It is a multiple activity, initially idea comes in mind then we put into
language to express it, we are focussed in hand eye coordination, eyes create
visual impression on brain which is recorded there. WRITING WORKS ARE
EMBOSSED ON BRAIN LIKE CARVINGS OF AJANTACAVES.

4. In exams my brain goes blank, but I can crack them at home. Home attempt is
your second attempt! you are contemplating about it while home back. You
do not behave differently in exam you replicate your instincts. Once a fast
bowler was bowling no balls. His coach placed a stump on crease, in fear of
• injuryhegotitright. CONCEPTUALIZATION, WRITING EQUATION, SOLVING,
THEN PROBLEM GETS TO CONCLUSION!

/ am an average student. It is a rationalization used by people afraid of hard


work. In their reference frame Newton's first law applies "if 1 have a
misconception I will continue with it unless pushed by an external agent
even 1 will surround him in my web of misconception yielding zero
resultant." AVERAGE IS NOT DUE TO CAPACITY LACUANE BUT DUE TO LACK
OF DETERMINATION TO SHED INERTNESS.

A famous cliche "I do not have luck in myfavour'' PRINCIPLE OF CAUSALITY:


CAUSE OF AN EVENT OCCURS IN TIME BEFORE OCCURRENCE OF THAT
EVENT i.e., cause occurs first then event occurs. SHINING OF LUCK IS NOT AN
INSTANTANEOUS EVENT IT IS PRECEDED BY RELENTLESS HARD WORK.
Sow a seed ofaspiration in mind, water it with passion, dedication it will bear
fruit, luck can give you sweeter fruit.
yL

I
I
Useful tips:
I 1. Do not take study as a burden actually its a skill like singing and dancing. It
/ hastobehoned byproperdevotion and dedication.

2. Without strong sense of achievement you can't excel. Before entering the
competitive field strong counselling by parents is must. Majority do not
know what for they are here. No strategic planning, they behave like a tail
ender batting in front of Steyn's bouncers.

3. Science is not a subject based on well laid down procedures or based on


learning some facts, it involves very intuitive and exploratory approach.
Unless their is desire and passion to learn you can not discover new ideas. It
requires patience and hard work, whose fruits may be tangible later on.

4. Somestudentsrealizeverylatethattheyarestudyingforacquiringskillsand
honing them. Their is a feeling that they can ride at the back of instructor and
achieve excellence. Study comes as torturous exercise enforced on them and
their is some mechanism that can take this burden ofthem.

5. Scienceis not about gaining good marks, up to Xthby reading key points good
marks are achieved but beyond that only those survive who have genuine
interest in learning and exploring. Self study habit is must.

6. IF YOU WANT TO GAIN LEAD START EARLY. Majority of successful students


try to finish major portion elementary part of syllabus before they enter
Coaching Institute. Due to this their maturity level as compared to others is
more they get ample time to adjust with the fast pace. They are less
traumatized by the scientific matter handed over. For those who enter fresh
'' must be counselled to not get bullied by early starters but work harder
initially within first two months initial edge is neutralized.

^ 7. Once a student lags behind due to some forced or unforced errors his mind
\ begins to play rationalization remarks like 1am an average student, my mind
\ is notsharpenough, Ihave low IQ etc. These words are mechanisms used to
'\ avoid hard work. These words are relative terms a person who has early start
*\ may be intelligent relative to you.
Intelligence means-cumulative result of hard work of previous years, that
I
hard work has eventually led to a development of instinct to crack things
\ easily.
Geometrical Optics
Optics (1); Nature of light (1); Rectilinearpropagation of light (2); Regular reflection (5); The formation
of images by a plane mirror (7); Object (7); Image (8); Image formation from a plane surface (9);
,Vector form of law of reflection (9); Comer reflector (10); Angle of deviation (14); Extended object
(15); Number of images formed by two inclined plane mirrors (19); Spherical mirrors (27); Relation
^ between focal length and radius of curvature (29); Ray diagram for concaye mirror (31); Velocity of^
image in spherical mirror (36); Refraction (46); Huygen's principle (50); Reversibility of light rays (50);
Partial reflection and partial refraction at the boundary of two media (55); Glass slab (58); Total
internal reflection and critical angle (61); Some explanation of some refraction phenomenon (63);
Optical fibre./Optical pipe (68); Variable refractive Index (74); Dispersion (86); Single surface
refraction equation (102); Thin lenses (109); Optical (111); Important points (114); Optional (116);
Analysis of graphs (118); Displacement method (123); Methods for determining focal length of a
convex lens (129);, Optical instruments (163); Simple microscope (166); Compound microscope
. (.167); Photometry (171); Subjective problems (176);

Level-1: OnlyoneaItemativeiscorrect(187);Solution(206).
Level-2; More than one alternative is/are correct (222); Solutions (224).
Level-3 ; Comprehension based problems (226); Matching type problems (231); Assertion and
reason (234); Solutions (237).

Wave Optics
Interference of light waves (244); Young's double slit experiment (248); Displacement of fringes
(253); Fresnel's mirrors (263); The interference condition by division of wave front and amplitude
(265); Billet's lens (266); Fringes of equal thickness (284); Newton's rings (285); Diffraction (288);
LimitOfresblutidri(292), Polarization (294); Subjective problems (297);

Level-1: Onlyonealternativeiscorrect(307);Solutions(311)
Level-2: More than one alternative is/are correct (315); Solution (317)
Level-3: Comprehension based problems (319); Matching type problems (322); Assertion and
reason (323); Solutions (325).

Previous Year Problems


Gr^^n
Vencal
seal?

dys

Otject a? ^

GEOMETRICAL OPTICS

OPTICS
Optics is the branch of physics ±at studies the
phenomena and laws associated with the generation,
propagation, and interaction with a substance of
electromagnetic light waves. Light waves occupy an
enormous range on the scale of electromagnetic waves. It is (a) (c)
after the ultrashort millimetre radio waves and extends up
to the shortest electromagnetic waves known at present-
t- T •'t •^c
gamma rays with a wavelength X, less than one angstrom (A)
aA= 10"^° m).
Geometrical (ray) optics treats the laws of (b)
propagation of light in transparent media on the basis of Fig. 1.1
I -
notions of light as a combination of light rays-lines along
which the energy of light electromagnetic waves propagates. When the light contains a single wavelength it is called
Geometrical optics does not take into consideration the monochromatic (literally 'one colour'). On each of the
wave properties of light and the diffraction phenomena sheets the electric field has constant magnitude, it oscillates
associated with them. For instance, when light passes between to -^min we move from sheet to sheet. A
through a> lens with a mount diameter of d > X , given sheet moves to the right at the speed of light. The
where X is the length of the light wave, we may electric field is different in planes located at different
ignore the diffraction at the edges of the lens. positions along the propagation direction. For simplicity we
take a particular sheet over which the electric field has a
NATURE OF LIGHT
particular phase of oscillation; e.g., we choose the phase
Light is a transverse electromagnetic wave. Light waves where E has maximum value and is pointed upward. The
spread out in space in three dimensions; this makes rectangular sheets of Fig. 1.1 (b) represent a wavefront,
visualization and representation of wave difficult. Fig. 1.1 on each of them the wave has an equivalent phase. When we
(a) shows representation of simplest light wave, a move from sheet to sheet, the phase increases or decreases
monochromatic plane wave. by 2k rad. Figs. 1.1 (c) and (d) show spherical and
cylindrical wavefronts.
:i

Wavefronts travel with the speed of light. The direction Beams of light rays upon intersecting do not- interfere
of energy flow is given by Poynting vector A line tracing and propagate independentiy of each other after
the flow of radiant energy through space is called a ray. In an intersection.
isotropic medium rays are perpendicular to wavefronts. We If a piece of wood, tissue paper and glass, etc., are held
assume that the direction of motion of light is represented in succession to a source of light, it is seen that some
by rays. Rays do not travel; they actually indicate paths of substances allow no light to pass through them, whilst other
light. In the ray approximation we assume that a wave substances do so to varying degrees. Hence we apply the
through a medium travels in a straight line in the direction terms —
of its rays.
opaque, to substances which allow no light to
1. Parallel rays indicate plane wavefronts. pass tiu'ough them,
2. Diverging rays indicate expanding wavefronts. transparent, to substances which allow much
3. Converging rays indicate collapsing wavefronts. light to pass through them,
When a wave meets an obstacle with circular opening translucent, to substances which allow a
whose diameter is much larger than the wavelength, the smaller portion.of light, falling on them (or as we
wave emerging-from the opening continues to move in a say, incident upon them), to pass through.
straight line, hence the ray approximation is'valid. If the The light which passes through is called
diameter of the opening is of the order of the wavelength transmitted light. But what happens to the light falling
[Fig. 1.2 (b)].the waves^ spread out fromsthe opening in all on an opaque body ? Since fiie body is visible it must
directions. ^ ^^^ ^ . throw off, or reflect, light just as a wall or floor
throws off, or reflects, a ball thrown on it. But there
is a difference, for whereas a ball is only reflected in the one
direction, light must be reflected in all directions. An
object (C^ be seen., ifrom ^ parts of a room,
despite the fact that only a single source of light is
illuminating it. This process by which. a body
reflects incident light in all directions is called
X«d
X»d irregular or disuse reflection or scattering, of
light, and it is due to this process that we
Fig. 1.2 ordinarily, see objects.

Finally, if the opening is much" smaller than the RECTILINEAR PROPAGATION OF LjGHT
wavelength/the opening can be approximated as a point Light Rays ' . vv ./-! '" r'' . mv
_ 1 . ( . •
source- of waves," each incident light ray subsequently travels Observations show that in homogeneous media, light
in a unique direction. Well defined shadows exist'ih regions also propagates, along s^traight lines. When an object is
where there, are no rays.. When a wave encounters ah object
of dimensions d arid if X « d, the object casts^a sharp
shadow. In'this'chapter 'the ray apprb>dmatibn'and die
assumption that X « d have been used.
A medium is
defined as optically
homogeneous if its
refractive index is
everywhere . the
same. In an optically
homogeneous . . .
medium, the rays are Fig. 1.3
straight: in . such .a , , . . .
medium light propagates rectUinearly. This IS
Fig:1.4:When-a point source's illuminates
confirmed by die formation of shadows. If 5 is a point source a pjane object O parallel to a screen,
of light, and K is.a body in the.path of the light from the a sharp shadow (umb/a) O' formed on i
source, then a cone of a shadow is,formed beyond the body the screen Is similaf .to the object. •,

K (Fig. 1.3). Noipoint inside this cone receivesTight from the


source. A well-defined shadow of the body K is obtained on a illuminated by a point soturce, a sfiarp shadow (umbra)
screen placed at right angles to the axis of the cone. is formed (Fig.1.4), whose shape is similar to the shape of a
GEOMETRICAL OPTICS

crossrsection of the object, which is parallel to the screen. position of the screen, these beams have the same direction
The size of the umbra is determined by the mutual pointing to the Sun.
arrangement of the source, the object and the screen in
conformity with the laws of projecting by straight lines.
Partial shadows (penumbras) observed sometimes are due
to the 'finite size of the light source rather than due to a
deviation of the direction of propagation from a straight line
CFig.1.5). Joiners employ a well-known method of verifying

Fig. 1.7:Beams isolated from a plane light wave (the light source is
the Sun) by a screen with small holes.The diameters of
the holes are large in comparison with the lightwavelength.

If in a dark room we place a bright bulb with a small


Fig. 1.5:When ball O is illuminated by an extendedi filament into a dark box
source SS, the umbra 0' on the screen Is. with several holes, the
surrounded by penumbra O".
path of emanating rays
the straightness of a planed board " by ray" (Fig.1.6). will be seen in the dusty
Phenornena associated with the rectilinear propagation of air of the room as a
light are similar to those described in the previous section. If narrow beam diverging
in all directions
(Fig.1.8). Having
marked the position of
the bulb on the box wall,
we will easily see that
these beams intersect at
the point wheretie bulb Fig. 1.8: Beams of rays isolated from a
filament ' "is' ^ located.' spherical lightwave.
Proceeding in the same way as in the case of the waves on
water surface, we draw the lines indicating .the direction of
light propagation. If the isolated beams are narrow, the
construction of such lines presents no difficulty. These
geometrical lines are light rays. In the cases considered
above, they are nearly parallel line directed to the Sun, or
the radii pe^)endicular to the surface of the sphere with, the
centre at the point where the light source (bulb filament) is
located. A light wave propagates along these straight rays.
Ftg. 1.6: Verification ofthestraightness ofthe edge by the "lens ofsight".^ So that they exceed the details of the pattern, the blurring
we make the paths of solar rays "visible" with the help of becoming stronger with a further reduction of the aperture
cigarette smoke, the experiment with partitions will be size. But since these spots are the traces of light beams cut
repeated for light. If we interpose a light ray with a by the aperture, die experiment denionstrates the
cardboard screen having one or several small holes (which divergence of a light beam upon a considerable reduction of
are naturally much larger, than the optical wavelength), we the aperture size. Therefore, it is physically impossible to
will see the path of light in the room in the form of narrow isolate an infinitely narrow beam. We haVe to confine
beams emanating from the holes (Fig.1.7). With any ourselves to the isolation of finite width beams and then
.4 OPTICS

replace them by the lines representing the axes of these At an interface between two materials, light can be either
beams. Thus, a light ray is a geometrical concept. reflected or refracted.
The merit of this concept consists in the possibility of Within a material, light can be either scattered or
establishing the direction of propagation of luminous absorbed.
energy. The laws governing the change in the direction of
Material I
rays allow us to solve very, important optical problems Material 2

concerning the change in the direction of propagation of Reflection


luminous energy. For solving most of such problems, it is Refraction
sufficient to replace a physical concept of light wave by a
geometrical concept of light ray and carry out analysis in
terms of rays. Scattering
Absorption
However, the concept of light rays cannot always be
used for determining the nature of propagation of light
waves. There is a large number of optical phenomena which
can be grasped only by analyzing light waves. Naturally,
optical phenomena can be considered in the framework of
the wave theory for simple cases when the ray method also
provides quite satisfactory results. But since the ray method
is much simpler, it is normally used for solving all problems
where it is applicable. Therefore,we must outline the range
of problems and the degree of accuracy with which
geometrical rays can be used to distinguish these problems (iv)

from those where the ray method leads to considerable Diverging bundle of rays
errors and hence is inadmissible.
Thus, the method of ray optics, or geometrical
optics, is an approximate technique for solving
problems, which is adequate for analyzing a certain scope
of questions. For this reason, it is important for those
studying optics to learn how to apply the ray method
correctly and to establish the limits of its applicability.
Fig. 1.9
Concepts: (1) Light Rays Travel in Straight
Line: Light travels through a transparent material in (4) An Object is a Source ofLight Rays: An object
straight lines called light rays. The speed of light is v = c/n. is a source of light rays. Rays originate from every point on
where n is the index of refraction of the material. the object, and each point sends rays in all directions. We
make no distinction between self-luminous objects and
(2) Light rays can cross: Light rays do not interact
with each other. Two rays can cross withoufeither being reflective objects.
affected in any way. (5) The Eye Sees by Focasing a Diverging
Bundle ofRays: The eye "sees" an object when diverging
rays from each point on the object enter the pupil and are
(i)
focused to an image on the retina. From the movements the
eye's lens has to make to focus the image, brain "computes"
the distance d at which the rays originated, and you perceive
the object as being at that point.
(6) Objects: Fig. 1.10 illustrates the idea that objects
(ii) can be either self-luminous, such as the sun, flames, and'
Fig. 1.9 light bulbs, or reflective. Most objects are reflective. A tree,^
unless it is on fire, is seen or photographed by virtue of
(3) A Light Ray Travels Forever Unless it
reflected sunlight or reflected skylight. People, houses, and
Interacts with Matter: A light ray continues forever
unless it has an interaction with matter that causes the ray to
this page in the book reflect lightfrom self-luminous sources.
change direction or to be absorbed. Light interacts with
matter in different ways;
GEOMETRICAL OPTICS

The sun is a self--.,


luminous object.
These are just a few of the
infinitely many rays leaving
the object.
Emitted light

Figure 1.12: A ray diagram simplifies the


situation by showing only a
few rays.
Reflected light

The camera "sees"


light rays reflected Note: Ray diagrams are the basis for a pictorial representation a
The tree is a by the tree but not ray diagram does not shows all of the rays. They rays
reflective object the rays from the sun shown on the diagram are just a subset of the infinitely
many rays leaving the object.
Fig. 1.10 Self-luminous and reflective
objects.
REGULAR REFLECTION
(7) Light rays from an object are emitted in all^ A special form of reflection of light—reflection from a
directions, but we do not feel it unless they enter the pupil of polished surface, i.e., a surface so smooth that the surface
your eye. Consequently, most light rays go completely particles are all in the same plane, and so all the rays in a
unnoticed. For example, light rays travelfrom the sun to the parallelbeam falling on them are reflected in parallelpaths.
tree in Fig. 1.10, butyou're not aware of these unless the tree In this case there are definite laws regarding the reflection of
•reflects some of them intoyour eye. Or consider a laser beam. the light, and so we call it regular reflection. The
It's almost impossible to see a laser beam from the side unless reception of a regularly reflected beam of light from a
there's dust in the air. The dust scatters a few of the light rays surface gives the impression of a brightness at the surface,
toward your eye, but in the absence of dust you would be which is thus said to be polished. Surfaces specially
completely unaware of a very poweiful light beam traveling made to reflect light regularly are mirrors. Ordinary mirrors
past you. Light rays exist independently of whether are made by pressing mercury between glass and
you are seeing them. tinfoil. A mercury-tin amalgam adheses to the glass and, in
(8) Fig. 1.12 shows two idealized sets of light rays. The time, hardens. It is coated with shellac varnish and backed
'diverging rays from a point source are emitted in all with red oxide of iron paint, to preserve it. For very good
directions. It is useful to think of each point on an object as a mirrors a fine layer of silver is deposited on specially smooth
point source of light rays. Aparallel bundle of rays could glass by a chemical or electrol5T:ic process.
be a laser beam. Alternatively it could represent a distant A mirror is not a perfect regular reflector, i.e., there is
object, such as a star so far away that the rays arriving at a loss of intensity on reflection. If a beam of light be
the observer are essentially parallel'to each other. directed on to a mirror in an otherwise dark room, the
mirror is visible from all parts of the room, i.e., it does
scatter some of the light.
Diffuse Reflection
Most objects are Each ray obeys the law of reflection
seen by virtue of their at that point, but the irregular surface
Point source Parallel bundle reflected light. For a • causes the reflected rays to leave in
many random directions.
Figure 1.11: Point sources and parallel bundles "rough" surface, the
represent idealized objects. law of reflection
0J. = 0j is obeyed at
(9) Ray Diagrams: Rays originate from everypoint on
each point but the
an object and travel outward in all directions, but a diagram irregularities of the
trying to show all these rays would be hopelessly messy and surface cause the
confusing. To simplify the picture, we usually use a ray
reflected rays to leave Magnified view of surface
diagram showingonly a few rays. For example, Fig. 1.12 is,
in many random Fig. 1.13
a ray diagram showing only a few rays leaving the top and directions. This
bot torn points of the object and traveling to the right. These situation, shown in Fig.1.13, is called diffuse reflection.
rays will be sujficient to show us how the object is imaged by It is how you see this page, the wall, your hand, your friend,
lenses or mirrors.
and so on. Diffuse reflection is far more prevalent than the
mirror like specular reflection.
OPTICS
"Rough" surface, means a surface that is rough or
irregular in comparison to the wavelength of light. Because
visible-light wavelengths are « 0.5|im, any surface with
irregularities larger than 1pm will cause diffuse reflection
rather than specular reflection. A piece of paper may feel
quite smooth to your hand, but a microscope would show
that the surface consists of distinct fibers much larger than N////////////////1B
0
1pm. The irregularities on a mirror or a piece of polished
metal are much smaller than 1pm. The law of reflection is ; Fig.1.15
equallyvalid for both specular and diffuse reflection, but the The angle of incidence is the angle a between the normal
nature of the surface causes the outcomes to be different. and the incident ray, while the angle ofreflection is the angle
When light is reflected from a rough surface, rays are P between the normal and the reflected ray.
scattered more or less randomly in different directions. This Light rays are reversible. This means that a ray passed
type ofreflection iscalled diffused reflection. When light against another ray that has covered the distance between
rays strike an optically smooth surface (a surface with points 1 and 2 propagates along the same path but in the
irregularities of a size much less than the wavelength of opposite direction. In particular, if the raypropagating along
light) the reflection of the wavefront is quite regular called 50 is reflected in the direction OS', the ray incident along
specular reflection. S'O is reflected along OS.
A line perpendicular to the surface at the point where The image of a point source formed by a mirror ( or
the incident ray strikes the surface is called a normal line or lens) isfound at theintersection or continuation oflight rays
simply the normal to the surface. The angle i that the emerging form this point- and reaching the eye of an
incident ray makes' with the normal is the angle of observer.
incidence. The angle r that the' reflected ray makes with
the normal is the angle of reflection. If the rays emerging from a point source and getting to
the eye do not intersect but their continuations do, the
Let S represent a
Normal image is called virtual.
unit vector pointed in A

the direction of energy


N - Fig. 1.16 illustrates the construction of the image of a
propagation. Then is
pointsourceS formed by aplane mirrorAB. For this purpose,
in the direction of
it is sufficient to take (see Fig.1.16 two rays SO^ and SO2
incident ray that makes incident at points 0^ andO2 ofthe mirror. The reflected rays
an angle i with a normal 0 not intersect, but their continuations intersect at point S\
N drawn to the The triangles SO1O2 and S'0i02 are equal since O1O2 is their
reflecting surface. The • ^'9- 1-14:. In.specular reflection the common side.
plane rormea bv
nlanp fnrmpH DyS»,- and., normal to the
S .flnH. ray,surface alllie ray
reflected in theand
samea AS=AS''' ^ B ' " ' '
NIS called theplane of plane. The angle of incidence (Sj),
incidence. According.. ' equals the angle of reflection (0^).
to law of reflection
N2
lies in the plane of incidence and makes an angle i = r,
shown in Fig. 1.14.
When light reaches the interface between two 0.-.'
Pi 1 Pi

transparent media, a part of it passes to the second medium S' 90°^ ^90°
N
' S

(is refracted), while the other is reflected to the first A

medium. Fig. 1.16


The law of reflection of light states that the reflected < SO1O2 = (90°-kxi) =< S'OiOs = (90°+Pi) and
ray lies in the same plane with the incident ray and with the
normal to the reflecting surface at the point of incidence, the < SO2O1 = (90°-a2) =< S'OsOi = (90°-P2).
angle of reflection being equal to the angel of incidence Consequently, the heights of the triangles are also equal, and
(Fig.1.15). the image of a point source (object) fromed by a plane
mirror is virtual' and is symmetric to the point source
(object). > . , .
GEOMETRICAL OPTICS 71

If a surface is not a mirror shown in figure. The


Object point Extended Object
for given rays, diffuse reflection image at point P' is called
is observed since rays incident a virtual image point. In
on such a surface are reflected general, images are
in all directions from the referred to as either real
nonuniformities Fig. 1.17. or virtual.

Fig. 1.17 A real image


point is defined as a
point at which
converging light rays
intersect, images from
A light ray is incident at angle 0 on the reflecting surface Fig. 1.18
which all the light
shown in Fig. lE.l. The reflecting surface is now rotated rays actually do
through an angle ([) as shown in Fig. lE.l. Find the angle emanate. A virtual image point is defined as a point
between the incident and reflected ray. from which rays of light appear to be diverging, all the
Original position of rays of light do not actually emanate from the
jOriginal position of normal line image.
normal line N New angle of .New position of,
incidence ' normal line

New angle of
reflection = 0+(t)

—Original mirror
position

Object Image
point point
(a) (b) Optical device
Fig.1E.1
Fig. 1.19
Solutloni In the initial position of the mirror, the
angle between the reflected ray and the incident ray is 29, OBJECT
according to the law of reflection.• Point of intersection of incident rays is called object.
As the mirror rotates through an angle (j), the normal line Real Object
also rotates through the same angle The angle ' of An object is real if two or more incident rays actually
incidence now is 0 + cj), as shown in Fig. lE.l (b); from law of emanate or seem^to emanate from a point.
reflection angle of reflection must be 0 + (j). The angle
between the new reflected and- the incident ray then is
2 (0 + (j)). Therefore the'change in the angle that the reflected Real
Object
ray makes with the incident ray is
2(0 +^-20 = 2(1)^',
which is independentof angle' of incidence 0. This device
is called an optical lever.
THE FORMATION OF IMAGES BY Optical Element
A real object placed in
A PLANE MIRROR front of an optical element
When a given object is placed infront of an optical Fig. 1.20
device, we imagine each point on the object as an object
point. Each object point is a source of many diverging rays Virtual Object
in different directions. An extended object consists of Now consider a converging set of rays as shown in
many different object points. If the light rays intersect .or Fig. 1.21. If not intercepted, the rays will meet at a point.
appear to intersect at some point, that point is the image However, if the rays are intercepted by an optical element
point of the object point. ' placed as shown in the figure, then the point of convergence
The drawing of reflected rays in Fig. 1.18 indicates an is a virtual point behind the optical element. This point is
image point directly behind the mirror and at the same called the virtual object for the optical element.
distance from the mirror as the source point; this result is
OPTICS

appear to emanate form a point then a virtual image is


Virtual formed.
Object

Virtual
Object

Optical Element
A virtual object that appears
to be behind the optical element
Fig.1.21

A virtual image formed by


IMAGE a real object in front of
the mirror
An image is tiie point of convergence or apparent point
Fig.1,23
of divergence of rays after they interact with a given optical
element. An object provides rays that will be incident on an Object/Image \ Criteria
optical element. The optical element reflects or refracts the
incident light rays which then meet at a point to form an Real object When incident rays are of diverging nature.
image. Virtual object When incident rays are of converging nature.
Real Image Incident rays converge to a point behind the
optical element, ' •
Real images are formed when the reflected or refracted
rays actually meet or converge to a point. If a screen is Real image When rays after interacting with the optical el
placed at the point, a bright spot will be visible on the ement are of converging nature.
screen. Thus a real image can be captured on a screen. Virtual image When rays after interacting with the optical el
Examples of a real images are shown in Fig. 1.22(a) and (b). ement. are of diverging nature.

A real image can be captured on a screen. Preferably the


surface should be white in color so that we can see the image
clearly. This image on the screen can be perceived by the
eye.
Real Real
Object Image How about virtual images ? Since they cannot be
captured on a screen is it possible to see them. From
practical experience we, know that we can see ourselves in a
(a) Areal image formed by a real plane mirro^which is indeed a virtual image. So it is possible
object placed in front of the' to,see a virtual image. But how is it done? There is a lens in
convex lens
our eye that focuses the diverging set of rays on to the retina
forming a real image inside our eyes.
In a similar manner, a camera can also convert a virtual
Virtual image into a real image which is captured on the film (Fig.
Object
1.24).

Final
(b) A Real image formed by Real Image
a virtual object that appears Object
to be behind the mirror

Fig. 1.22

Virtual Image Camera


Fig.1.24
When light rays, after interacting with the optical
element, actually meet at a point the image formed is a real
image. However, if the rays do not meet at a point but a
GEOMETRICAL OPTICS

- IMAGE FORMATION FROM


A PLANE SURFACE
]e^ = xi + yj + zk e, = xi + yj - zk
Point Object
(i) Distance of object
from mirror = Distance of
image from the mirror.
Fig. 1,28 _
All the incident rays'
from a point object will VECTOR FORM OF LAWS OF REFLECTION
meet at a single point !object image,
after reflection from a
plane mirror which is
called image.
(ii). The line joining a Fig. 1.25
point object and its image is
normal to the reflecting surface.
Fig. 1^2?
(iii) The size of the image is the same as that of the
object. ^2 = Ci - 2(^1 11)11
(iv) For a real object the image is virtual and for a virtual where = unit vector along incident ray
object the image is real. n = unit vector along normal
(v) The laws of reflection are valid for any smooth 62= unit vector along reflected ray
reflecting surface irrespective of geometry. Form triangle law of addition of vectors
zi =:zr Consider two vectors a and b as
shown in Fig. 1.30.

Here we can express a as its !


components a]| and aj^
^ ^

Such that a = ay + aj^ Fig. 1.30


Fig. 1.26
-* -*

(vi) Whenever reflection takes place, the component of a- b -


incident' ray parallel to reflecting surface remains
unchanged, while component perpendicular to reflecting |b|
surface along norinal) reverses in direction ' also
->

a I =
-i

a— ail

ei =ei| + e^

where, ^ = component of e parallel to mirror


= Gj - (Ci n)A

miuiiwmnwwwit/iw .

Fig. 1.27

Consider incident ray along^ unit vector given


ei=- + similarly e3 = xi + 3d unit vector along
reflected ray will be given by 63 = -a! + yj diverge.

Similarly consider on incident ray = :)d H- yj + zk on a ej_ = component of perpendicular to mirror


mirror.
= (61• A)n
10 . - -
OPTICS j
Using remark we get;
1 Concepts: (1) Plane mirror changes right handeS
62 = e||- e_L = Cj - 2n (e^ •n) co-ordinate system to left handed co-ordinate system ^and
vice-versa.
CORNER REFLECTOR
When incident ray suffers two successive
reflection.

From vector form of laws of reflection


s y'

Fig. 1.33

(2) The image of a horizontal object becomes'


vertical and the image of a vertical object becomes horizontall
if plane mirror is inclined at an angle 45° with horizdfital

(a) (b)
!_ _F'9'''-32

§2 = Ci ~2(ei-j)j ...CD
Simil^ly, 63 = 62 - 2(63 •i)i ...(2)
From eqns. (1) and (2), we get;
63 = Ci - 2Cei. j)j - 2[{e2 •i)} •i]i
(a) (b)
= Ci - 2(ei •j) - 2(ei •i)i + 4[(ei •j)j •i]i Fig.1.34

= §1 - 2[ei •i)i + (ei •j)j] (•.• i •j = 0) (3) The minimum height (or length) of a
or 63 =ei -26] mirror required for a person to see his complete'
03 = -ei
image in a plane mirror:
or ...(3)
Hence, 63 is antiparallel to e^. AB is the person with E as his eyes. M1M2 is a plane,
mirror in front of him. (Fig.1.35) .
Alternatively:
A'
Let, ei =xi + yj

then,
and 63 =-xi-yj
Clearly, 63 = - e^ ' ,
Such a combination of mirrors is used in periscope.
The result can be extended to three plane mirrors
arranged mutually perpendicular to each other. This h-
arrangement of three mutually perpendicular mirrors is also Flg.1.35
knovm as comer reflector. If the incident ray is represented
byxi + yj + zfe then after three reflections final reflected , The rays coming from the extreme and portions of his
ray is given by -xi-yj-xfe. Thus we always see our body, Le., A and B, should enter his eyes after reflection.
reflected image (put only upside down), independent of our The light ray AMj is incident ray and M-iE the reflected
•position. ray

MjFi = x(say) = N^E = -(aE) •M)


2
GEOMHRICAL OPTICS 11

I Similarly the light rays BIA^ and M2E are incident andl or AN^+N-^N2+BN2=H
^^reflected rays respectively ! or ANj +(MiE2 +£2^2)+ 6^2 =H
MjEi =j(sayD (BE). ...(2) ' or ~ +(x+y) +^ =H
ri ri
Adding eqns. (1) and (2) x + y = height of the mirror
x+y=length of mirror = —(AE + BE) = —(AB)
2' 2 U+y) =
(•n + i)
= —(height of the person). r\H
2 Height of mirror -
(11+ 1)
H i I Note that the height of mirror obtained above is'
Note: For a person of height H, with a plane mirror of height —, to)
2 j ^minimum since, light comingfrom the extreme edges of the1
view his complete image, the distance between mirror and jroom A and B is just able to enter the person's eye after\
person does not matter. However, the upper edge of the •reflection from the mirror. i
mirror should be at a level exactly midway between that of;
his eye level and the top most level or the lower edge of the^ r
mirror should be at a level exactly midway between that of Note : (1) Result is independent of the height or the eye level of)
his eye level and.the bottom most level. \ the person. , i
(2) E2M2=EP2=y. ' ' ' ' -i
y . I ^ r' 1 . I ! - "
I (4) If a plane mirror be fitted on a wall and a| But / + — = /?, height of eye level above the floor of the'
\pefson gazing at it, intends to view the complete^' n • . , . . . 1
jimoge ofthe rear wall ofheight H. | room

t\h
I If'r\ is the fraction of the distance ofthe person from the| y =
(Tl + 1)
jmirror, as compared to the length of the room. (Fig. 1.36). '
Fora person to see the complete imageofthe rear wall the)
From-similar triangles AM^Ni and EMiPi ' T\h i
lower edge of the mirror, i.e., M, should be at a level ~—
'"a. ' ^ Tl +lj
lower than his eye level. ' 1
(3) If the person stands at the middle of the room, then!
1 - - . • j

rrrrr: b-1

Fig.1.37
Fig. 1.36
Height (minimum) of mirror required is
•f
AXT X
or AN, = —
j]l I 11
...(V 2" H
Similarly, from similar triangle BM2N2 and EM2P2 + 1

y _BN2
or. BNn = - .••(2)
r\l I Ti The Minimum Width of a Plane Mirror Required for
Adding eqns. (1) and (2), we get a Person to See the Complete Width of his Face
ANi+BN2 =Cx + y)/rf. ...(3) The minimum width of a plane mirror required for a
Now, total height of the room = AB = H person to see the complete width of his face is (D-d)/2
where, D is the width of his face and d is the distance
between his two eyes. (Fig. 1.38)
12 OPTICS^

Field of view
'of Image I

(b)

Field of view •
of image I-,

Fig. 1.38

MM, =- D-±(D-d)
• 2 2 '

(D + d)
...(1)

CD + d)
and MM2=D-
4
Field of view
_ (3D - d) of Image I2
...(2)
4 (c)
Fig. 1.39
Width of the mirror = M1M2
= MM2--MMj Virtual object has no field of view i.e., it cannot be seen.
2D-2d lUustration 1: A point
'[ From eqhs. (1) and (2)]
source of light S, placed at a
. .. m-d} , distance L in front of the centre of 'I
• 2. -r. a mirror of width d. A man walks j l-«—L—H
in front of the mirror along a line ' ' N 2\r-
Field of View • . '
parallel to the mirror at a 'distance _-' _ Fig^l .40 (a)
This is the region where diverging rays from object or 2L from. it • as shown" in Fig. • ' " ' •
image are present. If our. eyes are present in field of view 1.40(a) Find the distance over which he can see the image of
and receivedivergingrays^then only we can see image. Field the light source in the mirror.
of view of image is decided by drawing extreme reflected or r

Solution: Man will see image


refracted rays. . . .'. • ,. . . , .
as long as he is in the field of view
of image.
From similar AMM' I and APQ7
Fov of image
d_m
L~ 3L
or PQ=3d
Fig. 1.40 (b)
Required distance = 3d
Field of view of image
r , Fig. 1.39(a)
13
GEOMETRICAL OPTICS

^^(4,3)
Fig. lE.2(a) shows a point object A and a plane mirror MN.
Find the position of image of object A, in mirror MN, by.
M ^(4.2)
drawing ray diagram. Indicate the region in which observer's'
must bepresent in order to view the image. (The region is, A
called field of view.) . , (2.0)
• M~ " Fig.1E.3 (a)

Solution: The image of point A, in the mirror is at


A' (6,0). Join A'M and extend to cut 7-axis at M (Ray
originating from A which strikes
; N , the mirror at M gets reflected as
S :Fig.1E.2(a)f ^ j the ray MM' which appears to
come from A'). Join A'N and
extend to cut Y-axis at N (Ray
Solution: See Fig. 1E.2 (b), consider any two rays originating from A which strikes
emanating from the object and N2 are normals; the mirror at N get reflected as
i-i = ri and 12 = ^2 the ray NN which appears to A"
come from A'). (6.0)
.A'
Fig.1E.3 (b)
From geometry.
M'=(0,6)
N = (0,^. AfAT is the region on Yaxis in which reflected
rays are present.
Illustration 2: A ray of light is incident on a plane
mirror at a certain point. The mirror is capable of rotation
about an axis passing throughX as shown in Fig. 1.34. Prove
Fig. 1E.2 (b)
that if the mirror turns through a certain angle, the reflected
ray turns through double the angle.
The meeting point of reflected rays Solution: LetXYbe the position of mirror initially and
Rj and R2 is image A\ Though only two mirror be rotated to a new position XY through an angle 6
' A'' about an axis passing through X. ........
rays /ace considered it must be
understood that all rays from A reflect
frommirror such that their meeting point
is A'. To obtain the region in/which
reflected rays are present, join A. with
the ends of mirror arid extend. The
following figure shows this re^on as
shaded. In Fig. 1E.2 (c) there are no Q

reflected rays beyond the rays 1 and 2, Fig.1E.2 (c)


therefore the observers P and Q cannot
see the image because they do not receive any reflected ray. Fig.1.41

5i = Deviation in first position of mirror


= 7C - 2(j)

'•Find the region on Y-axis in which reflected rays are present. $2= Deviation in second position of mirror
'.Object is at A (2,0) and MN is a plane mirror', as shown in = It - 2(^ - 0) [••• Angle of incidence in
\Fig. lE.2(a). , __i second position = (j) - 0] '
5i - 62 = 7t - 2((t) - 20) - (Jt - 2^) = 20

Tr.-2.43 -"^0
•14

Hence, reflected ray rotates twice the angle turned by = 120° anticlockwise
the mirror.
62 =180° - 2 x30°
Remarks
= 120° ;
(2) The angle turned by the reflected rayis twice the angle
' turned by the mirror. So, if a mirror turns through a certain .-. Total deviation= 6j + 82
angle, keeping incident rayfixed the reflected turns through = 240° anticlockwise or 120° clockwise
double that angle in same sense i.e., if mirror rotates clockwise Illustration 4 : Two plane mirrors are inclined to each
then reflected ray will rotate clockwise. other at an angle 0. A ray of light is reflected first at one
. .(2) The above statement-is mirror and then at the other. Find the total deviation of the
valid for rotation of mirror ray.
about any' point. (See Solution: Let
illustration 2).' '
a = Angle of incidence for Mj
(3) If mirror is kept fixed
and incident ray is rotated then
p = Angle of incidence for M2
inddent rayj Sj = Deviation due to
reflected ray will rotate in
.opposite-sense by same angle. - 82= Deviation due to M2
(4) If mirror and incident ray both are rotated then net
rotation suffered by reflected ray will be algebraic sum of
rotation suffered^ by reflected-ray due to mirror rotation and
incident ray rotcitiqn separately keeping sense, of rotation in
mind. ' " , . . . ,

ANGLE OF DEVIATION
When a.light ray strikes a ; --.r -
plane mirror- at an angle of
incidence i,the deviation of the O wWTOTOTOTOTTOTOWTOWTOTO M
reflected ray -is (7t-20. The
angle of deviation is the angle
-made-.by. the reflected ray with Flg.1.45
the direction of incident ray.- ..
From Fig. 1.45 - '• • •
.• If several plane mirrors-are,' pig; 1.43
placed on the path of a-light ray, ' 81 = Tie 3a, 82'= 71-2p
each mirror deviation ray given by the deviation can he Both deviation are iii same sense i.e., anticlockwise
clockwise-or anticlockwise. In thc above^case, the,deviation 8net'= Total deviatioii= 81+82
is anticlockwise. In case of multiple,re^^ctipnsi-clockwise • ='2jc.L2(a + P)'
and anticfbckwise deviations are given opposite signs, net
deviation is the algebraic sum'of the deviations due to the NowinAOBC,
individual mirrors. or .,(9p°-a)' +C90°-p) +0=180°
Illustration 3: Two plane or ' ^ a +p =e
riiirro'rs are inclined to each other Hence, S^et = 2;c - 20
such that a ray of light incident on Illustration 5: Calculate deviation suffered by
the first mirror and parallel to the incident ray in situation as shown in Fig. 1.46 (a) after three
second is reflected hom the second
successive reflections. ' ' •
mirror parallel to the first mirror.
What is the angle'between the two
mirror and find' total deviation^ •TOVWVWWW

producedin the incidentray due to


the two feflectioiis. Fig.1.44
Solution: From Fig. 1.44
30 = 180°
0 = 60° Fig. 1.46 (a) •

5, =180°- 2x30°
GEOMETRICAL OPTICS 15

Solution: §2 =it-2(a-0i). (anticlockwise sense) ...(2)


Total deviation produced is
^Net — -|- S2 ~ 2(71 —C(),
which evidently depends only on a, the angle between
the two mirrors.

Some Applications:
(1) If the mirrors be mutually perpendicular, then net
deviation Is n. The light ray reverses its direction,
i irrespective of its angle of incidence.
Fig. 1.46 (b] (2) if the incident light ray and the emergent light ray are
parallel to the two mirrors, then net deviation = (k -i- a).
From Fig. 1.46 (b) 7c + a=2(7c-a)
7C
5i = 180°-2 X40° = 100° anticlockwise or 3a = 7c or a =

82 = 180°-2x 20°= 140° clockwise


The two mirrors are inclined at an angle —(or 60°) to each
S3 = 180°-2xl0°= 160° clockwise * 3.1
6 =Total deviation 100°+140°+160° ! other. _
= 100° clockwise or 260° anticlockwise
EXTENDED OBJECT
An extended object like ' AB shown
combination of infinite
number of point objects from
\Show that the angle of deviation of a light ray striking two
'plane mirrors, successively, kept inclined to each other, A to B. Image of every point
{depends only upon t^ angle between the mirrors. , object will be formed
individually and thus infinite
Solutioni Let the two plane mirrors and M2 be image will be formed A will
kept inclined at an angle'a to each other. Let the incident ray be images of AC will be Fig. 1.47
strike the plane mirror at an angle of incidence 0^. (Fig.lE.4) image of C,B' will be image of
B etc. All point images together form extended image. Thus
Original ray "extended image is formed of an extended object - • •"
Properties of image of an
B'
extended object, formed by a
plane mircor:. .
Final ray ' ' (1)' Stee of'exfehded-bbject' =
Size of extended'image.'
'A'
' ' (2/ The image_ of erect, if the
extended'object is placedparallelto L . —Ji'St.!:!?.
the mirror.

. (3) The image is inverted if the extended object lies


perpendicular the plane mirror.
Fig. 1E.4
B B'

Deviation produced at first reflection is -


= (jc- 2Bi), (anticlockwise sense) •••(1) IB.
0= ^•Q
A'i •B'

When the light ray strikes the mirror M 2 at an angle of


incidence 02 given by - - Object
Image:
02 - tc-[ZMiFM2 -t-0i]
(a) (b)
= M1PM2) - 0i] = a - 0.
Fig.1.49
[ M1OM2P is apyclic quadrilateral so M1PM2 = 7t - a]
The deviation produced due to the reflection on the
mirror M2 is
16
OPTICS

iNote: It an extended horizontal object is placed infornt of amirrorj ^o/M ~ ^ co-ordinate of object w.r.t. mirror
inclined 45® with the horizontal, the Image formed will be;
vertical. See Fig. 1.50. ^i/M = ^ co-ordinate of image w.r.t. mirror
Horizontal incident"
rays on the mirror
For plane mirror
get reflecte In vertical i X.O/M = -XI/M
direction. So the image
l~"
of extended object M
wiil be vertical

lights fr^^pointsAand B
Common direction of incident

Xi
O §

BE=EB
AF=FA'

Fig.1E.5(c) .

Differentiating both sides w.r.t. 'tf


Fig. 1.50
— CXo/m) =

rv. rlx

Fig. 1E.5shows an object AB and a plane mirror MNplaced K -


iparallel to object. Indicate the mirror length required to see
the image of object if observer's eye is at E. • • •
Sirndprly, YyM ^^oim ^
M •
Differentiatingboth side w.r.t. 't' we get
(V i/M )y
Jy = (V 0/m)y
In nutshell, for sloving numerical problems involving
calculation"of velocityof image of object with respect to any
observer, always^ calculate velocity of image first .with
respect to min-br. using following points^
(V.
N r
Fig. 1E.5 (a)

Solution; ^I/Jw: - C^I/M )i| + (y


Velocity, of image with respect, to required observer is
A'
then calculated using basic equation for,relative motion.

^A/B ~ Velocity of A with respect B

; PartCD isusedtoview theimage = ^a-Vb


B' (i) If an object moves towards.(or away from) a plane
mirror at speed Uj.the image will'also approach (or recede)
at the same speed v, and the relative velocityof image with
Fig. 1E;5(b) respect to object will be 2v, as shown in Fig. 1.51.
Velocity of Imaged "
Let, X^'^= -X co-ordinate of object w.nt. mirror
~ ^ co-ordinate of image w.r.t mirror
GEOMETRICAL OPTICS 17

5 -v^ .1 The object and its image V|| = Vyj + = velocity component of mirror in the
(A) move towards each other
with same speed v plane of mirror.

Rest M/g = V I + Vi

The Image of a stationary j Velocity if image is not effected by motion of mirror in


jo —I object move with 2v if the | its own plane i.e., V[| + does not change the
(B) mirror moves toward it with v!
i Rest
velocity of image.
Fig. 1.51
^ A
Only the Vj^ = v^^ihas effect an velocity of image. Thus
(ii) If the mirror is moved toward ( or away from) the
in the above situation
object with speed v, the image will also move towardfor
away form) the object with a speed 2v, as shown in figure. V,/„ = - V
Vs Olg+^^Mjg
Case (i) i Fixed Mirror
Case (iv) General case

'""C/M %Mi

"O/M - 1^1/M

Fig. 1.52

If mirror is fixed and object moves in plane parallel to


mirror, velocity of object is equal to velocity of image.
Fig. 1.55
Case go r '
Given = -2i + 2j + 2fe

Fixed VmI
object Velocity component Velocity component
Fig.1.53 ofmirrorA. to mirror ofmirrorin its plane

Given Vo^g = + j
= -V 0/g Mlg

V 01 Vo,|
as
o/g = 0 velocity, of object Velocity component of

^l/g'
perppdicular to mirror object parallel to mirror

Case (iii) Note that v has no effect on velocity of image.


/:
[v//g]x =-[Vo/g]i +[2nj/g]x
Component _L to mirror
= -i + 2[-2i].
Fixed
A

objet = -51-

; Fig.-1.54

If mirror has velocity Component |[ to mirror, - . .


Thus
P/g
= -5i +j + lc . -
Vm/s + •
A

Vxi = velocity component of mirror-perpendicular to Illustration 6 : A point object is ihoyihg with a speed v
before an arrangement of two mirrors as' sHown in Fig.
mirror.
1.56(a). Find the velocity of image in mirror with respect
to image in mirror M2.
OPTICS

2g
ar =

3ct
Accelerate of image in AjB= 2a^ =

Acceleration of image in AB w.r.t. that


Mo

Fig. 1.56 (a)


CD =^ +^ =^m/s^
2 3 6

Solution. Velocity of image in mirror Mj iandM2 is as Image formed by two Plane Mirrors
shown in Fig. 1.56(b). If rays after getting reflected from one mirror strike
second mirror, the image formed by first mirror will function
as an object for second mirror, and this process will continue
for every successive reflection.

W
'Fig. lE.7(a) shows dpoint object placed between two parallel
mirrors. Its distance from is 2cm and thatfrom M2 is 8j
•cm. Find the distance of image from the two mirrors^
considering reflection on mirror Ml first.
'Ml M2 i , f
Fig. 1.56 (b

2cm
Vy2 = Velocity of I^w. r.1.12 •O

8cm
= Vi-Vo
' Fig.1E.7(a)
or 'vy2 = 2vsin 0
- Soiutioh: To understand how images are formed see
the following figure and table. You will required to know
Two blocks each of mass m lie on a smooth table. Th^ are - that symbols like.Ijai.st^ds for. See the Fig., IE.? (b)
attached to two other masses as shown in the Fig. 1E.6. The
{pulleys and strings are light. An object 0 is kept at rest on the, ). i
'12

table. The .sides AB and GD of-the two blocks are made I . '
T^This last number'T indicates
^"''thallight'ra/s are reflected from
reflecting. The acceleration of two images'formed in those two • rnirror 'V i.ei M^ . .
reflecting surfaces w.r.t. each other is:',-^ —7*'-li2is object-in this case •

Vt\\\tR"\n\U
2 \

Fig. 1E.6

'(a) 5g/6 (b) 5g/3 '121 12 '1212


.i

\(c) g/3 (d) 17g/6

Solution: We ^ow Aat


• V/=2Vm+Vo
differentiating Qj = 20^ + cio , Rg. 1E.7(b)

flo =05 Ot = 2aM


GEOMETRICAL OPTICS 19

Incident \ Reflected \ Reflected \ Object \ Image


rays \ by \ rays \ Object Image
distance \ distance

Rays 1 Ml Rays 2 0 h AO = 2cm All ~


Rays 2 M2 Rays 3 ' h hi B/j = 12cm Bli2= 12cm
Rays 3 . Ml Rays 4 hi hii ^7^2 = 22cm AI121 = 22cm
Rays 4 M2 Rays 5 hii hill Bhi - 32cm
Ans so on ' •
Similarly images will be formed by the rays striking mirror M2 first. Total number of image =~.

NUMBER OF IMAGES FORMED


BY TWO INCLINED PLANE MIRRORS
When mirrors are
parallel : In this case,
infinite images are formed
due to multiple reflections. : Extension of mirror umUMUimMinn
Image formed " after
reflection at is object for
reflection at M2. Thus each
Fig.1.57.when mirror are parallel]
image acts as object for
second mirror.
Flg.1E.8(b)
8 For incident rays 1,2 object is O, and reflected rays 3,4
form image J2-
•Consider twoperpendicular mirrors. and and a pointi Now rays 3,4 incident on (Object is I2) which reflect
object O. Taking origin at the point of intersection of thel as rays 5, 6 and fomiTmage 121- Rays 5, 6 do not strike any
[mirrors and the coordinate ofobject as(x,y), find thepositionl mirror, so image formation stops.
and number of images. 1
I2 and J21, are equidistant from Mj. To summarize see
Solution: Rays 'a' and V strike mirror only and the Fig. 1E.8 (c).. . • .• .. 1 , •
these rays will form image at (x,-y), such that O and are •t .• -x. • ! First step •
. (M2 forms image12,'
equidistant from mirror M^. These rays do not form further ofobjkt p)
image because they.dq notsfrike anyihirfdr'again. Similarly '(M, forms image 12
^ ofobject j2)-^r, .
rays'd' and 'e' strike mirro'rMa onlyand these rays will form
image /2 at such that'D and 12 are equidistant from
(x.y)
mirror M2.
Extension of mirror M,.„

0 x,y a
^ F'9-.1E-_8.(c) !
Forrays reflectingfirst from and then from M2, first
image J^Cat will be formed and this will function as
object for mirror M2 and then its image 7^2 [at (-x,-y)] will
be formed. '
Fig.1E.8(a
Ij2 and 721 coincide.
Now consider those rays which strike mirror M2 first .*. Three images are formed.
and then the mirror Mj.
20 OPTICS

Concept: (i)Note that the third image isformed due to |


rays undergoing two successive reflection. Also, object and its'
images lie on a circle whose equation is given by\ 'Two mirrors are inclined by an angel 30°. An object is placed'
+ i making 10° with the mirror My Find the positions offirst two[
(ii) When an object is placed injront of arrangement of iimagesformed by each mirror and the total number of images',
three mutually perpendicular mirrors, then total seven imagesl \using (i) directformula (n) wui^ng the images. J
'are formed. Further, object and its images lie on a sphere
whose equation is given by + y? + = a^+ b^ + c^ Solution: Figure is self explanatory
where a, b and c are co-ordinates of object. Number of image
' - •
Locating All the Images Formed by Two Plane
Mirrors :
Consider two plane mirrors and M2 inclined at an
angle 0 = a + P as shown in Fig. 1.58.
Point P is an object ^andsoon|^^(jj+2p)
kept such that it makes
angle a with mirror X''' '
and angle p with mirror ..iP (object) Fig. 1E.9
1^2- Image of object P
360°
formed by denoted by /l2(P) (i) Using direct formula ; = 12 ( even number)
30°
Jj, will be inclined by .•'''li2(2a+P)
angle a on the other side number of images = 12-1 = 11
V- •*
and so on
of mirror . This angle is (ii) By counting. See the following table
written in bracket in the __ _
Fig.1.58
figure besides I Similarly Image formed by Mirror Images formed by Mirror M2
image of object P formed by M2, denoted by I2, will be (angles are measured from (angles are measured from
inclined by angle p on the other side of mirror M2. This the mirror /W^.) the mirror A72.)
angle is written in bracket in the figure besides I
Now I2 will act as an object for which is at an angle
(a + 2p) ft-om Ml. Its image will be formed at (a + ^);on the
opposite side of M^. This image will be denoted as 121, and
so on. Think when hsi will process stop." '
Hint: The virtual image formed by a plarie mirror must
not be in front of the mirror or its extension. 100'
110°
i305.
Number of Images formed by Two Inclined
140'
Mirrors:
360°
(i) If = even number ; number of image
360°
-1 Stop because next angle Stop because next angle
'will be.more.than 180°. _will.be more than_180°_ !
360°
(ii) If = odd number; number of image To check whether the final images made by the two mirrors coin
0
cide or not: add the last angles and the angle between the mirrors!
360°
—-1, if the ^object is placed on the angle If it comes out to be exacdy 360°, it implies that the final images
0 formed by the two mirrors coincide. Here last angles made by the
bisector. mirrors + the angle between the mirrors= l60°+170°+30°= 360°.
"^60° "^60° Therefore in this case the last images coincide. Therefore the num
(iii) If = odd number; number of image = , if ber of images = number of images formed by mirror + number
0 0
of images formed by mirror M2 -1 (as the last images coincide)
the object is not placed on the angle bisector. = 6-1-6-1=11
360°
(iv) If t integer, then count the number of images
0
as explained above.
21
GEOMETRICAL OPTICS

le. 10

In Figs. lE.lO(a) and (b) two mirrors are inclinedat 90° and
120° respectively. Trace the rays as it is reflectedfrom the two
mirrors^ Mi,M2.

M,g Incident
ray

Fig. 1E.11

The rays leading to the formation of image are


shown. Notice that/3 would be an image of/j if M2 were
(a) (b)
extended downward. Similarly, I2 is an image of/2 in the
'virtual mirror' formed by extending to the left.

12

(a)What is the minimum length of mirror required for a


person to see his or herfull height?Assume that the eyes are a
distance a below the top of the head and a distance b above
the feet, (b) Does the person's horizontal distancefrom the
mirror matter ? (c) Does the vertical position of the mirror
matter ?
(C) (d)
Fig. 1E.10
Solution: (a) Rays enter the eye from the feet and
the top of the head after reflection at the mirror, as shown in
Solution: The law of reflection is applicable at each Fig. 1E.12. We know that the angle of incidence is equal to
surface. From the figure. the angle of reflection. Light from the feet reaches the eye
p + 9O°+9O°-0 = 180° a/2

P-e ..
Therefore the reflected ray is antiparallel to the incident
ray, whatever be the angle of incidence.

An object is placed between two perpendicular plane mirrors.


How many images are seen ? Fig. 1E.12

Solution: in Fig. lE.ll we use the tail of an arrow


as the object. Its images in mirrors and M2 are and /2 after reflection at point B, which is b/2 above the floor. Light
as shown. In these cases, light reaches the eye after one from the top of the head reaches the eye after reflection at
reflection. In addition, light can enter the eye after being point A, located a/2 below the top of the head. The person's
reflected by both mirrors. total height is a + b, and the required length of mirror is a/2
+ b/2, which is 50% of the full height of the person.
(bj No.
Cc) Yes, the bottom of the mirror must be at a height b/2
22
OPTICS

'Rearview mirrors in cars have a day setting and a night\


\setting. The night setting greatly diminishes the intensity of
\the image in order that lights from trailing vehicles do notj
blir^ the driven How does such a mirror work ? ]
Solution: Fig. 1E;13 shows a cross-sectional viewof a Fig. 1E.14
rearview mirror for each setting. The unit consists of a
reflective coating on the back of a wedge of glass. In the day Solution: (a) The image will appear to be at restw.r.t.
setting [Fig. lE.13(a)], the.light from an object behind the the particle, at the instant the velocity of the particle is
car strikes the glass wedge at point 1. Most of the light parallel to the mirror.
enters the wedge, refracting as it crosses the front surface, usina-gt
i.e., — = tan0 or - tan0
and reflects from the back surface to return to the front ucosa
surface, where it is refracted again as it reenters the air as
Vcosa(tan a - tan 0)
rayB (for bright). In addition,.a smallportion of the light is or t =
reflected at the frontsurface ofthe glass, as indicated byray
D (for dim). (b) Path will be a straight line. Because at any instant
Reflecting — - -- --- velocity of the particlecan be resolved into two components,
side of mirror one parallel to the mirror and the other perpendicular to it,
parallel components of the particle and its image are
identical. . Hence the path will be a straight line
perpendicular to the mirror.

[An oblong object PQ of height h stands erect on a flat


Incident incident
light light \horizontal mirror. Sunrays fall on the object at a certain
angle. Find the length of the shadow on a screen placed
Day-time setting Night-time setting \beyond the shadow on the mirror.
(a) (b)
Flg.1E.13

This dim reflected light is responsible for the imagethat


is observed when the mirror is in the night setting [Fig. \> Q
1E.13 (b)]. In this case, the wedge is rotated so that the path
followed by the bright light (rayB) does not lead to the eye.
Instead, the dim light reflected from the front surface of the
Fig.1E.15(a)
wedge travels to the" eye; 'and the bfighttiess of trailing L„
headlights does not become a risk.
Solution: The
formation of shadow on
14
the mirror and on the
2a a
'A plane mirror is inclined at an angle 0 with the horizontal, screen is shown in Fig. h sec a
1E.15 (b) in compliance
Surface. Aparticle is projectedfrom point P(see Fig. 1E.14) at with the principle of
f = 0 with a velocity v at an angle a with the horizontal.The^^ Fig. 1E.15 (b)
jmage ofthe particle ispbservedfrom theframe oftheparticlei rectilinear propagation of
'projected. Assuming the particle does not collide the mirror,' light and law of reflection.
^nd the PS is the shadow on the mirror and P'Q' is the inverted
fa) time when the image will come momentarily at restw.r.t.' shadow ofPQon the screen. Leta = angle of incidence. Then
[the paiticle, (^) path ofthejmage as seen by the particle, i PS = h tan a andQS = h seca. From the property oftriangle,
P'Q' = 2(/z seca) cosa = 2h
GEOiVlETRICAL OPTICS 23

i Example 16 i can only see the entire image of the object if it is at one of
the points enclosed between the rays AB and CF.
Determine graphically the positions of the eye when an
observer can simultaneously see in a flat mirror of finite
dimensions the image of a point and a section of a straight
A mirror 1 m high hangs on a wall. A man stands a distance
line placed w.r.t. the mirror as shown in Fig. lE.l6(a). of 2 m away from the mirror. What is the height of the
portion of the opposite wall in the room that can be seen by
/ the man in the mirror without changing the position of his
head? The wall is 4 mfrom the mirror.
Fig. 1E.16(a)
Solution: The image of the
wall will be behind the mirror at a
Solution: observer can see only when the eye is distance I2 = 4m. If the eye is
placed inside the triangle DEHlimited by the rays DG and EF placed at the point A (Fig. 1E.18) it
[Fig.lE.16(b)] will see only the rays coming from
F
all the points in the section of the
Fig. 1E.18
wall image DE after the reflection
in the mirror EC. Thus, the section of the wall visible in the
mirror will have dimensions

X = = 3m

19

A narrow beam of light S is incident on a dihedral angle


Fig.1E.16(b)
a = 60° formed by identical flat mirrors OM and ONsecured
on axis 0 [Fig.lE.19(a)]. After being reflected from the
17 mirrors the light isfocussed by lens L and gets into stationary
receiver R. The mirrors rotate with a constant angular
An object O'O and a mirror AC are placed as shown in Fig. velocity.
lE.17(a). Construct the image of this object in the mirror.
Where should the eye be placed to observe the image of the _
entire object ?
O' '

\.

Fig.1E.19(a)
Fig. 1E.17(a)
What part of the light energy of the beam will reach the
Solution: See Fig. 1E.17 receiver during a time that greatly exceeds the period of
(a). The rays coming from the rotation if the beam passes at a distance a from an axis equal
point 0' will be propagated inside to half the length of mirror OM ?
the band restricted by the straight
lines AB and CD after reflection Solution: The beam reflected from mirror ON forms
from the mirror. The rays coming with the incident beam an angle 6 [see Fig. lE.19(a)] that
from O will be inside the bands AE does not depend on the angle of incidence i. From triangle
and CF. ABC, we have ^ = 180°-2(i + r) and in triangle OAB, we have
The rays coming from all the a + (90°-i) + (90°-r) = 180°. Hence, 0 = 180°-2a = 60°.
points on the object •will only arrive When the mirror rotates, the direction of the reflected beam
at each point in space between the Fig.1E.l7(b) does not change.
straight lines AB and CF. The eye
!24
OPTICS-

Thus, if the beam that fell on mirror OM is reflected Similarly, it is easy to show that DE is the reflection of
from mirror ON, it will always get into the receiver. As can CD, etc. i.
easily be seen in Fig. 1E.19 (b) showing two extreme This solution of the problem is not unique, since thd
positions of the mirror at which the beam gets into the beam should not necessarily be sent initially to mirror ab)'
'Cm

MN'

(a)Inanexperiment light is repeatedly reflectedfrom moving]


mirrors (Fig. 1E.21) The mirrors were placed on discs
revolving in different directions. I

Fig.1E.19(b)

receiver (OM, ON and OM', ON'), this occurs during


one-sixth of a revolution. For this reason one-sixth of all the
energy of the beam will get into the receiver during one
revolution which is a sufficiently large interval of time. . Flg.JE.21^ • I
|(D The angular velocity oj of rotation of the discs being\
lknown,find the angular velocity of rotation of a beam that
In what direction should a beam of light be sentfrom point Ai 'is consecutively reflected n times from the mirrors. , •'
[Fig. lE.20(a)] contained in a mirror boxfor it to fall onto! \(ii) Determine the linear velocity of the n'^ image at the
point B after being reflected once from all four walh ? \ '^moment when the mirrors are parallel to each other and thdr
•reflecting portions move with a velocity v in different
A* [directions.
•B \(b) Solve thejnybl^ ifthe discs rotate injhesame direction.
Solution: (a) (i) The beam reflected from the first
r Fig. 1E.20(a) ? i mirror forms an angle 2a with the incident beam (a is the
' I _ • ; j
1 angle of incidence). During the time t the mirror will turn
PointsA and B are in oneplane perpendicular to the walls of\ through an angle cot and the new angle of incidence will
\the_box (i.e. in the plme of the drawing). j become equal' to. a.-i-cot, as will, the angle .of reflection!
Therefore, the angle between ihe,incident and reflected
Solution: Let lis ' 0 I beams.will increase .by.2cpt, L.e., the reflected hearn will turn
plot the image of point' I '
through an,angle 2a)t-.' ^ - , . ' ..
B in mirror bd [Fig. J
IE.20(b). Let us th'en_ I j . ' -In view of this,^the'angle'dfdncidence.on the second
construct image Bi'-in | • mirror, provided it does,not rotate,.would be'p -f- 2cot, where
mirror cd. Also, B3 is P is the angle of incidence with immobile discs. But the
mirror also revolves through the angle cot during the time t,
the image of B2, in EX.d and therefore the angle of incidence becomes p + Scot. The
mirror ac and B4 is the angle of reflection will be the same. Thus, after two
image of B3 in mirror B, '2
reflectionsthe beam will turn through the angle.Scot from its
ab. Fig.1E.20 (b) direction with immobile mirrors. After there reflections the
Let us connect ~ 7T ' ^ beam-will turn through Scot and ^e'r n reflections through
points A and B4. Point C is the point of intersection of ab (2n-l)2cot. In this way its angular velocity will be
with line AB4. Let us now' draw line B3C from B3 and Q = C2n- l)2co.
connect point D at which this line intersects ac with B2, E (ii) When the mirror moves from the source with a
with Bj and F with B.. velocity v, the image will move away from the source with a
velocity 2v and from the second mirror with a velocity 3v)
It can be stated that broken line ACDEFB is the sought Therefore, the second image moves.with a velocity 3v w.r.t.
path of the beam. Indeed, since B3CB4 is an- isosceles the second mirror and with a velocity Av w.r.t. the source.
triangle, CD is the reflection of beam AC. The velocity of the third image w.nt. the source will be 6u
and the velocity of the image 2nv.
GEOMETRICAL OPTICS 25

• Cb) (i) When the first mirror turns through an angle cot H
a sina "x p a sin^ a = Hp
the reflected beam will turn through an angle 2(0t. Hence, sina
the angle of incidence on the second mirror will also
H =
_ asin^a _ a.h^/a^ _ ^
increase by 2cot, and if the mirror did not revolve, the angle ^P P " ap
of reflection would also increase by 2cot. After two
10' 100x108
reflections the beam would turn through 2a)t as compared Here H = - 900 cm = 9 m
12
with the case of immobile mirrors. 12 X
Since the second mirror does rotate, however, the angle 108

of the beam incident on it decreases by cot during the time t.


The angle of reflection decreases by the same amount and
for this reason the reflected beam will travel in the same
direction as with immobile discs. •A plane mirrorisplaced along the xz-plane and an object Pis^
Since this line of reasoning may be adopted for any two placed at point (0,a) the mirror rotates about z-axis with',
consecutive reflections, the angular velocity of rotation of constant angular velocity co. Find position and velocity of
the beam subjected to n reflections will be Q = 0 if n is even image as a function of time t < —
and Q = 2(0 if n is odd. I 2(0
(ii) The first image moves away from the source tvith a
velocity 2v and from the second mirror with a velocity v.
Therefore, the second image moves w.r.t. the second mirror P*(0,a)
with a velocity v, i.e., it is immobile w.r.t. the source.
Reasoning similarly, we find that the sought linear
velocity of the image is zero if n is even, and 2v if n is , \\\\\\\\\\\\\i\\\\\\\()
odd.
U 22 Fig. 1E.23(a)

IWhen sunrays pass through a small opening in thefoliage at


Solution: to = e/t => e = (ot
yhe top of a high tree, th^ produce an ellipticalspot on the'
'.ground. Themajor and minor axes of the ellipse are a = 22 cm y = -(2acos^6- a)
land h = 10cm respectively. What is the height of the tree ? y = -a(2cos^0.-1) => y = -a[2cos^ (Ot -1]
\The angular dimensions of the sun's disc are p = rod; i y = -acos2(ot
20_8 ., •.

Solution: The cdne of rays through'the foliage "will (Oa),


have elliptical section by the ground plane. The section" of
the cone perpendicular its axis is a circle-of diame'ter b.
While the length of the section by the plane through the axis
of the cone and the foot of the'tree is elongated to ol the side CN

perpendicular,to it remains.unchanged. ' .

X= 2acos0sln0
X= asin20
Foliage
hole

(O.-a)

Flg.1E.23(b)

X = asin20 = asin^t
Fig. 1E.22
^ A /V

= (asin2(0ti-acos2£0tj) •
sina - - where a = ZOAD
a dx
VI = — = acos2(ot X 2(0 = 2(ocos2(ot
Now D£=ODxP
^
=-^p
• • sina
dt
dv
Vv = — = 2a(0 sin 2(ot
Also DE = a sina dt
26
OPTICS

24
im 26

light ray parallel to the x-axis strikes the outer reflecting An elevator at rest which is at floor of a building is\
\surface ofasphere atapoint (2,2,0).lts center is at the point having a plane mirrorfixed to itsfloor. Aparticle isprojectedl
|(0,0, -1). The unit vector along the direction of the reflected' [with aspeed 42 m/s and at45° with the horizontal as shown]
iraysis xi +yj + zk. Find the value of •in thefigure. At the veryinstant ofprojection, the cable of the
X'
\elevator breaks and the elevator starts falling freely. What^
iwill be the separation between theparticle and its image O.Si
Solution: n =^i±^i±^ (2,2, 0
[second after the instant ofprojection ? ;:l
3 mmininiiin

e = -i
Using r = e - 2(e. n)n
0,0,-1)
2(-2)(2i+ 2j +fo u=V2 m/s

3 3 45
UVUWWWWWW

..=-i+|c2i+2j +fe) Fig. 1E.24 j . Mirror [


; Flg.1E.26{a) {
-i + 8j + 4k
Solution: = -gtj;^p/g = i+ci-gt)j
25
usin0 = V2sin45°= Im/s r mm/

uCOS0 = 42 cos45° = 1m/s


{The mirror of length 21 makes 10 revolutions per minute'
•about the axis crossing its midpoint O and perpendicular to ^p/E =i +j;|^p/El= V2m/s
\the plane of thefigure. There is a light source in point Aand. in O.Ssec= 42 x 0.5 = m
an observer in point B of the circle of radius R drawn around, 42
centre O (AOB = 90°). y 1
sin45° = —rT=-'>y = — = 0.5m I P'
!(a) Along what curve does thevirtualimage of thepoint r like a/V2)2
\light source A move ? I Fig. 1E.26 (b)
PP' = 1 m
^(b) At what speed does the virtual image of A move ? i
fc) What is the. ratio l/R if the observer B fir^t.sees^the light n p m j Z7
the diigle of the minyr is.^ = 15%^? -
; I. I . ' '
Two largeplane mirrors OM and ONare arrqnged as shown.
Solution: (a) Circle
Find the length of the part of large screen SS\ in which two
(b) = lOx—= —rad/s'' [images of the object placed at P can be seen ?
. 60 3

. O
0)1=2(0^ '271 J ,
=~vad/s

Vi =o)iR = —ilm/s

(c) In AOA'C Fig. 1E.25


sin 30° sin 135°
S'i
I 'R 1m

l._ 1 Fig. 1E.27 (a)


21 r42
1 = J-
'R 42
GEOMETRiai OPTICS

Solution:, WSVVWVNVWW

Solution: (c)

__Fig.1E.29Jb)
Fig.1E.27 b) SA = 2x, SB = 2(x + I)
AB = SB-SA = 21
Length of the patch,will be = 21
X - ltan30° = —= m
V3 wwwwwww

Similar ray diagram for mirror N will give total part


= 2x = A
V3

;A man 2 m tall stands 5 min front on a large vertical mirror. |


[Then the angle subtended at his eye byhis image in theplane'
'mirror is nearly. j SA = 2x '
.(a)0.4rad (b) 0.2 rad (c) 0.2° (d) 0.4° ; SA=2(x + d) ' '
SA'-SA = 2Cx + d) - 2x = 2d
So if plane mirror is moved then patch will move 2d in
n
same time so velocity of patel with be 2v if velocity of mirror
2m 2m
Solution: (b) SPHERICAL MIRRORS
< 5m ^
U-5m-» Images Formed by Curved Mirrors
• _ _ Fig.1E.28 A spherical
2 mirror is a reflecting
tan0 = —,tan0 =
10 surface whose shape is
a section of a sjpherical.
0 = 0.2
surface. If the inside' 'Sbherlcal
- , , 1 i . r mirror
surface of the mirror is
polished, it is a
concave mirror./^ If
A point source of length S is located on a wallA plane mirror the outside surface is
iM having length I is moving parallel to the wall with constant polished, it is a convex
Concave mirro
\velocity v. The bright patch formed oh the wall by reflected'' mirror. The radius of Light ray
'light will ' { the spherical surface is Principal axis
1(a) move with uniform velocity v and will have a length 21 ' the radius of
curvature R of the
\(b) move with uniform velocity 2v and will have a length I,
mirror. The centre of
'(c) move with uniform velocity 2v and will have a length 211 the sphere of which the
\(d) move with uniform velocity but will have a changing' mirror is a part is called Concave mirro
\length. the centre of
•V I
curvature. A plane Principal axis
wwwvwwwwwwwww

-m I mirror can be treated


as a special case of a
S Wall spherical mirror: one
which has an infinite
Fig. 1E.29 (a) Fig.1.59
radius of curvature.
28 OPTICS

A mirror with its reflecting surface shaped like a normals drawn at other points of the mirror are called
paraboloid revolution is called a parabolic mirror auxiliary (or secondary) optical axes. The rays
(mirror formed by rotating a parabola about its axis of passing in parallel to the principal optical axis near it are
symmetry). All rays travelling parallel to the axis of the called central rays.
parabola will go through the focus of the parabola. This
point is known as the focal point of the mirror. The
converging rays are said to be focussed. Spherical mirrors
behave similarly to parabolic mirrors for rays that travel
close to the mirror axis, paraxial rays. A spherical mirror Incident central beam

is not perfect in focussing the parallel rays. The farther a ray Principal optical axis
is from the mirror axis, the greater is the error in focus. This
'defect' of spherical mirror called spherical
aberration.

Fig. 1.62

The relation F = R/2 given above (where F is the focal


length and R is the radius of a mirror) is not exact. It is the
= Vertex
more exact, closer the incident central ray to the principal
optical axis of the mirror. Indeed, for a concave spherical
mirror in Fig. 1.62 in AAFC, the angles p = a and y = a, i. e.,
p = Y- Consequently, AAFC is isosceles and AF = FC.
The focal length OF = AF cosZAFO = FC cos 2a, i.e.,
F = (R - F) cos 2a, whence

F = R
cos 2a cos^a-sin^a
= R
l + cos2a 2cos^a
Fig.1.60
"i p-p_l-tan^a
The equation for the parabola is or

" y^- = 4fx • • • • • Thus, the spherical mirror does not exactly converge
. , where / is the distance central rays at a single point (focal point). For a concave
from the vertex of the mirror, the further is a ray incident on a-spherical mirror
parabola, to the-focal point from the optical axis, the larger the displacement of the
of the . parabola. This reflected ray from the source, for a convex mirror, the
dis'tarice is called focal opposite is true.
length. The equation of The impossibility to converge the reflected rays at one
the circle is point in this case is knowii as the spherical.aberration of
Cx-R)^+y2 ='r^ the mirror, which makes the image 'blurred. Spherical
aberration can be reduced by using parabolic mirrors in
where R is the radius of
which the radius of curvature increases for points of the
the circle. On simplification
surface at larger distances from the pole. But since the
we get
= 4fx -x^ manufacture of parabolic mirrors involves considerable
= 2Rx-x^ difficulties, spherical aberration is normally reduced by
For X«R we can combining converging and diverging mirrors into a single
ignore compared to optical system.
In this case the equation for The point F at which ray SA parallel to the principal
the parabola and the optical axis intersects it after having been reflected from the
Fig. 1.61
equation for the circle are mirror is called the focal point (or ptincipal focus) of
identical if we take R = 2f. the mirror. The distance OF from the focal point to the
The centre C of the spherical surface is the optical mirror pole is known as the focal length of the mirror and
centre of the mirror. The point 0 at the middle of the mirror is denoted by F. The plane passing through the focal point
is called its pole. The normal to the mirror surface passing normally to the principal optical axis is called the focal
through the pole is called the principal optical axis. The plane.
GEOMETRICAL OPTICS 29

The focal point of a concave spherical mirror lies at the (ii) Parabolic mirrors do not suffer from this defect
middle of the radius of the mirror, i.e., its focal length is because the normal at any point on a parabola bisects the
' F =R/2 angle between the line joining the point to the focus of the
parabola and the line through the point parallel to its axis.
The focal point of a convex mirror is virtual and lies on
the principal optical axis behind the mirror at a distance Paraxial Approximation:
F = R/2 from its pole. -Paraxial approximation assumes that all rays strike the
RELATION BETWEEN FOCAL LENGTH AND
mirror very close to the principal axis and have very small
angle of incidence. Thus we can approximate
RADIUS OF CURVATURE
Consider a ray of light parallel to the principal axis, tana « a tanp ~ p
incident on a spherical mirror at A. If the mirror is concave, We can further approximate.
it meets at F after reflection. If the mirror is convex it QC == PC, QF ^ PF
appears to come from F after reflection.
As p=2aor^
PC
=2x^
PF

or PF = -PR i.e., / =- ...(1)


2 '2
where / is the focal length of the mirror

(a) Concave mirror


(b) Convex mirror
Fig.1.63
Fig. 1.65
FC
From AFMC, = sect
CM The external angle equals the sum of the opposite two
R R internal angles. Therefore.
FC = — sect CM = MA =
2 P=a +0 ...(2)
and Y= p + 0 • • •/ . ,...(3)
Now, PF = PC- FC
Eliminating the angle 0 from equations (2) and (3), we
f = R sect ...(1) get ...
. , 2
2p = a +y "• • • " '.'..(4)
For paraxial rays, i 0- AQ, . ' AQ • AQ ' '
Also tana = —tanp =,—-, tan7 = .— -- ,
...(2) • Qo'; ' , .QC • • • Qr/ ;
• 2 As stated eaflier> we assume all angles a,P,y,0 are,very
(i) The dependence of focal .length on angle of small. ' . , . -
incidence is a' defect in spherical mirrors which'is known as We can approximate •
spherical abeiratibn'. This defect can't be conipletely tana = a, tanp « p and tany = y
eliminated from spherical mirror but it can be minimized by QO = PO,QC = PC and Q1 ^ PI
using mirror of small size or blocking marginal or paraxial Substituting these approximations in equation (1) we get,
rays using stops in case of large size mirrors. 2 1
...(5)
Marginal PC PO PI
rays
Paraxial / or 1-i +i
rays R V u
From equation (5), the position of the image can be
calculated knowing the position of the object and the focal
length of the mirror.
Mirror Formula
In terms of Cartesian sign convention mirror formula
(a) Spherical aberration (b) Parabolic mirrors
may be expressed as:
Fig.1.64
' . Ill
(i) - +— -
V u f
30 OPTICS1
where u is the object distance from the pole Rules for Ray Diagrams
Vis the image distance from the pole, and We can locate the image of any extended object
/ is the focal length of the mirror. graphically by drawing any two of the following four
(ii) Lateral magnification ratio principle rays.
hi ~v 1. A ray, initially parallel to the principal axis is reflected
m = — = —
.ho u through the focus of the mirror.
where /i,- is the height of image, and 2. A ray, initially passing through the focus is reflected
parallel to the principal axis.
ho is the height of object.
3. A ray passing through the so line joining point object
The Cartesian Sign Convention and its image cuts principal axis at centre of curvature.
A sign convention facilitates the computations of the 4. A ray incident at the pole is reflected symmetrically.
object and image distances, assesses the nature of image So line joining O and I' or 0' and I will cut principal axis at
(re^ or virtual), its magnification and orientation. pole.
The object is placed to the left of the optical surface (a (i) Spherical mirrors bring paraxial rays to an
•mirror or refracting surface). The light is incident from left approximate focus at a point on the mirror axis.
tcTright. The centre of the optical surface is called vertex. (ii) The focal length of a spherical mirror is equal to half
The vertex is taken as origin. The horizontal axis is called the the radius of curvature of the mirror.
optic axis. The mirror equation for spherical mirror is

, ^ .
V u f R- . .
When the object point O is located infinitely far away
j _ • Positive,.
• • \ distances
from the mirror, thenu = -=« and the position of the.image is
called the focal length. If we substitute u = - ~ into the
j Negative"^ Positive
mirror equation,
distances
' distances •
1 I1_ 2
-oo V R
• Optical Origin (vertex) Optical
' axis axis Since v = f,
!
( / r 2
<

Parallel incident rays intersect in case of concave mirror


1 Negative Optical surface
and appear to intersect in case of convex mirror.
distances
i
1
s
^ 1 Convex mirro'r

1 - .Fig.1.66 . ! .
Light from F c
' • (1) Distances measured to the right of the origin along object point
a

optic axis are positive distances,-since^they are along the at u = -» m -p'' f>Om
positive axis of the standard cartesian coordinate system. f=|>Om
(2) Distances measured to the left of the origin along
the optic axis are negative distances, since-they are along
the negative axis of the standard cartesian coordinate
system.
(3) The sign convention for magnification:
Convex
Magnification is defined as the ratio of the size of image to mirror
the size of object.
. , Imagesize
|m|=
Object size c
object point i O
This is referred to as lateral magnification. If the at u = -co m f < 0 m
magnification m is positive the image of the object is erect f=-|->Om
(upright),^ meaning that the image has same orientation as
the object. If the magnification is negative, the image is -

inverted (upside down). (b)


Fig. 1.67
GEOMETRICAL OPTICS 31

The magnification is Note: If object is moved towards pole magnification goes on,
image size decreasing at pole magnification m tends to unity. i
m
object size u Region 2:
Note that the law of reflection is independent of the
medium in which light is travelling. The mirror and
magnification equation can be applied irrespective of the
medium surrounding the mirror.
(1) A ray parallel to axis is reflected back through focal
point as shown in Fig. 1.67 (a).
(2) A ray that passes through the focal point on the ray
to the mirror is reflected back parallel to the mirror axis as
shown in Fig. 1.67(b).
Fig.1.70
(3) A ray from the object is directed toward the centre of
curvature of the mirror, after reflection the ray retraces its Object placed between focus / and centre of curvature.
path because it strikes the mirror along the normal to the Image is real, inverted, magnified.
mirror.
' Note: If object is moved from focus towards centre of
(4) A ray that strikes the vertex of the mirror reflects at curvature magnification goes on decreasing" at centre Of
an equal angle on the other side of the mirror axis. curvature it becomes unity. _ _ »
Any two of these rays are sufficient to locate the position
of the image. Region 3:
We can assess two things from ray diagrams :
(a) If the image is infront of the mirror, it is real; if the
image is at the back of the mirror, the image is virtual.
(b) We are able to guess if the magnification is positive
or negative and greater or less than unity.
RAY DIAGRAM FOR CONCAVE MIRROR

Fig.1.71

, F Object placed between centre of curvature and infinity.


Image real, inverted, smaller (diminished)
/ 'Note: Ifobject is moved away from centre of curvature towards,
infinity magnification goes on decreasing. , , , '
Fig.1.68
Region 4:
Fig. 1.68 shows a concave mirror with objects placed in
form different regions. We now analyze the nature of image
from ray diagram.
Region 1:

Virtual
object

Flg.1.72
Fig.1.69
Object,is virtual point object. Image real, erect, smaller
Object placed between pole and focus. Image is virtual, (diminished)
erect, magnified. Note: As object moves away from pole magnification goes on'
decreasing.
32 OPTICS

Concept: A concave'mirror always forms real image of Vutual object placed between pole and focus. Image
a virtual object. \ formed is real, erect, enlarged.
\ Note: As object is moved away from pole magnification'
Convex Mirror j increases. |
Region 3:

L. Fig.1.73

Figure shows convex mirror with four different object Fig.1.76


positions as shown in Fig. 1.73. Now we analj^e the
corresponding image formed. Virtual object placed between F and C. Image formed is
virtual, inverted, enlarged.
Re^dn 1:
i As object is moved towards Cmagnification decreases.^
Region 4:

Flg.1.74

Object placed infront of'mirror. For all the positions of Flg.1.77


object infront of mirror. Image is virtual, erect, smaller in-
size. Virtual object is placed beyond C. Image is virtual,
inyerted, smaller.
Note; As object moved towards pole.'magnification increasesl
and.tends,to unity at pole. Notei'As object is'moved-away from C image size further
decreases magnification decreases. '
Region 2:

Concept: A convex mirror can form virtual image of a


virtual object Also this virtual image is inverted',

F C

Virtual object

Fig.1.75
GEOMETRICAL OPTICS 33

uvsv graph Concave Mirror ci


i Concept: For all the points — represents longitudinal]
u < O [Rq] u > 0 [Vq]
\ du ;
I I
u > 0 [V|] Imagnification for small linear object. Here — representsl
i du • i
Region 1 Aslope of curve at any point i

vvsu graph Convex Mirror


Region 3

Region 4
Region 4 ;

Region 1 +2f
+f

Region 3

u < 0 [Rq] u > 0 [Vq]' u ;


Region 2 O : :+f +2f
u < 0 [R|] w<0 [R|]''
Fig.1.78

For interpreting graph we use sign convention. Region 2

Real object [Rq],' Virtual object [Vq]


Real image [R|]' Virtual image [V]] Fig.1.80

For interpreting graph we use sign convention.

Real object [Rq] Virtual object [Vq]


Real image [R|] Virtual image [V|]

Fig.1.79

m> 0 means - erect image


m<0 means - ' inverted iniage -
|m|>l means enlarged; image-"
]m|< 1 means ' smaller'image • Flg.1.81

For point (1) in region, (1) For point (1) in region-(1)


m > 0becauseu andsuhave opposite-sign".i.e., image is • • m > 0 because u and u have opposite sign i. e., image is
inverted.
erect;
]m|> 1 because ]u|>|u[ image is virtual"because u > 0 [m] <1 because lu[ < |u| image is virtual because u > 0
/

For point (2) in region (2) For point (2) in region (2)
m< 6 because u and -v have same sign, i.e., image is m > 0 be cause u and v have opposite sign i. e., image is
inverted.
erect | m| > 1 because |v| > |u\ image is real because u < 0.
Im|> 1 because | u| > |u| image is real because u < 0 For point'(3) in region (3)
For point (3) in region'(3) •' m < 0 because v and u have'same sign i. e., image is
m < 0 because u and v have same sign i.e., image is inverted [m[ > 1 because |u|>|u|.-'lrnageds'virtual because
inverted ]m|<lbecause Iv] < |u| image is real because v < 0. i;>0. . -
For point (4) in region'(4) For poiht'(4) in,region (4)' \
m > 0 because u and v have opposite sign i. e., image is m < 0 because v and u have same.sign i.e., image is
erect. inverted:• 17n|< 1 because [u|<|u]. Image,is virtual because
Im| < 1 because |u] < |u|. Image is real because u < 0. u > 0.
34 OPTICS

RAY DIAGRAMS FOR MIRRORS


Concave Mirror Convex Mirror

Optical axis

(a)

. ' ©1
©

(d) j... , ,

^ • I i . i . ' *
Fia.1.82.(b). ,
\ ;Fig: 1.82(a)^ ; ^ . ! - ' '.1 v'/'
Four Construction rays used to locate the image by concave mirror Five construction rays used to locate the image by a convex mirror
CHARACTERISTICS OF IMAGE- ' .< S ( t

Concave Mirror Covex Mirror

u — °<> u = R = 2F u = F
Image at F Image at C Image at^o
real Inverted real inverted real inverted
very small enlarged , . very large . Image at F
virtual erect -
Real Real very small ,
Virtual
inverted inverted "upright
enlarged Virtual
small enlarged
erect
small

! OBJECT: Between . Between Between


; ~andC F and C F and C
OBJECT: For alj positions infront of rpirror
' . u > R R> u > F u< F
i IMAGE: Between F and C Between C and Behind IMAGE; Between F and P
mirror

Fig. 1.82 (c) Fig. 1.82 (d)


GEOMETRICAL OPTICS 35

Graphical method for determining the focal Since mi is always negative image is always
length of a concave mirror : It forms real and inverted longitudinally inverted,
image of an object placed beyond its focus. From mirror -V
\2

equation, m, = - (For small object)


u ,
i +i =i mi = -Cmt)
V u f
Using cartesian sign convention, we have For Large Object
1 11
u - -x; V = -y and / = -/
1 11 V u f t "2 %
or _ + _= .py- Lli ^
-y ' Put u^,U2 and solve for • h-
A. . . - . 2', 2 •
A graph between — and —is a
bi,V2,
\Vt\-\V'
O2 '1 1
-D -u
mi =
straight line, as shown in Fig. * Vi *''
1.83 (a). Fig.1.85
Note that the slope of the Numerically [ui| >|i'2l or [uil < [t;2| decides whether
straight line is -1 and the intercepts image is inverted or irrect.
on the horizontal and vertical axes
Fig. 1.83 (a) Splitting of Mirror -. — .
are equal. It is equal to
A mirror is split into, two parts that are displaced by
small distance 2a as.shown-in figure.-. - '
Astraightline OP at an angle 45° wtb the horizontal axis
is drawn which intersects .the line AB. at P'The coordinates of
1 1
the point P are : Principal axis
-2f'-2f') of Mi
Thtfocal length of the mirror can -V.

be calculated by measuring 'the \


coordinates of either of,the points
A,BotP.
-2f t «
Principal axis
Alternatively, .a graph beipAreen -v.
and -u can also be plotted^ ,which is a .<45° ; %
curve as shown in Fig. 1.83 (b). A .^0 -2f -u

line drawn at angle 45° from the Fig. 1.83(b) Fig. 1.86 (a): Distance between I2= 2ma + 2a
origin intersects it' at the point P
whose coordinates are ' (-2/,-'27). By measuring the A point object is plqced. between F and 2F. Each part
coordinates of this point, the focal length of the mirror can forms image that is of size nid from its principal axis.
also^be.measured:''' ' • ^ ' ' ' • i - 1 •- ' •
Longitudinal Magnification for Very Short Object'
. ,,

dv
mi= —
du

1 +i -1
V u f
Differentiating both sides
with respect to 'u'.
1 dv 1
Fig.1.84
du
' ' Fig. 1.86(b)
"

dv
771, = — : (applicable only' wheri ohiject is' veiy Object is placed between focus, and.distance between
du
I1I2 =-2ma —2a.
small)
36

(i) Mirror is cut along its principal axis and two parts are Newton's Formuia
separated perpendicular to the principal axis. If the object and image distances are measured from the
focus instead of the pole of the mirror. Then, the mirror
formula reduces to a simple form called the Newton's
formula.
C 0 F cy : Vi XoXi = f
y2
where is the object distance from the focus.
Xi is the image distance from the focus.
(a) (b) In case of spherical mirrors if object distance (x^) and
Fig.1.87 image distance (X2) are measured from focus instead of
pole, then u = f + x and u = / + X2 the mirror formula
V
m, = — reduces to
u 1 1
1
s= +yi+m^y2 +y2J s = +l)(yi +y2) / + X2 / +
Where 's' is the separation distance between two images
or X1X2 -
and 12 of the same object O formed due to two halves of
the mirror. which is known as Newton's formula.
(ii) Mirror is cut along its principal axis and two parts While using mirror equation known quantities are to
are separated parallel to the principal axis. substituted with proper sign and quantities to be calculated
(unknown quantities) are not be given any sign. .
VELOCITY OF IMAGE
c 0 F IN SPHERICAL MIRROR
•••& '=•^-•^2! Consider pole of mirror to be origin of co-ordinate
u+d
system and X-axis as the principal axis of mirror and Y-axis
is perpendicular to principal axis. Incident rays point along
(a) (b) X > 0.

Y
. (Vo/m)
Upper,Half . . Lower Half
' -A, • . .-1. -1. --4(Vo/ni-)y.
^ •. -•/' • r 'V_2 i ' O,.
. Vi =? • -.',w--2'=?- • ' ' <r
Mirror is turned through small angle 0

Fig. 1.90

From mirror equation, we have


1 1 1.
+
X I/m X O/m 7
Differentiating both side w.r.t., 't' we get

4m, ''t Xl.^


O/m dt
Fig.1.89
2
X I/m
- ^(4m) =- X o/m

C^I/m
UCOS0
GEOMETRICAL OPTICS

_ / _ Height of image
f -u Height of object
f
^I/m =

Differentiating w.r.t. 't' we get


_d
^O/m) Fig. 1.91 (b)

dt dt f-u
As object lies on principal axis case (i) applies. For
/ velocity component parallel to optical axis
• CVl/m^r - f-u)

(Using product rule)

or (Vi/n,) =
/
CVb/m^y +C^O/m)
O/m-
/ du (Vi/„)|| =-C-2)"lli =-44im/s
f-u (.f-uf dt For velocity component perpendicular to optical' axis
Case (I): If object is on principal axis, then Yo/m ~ 0 ^ ' i f
CVi/m)x = (Vo/m), • '•••'
' /•'• (Vo/m)
O/mJY '
.- . . ,

' * >U-" = (-2)12j-=-24jm/s


Case (n)V If object is not on principal axis but moving
parallel to principal ajds then (Vo/mOy ^-O
Vj/m =Velocity ofimage w.r.t. mirror
' '-'(Y 1 • ^
\= (-441 - 24j) m/s
. i:-: Note that — is negative ifu is decreasing with-time.and
dt Also, V,/..,
I/m = Vt-V,
it is taken positive if u is increasing with time. —»

Case (III); If object is on principal axis and moving or Vj =(-44i-'24j)-2l


along it then Yo/m = ^ arid,(Vo/n,)i^ = 0(Vj/m) = 0 •I'.', '.N- ~'=(-46i-24j)rii/s v.
Illustration 7: Find the velocity of image in situation Measurement of refractive Index of a liquid by
as shown in Fig. 1.91 (a). - - 1. a concave mirror : A concave mirror of large radius or
curvature is placed .Ci
15 m/s^ on a table with.^its, , ,',f -om
53° ' principal axis j
vertical, as shown | .'
in Fig.1.92.. A i
f = 20 cm- horizontal pin is L-
placed with its tip |
30 cm
on the principal j
Fig. 1.91 (a) axis of the mirror, i
The pin is moved
Express velocity and .mirror of object ,in unit, vector till there is no ; Fig. 1.92: Concave mirror with a drop of liquid
parallax between
notation Vq =Velocity of object = (91 + 2j)m/s
the tip of the pin and its image, when the pin lies at the
centre of curvature of the mirror.
Vm= Velocity of mirror = -21m/s
Asmallquantity of liquidwhose refractive indexis to be
/ - :— -20 _—.
m = • • measured is poured into the mirror. The pin is moved down
f-u -20-(-30)
in order to remove the parallax between the tip of the pin
and its image.
38
OPTICS

In Fig. 1.92, Cjis the position of the pin when its image Because the image distance is negative, the image is real
coincides with itself, without liquid and C2 is the position of and is formed infront of the mirror, the reflected rays
the pinwhenits image coincides withitselfafterpouring the actually pass through the image.
liquid into the concave mirror.
The lateral magnification is
The ray BO that is normal to the mirror passes through
Cjbefore pouring the liquid. It is refracted away from the m = — —3
u (-20)
normal when theliquid ispoured, now it passes through C2.
In Fig. 1.92 shown,
The lateral magnification is negative indicating that the
image is inverted with respect to the object.
CD . CD
sini = , sin r =
OCi DC.

T? r 1law, \i = OD x 0C-,^ OC,


From Snelrs
DC. OD OC. An object is located 6 cm infront of a mirror. The virtual
'image is located 4 cm awayfrom the mirror and diminished.
Taking paraxial ray assumption, iFind the focal length of the mirror.
OCj -DCi; OC2 ~DC2
DCi Solution:
Thus = • The first question arises: is the mirror concave or
DC.
convex or either is possible? Since both the mirrors
If we neglect the depth of the liquid, we have form virtual images. The answer is, either .type is
possible. Aconcave mirror canform a virtual image if
1^ =
AC. the object is within the focal "point of the mirror. A
convex mirror always forms a virtual image.
• The second question arises that the image is
diminished in size as well as virtual, do these
A pencil is placed 20.0 cm infront of a concave spherical characteristics together indicate a concave or convex
mirror offocal length f = 15.0 cm. Find the location of the^ mirror? a concave mirror produces a diminished
image. State whether the image is real or virtual, erect or' image only when the object is located beyond the
inverted and-give its.lateral magnification..
centre of curvature of the mirror. However, tlie image
in this case is real not virtual. Aconvex, mirror always
Solution: The ray diagram shows that the image is produces an image that is virtual and smaller than the
real and that, relative to the 6bjett,''it isfarther away from object.
the mirror, inyerted-and larger. • The focal length of a convex mirror is positive. The
• given p'ara'meters'are ^ ' •
""T "• u' = - 6cm' '1 ' ' ' • • ' -
V = +4cm

From mirror equation,


1
. •• ' • ..f .V ' u -. (4-4) ' (-6) ^ 12
- ' ' ! / ~ +12cni
As expected, focal length is positive.
Fig. 1E,30
I I
The given parariieters are "
'When the position ofan object reflected in a concave mirror of
,,/ =-i5cm_, \0.25 mfocal length isvaried, the position oftheimage varies.
•.' •r , ; u.='^ 20cm Plot the image distance as a function of the object distance,'
From'mirror equktion,^ ' ' •taking the object distancefrom 0 to +00. '
1-Jl • 1 1 •
aT<r "7 —
Solution: Fig. IE.32 shows the required graph
V f u -15 C-20). 60

f = - 60 cm
GEOMETRICAL OPTICS 39

V ,
• j—Real image
1.0

0.5 -y\
0
•1/4 -
-0.5 \ .4—Virtualimage
-1.0 • /
Fig.1E.32

1m;

'// the pencil in Example 30 is at 10.0 cm infront of thel


\concave mirror offocal length 15 cm, characterize the hew\

Solution: The ray diagram shows that the image is


virtual,"upright and larger relative to the object."
•"tlie'^ven parameters are;'
t' - • t I ' V/ i'
.' . . . u = -10 cm. , , . _
, 'f =.-15cm
From mirror equation, - " ' Fig.1E.34
• 1_ 1 1 1^ -1 _ 1
-v~ f u (-15) (-10);' +30 ' (+2) (-6)
u = +30cm - ,• .•
V = + —m
- The positive sign shows that image is behind the'mirror, 2
virtual; the rays appear to emanate from the image. '
The positive sign for v indicates that the image is behind
"the lateral'magnification is • ' the mirror and therefore is a virtual image. , _ , ,.
• ' The lateral^magnification is y . , . .>
•^ .. •5-1®.S,
The magnification is positive indicating ;that.the image is UJ (-6)
upright; m > 1 implies that the-imagejs magnified.
The positive, sign shows that the image is upright,
34
diminished in size,'relative to the object.
• The rear view mirror on the passenger side is often a
convex','mifrbV having, warning: "VEHICLES IN
Becauseof its shape a convexmirror givesa widerfield of view]
MIRROR ARE- CLOSER THAN THEY APPEAR." The
Ithando other types of mirrors. Such a mirror is mounted on]
reason is that the virtual image is reduced in size and
an automobile. The radius of curvature of the mirror is 4.00\
therefore looks smaller, just as the distant object
m. If a bus is located 6.00 mfrom the 'mirror, where is the would appear in a plane mirror.
image located ? ______
We judge the apparent distance, of objects by the angle
Solution: The focal length of the mirror is •' they subtend at our eye. If the viewer's eye is at 1 m from a
f =Rl2 = 2m ' plane side mirror and looking at a 1 m wide portion of a bus
The ^ven parameters are' ^ ' /, , ! .- behind him. Fig. lE.34Cb) shows that'the image is 7 m from
his eye and subtends an angle of 0 = lm/7m = 1/7 rad. In the
'/='+2!6o'hi ' ' y.,', , y. y i convex mirror the image is only 1.5 m-behind the mirror,
u =-6.00,in /. s which is at 1 m from the viewer's eye.iThe size of the image
From mirror equation, we have is 0.25 m, the angle subtended is 0.25/2.5 = 1/lOradian. The
smaller, angular size creates the illusion that the bus is
farther behind than it actually ik
OPTICS

1 1 1
cm
(-21) (-42) 42
A diamond ring is placed infront of a mirror of radius of u = - 42 cm
curvature. The image is twice the size of the ring. Final the' The image is 42 cm infront of the mirror. The image
•object distance of the ring. formed by the concave mirror is object for the plane mirror.
Now we use the mirror equation for the plane mirror with
Solution:
f = oo^u = -[42- 21] cm.
O The first question arises: is the mirror concave or
1 1 1
convex or either is'possible ? The image formed by a
convex mirror is always smaller than the object, ^"^(-21)
therefore the mirror must be concave. V = 21cm
n The next question arises : How many positions are The position of the final image is 21 cm infront of the
there infront of a concave mirror, where the ring can plane mirror.
be placed and produce an image that is twice the size
of the object ? There are two places : (1) When the
object is placed between centre of curvature and the
focal point, the magnified image is real and inverted. Aconcave mirror with aperture diameter 6 cm and radius of
(2) When the object is between the focal point and curvature 5cm is covered with an opaque paper except a chmj
the mirror, the magnified image' is virtual and strip at the edge. At what distance from the mirror- will qi
upright. " ' ^ parallel beam of rays converge ? i]
In first casethe image is inverted-, so the" magnification is
m = ~2 . In the second case the image is upright, so-the Solution: The small strip at the edge of the concave
magnification is m = +2 , ,, ,, mirror acts as a plane mirror; the law of reflection is obeyed.
• From mirror equation and magnification equation, . •. The rays converge at point F
' . 1 .,1 ; 1 , . V , - From figure, O'C = - 3^ = 4cm
—+ — = — and m = —
." ' ' V -'li f • • • u ' •
00'= 0C-0'C = 1cm
On solving magnification equation we have v = -mu; O'A 3
substituting this expressionfor v in mirror equation;we'get tani =
O'C 4
Hence. , i=37°
- 3. - " (-mu), / -m ' ,
ZO'FA = 74°'
Applying this result,.we obtain O'A 3
tan 74° =
f(.m-l) (-12) (-2.-1)-- r O'F O'F
m = -2, t1 = =-18cm
m -2
O'F =.—=.0.86cm
- -6m tan 74°
m +2 Hence OF = 00'+ 0'F = 1.86cm
Negative signs for object distances indicate that the
object is real, lies infront of the mirror.

,The co-ordinates of the image ofpoint object P formed by a,


•concave mirror of radius of curvature 20cm (considerl
'A small plane mirror is placed 21 cm ihjront of a concave paraxial rays only) as shown in the figure is
mirror offocal length 21 cin.An object is placed 42 cm infront ' "y, ^ ,
of the concave mirror. If light'frofn the concave Thijrorstrikes •p i
the plane mirror, where is the final image ? (40 cm, 3 cm)
'• b; ^ X

Solution: First we will obtain image-position-from


concave.mirror,by;mirror.equation. '
The given parameters are *•- ; Fig.1E.38 ' |
/ - -21cm (a) 13.33 cm, -1 cm (b) 13.33 cm, -1-2 cm
, I ' * ' .5^
u = - 42cm ,(c) -13.33 cm, +2 cm (d) -13.33 cm, -1 cm
GEOMETRICAL OPTICS

Solution: (a) — Solution: — = e, = o.i rad


V 40 -10 10 ^
1-Zl 1 _ -4+1
u ~ 10 40 ~ 40
_ -40
" 3
u = -40; / = -10 . 5cm
uf _ -40 X-10 -40
Fig. 1E.40
V = cm
u~f -40 + 10

m = -
3 J -1
3 -40 3

hi - -1 cm
5 I 5/ 25 +10/ ^2
'5+./ .
A spherical mirror is polished on both sides. When the convexj 2f = 2.5,+ /
•side is used as a mirror the image is erect with magnificatiow / = 2.5m
\l/4. What is the magnification when the concave side is used\
ios a mirror, the object remaining the same distance from the\
^mirror ? \
A bright point S is on the principal optical axis of a concave\
mip-or of radius R = 40 cm at d = 30 cmfrom its pole. At^
\what distance (incm) infront ofthe concave mirror should aj
Iplane mirror be placed so that after two reflections, the raysj
Solution; iconverge back at points S. ' I
Fig. 1E.39
Solution: For minor 1
~R
f L'
1 +i - i y =-30, ,
m - V u f / = -20 • •{' = 40,
f-u -R'3R
• - -2 . ,2 i_A = _±
v .30 ' -20

f'-u - =^=-4 . - V 30 '20 X d = 30


. 2 ' V = -60 cm Flg.1E.41
4R = J? + 2x . so X = 15 cih
3R
X = — SO distance between two minors = 45 cm.

\Ca) A ray of light falls on a concave spherical mirror, as\


U large convex spherical mirror in an amusement park isj, ^shown in Fig. 1E.42 (a). Trace the path of the ray further.
facii^ a plane mirror 10 m away. A boy of height 1ml
•^standing midway between the two sees himselftwice as tall as|
'in plane mirror as in spherical one. In other words, the angle''
^subtended at the observer by the image in plane mirror iSj
\twice the angle subtended by the, image in the spherical]
'jnirror. What is the focal length ofthe convex mirror ? [
Fig. 1E.42
42 OPTICS

1^^ A ray of light falls on a convex mirror, as shown in Fig.i The optical centre C of. the mirror can obviously be
\1E.42 (b). Trace the path ofthe ray further. J found as the point of intersection of ray SS' with axis NN'.
The focus can be found by the usual construction of ray
Solution: (a) Consider a beamof ra)^ parallel to the SM parallel to the axis. The reflected ray must pass through
ray MN. The beam reflected from the mirror will converge at
focus F (lying on the optical axis of the mirror) and through
the secondary focus F, which lies in the focal plane. Drawing
S\ ^ •
the ray D0| [MN parallel to the centre of the mirror, we find
the secondary focus F\
The ray NK is the one we are looking for [Fig.lE.42(c)]
\The positions of optical axis N1N2 of a spherical mirror, the
M ^^source and the image are known [Fig. lE.44(a)]. Find by
F \construction the positions of the centre of the mirror, itsfocus
1^0 land the pole for the cases : (a) A- source, B- image; (b) Br
F' [source, A - image. ' -

(c) (d)
Fig. 1E.42

Method 2. Choose an
arbitrary point M on the ray
MN and with the aid of Fig. 1E.44 (a).
F'
characteristic rays construct
its image M'. The required .q Solution: (a) Let us construct; as in the previous'
ray NK passes through this ' example, the ray BAC and find point C (opticaTcentre of the
point [see Fig. 1E.42 (d)] 1 Fig. 1E.42 (e) mirror) [Fig. IE.44 (b)]. Pole? can be found by constructihg-
(b) The paths of the the path of the ray APA' reflected in the pole with the aid ,of
rays are shown in Fig. 1E.42 (e). symmetrical point A'. The position of the miiror focus F is
determined by means of the usual construction of ray AMF
parallel to the axis. .
, , ^ ^ .... - - ~ -

Point S' is the image ofa point source oflight.S in a spherical^


mirror whose optical axis is N1N2 [Fig. lE.43(a)]. Find byl
constmction the pqsition^of the centre, ofJhe mirror and its'<
\focus. ' i

Ni N2
P' ' -^N^Vc - F-'fvP ' NiV

Fig. 1E.43 (a) '

Solution:
Since the ray Flg.1E.44
incident on the
mirror at its pole (b) This construction can also be used to. find the centre
is reflected C of the mirror and pole P [Fig. 1E.44 (c)]. The reflected ray
symmetrically BM will pass parallel to the optical axis of the mirror. For this
w.r.t. the major reason, to find the fociis, let us first defermine point M at
optical axis, let which straight line AM, parallel to the optical axis, intersects
us plot point the mirror and them extend BM to the point of intersection
symmetrical to S' with the axis at the focus F.
and draw raySSi ' Fig. 1E.43 (b)
until it intersects ^
the axis at pointP [Fig. lE.43(b)]. This point will be the pole
of the mirror.
GEOMETRICAL OPTICS 43

-1 J_
10 -50

U point source S is placed midway between two converging which on solving gives / = 12.5 cm.
mirrors having equalfocal length f as shown in figure. Find Radius of curvature R = 2f = 25 cm.
\the values ofdfor which only one image isformed.

Solution:
A converging mirror Mj, a point source S and a diverging,
mirror M2 are arranged as shown inFig.lE.47. The source is
•placed at a distance of 30 cmfrom M^. The focal length of
each of the mirrors is 20 cm. Consider only the imagesfonhed
by a maximum of two reflections. It isfound that one image is
formed on the source itself.
(a) Find the distance between the mirrors.
(a) (b) (b) Find the location of, the image formed by the single
Ftg.1E.45
reflectionfrom M2.
When S is placed.at the common focus of mirrors, the
rays after reflection from one mirror incident parallel on to Solution: Formirror M^:
the second mirror, which finally intersect at focus of the u = -30cm,
miiror.Thus there will be only one image. In this case the / = - 20cm
value of d will be 2f. When S is placed at the centre of
curvature, the image will form at the same point, so in this By mirror formula, —+ —= we have
case the value of d will be 2/ + 2/ = 4/ (see Fig. 1E.45) u V f
1 1_ 1
-30 V -30

which on solving gives, v - -60 cm.


•An objectisplaced injront ofa convex mirror at a distance of
\50cm. Aplane mirror is introduced covering lower halfofthe\
\mirror. If the distance between the object and the plane'
imirror is 30 cm, it isfound that there is noparallax between]
\the images formed by two mirrors. What -is the radius of
[curvature of the convex mirror ? . t i
Solution: The distance of the object from the plane
mirror is 30 cm and so the distance of its image is,also 30 cm
from the mirror. As images formed by both the mirrors I ;*."30cm (30-x)
concide, so distance of image for convex mirror is = 10 cm.
I- 60cm
Fig. 1E.47

For mirror M2I The image formed by mirror M^is a


virtual object for mirror M2. Let it is a distance x from the
pole of mirror M2. Thus — • • . ,
^ _ _U2 =+X
>2 = -C30-x) '
Again by mirror formula, we have
20cm 1 1
— +
30cm 30cm X -(30-x) 20

Fig. 1E.46 which on solving gives x =10 cm or 60 cm,. •; ^ .


X = 60 cm is not possible, thus x = 10 cm. - .1 '
By mirror formula —+i =—, we have
u V f
44 OPTICS

(a) Thus the separation between the mirrors


49
- 60-10 - 50cm.

(b) The image formed by mirror M2 is at a distance


10cm.
Two concave mirrors of equal radii of curvature R arefixedon
a stand facing opposite directions. The whole system has a
mass m and is kept on a frictionless horizontal table [Fig.
1E.49 (a)]

A gun of mass M fires a bullet of mass m with a horizontah


speedV. Thegun isfitted with a concavemirror offocal length] m
f facing toward the receding bullet Find the speed of
•2R. .2R- H !

Fig. 1E.49 (a)


Solution: if u' is the recoilvelocityof the gun, then by
conservation of linear momentum, 'Two block A and B, each of mass m, are placed on the two
' ' O = mvl+Mv'
Isides ofthe stand. Att = 0, the separation between Aand the
'mirror is 2R and also the sepdration between B and the mirror
mv
- V =11-.- •is 2R. The block B moves towards the mirror at dspeed v. All
M Icollisions which take place are elastic. Taking the drigihdl
The velocity of,the bullet with respect to the mirror •position of the mirrors standard system tobex = 0 and x-axis
-» -» -» lalong AB,find the position of the images of A dn'dB at; ;
V •
^bullet 1 —
^ bullet. —

• mirror
.
mirror '• " 1 g- . . :(a) t = - , (b)t=i —> (c)t = —
I V '• V V
-U
. =,v - V

mv R
= u- - Solution: (a) Att = —.
M • - V ••

For block A, u = -2R.


1 1 2 '
V -2R -R
du •t- ^ I
or 1 -+ — \v -2R
dt M or V =

Bythe defination, the image velocity ' R.


For blockBiThe distance travels by block B in time —is R

Thus u = -R

+ •
.-R ^-R
I / M

Fig. 1E.48 or I' v'= -R' ' :


The x-coordinate of the image of the block with respect
"l/M - C^^O/m) •- •• to the mirror will be +R.

At the instant of firing, bullet and its image are at the (b) At t =
pole of the mirror, so m = 1. V

2R
The velocity of image w.nt. mirror = velocity of bullet The block B will collide with the stand after time —.
w.r.t. mirror ' '., - ' • " • " - • . . , ... V

The speed of separation between bullet and its image After collision block B becomes at rest and mirror starts
moving with the' same velocity v. In the remaining time R/v,
du 1 the distance moved by the mirror is R...
/ ^
J. The poisitioh of blocks and mirrof' are shown in Fig.
m
= 2 1 + 1E.49 Cb)
V M

X..ji
GEOMETRICAL OPTICS

—1 —

m m
m
60 cm 40 cm
0
..axis of Ml ,
axis of Mo

At this time the blocks lie at the centre of curvature of


the respective mirrors. Their images will form at the centres
Mj
of curvature. So their co-ordinates are:
Fig. 1E.50 (a)
For block A, x = -R
For block B, x =+R Solution: Using mirror formula for first reflection:
5R
(c) At t = 1 =1 +1^-L =1 +J-
V f V u -20 V -70
2R
The block B will collide to the mirror after a time 1

1 a on'
=^v = -30cm
' '
u 60 20
Thereafter mirror starts moving towards block A with
' ' 4R ' - Using mirror formula for second reflection
velocity u.Att = —, the mirror will collide with block A and
V 1 _1 +1 ^ J_ =1 +J_
stops after collision. The positions of blocks and mirror are / v 'li -20 V• -70
shown in Fig. 1E.49 (c). ^
V 70 20

=-28cm
x:
w- R- 2R- 2R-

Fig.1E.49 (c)

For block A; Its image will form on the same place.


60cm 40cm
Therefore the positions of the blocks are
Xa = -SR
For block B; u = -2R 30cm
28cm
'X ' 1 2'
70cm
V -2R -R

V = -• Fig.1E:50.(b) J-.V-
-3

The co-ordinates of B are - 2R-^


3
Height of/2 => m=I =-^
• " -4R => Ji = — cm
2
1 3
Height of first image from x-axes = 1 - = - cm . -
^4 50 2 2

Two concave mirrors each of radius of curvature 40 cm are] Height of J2 => m =


placedsuch that theirprincipalaxes are parallel to eachother\
-70) 3
3x28 ' ^
and at a distance of1 cm to each other. Both the mirrors arej ^ f2 ~ I2 = -0.6cm
ata distance of100 cm to each other. Considerfirst reflection\ 2x70
lot and then at M^, find the coordinates ofthe image thus^ Co-ordinate of I2 = (12- 0.6)
^formed. Take location ofy^ect as the origin. . , >j
'Apoint object isplacedat the centre of curvatureof a concave\ jA convex mirror offocal length f = lOcmisplaced infront ofi
bnirror (taken as origin)^ A plane mirror is also placed at a! ]a glass slab of thickness t = 5cm at a distance (nearer!
idistance of 10 cmfrom the object as shown. Consider tivoj Isuiface) x= 5cm. An object lies on the common axis of the'
irejlection first at plane mirror and then at concave mirror.] •system at a distance d = 15 cm from the mirror. Find the\
'Find the coordinate of the image thus formed. ^position of image. Draw the ray dia^am showing t/zei
formation of the image. of slab = 1.5. I

1^91° \ \

1 0 1
« ^
20 cm 0 i

Fig. 1E.51-(a)
Fig. 1E.52
Solution:'Distance of'from mirror = 40 cm
Solution: Since the rays are diverging^ the functiorl
of the slab is to bring the object closer by t - —= 5 - — ='—
1.5 3
So we may consider the object is at a distance
-10 V -140
1c 5 40
15 — = — cm.
3 3
V 40 10
Considering reflection at the mirror,
1 1 . 1 . 40
—+ — => v = +—cm
V -40/3 +10 , 7
• v= -40/3 cm
Sincethe rays are again diverging, the slab willbring the
image closer by 40/3 cm. •...••
40 40 4
10cm .-.Final image is at a distance — -r — = —40 x —
" ' ' . . ' ' • , 7 3 • ' ' ' 21
• s20 tan.1 20/3cm' ^ 40/3cm
♦ I * I lOcosI I , ' -w • f ^ -7.6cm
, The negative sign shows that it. will be infront of.the
20ctTl' mirror. > • i . . ' _ • - r- • »

Fig. 1E.51 (b) REFRACTION


When a light ray obliquely encounters an abrupt change
Using magnification formula
in medium, the direction oflight ray changes permanently at
' /' V
m = — = — the interface; this bending is called refraction. At the
0) u
L boundary the media are assumed to •be homogeneous
1 . -40/30 isotropic and transparent. A homogeneous medium
-20 tan 1° -40 means uniform-composition and structure throughout the
^ , ,20 tan 1° 20 n material. An isotropic .medium means identical or
=> I = —r —— tan invariant in all directions; otherwise it.is anisotropic.
. . _ 3 3 180
Refractive Index
,^i' •1"' i
Refractive index depends on the opticalproperties of the
medium into which light penetrates. The refractive index
Co-ordinate of image = — ,~ obtained when Tight gets from vacuum into a medium is
I 3 27
known as the absolute refractive indexfor a given medium.
GEOMETRICAL OPTICS 47

Let the absolute refractive index of the first medium be Table 1.1: Refractive Indices of Some
n-i and of the second medium be n2. Considering the Substances Relative to Air

refraction at the interface between the first and second Liquid Solid
media, we can make sure that the refractive index n for the
Substance n Substance ' ** i
transition from the first to ±e second medium (relative
refractive index) is equal to the ratio of the absolute 'Carbon disulphide 1.632 Diamond 2.417 i
refractive indices of the second and first media: Ethyl alcohol 1.362 Glass (heavy flint)* 1.80 ;
iGlycerine 1.47 Glass (light crown)* 1.57 i
n - ...(1)
ILiquid helium 1.028 Ice 1.31

(Fig. 1.93). On the contrary, when the light propagates Xiquid hydrogen • 1.12 Ruby 1.76 1

from the second to the first medium, the relative refractive Water 1.333 Sugar 1.56 .

index is •
We shall often deal with this phenomenon in the
n = — = ...(2) following chapters. The data contained in table 1.1 refer to
' n
yellow light.
These relations between the relative refractive index for It is interesting to note that the law of reflection can
two media and their absolute refractive indices could be formally be written in the same form as the law of
derived theoretically without new experiments in the same refraction. It should be recalled that we agreed to measure
way as it. can be done for, the reversibility principle "(See angles from the normal to the corresponding ray.
reversibility of light rays). Consequently, the angle of incidence i and the angle of
reflection i' should be taken with opposite signs. Then the
Vacuum
Vacuum law of reflection should be written in the form
i' = - I

sin I
Medium
or ,= -1 ...(4)
sini'
Medium
Medium I
Medium Comparing this formula with the law of refraction, we
see that the law of reflection can be treated as a special case
n
of the law of refraction for n = -1. This formal similarity of
Fig. 1.93
the laws of reflection and refraction is very useful for solving
practical'problems. , . . .
A medium"having a.lafger'fefractiVe index-is referred to In the above analysis, refractive index was regarded as a
as an optically denser medium. The refractive index of constant characterizing a-, mediurh',and independent of. the
various media is normally determined relative to air. The intensity of light propagating through, it.. Such*- an
absolute refractive'index'of ain^haj^ is- 1.003.'-Thus, the interpretation of refractive index is quite natural, but it is
absolute refractive index n^bs of a medium is connected with unjustified for high-intensities of radiation attained with the
its refractive indexrelatiyet0,air.(nj.ei ).through the formula help of modern lasers. In this case, the properties of a
. . ^"rel r l-.OP^nrer • • -.^-O) medium through which a' high-power optical radiation
propagates depend on the radiation intensity, The medium is
- . Table 1.1 contains relative refractive indices determined
said to become nonlinear. The nonlinearity of the medium is
for some cases of refraction of,light at the,interface between
manifested, in particular, in the fact that a high-intensity
air and a corresponding medium.
light wave changes the refractive index in it. The
Refractive index depends on the wavelength of light, dependence of the refractive index on the radiation intensity
i.e., on its colour. Different-refractive'indices'correSpond'to J has the form
different colours. This phShdmerioh, •which is known as . n-h-Q+aJ.
dispersion, play'shn important role in optics. * '
Here Uq is an ordinary refractive index and aJ is the
Relative refractive index for two media"; > ' ' ' ' ' • •'»
nonlinear refractive.index, where a is the proportionality
• - sini- - • sini ' ' sinr, A, ? • ' factor. The additional term in the formula can be either
—. . - , 3^2 —>^. - . ~r—• - . 1
smr^ • smr2 sin To Hi - • ' ' ' positive or negative. ,
1 >. I •• 1- -

Crown and flint are the grades of optical glass


48. OPTICS^

(a) A ray entering a material of larger index of


21-^^1 -1^21
refraction bends toward the normal. P-12

Incident
Metenai a Metenai b 1^21x1112=1
"6 > "a
General form of Snell's Law

Normal
The Snell's law describes the relation between angle of
incidence 0^ and angle of refraction 02-
Refracted
LReflected sin0i = n2 sin0g = constant .... (1)
. Fig. 1.94 (a)
Pi sinOa ^2 ^2
(b) A ray entering a niaterial of snialler index of
reifraction bends away from the normal. Pi sin0i = P2 sin02
£
Pi^i = P2f2 = l^3^3 = const. ...(2).
fIncident- 1
f Normal => U oc —
p
Reflected
•PiXti —P2^2 ~ ~ const. ..'.(3)
Refracted
'^Material a Materiajb ^ A oc
1

l_. Fig. 1:94 (b)


Eqns. (1) and (2) are'always applicable.
.Cc)'A ray oriented along the normal does hot bend,
regardless of the materials.' - med. (1)

Refracted
Reflected med. (2)

Fig; 1.94 (c)


med. (3)
, . Isotrppic optical medium: Refractive "index of
medium in alldirections issaihe (not- directional property).
med. (4)
.Anisotrqpic' medium: .Re'fra^ve index" of "same
medmrii in-different directions -is "different.- Fig. 1.96
•Optical'density-is, according to.velbcity pfpfopagation of
waye-intHe medium..vblo'cityof prbpagatibmdedreases.with ' p'l sin0i-=p2.sm02' = P3irin03 •=P4 sin04 =...=const.
increase irifOpticaLdensity. of medium'and vicerversa. Af^li'cable Only'when allrefractOry surfaces are simple
'plane and.pafallel to-'eadi^other.
med (1) ' • med. (1)
Vi , . Vi
med. (2)

med (2)
v'>
I92\. ^2 ' ^ ' ^2 - 0^63 med. (3)

Fig. 1.95 me'd. (4)

^ ' • ' '• '' J 'r r ' ' ,, , »^


Frequency of ihcideritland refracted ray is same. med. (1)
'',.V{ = V2 • Fig. 1.97
- . . •' ,'r- ' U-7
By convention^; "ifX2-=::)ri2i= —
' Pi sihi =p'2 sin02 =P3 sin03 =P4 sin04 =Pi sine
' V ti'- . I' ' . , .
R.I.'rofnhediumiC2)"'w.f.tT.medium (1) is'giveffby Pi sini,= Pi sine
' i = e
.•'Vv _ ^2 _ _ ' ^ 1
. •1^2 -M-12.. — .
M-1 c/Vi V2 If all refractoiy-surfaces, are simple plane and parallel
_ Xj _ sin01 and nature of mitial and'final'medium are same, then initial
incident and'final emergent rays are parallel to each other
X2 sin02 irrespective of' nature and nutnber of medium present
GEOMETRICAL OPTICS 49

between them. Final emergent ray may have some lateral =^ =SlJh. =Jh.
shift.
^2 ^2 "1
where and 03 are refractive indices of the two media. Thus we have
The refractive index is never less than 1 because v<c;n
"1^ 1 ~ "2^2 rijUi —n2U2 .... (5)
yaries with wavelength of light, except in vacuum. So a
particular wavelength is specified, for a given refractive From Snell's law,
index. sinGj _ "2 _
.... (6)
sin 02 1^2 ^2
Noima ine
Incident Angle of ; Angle of i Normal
ray incidence • reflection Reflection
ray (usually
weak) Normal
'1 ^ "2
Transparent medium 1
-• hterface
Angle of Transparent medium 2
refraction :Glass
V2>V.,

Reflection ray I !

Fig. 1.98 (a) (b)


Fig. 1.100
•-When, light travels from medium 1 to 2 its refractive
index is written as' • The path of light ray through a refracting surface is
_ "-2
reversible, e.g., in the figure the"incident"ray "travels
.... (4) from point A to point-B; If,the ray originated at B, it
V'2
woiild travel alongTine BA to reach A.
• . 1
l"2=
.2'^!
Medium of - •Mediurnyf'.
Incident ray refra'ctet^ray From eqn. (5) we have
sin0i j
1712 is refractive index of. mediurii"2:-w!r.t.. mediurn'l
sin0o ^>2
referred'tOias relative refractive'index.' If the medium.l
is air. or. vacuum the.refractive index relative to.it is referred., V\^en' light moves from a medium in which its speed is
to as absolute refractive index. "It is customary to drop high (rater medium) to a medium in which its speed is
the first subscript in case-of absolute refractive index. Thus lower, (denser medium), as shown in Fig. 1.100, the
"glass' stands for absolute.-refractive index of'glass,, with angle of refraction 02 is less than the angle of incidence 0^
incident ray in air or vacuum and refracted ray. in ghss. and the beam is bent toward^the normal. If the ray moves
When light travels .from, one from- a material in which'" it moves slowly into another
medium- to the other, its ;• medium in which it moves fast as shown in Fig. 1.100 (b), 02
frequency does not change but .: • c- is" greater than.0i\ ,
its wavelength and velocity Ij • Wheii light travels in air its speed is 3 x 10® m/s, it is
does. Consider- observers at reduced to 2x 10.®"m/s in glass with refractive index
points A and B as shown in.Fig. ' • 3/2. When,the light emerges from glass its velocity is
1.99. Wavefronts pass, the- [i_i restored. Inside the glass light encounters electrons
observer at pointA in medium! li bound', to/an atoih; this causes the electron to
with a-, certain frequency. The ': oscillate.' An. oscillating charge is a source' of further
Frequency with' which the electromagnetic radiations;, the neighbouring atoms
wavefronts pass the observer at ; absorb/this- electromagnetic radiations; The finer
B inrmedium 2 must.be equal to details of, these absorptions and radiations are the
frequency at which they pass Fig. 1:99 •
subject matter of quantum inechamcs. Overall, the
point A';. If this were not so, then the wavefronts'.wo'iiid process cpiisists"of'transmission of light from one
accumulate at the boundary or they would be destroyed or atom to Ae' other. Although light travels from one
created at the boundary; hence the frequency must be glass atom.t'o other" at 3 x 10® m/s, due to absorptipn
constant.
and radiation the average speed through the material
As Vi =vX,i and t»2 =vX.2 is slowed to 2x 10® m/s
50 J)PTI«
HUYGEN'S PRINCIPLE However, the wavelengths are different because of the
when a wave is incident on a boundary between media, different wave speeds (v = Xf)
some of its energy is reflected and some is transmitted. For The change in the direction of wave propagation is
example, when light travelling in air is incident on a described by the angle of refraction. In Fig. 1.101 (h),
transparent media such as glass, it is partially reflected and the angle of incidence is ©i and the angle of refraction is 02.
partially transmitted. We can analyze this phenomenon on From the geometry of two parallel rays, where d is the
the basis of a geometrical method developed by Huygens. distance between the normals to the boundary at the points
Huygen's principle states that : "Each point on an where the rays are incident,
advancing wavefront can be considered to be a source of . V-yt
sm0i = —^ and sinOo = —^
secondary waves or wavelets and the line or swface tangent to ' d ^ d
all these wavelets defines a new position of the wavefront. Combining these two equations in ratio form gives
Huygen's principle is applied to incident and ^ (Snell's law)
transmitted wavefronts at a media boundary as shown in sin02 V2
Fig. 1.101 (a). The wave speeds are different in the two This expression is known as Snell's law.
media. In this case, > ^2- C^he speed of light varies in
different media and, in general-is less in denser media. REVERSIBILITY OF LIGHT RAYS

•^1
While analyzing refraction observed when light is
Medium 1 Wavefront incident on the interface between two media, we assumed
at t
Wavefront that light propagates in a certain direction indicated by
att = 0
arrows. Let us now see what happens when light propagates
in the reverse direction. For the case of reflection, this means
that the. incident ray is-directed not from the left and
downwards as in Fig. 1.102 (a) but from the right and
downwards as'in Fig. 1.102 (h). ^
For' the "case of refraction, - we shall consider the
propagation of light not from the first to the second medium
as in Fig. 1.102 (c), but from Oie'second to the first medium
Medium Wavelent
(optically as in Fig. l.T02Xd).
from
denser) point A Accurate measurements show that both for reflection
and refraction, the angles between the rays and the normal
(a)
to the interface remain unchanged, and only the direction of
arrows is reversed.' • •
Normal Normal
-

Av '
\h

X B

Fig. 1.101 "" .

• The distances ihe'wavefronts travelin a timet are Ujt in


- -

. .c
medium 1 and'v2t in mhdium 2 [Fig. 1.101 (b)]. As a result W-- (d)
of the smaller wave speed in the second medium, the Fig. 1.102
direction of the transmitted wavefront is different from that
of the 'incident wavefront. The ;particles of the second Therefore, if a' light ray is incident along CB [see Fig.
medium are set in motion by the incident wave disturbance, 1.102 (b)], the reflected ray will propagate along BA, i.e.,
thus the frequency of wave is the same in both media. the incident and' reflected rays have changed places in
comparison with the former case. The same is observed for
rGEOMETRICAl OPTICS _„ L- 51

refraction of a light ray. Let AB be an incident ray and BC a


refracted ray [see Fig. 1.102 (c)]. If the light is incident
along CB [see Fig. 1.102 (d)], the refracted ray will
propagate along BA, i. e., the incidentand refracted rays have A parallel plate of thickness d and index of refraction n2 is|
•surrounded by a medium with index of refraction n^. A light
changed places.
'beam passes from medium 1 to medium 2, with an angle of.
Thus, both in reflection and refraction, light can follow incidence as shown in Fig. 1E.53 (a) Show that the.
the same path in the opposite directions (Fig. 1.102). This 'emerging ray is parallel to the incident ray. (b) What is the'
property of light is known as the reversibility of light rays. 'lateral shift ofthe ray as shown in the figure ?
The reversibility of light rays indicates that if the
refractive index is n for the transition from the first to the
second medium for the reverse transition it will be 1/n.
Indeed, let the light be incident at angle i and refracted at
angle r so that n = sini/sinr. If for the reverse direction of
the rays, the light is incident at angle r, it must be refracted
at angle i (reversibility). In such a case, the refractive index ie^
n' = sinr/sini, and hence n'=l/n. For example, when light
passes from air to glass, n = 1.50, while for the reverse
transition from glass to air, n'= 0.67 = 1/1.50.
The reversibility of light rays is preserved in multiple
reflections and refractions which may take place in any Fig. 1E.53 (a)
order. This follows from the fact that in each reflection or
refraction, the direction of a light ray can be reversed. ' Solution: (a) First we apply Snell's law at the upper
surface.- • \ • '
Thus, if a light ray emergingfrom any system of reflecting
and refracting mediais reflected at the last stage exactly in tHe rii sinOj = n2 sin02
backward direction, it will pass through the entire system in sin02 = ~sin0i '
the .reverse direction and'retum to its source.
Now we apply Snell's la\v at the lower surface.
n2 sin02 = Hi sin03
sin03 = —sin02 ... (2)
"i
Now substituting eqn. (1) in eqn^ (2), we have

Therefore03 = 0^; the ray emerges parallel to initial ray


however it is displaced. - .
InAAOV, ' t;=,AC sin(0i-02) ...(3)
InAABC, AC=—^ " ...(4)
cos 02
From eqns. (3) and (4),

I = —-—sin(0i -©2)
COS0'

=.dsin0i
^_ sin02 COS01
.:.(5)
V'
sin 0,1- COS 02

(.ni/n2 sin0i)cos0i
' = dsiri0i 1-

Fig. 1.103: Tothe reversibility (flight raysjn refraction.


sin0i Jl - —sin0i
52. OPTICS

COS0] Substituting these in Snell's


- dsinOj 1- ... (6) law.
(n| - nf sin^Gj)^^
0 = ^202
If 01 and 02 are small then sinGi =^0i and
COS01 - COS02 - I; thus we get -Ho -
d' d
2
= 1.50m
'd=2.0m
"2 (4/3)
= te, ... (7) Note that the above
Coin
derivation assumes that rays are
Cb) Solution of part (a) is true for any number of parallel very close to the normal line. Fig. 1E.54
media.

ni
Fig.lE.55 shows two perpendicular mirrors thafform two.,
02 "2 'sides of a vessel filled with water, (a) A light ray is incident,
normal to the water surface. Show that the emerging ray is;
03
"3 [parallel to the incident ray. Assume that there are two
reflections at the mirror surfaces, (b) Show that the result is'
n4 \truefor oblique rays. >

ni


Fig.1E.53 (b)_

sinGi = n2 rin02
sin02 = n^ sin03
sin 03 = n4 sin04
sin 04 - Hi sin0' (a)
Fig. 1E.55
Combining these, we get
01 = 0' Solution: (a) The normal rays are not deviated. The
• If rii =1 and n2 = n, the eqn. (6) reduces to angle of incidence at first mirror is 0i = 45®; it will be
1 reflected at 45°. This means that the ray is travelling
1= 1 1-
horizontally after reflection. The angle of incidence at the
n^' + l
• Eqn. (7) is valid for rays close to normal line. second mirror is0^ = 45°;the angle ofreflection will also be
45°. Thus the ray leaves the mirror vertically and it will
come out without deviation.
54
(b) From Snell's law at air-water surface,
•A swimmer has dropped a coin in a pool 2.0 rp. deep. The coin| sin01 =nsm02. The normal to the water surface and
[does not look that deep. Why ? How deep does the coin appear', normal to first mirronmake an angle 45°. If normals n^ and
;to be when he looks straight down into the water ? Take, n2 are produced, the triangle formed will have interior angle
135° [see Fig. lE.55(c)].
- J - •

Solution: The ray diagram shows that the rays


travelling upward from the coin on the.bbkbm are refracted
away from die normal as they exit' the Water. The rays
appear to be diverging from a point higher in the water.
From Snell's law,
9O'-04=9O'-(45'+e2)|
sin01 = n2 sin02
=45°-02 (
For small angles, sin 0 - tan 0 - 0i. With 0 in radians, we
have
X X Fig. 1E.55 (c)
011 - tanGi1 = — and Go2 ^tanGo2 - —
^
rGEOiVlETRiCAl OPTICS 53

From figure, 63 + 02 +135° = 180° apparent depth


83 = 45°-e2 real depth
The angle of reflection is also 45°-02 real depth.
or.
From the triangle formed by n2and n3, apparent depth
03 +84 +90° =180° or apparent depth = t/p
84 = 9O°-03 = 9O°-45°+02 = 45°+82 The image shifts closer to eye by an amount
The angle of reflection at the second mirror is also OI=PO-PI
45°+02. The angle made by n4With the inclined surface is PO-PI
135°. From Aefg, PO = 1- EL PO
PO PO
9O°-04 +135°+05 = 180°
(45°-02) + 135°+05 = 180° or. At =

85 " 82
Now we apply Snell's law Similarly if observer in P2 = p
sinOg = nsin05 and object in medium Pi = 1
As 05 = 02. p _ apparent depth
This means 06 = 0i 1 real depth
The exiting ray is parallel to the incident ray. or apparent depth = p(real depth)
Apparent Depth = Ft
A point object O is placed in a medium of refractive Case (i) Let object distance is u and image distance v.
index Another medium of refractive index P2 its Light travels from Uj to rij. All the distance are measured
boundary at PA. from surface. Distance measured in direction of light is
positive and apposite to it negative.
We can write above result as

• =o
V u
V n2
Let a coin lie at the bottom of a
pool, at a depth t.
U=-t, "2= "air-" f
til
••l — "water — n

1 n CD
= 0
(-t)
(a) (b) Fig. 1.105
Fig. 1.104
V = —
n
If i and r are small,
' The negative sigh shows that image is formed at same
.PA
sini == tani = — side as the object. The swimming pools appear more shallow
PO
than they actually are because when the light rays begin
PA from the bottom of the pool (ui = 1.33) and enter air
and sm r = tan r = —
PI (1X2 = 1), they are bent away from the normal. '
Thus,
P2 „ sini . Case (ii) Similarly if we have seen an object in air from
Pi sinr .water, it looks farther away, because when,rays from an
object change the medium they bend toward normal.
- (EL EL
~ ^Poj'[pAj PO
n 1
= 0
C-t)
Suppose medium 2 is air and an observer looks at the
V = -nt
image from this medium [Fig. 1.104 (a)]. The real depth of
the object inside medium 1 is PO whereas the depth as it
appears to the observer is PI referred as apparent depth.
Writing P2 = 1 snd Pi = p.
54
OPTICS

The negative sign shows that f—


Shift,
image lies on the side of the object.
• In general,
jii s = h + t~h~ —;
= 0 [sign substituted] P-
V (-u)

_
V = -
"2 s = t| 1 - -
u
P-

When n2 > n^, object is in rarer Condition (2)


medium, viewer in denser medium,
V >u.
^(h+t)-t

When n2 < rii, object is in ( ^ 1


denser medium, viewer in rarer
medium, v <u
Fig. 1.106
• For an object in water, viewer ' P1 " P2 0 1 *
in air, the shift in the position of object. * h * *s *
Shift = real depth - apparent depth = t-t/n H(h+t) '
Pr Md i^R
Similarly in case (ii). Shift = (nt -t)
Fig. 1.109
• When an object is placed near a glass slab and viewed
from the other side of the slab^ the object seems.to be ^ _\i(h +
+ 1t)-t r.
., . _ shifted by (t - t/n) in the direction of,light. Shift, S " ~h
P
• .When, a gjass slab comes, in the .way of converging
rays, the point of convergencejs shiftedby (t - t/n) in s = h + t: H
the direction of light, i.e., away from the glass slab. ' • p '
\ ' J

, , .1

'"n s = t
1_1^
Away from . P
Toward
normal Away from . j
iShift in the normal
normal
In both the conditions (1) and.C2) shift is same and in
direction
of light
. direction the direction of incidentray. Shiftdoes not depend on' h\ It
of light
Toward normal onlydepends on't'and ji.
Condition (3)

M-d'
' (•

h«-t-H
.(a).' ; ' • - .(b)
I
" ' , , Fig. 1.107

Condition- (1)

Fig. 1.110

Shift,

s = h + \it-t-h;
(nh +1) s = Qi - l}t

Fig. 1.108
GEOMETRICAl OPTICS 55

Condition (4) ^1 +^2 +^3"^ _ ^(0


l^eq
Hr
1^1 1^2 1^3

If tl —^2 ~ ^3 ~
1111

l^eq 1^1 \^2 ^3


' li 1 0
PARTIAL REFLECTION AND PARTIAL
, h ^
(h+t)/n REFRACTION AT THE BOUNDARY
OF TWO MEDIA
t

Fig. 1.111

Shift, 5 =

s = h-h~t + \it;s =,(}i - l)t


In both conditions (3) and (4), " " . jjq
shift is same and in the opposite t '
direction of incident ray. Again shift Fig. 1.114
does not depend on 'h'. It orily* -^2, ^2

depends on 't' and p. , ;13' 1^3


If partially reflected ray (1) and partially refracted ray
If there are number of media of
(2) .are mutually"perpendicular (this is possible-only for
different depths, one over o±er and
V-4 unique value of angle-Of incidence'iO' ' '
ko
their boundaries is/are simple plane ^k) "'1^2 •sihf -'' sirii ' sini '' ' '
Fig. 1.112
and parallel to each other; Pi sinr 1 sinC90°-i) ' cost-
Total Shift P.2 . —1 H2
tani =•—;i, I =
= tan
Laii
.S ='Sl-k-'S2'+53' + S4+:..'. 1^1
/' N
/• N / / 1

_ M-o i = tan •^
S = ti 1-^ + ^2 1 + ^3 1 +....
V-1
1 V P-2J k 1^3. -PV,
This angle 'i' is called "Brewster's angle" which will be
If combination" of media are replaced by a single further discussed in polarization. ' ' - .
medium of same outer physical size and shift produced is
Illustration 8:
same as above. Then R.I. of that medium is equivalent
refractive index of combination. Case (i) Interface stationary
"" .'H :
10cm

10cm

2m/s.

1 , Rl of water = -4/3. .
Fig. 1.113 - ] Fig. 1.11^. i
S = (tj ^"^2 "'"^3 + ^4+••••) 1-
\^0 Position ofbird as-seen by fish'-Xj^p— p^.- z) +,(/ - x)
•eq J

1^0
(ti + t2 + t3 + t4+....) 1-
^ . .....

eq Xb/f represents derivative of Xg/p wrt time.


* • • • •
\^0 1^0 -^s/F =P^[2-j] + ty-^] •
= tl 1- + t- 1-^1 +^3 1-
\^2 Hs
56
_ OPTlgl
= 4/3[-4] + (-C-2)
-10 ,
Similarly =(^ | +x
m/s

y
Note that z = -4m/s as z is decreasing ^FIB +

y = 0 as y is constant
#

x~-z m/s as x is decreasing.


= -9/4m/s
Method 2: Set reference point on the surface
Case (iii) Suppose base of container is silvered. Find
speed of bird as seen by fish in mmor and speed of fish as
seen by bird in mirror.

r 4m/s

2m/sn=
^B/F=l^ + y
mirror
Xpyp =px+y 77777777777777777777777777
Fig. 1.118'
. =|[-4]..
4 *
• Xp/p =li(z-y) + y.+ x
==15^/3 • from mirror

3 '
Xqip =-iL(2-y)+y+;x
Note that x have refers to speed of bird .relative to
surface. . =|[(-4)-0] +b+(-2)
o

X < 0 if X decreases -16


-2
3
y > 0 is y increases
-22
Caise (ii) Suppose interface is also moving find speed of m/s
3
bird as seen by fish.

m(z-y)

Fig. 1.117 Light ray


received
of fish iniiiNiunUJIIlllllHII
+ y
m(z-y)+y
Xq/p =irx+y r r /

• f .

=-^[-3] +[+l]
Fig. 1.119
= -3 m/s
Here x = -3 m/s as relative to surface bird is coming
near to surface.
GEOMETRICAL OPTICS 57

le 10
56 d' = = 7.5 cm.
15
1
See the figure
Refraction Across Multiple Slabs:
In Fig.1.123 an object is placed in front of two slabs in
36cm
contact. The thickness and refractive indices of the slabs are
air and t2,\i.b respectively. Where will the final image of
(n=1) the object appear to be ?
Slab I

water
36cm
(n=4/3r-----;
Eye

Fig. 1E.56

(i) At what distance will the bird appear to the fish.


(ii) At what distance will thefish appear to the bird. Slab 2
A object placed In front of
Solution: a) For fish: = 36+ 48 ^84 cm two glass slabs In contact
Fig. 1.123
dg = 36+ 48 = 84cm
(ii) For bird: dg = 27 + 36 = 63 cm A light ray emerging from O
now refracts at three surfaces. The
e I 57 first is between air and the
second between and p^, while
A concave mirror is placed inside water with its shining the third is between p j, and air. Let 0
surface upwards and principal axis of concave mirror. Find us solve the problem taking one
the position offinal image. step at a time. • •
1st interface Fig. 1.124
Li Air
Here, p^-l; P2=Pa
water
4/3
dj = -X, d2 = ?
30 cm
but = El
di d2
Therefore, the image distance d2 = -Pa'^ ...(1)
R = 40 cm 2nd interface
Fig.1E.57 (a) fib
Here, .. - ,
'Y

Solution: The
incident rays will pass Ii P2 X

undeviated through the di =-Cp^x + ti);


water surface and strike the d2=?
Fig. 1.125
mirror parallel to its 10 cm water
Since Ei = El
principal axis. Therefore for 4/3
30cm
the mirror, object is at «>. Its
image A [in Fig 1E.57 (b)] the image distance da = -p j, x + ...(2)
will be formed at focus P-a
which is 20 cm from the
R = 40 cm
mirror. Now for the
interface between water Fig. 1E.57 (b)
and air, d = 10 cm.
58 OPTICS

3rd Interface
Hb
,X
Interface 3 fY
X
interface 2

Interface 1 X

O •;
Fig. 1.126 Fig. 1E.5_8(b) !
Here, Hi =Hb, H2 =1 Water-Oil Interface:

+ t2,d2 =? dj = -8cm, Pi = 4/3, Pz = 1-5


di =-Hb x +
Ha !£i =
As £lL = E2
ni.
and the final image distance from the 3rd interface is di d2
tl t.
-1 +, ^2 H2
d2 - X + — or d2 = — di, we get d2 = -9cm
Ha H( Hi

Therefore, the net shift in the position of the image is Oil-Glycerine Interface:
di = -(9+9) = -18cm, Pi = 1-5,P2 = 2
s = - X + — + — -[-(x + ti +t2)]
Ha Hb As — = — or d2 = ~ di weget d2 - -24cm
di d2 Pi
J.
or •s = ti 1-^ + t2ll- — Glycerine-Air interface: •
Ha ^ Hb
di = -(4+24) = -28cm,Pi = 2,P2 = 1
Looking at the above result we realize that the net shift
in the position of the image is simply the sum of the As = or
individual shifts at each of the slabs if they were di
independently placed in air. we get da = -14 cm
Thus the final image is 14 cm below the clycerine-air
58
interface.

GLASS SLAB
'-A tank contains three layers of immiscible liquids [Fig. 1E.58
•(a)]. The first layer is of water with refractive index 4/3 and Single Glass Slab in Air
'thickness 8 cm. The second layer is an oil with refractive index (i) When a glass slab of thickness and refractive index p
3/2 and thickness 9 cm while the third layer is of glycerine is placed in the path of a convergent beam as shown in. the
Iwith refractive index 2 and thickness 4 cm. Find the apparent Fig. 1.127 then the point of convergence is shifted by
depth of the bottom of the container.-'
s = t 1-1
H

4 cm T p = 2

9 cm p = 3/2 'y

8 cm t ^1 = 4/3 0 1 0 1
1 ^ - 1 1^
P*~*1

•s s

! ' Fig. 1E.58(a) '


k
t

Solution: Case-l: Method of Interfaces: A glass slab delays the A glass slab brings closer
convergence of a beam the point of divergence
A ray of light from the object undergoes refraction at (b)
(a)
three interfaces. (1) Water-oil, (2) Oil-glycerine (3) Fig. 1.127
Glycerine-air. The co-ordinate system for each of the
interfaces is shown in Fig. IE.58 (b). (ii) When the same glass slab is placed in the path of a
diverging beam, the point of divergence is shifted by
GEOMHRICAL OPTICS 59

Solution: a
s = t
ray of light from the
Important It is important to note that the shift (s) is object undergoes
Y
refraction at three 1 • • • • 1

always on the direction of light


interfaces. (1)
If ±e slab is made of air and the surrounding medium is Air-Medium A, (2) X X ...
of refractive index fi, then the apparent shift would be Medium A-Medium ^
s = tOi-l) B, (3) Medium B-Air.
The co-ordinate
Concept: The refracting surfaces of a glass slab are system for each ofthe ^
parallel to each other. When a light ray travels through a interfaces is shown in Fig. IE.59 Cb).
glass slab, it is refracted twice at the two parallel forces and
finally emerges out parallel to its incident direction. The light Air-Medium A Interface:
ray undergoes zero deviation, 5 = 0. di = -i-14cn^ Pi = l,p2 = 1-5
Angle of emergence ~ Angle of incidence As ^ =
di d2
e = I
or ^2 = — d-,, we get = +21 cm
Incident 1^1
Rav Medium A-Medium B Interface:
Air
di = (21- 6) = 15cm, pj = 15,p2 = 2
As
1^1 ^1^2
did2
or d2 = 12, dj, we get d2 = +20 cm
Air

Medium B-Air Interface: ^


Emergent
Rav dj = (20-4) = +16cm, p^ = 2,p2 = l
1^1 1^2
As
A glass slab produces lateral
displacement
Fig.'1.128 or d2 = El dp we get d2 = +8 cm
The. lateral displacement ofCMthe ray is the perpendicular Thus the final image is 8cm in front of the medium B-air
distance between the incident and the emergent ray and is
il
interface.
t
given by sinfi - r)
cosr Concept: TheFig. 1.129shows three objectsOi,02 and
O3 located on a vertical line in a liquid of refractive indexp.
These objects are observed from air a long the vertical line.
e I 59
The distance between the images ofO^ and O2 is — and the
A convergent beam is incident on two slabs placed in contact
as shown in Fig. 1E.59 (a). Where will the rays finally distance between the images ofO^ and O3 is ^^ ^
converge? 1^

-A
/I
p = 3/2
Air

' 0
Tt

2 I
Air - - #02
3 :
6cm 4cm
^•03
14cm
Three points in a denser medium
Fig. 1E.59 (a) are observed from a rarer medium.
Fig. 1.129
,60 OPTICS

Note: That the distance between any two points in a denser


E Medium p
medium is reduced by a factory. -o-
•x? v/p 1 V 0

Example 60 x/p
K-

The image of an object in the denser


The image of an object kept at a distance 30 cm in front of a medium appears to move slower.
concave mirror isfound to coincide with itself. If a glass slab
Fig. 1.130 (b)
(a = 15) of thickness 3 cm is introduced between the mirror
and the object, then
61

A bird in air is diving vertically over a tank with speed 5 cm/s


base of tank is silvered. A fish in the tank is rising upward
along the same line with speed 2 cm/s. Water level isfalling at
Fig. 1E.60 (a) rate of 2 cm/s. [Take \i water = 4/3]

(i) Identify, in which direction the mirror should be displaced


so that the final image may again coincide with the object
itself.
(ii) Find the magnitude of displacement.

Solutioni (i) Since the


mmmmnm//
apparent shift occurs in the
Fig. 1E.61 (a)
direction of incident 'light,
therefore, the mirror should be Column-ll j
oc Column-!
displaced away from the objects. ol^ (in cm/s) t
(ii) The magnitude of
(a) Speed of the image of fish as seen by (q) 8
displacement is equal to the 30cm
the bird directly
apparent shift, i. e.,
Fig. 1E.60 (b) (b) Speed of the image offish formed after Cq) 6
1
S = t = 3 1- = 1 reflection in the mirror as seen by the
3/2 bird
cm
Cc) Speed of image of bird relative to the (r) 3
fish looking upwards
Concept: The Fig. 1.130 (a) shows an object 0 moving
toward the plane boundary of a denser medium. To an Cd) Speed of image of bird relative to the (s) 4
observer in the denser medium the object appears to be more fish looking downwards in the mirror.
distant but moving faster. If the speed of the object is v, then
the speed of the image will be |iu. Solution: ^ =-5
dt
Medium
Air dyi
. . . o- •>. • = 2
1 o V
dt

px dy2 = -2
The image of an object in the rarer dt
medium appears to move faster
(a) Distance of image of fish as
Fig. 1.130 (a)
seen by bird directly
The Fig. 1.130 (b) shows an object 0 moving toward the y2 - Ji
= (ys -y2) +
plane boundary of a rarer medium. To an observer in the
rarer medium the object appears to be closer but moving
slowly. If the speed of the object is v, then the speed of the speed _ 4X3 , ^72 I1 d/i (1
dt dt dt Ip. dt Ip
image will be —.
1GEOMETRICAL OPTICS ^
=-5- -1j-C2)| =~6cm/s mark is again focused. Finally, a little lycopodium powder is
sprinkled on the surface of the liquid and is focused as
before.
(b) Distance of image of fish as seen by bird after
reflection TOTAL INTERNAL REFLECTION
y2+yi AND CRITICAL ANGLE
X2(j3-y2) +
A material with a higher index of refraction than the

Speed
dx 2_dy3 _^fdyz -1
dyi other is known as optically denser, the medium with
~dt dt dt dt
lower index of refraction is optically rarer, quite
independent of the real relative density of the materials in
=-5 +C-2)| ^-1 1+2 kg/m^.
When light passes from an optically denser to rarer
3 3 medium, say from water to air, an interesting effect occurs.
= -5 — + 2 + — = -3 cm/s
2 2 Let the denser medium have refractive index n^ (here
(c) Distance of bird as seen by fish directly water) and the rarer medium, nj-in^^ > n^
^3 = Cy2 -Ji)+iiCK3 -jD
dx2 , ^73 I dyz dyi : f/
Speed Air '•/ ^2 Air JTH'SO" "2
dt dt dt dt
Water /k Water "1

=I(-5) +(-2)f-1 j~2=-8cm/s ' 7


W0ak
\ reflected light
Weak
^y^^reflected light
(d) Distance of bird as seen by fish after reflection
^4 =Cy3~y2)^l+ 72+^1 : // i No reflected light 1
Speed Air ''J^ n2 Air i' • .02
dt dt dt dt Water^ . ni Wate!>-:yp\,,_^^
=-^ +(-2)j^—ij+2=-4cm/s j; •\ Weak
4a \ reflected light
=' All llght"^^.
reflected

Measurement of Refractive Index of a Liquid by a


Fig. 1.132
Travelling Microscope
Using the fact that p=—real depth— From Snell's law, sinGj = n^ sin02
apparent depth
Since the light is travelling from denser medium to rarer
We may determine the refractive index of a glass slab. medium, the refracted ray is bent away from the normal
The travelling microscope is first focused on a fine mark line, that is, n^ > n/, we must have 02 > 0i.
on a piece of paper and the reading on the scale is noted as When the angle of incidence is increased, the angle of
Yi. Then, the glass slab is placed on the paper. refraction ©2. also increases. If we keep increasing ©i,
The microscope is now shifted up and is focused again eventually the angle of refraction ©2 becomes 90°. The angle
on the mark so that it is distinctly visible. The reading of the of incidence for which the angle of refraction is 90° is called
microscope is noted as 72- the critical angle ©g, the refracted ray grazes the surface.
Now, a little lycopodium powder is scattered on' the When the angle of incidence is equal to the critical
upper surface of the glass block The microscope is focused angle, Snell's law yields
on the powder and third the reading of the scale is noted as sinC = sin 90°

sinC - —
The real thickness of the glass slab is - 7^ and the
apparent thickness of the glass slab is 73 - 72- Thus,
If, e.g.,n^ =2 while =1, from Snell's law
2sin©£ = sin0r and provided sinOj < 1/2, the equation can
be satisfied. But for incident angles greater than this critical
angle C = 30°, the sin©^ would have to be greater than 1,
The same method can also be used to find the refractive
and that cannot be; hence there is no transmitted beam.
index of a liquid. Any mark on the bottom of a glass vessel is
first focused and then liquid is poured into it and the same
62 OPTICS

When the incidence angle is greater than the critical


angle, 100% of the energy in the incident beam is reflected
back into the incident medium and so the effect is called
total internal reflection.
At the critical angle, the refracted light propagates along
the interface, the wave is limited to the boundary, its energy
flows back and forth across the interface with no average
transmission into the second medium.

When light is incident on the interface between two


media, the luminous energy splits into two parts: one part is
reflected, while the other part penetrates through the Fig. 1.133: Total internal reflection : the
' thickness of rays corresponds
interface into the second medium. Considering the example to the fraction of the reflected
of light propagation from air to glass, i.e., towards the luminous energy or that trans
optically denser medium, we learned that the fraction of mitted through the Interface
reflected energy depends on the angle of incidence. In this
case, the fraction of reflected energy sharply increases with Let us consider again the incidence of light on the glass-
the angle of incidence. However, even at very large angles of air interface. Let a light ray be incident from glass "on the
incidence close to 90®, when the light ray almost slips along interface at different angles (Fig. 1.133). If we measure the
the interface, a part of the luminous energy still goes to the fraction of the reflected luminous energy and the fraction of
second me'diurh.' the luminous energy transniitted through the interface, we
If light propagating in.a medium is incident on the obtain the values compiled in table 1.2. (the refractive index
interface between-Ais medium and an optically less dense of toglass is n = 1.555). The value i^ of the angle of incidence
medium, /; e.ipnehaving a smaller absolute refractive index. starting
0
00 from which the entire luminous energy js reflected
ln,this^case, the ftaqtion of the. reflected energy increases from the interface is called the critical angle of total internal
wto tfre angle of incidence, but this increase obeys another reflection. For the grade ^of glass for which tabk 1.2 is
\swistartingfi-gm a certain value of the angle of incidence, the complied (n = 1:555), jhe' critical 'angle is. approximately
entire lurningus ^energy is reflected-from .the interfacei This equal to 40®.' , ' " ' ' " ' '' '' '• '''
0
0
phenomenon is loiown as, the total internal reflection. •

Table 1.2: Fractlon of Reflected Energy for Various Angles of Incidence of Light Propagating from Glass to Air.

Angle of incidence / 0° • 20° 30° 35° 39° 39°30' 40° 50° 60° 70° 80°

Angle of refraction r 0° 15M0' 32° 51° 63° 73°20' 79° -82° 90° - - . -
-

Fraction of reflected

Energy (in %) '4:7 ' 4.7 5.0 6.8 ' 12 23 36 47 100 100 100 ' 100 ICQ

, , .Totql,-internal reflectipn can rbe .observed- at the prism is 35-40® depending on the'refractive index of a given
boundaries of air bubbles in water. They shine since the solar grade of glass. Therefore, the angles of -incidence and
rays incident on them are vcompletely reflected, without emergence of light rays can easily be selected for such
penetrating the bubbles. This is especially noticeable for the prisms. Reflecting prisms, successfully play the role of
air bubbles which are always present on pedicles and leaves mirrors and are preferable since their reflecting properties
of seaweed and which; when Observed,in"solar rays, appear remain unchanged, while metal mirrors grow dull with time
as if made of silver, i. e.', are a good,reflector of light. beca.use of the oxidation of metals. It should be noted that
Total internal reflection is usddMn the construction of an erecting, pdsm._is ^simpler, in construction than any
X V (

glass reflecting 'and erecting prisms.'The .operation of these erecting mifror". system.. Erecting prisms', are used, in
prisms is explained by Fig. 1.134.,The critical angle for a particular, in periscopes.
GEOMETRICAL OPTICS 63

direct line from the heavenly body to the earth (as shown in
Fig. 1.136).
\
\ 2. The apparent A-^Appears to be at A
flattening of the sun and
\ moon when near the
horizon is due to refraction.

1 2 3
Rays from the lower part of
the sun or moon travel at
(a) (b) greater length through the
earth's atmosphere than
rays from the higher part,
Surface of earth
and so are more refracted.
Thus the lifting-up effect is Fig. 1.136
greater for the lower part of
Fig. 1.134 the sun or moon than for the top. The extremities of the
sides are equally affected, and so the moon appears elliptical
Critical Angle and Colours instead of circular.
When the light is just emerging into the air from glass, 3. Stars are a great distance away from the earth, and so
or water near the critical position, it is observed to be a very small proportion of the light radiated by them reaches
coloured. This is because the critical angle values are us. Light received from a star travels along paths which are
different for the different colours. It was shown that very variable, with a consequent variation in time taken.
=—3^, where cis the critical angle from the medium Thus the light received from a star by an observer- on the
sine earth is not uniform in intensity; at one instant little light-is
into the air. ' ' ' ' ' ' received, at another, light is received by more than one pathl
But PBiue light >lLRedWht' ^nd SO siu c, and hence c, is and so we get a twinkling effect. Planets are of greater
smaller for blue light than for red light. Thus when the light, angular size than stars, i. e., they subtend at the eye a'much
in passing out, is near the critical position, the blue portion larger angle than do stars, thus more light is received at any
reaches it first and passes along the surface whilst the red instant from a planet by an observer on the earth. Refraction
portion is being refracted into the air (see Fig. 1.135). effects are therefore inappreciable and so planets do not
I ' " • ' twinkle. This is borne out by the fact that stars, as seen in
large telescopes, do not twinkle, for the amount of light
Air
collected by the telescope and then viewed is approximately
t
constant.
Red
Surface
Blue
4. An observer under water sees (a) objects outside as if
Water they were in a cone of semi-vertical angle 48-°, and
.Critical angle
for blue light lifted-up, and (b) objects in the water away from him by
total internal reflection (see Fig. 1.137). If a glass of water
: I^ ^
: White
jlight
' •' Fig. 1.135
This effect is observable for all the colours in the usual
order violet to red. Water

\ SOME EXPLANATION OF SOME \


REFRACTION PHENOMENON / Observer

1. The altitude of a star or planet appears to be greater Fig. 1.137: Viewing objects from underwater
than it really is, for light from either is refracted in its path to be held in the air above the level of a bright light, the water
the earth, owing to a variation in optical density of air surface, viewed from below, appears like one of mercury of
layers. As the physical density of the air increases, or silver, and a coin or spoon in the water can be seen in this
decreases, so its optical density increases, or decreases. The surface by total internal reflection.
denser air being usually nearer the surface of the earth, the
path of light when received there is more oblique than a
;64
OPTICS

5. Hold an empty 8. After passing through a


test-tube slant-wise in Air • plane-parallel (parallel-sided)
water and look at it from
above (Fig. 1.138). Light In
/ /
plate (Fig. 1.141), the rays leave
it at the same angle i at which
travelling in water towards Critical
angle
they enterit. The plate displaces ®
±6 test-tube at an angle of a ray of light parallel to itself ^
incidence greater than the / / over the distance
critical angle cannot pass
into the air in the test-tube
' Water sin^i
A = dsini 1 -
and so is reflected upwards. n - sini

The total reflection causes Fig. 1.138


where d = thickness of the
the test-tube to appear as if silver-coated. Pour water into
plate
the test-tube; the effect disappears. The bright metallic Fig. 1.141
appearance of cracks in plate-glass windows when looked at i = angle of incidence of the
obliquelyis due to such total internal reflection in the glass rays
at the cracks. n = refractive index of the plate material relative to the
6. Mirages: Pools of water appear to be on the surface surrounding medium.
of a tarred road or a stone promenade on a hot afternoon. The luminous point S of a source or an illuminated
This occurs when the sun is shining along the direction of object seems to be brought closer to the surface of the plate
either of them and is at a certain height. Rays of light, by the distance
travelling through air layers of varying optical density,
gradually change in direction. As a result they strike some 1 - sin^i
A'=d 1 -
layers at angles greater than the critical angle. - sin^i
In hot countries the resultant effect is more common.
The layers of air near the earth become very hot and rare, With normal incidence ofthe rays (i = 0),wehave A= 0,
and, at the same time, less optically dense than the air and A'= d(n -l)/n.

4, More

Less
optically
dense
: 62

Investigate ho\y a thin film of water on a glass surface affects


Less
optically
the critical anglefor total reflection. Take n = 1.5 for glass-
e of light
HOT dense (Observer and n = 1.33for water, (a) What is the critical anglefor total
Inverted \ internal reflection at the glass-water interface ? (b) Is there
Image any range of incident angles that are greater than 9c/or glass
to air refraction and for which light rays will leave the glass
Fig. 1.139: Mirage produced in Hot Regions.
and the water and pass into the air ?
above. Thus some rays of light from a distant object travel as
shown in Fig. 1.139, giving an inverted image.
Solution: (a) Critical angle for glass-water interface.
Some light, when at the critical angle position, travels
along parallel to the ground and gives the appearance of
= sin ^ zzsin'M^ 1=62.5°
V 5 /
bright pools of water. This optical illusion is called mirage.
7. In arctic regions, and over the sea in hot weather, (b) Critical angle for glass-air interface,
mirages are seen in the sky. The air near the ground is cooler n. .-1
= sin"^ = sin = 41.8'=
and more optically dense than the air above and the effect is n. 1.5
as shown in Fig. 1.140. •
Image appears and for water-air interface.
upside down /• \
n.
= sin-^ = sin — I = 48.8°
:opLcally 1.33

Suppose a ray in the glass strikes the glass-water surface


'More
with an angle of incidence0^ < 0^'^, i.e., angle of incidence
optically is less than critical angle for this interface. Light refracts into
dense Object
water with an angle of refraction 0"'such that, from Snell's
Observer
Fig. 1.140: Mirage produced in Very Cold Regions. law.
GEOiyiETRlCAL OPTICS 65

sinO^^ = -^^sin0^ Solution: Let the refractive index of glass be n.


133 From Snell's law at face AB,
This ray will strike the sinGj = nsin02
lAir
water-air surface with the angle sinGi
of incidence e"'. If e"" > the sin02 = ... (1)
Water
ray will reflect at water-air
interface. If 0|'"' > 0^ > 0^'" we The normals at faces AB and BCintersect at right angles;
thus
might think that light will refract
02 + 03 = 90" or 03 = 9O°-02 ... (2)
into air.

For 0^ >0^'", we have Fig. 1E.62 If the ray has to suffer total internal reflection at face
BC, it must be incident at least at critical angle 03 = C,where
sin0"' =^^sin0^ >-?^sin( sin03 = —
133 133 ... m
15 1
X — From eqns. (2) and (3)
133 15 133

0^ > sin
-1
= 48.8°=0'^''' sin(9O°-02) =i or C0SG2 =—
n n
... C4)
133
On squaring and adding eqns. (1) and (4), we get
Thus the light is totally reflected at the water-air
surface. sin^Qi , 1
We can generalise this'result. We consider three media
such that
or n=^[sisin 01 +1
rig ^ JT-f ^ Hi
This is the required ri for.a given angle of incidence.'But
i3,2 _
sinG^^ = —, sinG^^ = — the problem^ states that irrespective of. 0^ the rays should
reflect at faice Bt. in such .cases-we niust work out-"what is.
.3,2
Let '3 ^ "C the leak angle of -incidence at BC. This angle is taken to be
The ray from medium 3 penetrates into medium 2 and the critical angle. ...
strikes the interface of medium 2 and' 1 with the angle of From eqn. (1), when 0^, is maximum, 02 is also
incidence Gjj. Now maximum.

, sin02 = — sin03 > — sinG^^ . From eqn. (2), 63 is minimum when 02 is maximum.
"2 n2 Thus least angle 03 is formed at face BC when the ray is
_ "3^1 _ incident at maximum angle at face AB. The maximum angle
as ineJ^=-^
sin
at face AB is 90°.
tt2^3 ^2 ^2
Hence 02 > 0c^ and so the ray is totally reflected at the Thus, n =V(sin90")2 -M = V2
interface 2-1. No light will penetrate into 1.
As sin03 - —
.63
if 03 is maximum, n must be least.
A light ray enters a glassslab at an angle of incidence Q-^.What Any ray that is incident at face AB at an angle less than
'can be the minimum value for the index of refraction of the 90° wiU make an angle greater than 03 at face BC; hence it
•glass if the ray does not emerge at face BC, irrespective of
[angle » will reflect at face BC. Ifn > V2, as sin63 = —, the angle 03
will decrease, i.e., the required critical, angle is decreased.
Method 2;,The condition for total internal reflection at
the second surface is
nsin03 > 1 • • , •
As 02 +03 = 90°, so ncos02 £ 1 ;
n^(l~sin^02) >1 ' '.
From Snell's law, sinGj =
Fig. 1E.63
66 OPTICS

sin^Bi
1- > 1

-sin^Oi > 1
idl-Qc
> 1+ sin^ 01
The largest value of n corresponds to Gj = 90°.
Hence n > V2 Fig. 1.144

i Concept: Deviation (8) - The Fig. 1.83 shows a light,


Area of Spherical Cap
•ray travelling from a denser to rarer medium at an angle
smaller than the critical angle 6^. The deviation 8 of the light Area (A) of die spherical surface
ray is given by 8 = 02-01. Since lip sinBi = pjj sin02/ within a cone whose vertex is at the
[therefore, centre of sphere and semi-vertex angle B
is given by,
i sinB2 =^^sinBi =psinOi;
; ! • A=jdA =2jrR^J%in0d0
02 = sin"^(p.sin0i) A = 2jtR^a-cbsB)
8 = sin'^Cp smBi)-Bi Solid angle (co) substended by this Fig. 1.145
! This is a non-linear increasing equation. The maximum cap is given by, •.' -
\value of 6 occurs when 0i = 0c, o.nd is equal to, A

CO = —- = 27i(1 - COS0)
TZ
max
R-

Circle of Illumination
circleof vision
Go rS: '

Medium

(eiOc)' 1
j. I FjgJ.142. . .j
If the light is incident at.an angle Gj > 0^, then the angle Denser

of deviation is given by, , '


; 8 = jt-^i ; Fig. 1.146

•• '1 1
• sinBj. = ; ' tanG^ = —
• •: ^ • • ••
Medium
=

P-R

, (0i>ec) tanG.. = —

Fig. 1.143
r = h tan0(
This is a linearly decreasing function. The maximum> •h
r =
value of b'occurs when Bi = 0^ and is equal to, '' ^ •
I Smax Observed at position 'O' in denser medium can see
I The variation ofbwith the angle ofincidence 0i isplotted, whole of world in rarer medium through this circle of
fin Fig. 1.144. ' ' " '' •
illuinination'only. According to reversibility of path of. light
observer in rarer medium can see the object 'O' in denser
medium only through rays coming out from this circle of
illumination; If this circle of vision is covered with opaque
i GEOIVIETRICAL OPTICS 67

disc, then above mentioned observation will not be possible. interface, then total internal reflection takes place. Now if
Radius of circle of vision of red colour is maximum and that transparent medium (III) of refractive index p 3 is placed on
of voilet colour is minimum this is because p depends on X. the interface as shown in Fig. 1.148 (b), then for any value
Only those light rays emerge from denser to rarer which of p 3 all light will ultimately be reflected back again into
are confined in the cone whose vertex is at '0' and forms medium (II).
vertex angle 20^ are transmitted. Med (I)
If a point source of light is placed at position 'O' in Med (I)
Med (111)
denser medium then fraction of light energy produced by
source emerges from boundary and it is independent of !Med([l)
depth 'h\
Med (II)
Because of total internal reflection, ray will be reflected
back into denser niedium if 0.> 0^, only that portion of
incident light rays will emerge through boundary which are Fig. 1.148
confined to cone ofVertex angle 20^ at the position of point
source. Condition-I

Fraction of light emerging from boundary 0^3 <!^l)'


. _ Area of spherical cap Initially
Area ofwhole of-the sphere > .<;in

_ 27cR^(1-cos0j.) ' . ' •


^ ' • » » , • . • • •

4kR^ Now in the presence of III medium.


_ 2jc(i - cos0c) -1 El
0' = sin
47C Med (I) V-i
.^2
_ solid angle at the, position of point source due to cone Med (III)
Since 1^3
total solid angle at the position point of source
So
M-3, .
• =-(1 VcpsQ* J =l(r- Vl - sin^ 0c 1 ^•2 1^2 ' j Med (II)
2 > 2v." / /
sin M— I< sin ^ El Fig. 1.149
1^2 4^2
Angle of incidence0 is greater than 0^ and 0^ is greater
than 6c, so 0 is greater than 0^.Total internal reflection takes
place at the interface of medium II arid III and light ray will
be reflected back to medium II.
circle of vision
r Condition-II
Med (1) i Hi

Initially
(4^3 >1^1)
0 > 0c
Med (11)"^"
H3
0 > sin
•-I
"El
Denser >2
/ \

= ,sin Med (If) H2


Fig. 1.147 Fig. 1.150

Since its >p-i'


% of light energy emerging from boundary,
p 3 1" ; -I
So sin_^ ' > sin ^
xlOO 1^2 1^2 .^27 \P'2y
. . 0'c>0c
, 1 Illustration 9: If monochromatic light is incident on a Since 0 is infinitesimally greater than 0^, so 0 is less than
plane interface to two media (I) and (II) of refractive index e'c • '
I^Vland p2Cp 2>Pi) atan angle 0as shown in Fig. I.i48 (a) Ray refracts to medium II as shown in Fig. 1.150. at an
and 0~is infinitely greater than the critical angle for this angle' r',
68 OPTICS

ji 2 sin 0 = II3 sin r

sinr - —
llo
sinS vG, <0 9O®-0, >0c - • ; -
^^3 90° >20^
ilo
sinr>—sinG^; sinr>——
Uo Ui 0, <45° " '
1^3 1^3 ^2 sinG^ < sin45°;
]£l
I J_
sinr >
^^3 •

sinr > sinG'cJ r > 0" II > V2


Gj critical angle for the interface of medium II and Successive total internal reflections take place at curved
medium I. Total internal reflection takes place at the upper surface and finally ray emerges from other end of cylinder
interface and finally ray will be transmitted back to medium through flat surface.
II. Here

OPTICAL FIBRE / OPTICAL PIPE Minimum Angle of incidence Required


The optical fibres functions on the principle of total
internal reflection. It consists of very long fifie quality glass
or quartz fibre known of the thickness or the diameter of the
strand is approx. 10"^ cm. and the refractive index of the
main fibre is approximately 1.7. The fibres are coated with a
layer of material of lower refractive index (1.5).
•Coating Fig. 1.153

^1 >1^2
sini > sinG,

Fibre Matena smi > ...(1)


V 7 (0i+i = 9O°)
Fig. 1.151 sin0i>='|i2 sin02
}ii sin(90°-i) = 1I2sm02
When light is incident on one end of the fibre at small |ii cosi = 1I2 sin02
angle, it propagates down the fibre without emerging
P2 sin02
through its curved surface of lateral surface. cosi =

The angle of incidence at the .boundary of fibre and its


coating is larger than the criticaT angle of the fibre material
with respect to its coating. The light undergoes repeated sirii = |l- —sin^02 •••(2)
total internal reflections along the fibre and finaUy emerges V Hi
out at the other end, even if the fibre is bent. From eqns. (1) and (2) we get,
Minimum Refractive Index Required
' " ' " transpaTent cylindrical rod" '>
74^>E2 ^=,i_4sin^
' :of uniform crossection i V M.? tii Hi Hi

sin^ 02 < H?-Hi


Hi '• • - - Hi Hi
V2

dense? medT? sm'


H?-Hi = sin
-1
iil -1
'2(max)
Hi iHzj
Fig. 1.152 ' ' ; This is the maximum angle of incidence for which a ray
can be propagated down the fibre. 02(max) called the angle
0 + 0^=90° 0=9O°-0c of acceptance.
If ray does not emerge from cylindrical curved surface, Illustration 10: Solid sphere is made of transparent
then total internal reflection must takes place. material and •its refractive index w.r.t. surrounding is
69
GEOMETRICAL OPTICS

> 1). Maximum surface area of sphere visible from point As b increases from zero, cos(j) decreases. Thus (j) has the
A on the surface of sphere is given by, largest value when
Ug cos(j) = n
n.
or cosb -

visible area, / \
-1
or ({) - cos -1 = COS -1=^49=
3

(b) If the prism is immersed in water.


Fig. 1.154 -1
-1
^ = cos = cos = 29®
Visible area = 27tr^(l- cos20c)
47cr'
= 2nr^.2sm^ 0c = 65 .
7^
Illustration-11: Optical rod of uniform cross-section tConsider a glass slab of refractive index n covered by a'thin]
bent in the U-shape. Rays incident normally at face A will 'sheet, ofwet paper. When a beam oflight strikes the paper at\
eventually come out of face B provided, •normal incidence, a distinctive circular pattern is'formed]
R
'.This illuminated circular pattern issurrounded by a darkringl
and again a circular bright ring. Find the radius of the bright,
^ '
Working 0 > 0c; sin0 > sin0c
Solution: in the Fig. iE.65 shown rays incidentat an
sin0 > sin0 > — angle less than the- critical angle' pass Arough the upper
V- surface whereas that incident at ah angle greater than the
•ic
R
>
1

criticalangle are reflected. The rays incident at.criticalangle
R-l-2r |i 02 = C, graze the surface, imagine a cone with semivertical
rt angle 02, the rays enclosed in it come out the others are
liR>R + 2r (\i-DR.>2r
R
— >
2 w reflected back to the lower surface where they escape into
.the wet paper and are scattered, forming the brighter
r p,-l surrounding area. '• - •
Fig. 1.155 -J

64
r -

'a ray of light is incidentnormally at face AB of a glassprism]


in = 3/2. Find the largest value for the angle bso that the ray,
jis totally reflected atface AC if the prism is immersed (a) in
iair and (b) in water. ' ' ' . ' !
1 1 A ,
' ' ' ' '

90'-^"^;

90°-$ "77?^
n2-1

(b)
i _ , _ '"•a-- i- " : . _ _ , Fig^1E.65_

' Solution: (a) No deviation occurs at face AC; hence From Snell's law,
the' angle of incidence at surface^ AC is 90%(|). For' total nsin02 = lsin90''
internal reflection at the second surface,
sin02 = — ... CD
Ug sin(90°-(|)) > Hq
y y ' . "rig C0S(t)> n^ -• .
70
OPTICS

©2 = 90

L90° :
'A point source of light is placed a distance h below the swface]
,of water. What is thefraction of light energy that escapes
frirectly from the water surface ? Ignore any absorption or
scattering of light energy.

Solution: The r = R sin e


Rd0
light that escapes is
confined within a cone
of apex angle 2C,
where. C is critical
Fig.1E.65 (c)
angle for total internal
From the figure, radius of illuminated circle, reflection. Imagine a
sphere with source of
r,= Dtan02
light as its centre and
D
r = suppose that the
4^ surface area A of the
Similarly if an sphere is inside the
illuminated point object is cone. At each point of
kept at a depth h in water, the sphere the
Fig.1E.67
the light rays that are intensity is same.
confined in a cone of angle of. therefore the fraction of total energies passingout is
vertex 0 = 2C will emerge
/ = " -
and the circle of illumination
, • '.47cR^ . ,
will have radius
where R is the radius of the sphere. •• - ;
r = Now we will derive A in terms of critical angle 'C.
Fig. 1E.65 (d)
4h^:. Consider a differential ring of, angplar thickness d0 on the
spherical surface. From the•figure, ±e radius of this ring is
r = R sin0. Its thickness is R dQ,and circtimference 27tR sin0.
Hence its area,
Consider a solid glass cube of edge length 10 mm and dA - 2tzR^ siri0d0
refractive index n = 1.5. A black spot-lies at the geometrical\
centre of the cube. Whatfraction of the cubesuiface must be Hence A'= 27cR^.fJoSin0ci0
icovered so as to prevent the spotfrom being seen ? 1
- 2jtR^(l- cosC)
Solution: , The light The critical angle,is given by
that passes .through each face
' r'
,90'/
forms a circle of radius r. sinC, = —
• ' n"•
Applying the result of
previous problem, cosC = Vl - sin^ C = Jl-
I «2
n j
r =

Thus
27rR^a-cosC)
10
/ = 1-Jl-
j- -47cR^,-
27(1.5)^-1 ' Fig. 1E.68
= 4.47mm
Area through which light can escape = nr^. [An isotropic source oflight is deep inside a lake. The surfaceofr
, - ir-

Six circles of illumination are possible; thus the fraction •the lake is covered bya layerofoil of refractive index1.2. Findi
of the surface are through which light can escape, thepercentage of light (a) escapingfrom the lake surface and
>Cb) totally internally reflected in the oil layer. Assume that
f- - ^^(4-47)^ = 0.63 both oil and water are 100% transparent, refractive index of
6L^ ~ (10)2 water is 1.33. ;
GEOMETRICAL OPTICS 71

Ogir 1 Air :

>2 = 56.4° Glass mugs are often made with thick sides, so that they
= 56.4\y appear to hold more than they really do. Whenfilled with say
Hgil - 1.2 'apple juice it appears to extend all the way to the outer
Water Z '.surface of the mug. Fig. 1E.69shows aview from above of the'
64.5% _
64.5°
path of a rayjrom a point Pin the juice at the inner suiface of
48.8^ the mug. Take refractive index of glass and juice to be 3/2 and
4/3 respectively.
"^water~

/
_ Fig.1E.68

Solution: Light refracts at water-oil interface and


then oil-air interface in order to come out.
Critical angle for water-oil boundary is given by
. ^ 1.2sin90° 1.2
sm9i =
1.33 1.33

01 = 64.5°
Critical angle for oil-air boundary is given by
sinBo = —
^1.2^ /. • • • .
Fig. 1E.69 ^
02 = 56.4°
Let a ray incident at angle 02 at oil-air interface make an (a) Findt/r ' ' • " ,
angle 03at water-oil interface. - - >• ' ' (b) Find the ratio'ofthe apparent volume ofjuice to the actual
From Snell's law, . r - • 1 volume ofjuice in the mug.
^water sinGg =fioii'sine£ '
Solution: (a) The figure is drawn so that the ratio of
1.2siri56.4' 1 -
sin03 = the wall thickness t to the inner radius r is as large as
1:33 133'
possible. In this case the light from point P appears to be
03 = 48.8° coming from P' at the edge. For thicker walls light from
Proceeding similar to previous example, we can find the point P would appear to be coming from inside the edge. For
solid angle subtended by the vertex of the cone of angle a as thinner walls, light from point P could not be seen from this
(•a 27csin0rd0 direction, and light apparently coming from P' would-be
= 2^ (1 - cosa) originating at another point in the juice.
Jo

At" this point the ray is tangential and will refract at


Percentage of light escaping from the lake surface critical angle the glass; similarly at P' once again
=^Cl_cos48.8°)x,100 •' critical angle (fic-is formed.>.
471
Critical angle at juice-glass interface,
= 17%

Percentage of light totally internally reflected at water siiK^c = = 62.73°


oil surface - i ,i• Critical angle at glass-air interface,
=i(l-cos64.5°)xl00 - sin fc - fc = 41.8r
2

= 28.5% S- X
From AOPM, sin0 = CD
Therefore percentage of light totally reflected at oil-air
surface • ' •
From APMP',tan(J)c = ... (2)
= (2a5-17)%=11.5% S + t
72
OPTICS

where 5 = r-(ci + t) = r-(rcos0 + t) Normal

Angle (j)c is exterior angle of triangle OPP'.


Normal
Hence , 0 + (|)c = <j)c
Q= ^c~¥c=2-0.9r Normal

Dividing eqn. (1) by (2), we get


sin0 r-(rcos0 + t)
= l-cos0 + -
tan (fic r

£ sin0
+ COS0-1
r tan (l)c
On substituting numerical values, we get
- = 0.33
r

(b) Actual volume = nrH


Apparent volume = 7c(r+ t)^Z Fig. 1E.70 (b)
' j
Apparent volume _ + Hence the required condition is
Actualvolume
h - ^critical
As sin0critical — ~ - J
p 3

J 70 from geometry of figure, we have


R
sin0o =
d + i?
, A rod made of glass, ^refractive indei 1.5 and of square
cross-section, is bent into'the shape shown inFig.lE.70(a). A' So sinn03 >'sin0^ti,,i
parallel beam of lightfalls^ndrmally^on theplaneflat surface-, R ->2 •
A. Referring to the diagram d is the width of a side and R is' d+ R 3' f
'the radius of inner semicircle. Find the maximum value of
'ratio d/R so that all lightentering theglass thrbilgh surfdee A' or i<i
R 2
emerges from the glass through surface B. . •' '
Therefore
naviTrtiim
p'
^

71

Rays of light fall on the plane surface of a semi(ylinder of


refractive index n = 42, at angle 45° in the plane normal to]
the axis ofcylinder. Discuss thecondition thai the rays do not',
isujfer total internal reflection. |
Fig. 1E.70 (a)

, Solution: Fig. lE.yO'Cb) shows three rays 1, 2, 3


incident on plane face A. .We can see that angle of incidence
at curved surface is least for ray 3.' If ray 3 reflects at the
curved surface, then all the rays will reflect as their angle of Frg. 1E.71 (a)
„ JJ
incidence is greater than angle 03.
Solution: First we consider a ray incident at A. From
Snell's law.
GEOMETRICAL OPTICS : 73

lsin45° = V2sin0; For no total internal reflection,


ZA'CO < 45°
sin02 = —
^ 2 In AOA'C, ZOA'C + ZA'CO + ZCOA' = 180°
02 = 30° 12O°+04 +(180-{t)) = 180°
04 = (!)-120°
01=: 45 Thus, ^ -120° < 45°
(1) < 165°
Hence for rays to transmit through curved surface,
75°<(t)<165°

Normal

Normal
A fish is at a depth d under water, refractive index n. Showi
'that when thefish is viewed at an angle 0, the apparent depth\
Fig. 1E.71 b
d' of the fish is, d' = 1
, (n^~sinHf'^ , I
Let the angle (j) = ZAOC denote the position of the point
C on the curved surface.

ZCAO = 60°

The critical angle, for glass to air interface can be


determined from Snell's law.
nsinC = 1 sin 90°

sinC = —= i ^'
n ,42
Fig.1E.72 (a)
C = 45°

So|utioni We cannotuse formula for apparentdepth,


that is applicable for. small angles of vision. The object
appears to be shifted not only upwards but also laterally.
•30'
From figure, AS - dtana
AB'= d'tan 0
Z A'CO = So, l = AB'-AB
= d' tan0- dtana ...(1)
As lsin0 = nsina
(c) (d) sin0
sina =
n
Fig. 1E.71

cosa - Jl-
sin0Y
If total internal reflection has to take place at the curved
n J
surface, angle 03 must be greater th^ the critical angle,
C = 45°. sm0'
•t'aha'= ...(2)
t As 03 = 180°-<|) - 60°, therefore, for no total internal 4n^'- sin^Q
•4

reflection, dsin0
I = d'tan0- ... (3)
180°-(j) - 60° < 45° •v/n^ -sin^G
.'.."or ^>75° Let x(0) = tan0 ... C4)
When the ray falls at O, the refracted ray vnll move
radially out, without deviation. The normal rays do not and y(0)= ... (5)
suffer deviation. Next we consider a ray to the right of O. 4n^ - sin^ 0
Thus l = x(0)d'-y(0) ... (6)
74 OPTICS

The two rays incident at angles 0 and 6 + d0 reach the [Image due to
image point S'. Hence for the other ray first refraction
through glass]

»• I s
B' B S object
-nh

a rf.v.-.v.v.v.v.v.v

Fig.1E.72 (b)

at an angle 0 + d0, we have


*.!• ' [image due to ^
I = x(0 + d0)d' - y(0 + d0) (7) nh+2d • li reflection at
\ ; firstsurface]
From eqns. (1) and (2), wet get
x(0 + d0)d'-x(0)d' I4 _[Final image] •
nh+d

= yCe + d0)-yC0) '


or d' =
y(0 + d0)-y(0)
x(0 + d0) - jc(0)
C8) 1 1 i [Image due to ^
'• I3 reflection at
mirror]
If we take limit A0 0, we will get the desired result Fig. 1E.73
d' =Lim fy(e +d0)-y(8)]
A6->o\ d0
/|'x(0 +d8)-x(0)]
j/ 1 d0 J |(dj Find the position of the image ofyour face that you see
\due to reflection from the front surface of the glass, (b) Find
, . , ,- dy.(0) /dx(0) ..... C9) [the position of the image ofyourface that you see. due to
d0 / d0, , Ireflection from the mirror surface behind the glass. Take into
dy(0) n^dcos0 •account the effects of refraction at the glass-air interface, (c)
From eqn. (5), How far apart are the images'\n parts (a) andjbj ?
d0 (n2-sin^'0)^2
From eqn. (4), = sec^ 0 Solution: (a) First image is formed due to reflection
d0 at upper surface formed at h distance below upper surface.
n^dcos^ 0 Cb) Second image 12 is due to refraction at air-glass
Hence d' =
(n^-sin2 0)^2 interface. From single surface refraction equation, we have
dsin0 — ^ = 0 or v=-nh ... (1)
Also I = d' tan0- (.-h)
(n^-sin^0)^^
Image is virtual, formed a distance nh above the upper
_ -(n^ -l)dsin^0 surface. Image 12 is an object for reflection at the mirror. The
(n^-sin^0)^^ image J3 is formed a distance (n/i -1- d) below it. Image J3
n When the object is viewed at grazing angle. acts as object for final refraction at glass-air interface, when
d the rays come out of glass. From single surface refraction
0-jc/2,d' - 0 and I - equation, we have ;
Vn^ - 1 n
= 0
the object appears almost at the surface but laterally v' (nh + 2d)
displaced by 1. . ' '
, 2d
V = h + —

VARIABLE REFRACTIVE INDEX


|77ie primary reflecting surface of a bathroom mirror lies>
behind a thin sheet ofglass. This sheet is used to protect the\ If refractive index of medium is variable, then ray
travells along curved path.
reflecting surface from tarnishing. Consider a'bathroom]
tmirror with a glass sheet ofthickness d and index ofrefractionl Consider a case where refractive index is function of y
[n. Yourface is located at adistance hfrom thefront surface of If • \i = f(y) ,
\the glass sheet.
75
GEOMETRICAL OPTICS
The integral yields required equation oftrajectory.
y = fiM
Secondly refractive index may vary as function of x
If i.e., li = /U)
Pq sin0
sinGo =
^0

Posin(9O°-0o) = /U)sin0
. «
sin0 =
k ...(3)
I
Normal is along Y-^s and boundary is parallel to X-Z
plane.
Tangent drawn at a point (x,y) to the trajectory
followed by ray gives direction of incident ray atthat point. : x,y ! AG ,
\La sinOo = lisinO a = f(x)
^L„sin0o =/(y)sine
sin0 = ...(1)
/(y)
fc = p.aSin0o (say)
Fig. 1.158 '
If equation of trajectory followed by ray is given by, ,
Normal inside medium is along or paralleTt'o.:i:-axis and
boundary is along or parallel to y-z pldhe.
' —• = tan'0'
dy-./ .•- ^ =.taA0
dx '/ •
• (dx/dy) ' ' ...(2)
sin0 = . ^ (dy/dx) . .,..(4)
sin0 = ==-.

••'II
Equating eqns. (3) and (4) we get,
(dy/dx) . _ ,fc , .
f(x)

\ dx

Fig. 1.157

Equating eqns. (1) and C2)^we get, -. •


(dx/dy) k
dx^ 2 /(y)
1 +
dyj dy k

^1 {/(y)>^ + dyJ.

1- ,-/• The integral yields required equation of trajectpry,


' "r._ • k. .
dy' ^ =/(x) =tan (J)
dx

r'dx =fcj" ^=4= (|) = tan"^{/'Cx)}.


76 OPTICS

If angle (90°-(j)) is greater than critical angle at the' COS0 = Vl-sin^0 =. 1 [Ay ^2 + 1]
uppermost boundary then total internal reflection takes
place, otherwise ray immerges at an angle p to normal.
sin(90°-(l)j _ M-g
l/[Ay^2+i]i/2
sinp /CO
From above formula you can find p. From eqn. (3),
sin0 = = C0S(|)
•^.1 74 n(y)

'A ray of light travelling in air is incident at grazing angle 7l +tan^(ft ' -^l +(dy/dx)'
•(angle of incidence -90°) on a long rectangular slab of a . 9lV2
transparent medium of thickness t = 2.0m. The point of, n(y)= 1+1^
- ., .
dx
incidence is the origin A(0,0). The medium has a variable-
\index ofrefraction given by '
[1 + Ay^2] = l +
! = •' ' ydXy

\where k = 1.0(my^^^. The refractive index of air is 1. (a)'


dx
Obtain a relation between the slope of the trajectory of the ray\
at a point B(x,y) in the medium and the incidence angle ati
4y'^''=Vfcx +C • ' ...(4)
khat point ' ' j Now we substitute boundary conditions in eqn. (4),
X = 0, y = 0, hence C = 0. The required trajectory is
(b) Obtain an equation for the trajectory y(x) of the ray in^
the medium, (c) Determine the coordinates (Xj.,yj) of the,
point P where the ray intersects the upper sujface of the',
slab-air boundary, (d) Indicate the, path of the ray' (c) At point'P,y = Im^ k = we get
,subsequently.. , ' 'X = 4m ' ' '
The coordinates of P are (4,1). ' ' '
(d) From Snell's law,
Air P"(xi.yi)
Normal n'^ sini^ = Up sinip
atB
t=1.0m
As = Hp •= 1 _
so ip = = 90°-
A (0,0) • The ray will emerge parallel to boundary.
Air "
e 75
Fig. 1E.74

Due to a vertical temperature gradient in the atmosphere the,


SolutioniCa) The trajectory of the ray is shown by index of refraction varies. Suppose index of refraction varies]
dotted curve. The slope of tangent at point B is as n = noV^ + ay whefe 'hQ is the index of refraction at the.
dy •suiface and a = 2)0 x 10~^m~^. Aperson ofheight h = 2.0m!
tancf) =
• dx stands on a level surface. B^ond what distance he cannot see,
The angle of incidence at B is 0 = 90°-,(t>. .the runway ? _ ^ -

Hence -tanC9O°-0) =-^ Solution: Let O >be the Trajectory of


dx distant- object just-visible to light ray
the man. Let P be a point on '
or cote =^ (1) the trajectory^of the'ray.'From
dx
Fig. 1E.75, 0 = 90 -i. • J
(b) From Snell's law at A and B, we have ' • y
The slope of tangent at ,±
1 sin 90° = n(y j sin 0 (2)
pointPistan0 = dyfdx= coti. • Fig. 1E.75
1 1
sin0 = (3) From Snell's law, ^
nCy) + nsini = constant
GEOMETRICAL OPTICS 77

At the surface n = jIq and i = 90° a-x

Rq sin90°= nsini = (hq^I + ay)s'mi dx - (a-x)^


1
smi =
y=\dy=j-j. a-x
dx
^|a•^-ia-xf
coti =-^ =-fiy
ax
To integrate the above expression we substitute
-(a-x)^
2tdt = -2(a - x) (-dx)
t dt = (a - x)dx
x'=
y=l(^ =t=.Ja'-ia-xV
tdt

Onsubstitutingy = 2.0manda = 2x 10 \wehave Thus y^ = a^ -(a-x)^


X, = 2 = 2000m y^ +(x-a)^ = a^
-6
2x10
This is the equation of a circle of radius a with centre at (a, 0)
76 As y-coordinate of point A is ct hence
d^ + (x-a)^ =a^
iFig. 1E.76 shows the cross-section of a thin slice of
\atmosphere in the vertical plane, the thickness ofthe layer is x = a±-Ja^ -d^
As X cannot be greater than radius of circle at x = a, the
](L Therefractive index of air varies as n = with the ray
} 1-x/a ray grazes the upper surface.
entering at origin; Rq and a are constant, (a) Obtain an
i 77
lequationfor the trajectory y(x) of the ray in the medium, (b)
Find coordinates of the point A where it emerges, (c) What is
Mdex of refraction at A if it emerges at an angle a with the A thick plate is made of a transparent 'material whdse
Ivertical ? ,, . ' refraction index changesfrom n^ on its upper edge to n2 on'itsl
; j -y^ lower edge. A beam enters the plate at the angle a At what
angle will the beam leave the plate ? i

Solution: Let us divide the plate'into many plates so


T y.S"" :P (x.y) Normal
thin that their refraction index can be assumed constant
d within the limits of each plate [Fig. 1E.77 (a)].
•y y no
•X 1
90

5 Flg.1E.76(a) , ' n'

Solution: The refractive index varies along x-axis.


The. normal at any point P (x, y) is parallel to the x-axis. I
k
Slope of tangent at point P = tan0 = —
dy
dx
^ n(")

• "2
^
From SneU's law,
no sin 90°= nsin0 •
a "3
X
nr, =
n.
sine • y
1 - ix/d) . d 'p(x. y)
Assume that the beam enters the plate from a medium
a- X
sin© = with a refraction index of ngand leaves it for a medium with
a 90
« - -
a refraction index of ^2.
a-x
tan0 =
Fig. 1E.76 (b)
Then, according to the law of refraction,
^la^-(.a-xf sin a _ Ri
sinP Rq
OPTICS
sinp n'
sinX
Solution: Fig. ie.78 (b)
"i shows a light ray passing through
siny n" successive planoparallel plates of
sinS n' different refractive indices.
According to Snell's law,
sing2 _ singg _ ^2
singj n2'sing2
sinb . "2 The product of the sine of the . ^ 1E.78 (b)
Fig. , ,
sin^ angle of incidence and the absolute refr'active indei has ie
sin^
same value at all interfaces, i.e., nsing is.constant along the
"3 light ray's trajectory.
smx
"2 •
This relationship is also valid for a medium whose
refractive index continuously changes in one direction, since
Upon multipljnng.these equations, we get the medium can be considered as consisting of thin
sing _ rig planoparallel plates. Place the origin of the coordinate
sinx Rr.
system at the point where the light ray enters the medium.
In this case, the angle ofincidence for the first 'plate' starting
3ngle at which the beam' leaves the plate, aty =0is 90° and the refractive^ index is rig, which gives the
X-arc'sinj^^singjdepends onlyontheangleofincidence above constant as nCyjsmg =Uq.
The light travels along a
beam bh the plate and on the refraction indices of the circular arc of radiusR and,we' first
media-on both sides of the plate.'In, particular, if n. =Un examine its relationship to
then g.,- ^^ o' " coordinatey. FrOm Fig. 1E;78 (c); it
Note that the angle 0at' ^ is clear that ' • ' - i'
which the beam is inclined !\ "o * -
- • 1
Uq =nsing = n(y)
D _
•/
-w ,
. .

to the vertical is'related .to R


the refraction index n at any n
1
This gives. the\-,-space-
point on the plate by'the' Fig. 1E.78 (c)
dependence of the.refractive index
ratio nsin6= constant . as - " ~ ^ ^v
= Hg sing. If theIrefraction
n" R . \ -•
index reaches a value 'of- ri(y) =
n = Uq sing* .anywhere . _J R-y*^0 ,
inside the plate, full internal reflection will take place. In The matenal with the greatest known refractive index is
this case the beam wiU leave the plate for the medium atthe diamond, but even the refractive index of this material does
same angle g atwhich itentered the plate [Fig. lE.77(b)] not reach the value = 2.5. It.is this limit that sets the
maximum angular size ofthe arc the light ray can cover If
the refractive index changes from Hq =1to =Z5, 'then
77ie refractive index of the medium within a certain region • '.the maximum value ofyis |r, corresponding to an arc of
rx-directioti
^ ^ strikes the medium
'I f"'" 'feht ray travelling in the angular size'66.4°. • '
'at'right angles and moves
ittirougn the medium along a-circulararc. 79

•A portion of a straight glass 'rod of^diameter 4 cm and


refractive index 1.5 is bent into an arc of a circle of radius R
cm and a parallel beam of light is incident on itas shown in
>Hg. lE.79(a). Find the smallest Rwhich permits all the light
Fig. 1E.78 (a) •to pass around the arc.
•How does the refractive index depend on y. ? What is the
\maximum possible angularsize of the arc ? '

Ftg.1E.79 (a) j
GEOMETRICAIOPTICS

Solutioni The necessary and


sufficient condition for all the rays to pass |
around the arc is that the ray with least i
A circular disc of diameter 7 cm lies horizontally inside a
angle- of incidence should get internally ; Fig. 1E.79 (b) |
reflected.
'fietallic hemispherical bowl of diameter 25 cm: The disc is
ljust visible to any eye looking over the edge. The bowtis how
* • Froni the fi^re,it becomes obvious that the ray with filled with a liquid. It is observed that the whole of the disc is
least angle of incidence is the one which is incident almost just visible to the eye in the Same position. Find the refractive
grazingly with the inner wall.
index \i of the liquid. •
,. For this ray,, if 'a' be the angle of incidence,
R —d
'sina = r, where d is the diameter of the tube.
R
R- d ^ 1
But a>C sina > sinC
R 'n
d\i
^<1-1 R > Fig. 1E.81 (a)
R \i- ' • R p-1
4(3/2) Solution: in the figure, •ri
R> =^*R > 12cm 1

.(1/2) is the disc and 0, the


centre of bowl. , ! 0
Hence the least radius required is 12 cm. (
Let ZAOM = e , ♦ ' \
80 Then ZAIB = 0
Now from' geometry of
A spider and a fly are on the suiface of a glass sphere. Where figureZAb/ = 2Z7RA ' ^ '
must the fly befor the spider to be able to seeit ^Assume that 1 Fig. 1E.81 (b) '
the radius of the sphere is much larger than the size of the i.e., . Z/BA --(9O?-0) . ) I 1

spider and the fly. The refractive index of'glass is\i = 1.5. .' t I • >
90®-0
XM' .Zr = ZIBA . 1' I

• Solutioni Let us- consider the


spider to be at a height 7i ^above-the 9O®-0 9O®+0'
Also- Zi = + 0 =
sphere. We will let h -> 0 to make it an r 2 • 2' .
object on the sphere. In Fig. 1E.80, A is y-yyc . (90®-f^Y- '
the position of the- spider. Draw
tangential rays from A to the sphere. Now, Snell's'law gives J
Then these rays are incident at,90°. The . f9o®-e'\
sm ^
, corresponding angle of refraction is C I 2J
'(critical angle). Let the refracted rays J • 0 .0
ciit'the sphere.at D and'D'. Then D and Fig. 1E.80 cos — + s m -
2 2
' b' define the extreme portions of the' fly from' where it is or
e 0
= 11-
'< visible to the spider because by the principle of reversibility c o s — sin—
2 2
of the path of light rays, any ray from the,region between D
and D' will reach the spider. l + sin0 2
or =

Let ZDOD' = p. Then l--.sin0

ZBOB' = 2y (say) = 2;u - P-2(7t - 20) - " or '• 'a =' ^--
sm0 ^-1
.=> Y= 2C-P/2.\ ^ -

, , Now cosy . . But . sin0 = —


2a
' '' As h —» 0, cosy —»1 =>'y^6=>p f ,4C^ . - 11.^-1
or d = 2a
Hence the fly must be within AC in the opposite part. LL^+l
• Now 4C = 4sin~^ —= 4x41.8 = 167® For d = 7 and 2a = 25, we get p. = 4/3
1.5

So the fly must be within 167° in the opposite


hemisphere.
OPTICS
cosa
tan0 -
n-sina

[A man standing symmetrically infront ofa plane mirror with^ Looking at Fig. 1E.82 (a),
\bevellededges can see three images of his eyes when he is 3 ft.
'from the mirror [see Fig. 1E.82 (a)]. The mirror is silvered on _5_
cosa =
the back, is 2 ft. 6 in. wide, and is made of glass of refractive 13
\index 1.54. What is the angle of bevel of the edges ? + (3)-

12
sma =
13

- hence tanO = = 0.625


1.54-(1^13)
= 32°
Fig.1E.82 (a

Solution: The-man-can
an only't'
only t •*-'
see an image of his eyes if light-leaves'
It-leaves' , J • ^ A ray of light is propagating in a horizontal direction close to]
theriij 'strikes the - mirror and the Earth's suiface where the gradient of the refractive index
reflected back along-the .same path." - is-^ =3xl0- m (z = height above the Earth's suiface).*
• I

T^e central image is thus formed by dz


'light"- traversing'thevperpendicular i • •• c [The refractive index of air = 1.0 at the surface and radius of
•from his'eyes to the,mirror The outer Jearth = 6400^ km.
images are fomied by light striking the bevelled.edges at the (i)Find the radius of curvature of the ray of light. I
•pointi4 [see Fig.-lE.82(b)].a't an angle of incidence such
(ii) For wlicitfralue 'of the fradient would the. ray of light'.
that the angle of refraction <[)' makes the refracted ray strike [propog^e olZ fhe waj^rnuncyihe'earth
the silvered surface normally. >
This must be the .case if the ray of light it to leave the Sblutlpn: Let us consider a vertical (v +'dv).5t
bevelled edge the same path with which it arriyed. The angle wavefront whose lower end is at a height
(j)' lies between the normal to the bevelled adge and the z and the upper end is at a height z + 62. In
normal to the back surface. Since ZSAX = 90° [ see Fig. a short time 5t,.the upper end will move 'sd
1E.82 Cb)] by (u + du)5t and lower end by u5t.
.^'-•i-ZDAX = 90° ^ So, the angle through which the
wavefront has turned is
• But ZDAX = 9O°-0
(Change in arc length) Fig. 1E.83
Hence . - (b' = 9O°-9O°+0 = 0 U — :
(Change in radius)
DrawBA, a construction line at A parallel to the back of
(u + 5u)5t -u5t
the mirror. Angle BAC is also equal to 0.
5z
But by Snell's law, sin(l) = nsin0', where is the
refractive index'of air (hj = 1) and n is that of glass. Then A
8z
sihb'= nsin({)= nsin0. Also a = 0 + (9O°-^) [see Fig.
lE.82(b)] u5f vdt
Now R = radius of"curvature =
• ,^in[9O°-(a,-7 0)] = nsin0 ^St
dz
But •cc.i^i^(9.P°.q;\|/) =, cost}/
-It
and .cos(a ^.0) ..= R = = -xlOV
dn/dz 3
By the trigqnprnetric relation for double angles,
.-cosa.cos0 + smasin0 = nsinO
If radius of curvature = Re^ta, the ray would propagate
cosa + sinatan0 = ntan0 all around the Earth.
cosa = tan0[n - sina]
GEOMETRICAL OPTICS 81

= 6.4x10^ Distance of insect from the centre,


dfi/dz X = {H - h +x-^^)-3
d\i _ 1 • -7_-l 4
= 1.56xl0"'m = X-, — x^ +1
dz 6.4x10^ 3

84 I " +1
3
~h
Afixed cylindrical tank of height H = 4m and radius R = 3m| + 1 ... (1)
jis filled up with a liquid. An observer observes through a\
felescopefitted at the top of the wall of the tank and inclined] From the equation of continuity,
atQ = 45° with the vertical When the tank is completelyfilledi
]with liquid^ he notices an insect which is at the centre ofthel
\hottom of the tank. Att = 0,he opens a cork of radius = 3 cm' From eqn. (1),
\at the bottom ofthe tank. The insect moves in such awaythat^^ dx
ft is visible for a certain time. Determine +4A-^~ = A2^|2gxAO.-x)
= i ... (2)
• dt

dx A2
or dt
4Ai
•45>
or -[2a-x/"]S =^xV8i'-t
4Ai
T
''H = 4 m

Cork
or Vl - X = 1 - ^2 ...(3)
8A1
Insect -
From equations (2) and (3), we get
; ; Fig;iE:84{a)
4Ai X = A2 X X 1-
^2
[(a) The refractive index of the liquid, dt 8A1
S) velocity of the insect as afun^ion oftme.
or '8g-t
Sblutioni From Snell's law, we have dt 4Ai ^ ^ 32Af
rij sini = n2'smr
or V =
'^2 •8g't
sini =.l • sin 45®
•4Ai 32Af
. . 3
as smi = —
5
•(9 X10~^) (9 X10"^X 80 t
4x9 "32x9x'9
5.V2
'therefore rii =
=•[2.24- 2.5x'lO~^t] x 10"'^ m/s
When we/open'the cork; the Water level decreases in the
tank. Let after the time t the water level be h.aiid thie insect
:s5
be at point R
An-opaque sphere ofrcidius R lies on a horizontal plane. On
[thepefpendiculaf through the point of contact there is a point\
[source of light a distance Ra^ve the sphere. i

H = 4 m

Fig. 1E.85 (a)


Fig. 1E.84
[82 optics]
(a) Show that the area ofthe shadow on the plane is StcR^. j Putting in eqn. (2), we get R' = 4^
(b) A transparent liquid ofrefractive index 43 isfilled abovel Area of the shadow = tcR'^ - 27tR^
khe plane such that the sphere isjustcovered with the liquid, j
\Show that the area of shadow now becomes 2kR^. i 86

Solution: (a) Radius ofshadowon the groundisMP \A light beam of diameter VSR is incident ^qnmetrically on 3
Let ZMSP = 0 => ZSPM = 90°-e ' fglass hemisphere ofradius R and ofrefractive index n = 43.\
InAOSQ, ZSOQ = 4 f. , 'Find the radius of the beam at the base of hemisphere.
Triangles OSQ and PSM are similar' . . - •J3!R •
air
r '• R •• ~ •
hence — -
/ \ / \
n = J3\l/' \
or - r = VSR
Area of the shadow - tu (VsR)^ = SnR^ Fig. 1E.86 (a)
I .

(b) From the geometry of figure, we have


P'N = P'Q' + NQ' .y _ . . Solution: Hi sini=^2sinri.
= R' + iVT • ' • • - 1X sin60°='->/3'x sin r .
= R' + Rtani'L - • • •• r = 30° •' '
cosr = =
! 1 2R'r, .•/ AMNP ' .4.44 ''l . 'it:
P'N ' + Rtani tan 30® = x/ (R/2) i = 60°
or' -Rtani • •. A ' ...(1) R "! 4^
—...

. )
''Z243 " !i Jr/2 \
ii/'
• nJ7 Ti
-R-' r
L. .
X r
• 2. 2 2V3
Fig. 1E.86 (b)
/Z
'3\p' A' A -^
•6, ^ 2V3V ••'-Vs
' (• °
^ \ j -.
'K ^ -

,• A

P' N- M 87
M. , P
. ;c'.' • A'-
-4^) ' . •. (c) . ,
' ';A ray oflight travelling in air is incident at angle ofincident
. • '-"i -» .
Flg.1E.85' ^ '.ISO® on one surface ofslab in which rep-active index vari^Y-
\ W-', ' 'A, "A '..'•'1' >' V. I'j ' "A
' "jwith y.The light travels along the curyey-= 4x4(yjind xdre
But R' = P'N' + N'M = 2Rtanr + Rtani ... (2) ' "fin metre) in the slab. Find out the refractive index of the slab
From eqns. (1) and (2), \at y ;=>l/Ztt\itithe'Slab.'\ , ^ \ a-
2secr- tani = 2tanr + tani' ' ' -'A.A'siab.of 5
air /variable
' "seer - tanr"= taili , r;efractive
.'index
tani = tan —- —I(using trigonometry)
.4 2J

2i = ^-L ... •A- ....(3)


2 2 ' • Fig. 1E.87 (a)
Also from Snell's law, '"'-a •
sini = V3sinr Solution: Let _R.1. at y = y is jt and corresponding
From eqns. (3) and,(4),'we get angle-of refraction'is 0: ;
. . ' 'f ' 1 '• - |i'sin0 =-lsm-3d® '' '...('1)-
sini = -;=sinr = - ... (4)
43 3 . and tanf—,-.0
• \2
],= —=> cot0
j ' dx- "•
=-8x
=> tani =-^, tanr =^=
42 2V2
GEOMETRICAL OPTICS 83

Sy V2 Solution; (a) The critical angle for material-air


cot0 =
interface

cot0 = .^11
smC = — - —
2
at y =1/2 => cot0 =-^ =2V2 C = 30°
V2 2J2,
Fig. 1E.87 (b) The rays are incident normally on
=> sin0 - ~
3
the spherical surface, so they pass
undeviated and then incident of plane
11
=> Ll X - = — face at an angle 45°. As^the angle of
3 2 • .
incidence is greater than critical angle
3 (30°), so rays get totally reflected.' '
^^=2 (b) For spherical surface :
Fig. 1E.89 (b)
' 88 u = 00 , - _

We have P-2
^-^1 = 1^2 -4^1 • • ' • -
'•A vessel, whose bottom isflat and perfectly reflecting, isfilled V ' u ' - R . ,
\with water (index= ^3)uptoaheight=' 40 cm.Apoint object or
2 1 _ 2-1'
\in air above is moving towards the water suiface wUh a V o" R •" -
\constant speed - 4m/s. Whatis the rdative speed of itsfinal V = 2R. , i . -
{image (in m/s), as seen by the object itself, at a moment when
Ithe object is 30 cm above the water surface ? Thus the image will form on diametrically opposite
point. ", ; , - - • •
Solutlbn: vq ^=-4m/s (c) Some of the rays get totally reflected and so they will
• ^I/M form the image at I2.
= 4/3x4 30cm

^i/M = 16/3
V,
40cm
^I/G - \(a) A ray of light suffers an internal reflection inside a water
t - \
'idrop. Firid the conditionfor.minimum: deviatioTt, the angle of
^I/G =
- +16/3 _ . Plane j incidence at minirnum d^dtion and the value of miniihum
.4/3 mirrorj/,' deviation. .''. " ..... l -
^i/o = 4-(-4) = 8m/s 'Flg.1E.88 (b) A source and a scfeen'are held fixed at a distance Ifrom
each other: A thin leris is placed between them ^ch that the
source is focused on the screen..For, 'what values of focal
.89 .ilength of the lens there are one, two or no positions for the
Mens? • • -w ^ \ ' i
iFig. 1E.89 (a) shows a transparent hemisphere' of radiusj- Solution: (a) .
•3.0 cm made of a material, ofrefractive index 2.0;
D(deviation) = (i - 0) + (tc - 20) + (i - 0) = ji -1- 2i - 46
•! " Air.' -X
^=2-4!^^'
di di

By Snell's law, |X sin0 = sin I
Differentiating w.r.t.'i'we get, .
3 cm •
. de . •
PCOS0 = COSl -c'/
Fig. 1E.89 (a) di

(a) A narrow beam of parallel rays is incident of the


hemisphere as shown infigure. Are the rays totally reflected at di ' [p. COS0
\plane surface ? • . dD'
When D is minimum — = 0 <"
\(b) Find the image formed by refraction at the first surface.' di
\(c) Find the imageformed by the reflectionor by refraction at
]the plane surface.
[84 OPTICS

Solution: in the triangle OAB


r R

sin02 sin(9O + 0i-02)


sin 02
r = ...(1)
cos(0i -82)
Also, from the figure
Tq = R sin0i ...(2)
From Snell's rule
Fig.1E.gO
sin0i = nsin02 -.(3)
Eliminating sin0i and sin02 from equations
[I COS0 (1),(2) and (3)

2 =
4cosi ^_ ^0/"
\L COS0 COS(01 -82)
\x COS0 = 2cosi , since,cosC0i -©2) = cos0i cos02 + sin0i sm02
ji•Vl - sin^0 = 2cosi / N2
and COS01 = -sin^ 02 ~-yl ~ Tn

[ sin^i
COS0 =Vl-sin202
=4- • - sin^ i = 4cos^i = 4[1'- sin^i]
= 4- Ssin^i

4-II'
sin I =

-1 4-[L-
I = sm

-1 4-]x' 1 4-^1 01^02


Dmin = Jt + 2sin - 4sin — : /9O+01-92. LSI/
.

91
^ Fig.1E.91 b)

A beam of light is incident verticallyon d glass hemisphere of . -rn n\\


r -
radius R lying with its plane side of table. The axis of the / \2
beam coincides with the vertical axis passing through th'e^ ii-i^ Iri-R 1
+-
centre of the base of the hemisphere, and the radius Tq of the' '"n R-
cross-section of the beam is smaller than R. Find the radius of
the luminous spot formed on the table. 92 k"

'Consider the situation sketched. An illuminated object lies


j40cm under the surface of water (n = 4/3), 80cm above a
concave spherical mirror whose radius of curvature is 120cm.
'An observer looking down from above in the air (n = 1.00)
""O
sees two images. Find distance (in meters) between these two
•images.

Fig. 1E.91 (a
85
GEOMETRICAL OPTICS

imnimmiiiimiiiimimmi ^

A medium of |j.=1.2
69cm
n = 1.00

n =4/3
40 cm^
•O
Water;
1.1=4/3 ; 90cm
80 cm

R = 120 cm

Fig. 1E.93 (a)


Fig. 1E.92(a)

Solution: Position of object seen directly Solution: Distance of first image from water surface
40
^90^^90x6/5^3^^^
— X1 = 30 cm below
4/3
It. 4/3
Distance of from mirror = 69+ 81 = 150 cm
Position of image seen in mirror there are two events
Reflection and refraction
=> Distance of final image from mirror = 150 cm

150cm
120 cm

90 cm
11=1.2
69cm

120 cm
81cm
\lzz li ±ZZZZZZ

1i 1 Lzzzzz't
1
1
Fig. 1E.92 (b) 1(
It
H=4/3
Mirror image is at v Fig. 1E.93 (b)
(-80) (-60) 80x60
V = = -240 cm
(-80) - (-60) -20 l e ' 94
Real image mirror is at'water surface 120 cm above
Thus for refraction at mirror image is formed at Aperson on a bridge spanning a pool looks over theside and
120. seesa small object vertically below. He moves along a distance
•= 90 cm
4/3 Xtill his line of vision makes an angle of 45° with the swface
of the water. If h is the height of eye level above the water
' Example ^surface, then find the depth d of the pool taking ii as the
refractive index of water.

Aparticle lies on thebottom ofa tank Tfilled withwaterupto


a height of90cm. The medium above thesurface ofwateris of A'

Ir.L =1.2 above which there is mirror M. Beyond the mirror M


.the region contains air (|X = 1). The distance of the image•
45
formed by the mirror after reflection ofthe rays comingfrom
P is :

Fig. 1E.g4 (a)


186
1 • • OPTICS
Solution: Using Snell's law Object is placed at u = -10 cm its image is virtual and
magnified . • ,

Finally after refraction


_ "o/s
^j/s =
M

or
=-C"o-"J

Uf-(1) =^(-20~1)
4
Fig.1E.94
63 ,
sm45 Vi +1
. 4

= —cm/s upwards
4

/> •- . cos6 =•- DISPERSION


For most transparent dielectrics the index of refraction
tan0 = —=^d = (x- h)tahi9 = (x- -1 gradually decreases as" the wavelength changes from shorter
•visible wavelengths (blue light) to the longer visible
wavelengths (red light). So white light, composed of a
•95; combination of wavelengths, when refracts through an
air-glass boundary, the blue light is bent more toward the
jWater level in the tank is decreasing at a constant rate o/j normal than the red light.' Aprism can be used to separate
ilcm/& Asmall metal sphere, is Amoving downwards with aj the wavelengths into a spectrum in which' the colours are
[constant velocity Scm/s Base of the tank is a concave mirror arranged according to wavelength. When an index iof
\of radius 40 cm. Find the velocity of the image seen refraction n decreases widi increasing wavelength, the
[Take =4/3] phenomenon is known as normal dispersion. . , -,
Prism Geometry
Angle of deviation 6 is the- angle between the
'' ' -
.incident ray and the emergent ray. -
^ ~ ^firstsurface "'"^secondsurfece
j-i^l.cm/s:
••• • =Ci-ri>+(e-r2) . .-
::5ctp/s^
niiHuWrm^ 'Deviation at ofPf'Sfii'or refracting angle
R=4bcm first surface
Fig.iaw •J. . Normalaf second surface
Directly ' ^ ' Angle of •g Angle ofemergence
{(b) After reflection at the mirror. Incidence g-"" ( Deviation at ;
' second surface)'
Solution: (a) Here =we calculate the velocity with
respect to surface and then convert it to ground frame. Normal first surface

Directly u^5 = Fig. 1.159

Fromfigure, ^ = ti + r2 ... (2)


•O f.v .v-r ,
Snell's law is applicable at both refracting surfaces.
Uj = 4 cm/s downwards lsini = nsinri ...(3)
/ -20 nsinri=lsine •••(4)
Cb) m = = 2
f-u -20-(-10) • Normal Incidence : In this case, i = 0, =0.
From geometry we can see that rg = A.
iGlOMmCAl OPTICS ®LJ
IThe ray of light .willemerge at the second surface only if • Prisms of small angle : Consider a small angle
r2 < C or A < C • • prism shown in Fig. 1.162.
The deviation,
where C is the critical angle for glass-air interface.
Deviation at the first surface,
5 = (i-ri) + (e-r2)
5i = 0 and A = rj 4- r2
If the refracting angle (A) is
Deviation at the second surface,
small, then i, e, and r2 are small,
52=e-r2 = e-A
and so
Net deviation, 5 = 5i+S2=e-A
i = nvi and e = nr2
5 = nr^ + nr2 - A
= n(ri+r2)-A
ACHr)e = uA-A '

= (n-l)A Fig. 1.162

Fig. 1.160 The deviation is therefore independent of the angle of


incidence and depends only on the refracting angle of the
In this position, deviation is,maximum., . - prism and the refractive index of light.
I •! From Snell's law, we have. . , • .. 1. Condition of no Emergence: A ray of light
. • ' sine = nsinA''' incident on a prism of angle A and refractive index \i will not
-'or' - " ' ' 'e = s'in~^(risinA) ' . ' emerge out of a prism- (whatever mayibe the'angle of'
incidence) if A > 20^, where 0,. is the critical angle.
• Grazing Incidence': In this case, i = 90°, =C
. - . - - 1 .
.C+-.r2-A, I.e., ' • p. >^ • . . . V.
sin(A/2)
. or • f ^2 = A -C .. • . /A
If the ray of light'is to' :
emerge- 'from the second i j^ggol
surface,
r2 < C.
or 1 A - C < C
or . A < 2C^ ,
If r2 = C, the ray: of light •] Fig. 1.161 ; Condition of noVmergencej
will emerge grazing from the - ^ I - Fig. 1.163>* 1
second surfac^j.pas^ referred to as grazing incidence and
grazing emergence, if A> 2C;vthe'ray \yill not emerge from 2. Conditibn df'Grazing'4
the second surface. Emergence: By the conditioii of |
Deviation at the first surface, \' grazing emergence we mean the
angle of incidence i at which the
'

angle of emergence becomes e = 90°i


Deviation-at the second surface e = 90°.
§2 = e-r2 = e-(A-C) i = sin ^ •yj\!L^ -1 sin A- cosA
Net deviation Grazing emerging
= (9q-7,C) + (e-A + C) ray in a prism
Flq. 1.164
= 90-A+ 6
From Snell's law, we have' Note: The light will ernerge out'of.a^ given'prisrh only if the;
sine = nsih(A-C) ' angle of incidence is greater than the condition ofj
grazing emergence." . J
or e = sin"^[n siriCA - C)]
88
OPTICS

3. Condition of Maximum (c) When 5 = the angle of minimum deviation,


Deviation: Maximum deviation theni = e andr^ = r2,the ray passes,symmetrically w.r.t. the
occurs when the angle of incidence refracting surfaces. We can show that = 21^^^ - A
is 90°. where = angle of incidence for minimum deviation, and
5max = 90°+e-A r = A/2
where 'A + 5„i
sin

e = sin sin(A-Gj.)]
Condition of "rel -
maximum derivation sin

Fig. 1.165
pnsm
4. Condition of where '^rel
Minimum Deviation: The ^surrounding
minimum deviation occurs when
Also = (n -1) A (for small values of Z A)
the angle of incidence is equal to
the angle of emergence, i.e., - Illustration 12: A prism with angle A = 60° produces
a minimum deviation of 30°. Find the refractive index of the
i = e material.
• 5min=2-A Solution: We know that
Using Snell's law Light ray passes through a 'A + 6.
sm
prism symmetrically in the
sin
^min condition of the minimum
2 deviation
l^ = sm

sin
A Fig. 1.166 u
2 Here A = 60°,8^„=30^
Note: In the condition of minimum deviation the light ray; 60°+30°'l
passes through the prism symmetrically, i.e., the light sin 45°
rayjn the prism becomes parallel to Its base. ! = V5
sin 30°

Characteristic of a Prisrn
(a) Variation of 5 versus i (shown.in diagram).
For each value of 5 there are two values of angle of
incidence except for angle of minimum deviation.
Find the co-ordinates ofimage ofthe point object ^O'formedi
Ifi and e are interchanged then we get the same value of •after reflection from concave mirror as shown inTig. iE.96^
5 because of reversibility.principle of light.' ,• fa) assummihg prishi to be thin and'smdll in size ofprism]
' "'h !angle_-2°. Refractive ind^ ofprism materialis 3/2: ' -J
f = 30cm I
max

i
' 0(0,0)

5cm 20cm

Fig. 1E.96 (a)

Fig. 1.167 Solution; consider image formation through prism.


(b) There is one and only one angle of incidence for All incident rays .will be deviated by
which the angle of deviation is minimum. 5 = (p-l)A = --l|2°=l°
2 J
=—rad
180
Now as prism is thin so object and image will be in same
plane as shown in Fig. 1E.96 (b).

r\
GEOMETRICAL OPTICS 89

2. If a triangular
prism is placed in a
medium for which its
1
.-"'U relative refractive index
is below unity, the ray
AOi [see Fig. 1.168(b)],
d
after passing through
'
' the prism, will be
0
deflected by the angle 6
not towards the base but

5cm
towards the apex of the
prism.
3. Calculation
Fig. 1E.96 (b) (b)
shows the critical angle Fig. 1.168: Path of monochromatic ray
for glass to be about 42°. through triangular prism made of material
It is clear - - tanS = 5 (.-.5is very small) of(a)greater optical density than medium
5 Therefore it is easy to and (b) lesser optica[density
obtain total reflection in
j
or d = — cm a triangular glass prism with two angles of 45° Fig. 1.169
36
(a) shows a change in direction of light rays of90°in such a
Now this image will act as an object for concave mirror, prism, while Fig. 1.169 (b) shows the reversal of an image
u = -25 cm, / = -30 cm by the same prism. '
V = —-— -= 150 cm
u-f

Also, m = — = +6
u

Distance of image from principal axis


% , n
= — X 6 = — cm
36 6
Hence, co-ordinates of image formed after reflection.
from concave mirror are 175cm, —cm
6
i>42°
The two faces of the prism through whichrays.enter and, ^ • '(c) • . ^ -
emerge are termed refraction faces,.the termfpr the angle (j) . Fig. 1.169: Path oflight rays in total reflecfon'prism" * ' ' '
between these faces being refraction angle of the prism (Fig;. \-
1.168). Fig. 1.169 (c) shows a direct-vision prism and the path
of rays in it. The upper and lower rays are seen to change
Important Points places, but they continue to propagate in the original
1. Let a narrow beam of light AOj of a specified colour direction.
fall from the air on the prism with a refraction index n. 4. There is one more point to be noted. Since the index
Inside the prismit'follows the path O1O2. Emerging from the of refraction n depends on the wavelength X, the deflection
prism it moves away from the normal to the face and follows of the rays in a prism also depends on their colour. For
the path O2B. Hence the effect of the prism is to deflect the instance, the angle of deflection5,for red rays is less than for
beam towards its base. Since the direction of the light before blue.
it enteredthe prism wasAOj and after it leftit became O2B,
5. Prisms have the property to change the direction of
the beam was deflected by ±e angle 6 [see Fig. 1.168 (a)],
light. Figs. 1.170 (a) to (e) present some of the useful
termed .the angle of deflection. This angle is the greater
applications.
refractive index of the material of the prism or its refraction
angle (j).
[90 OPTICS

Screen

Red
Orange
i Yellow
Green
Primary |
' spectrum 1
Blue
Indigo
Violet

(a) (b) (c)


Fig. 1.172

For a small angle prism,


mean deviation, 8^ - (n^ - 1)A
angular dispersion, D = 8„ - 8^
(d) (e)
= -1)A-Cnr -1)A
Fig. 1.170
= (n^-nJA
6. The right-angled prism in Fig. 1.170 (a) turns the Dispersive power of a medium is defined as the ratio
light through 90°. A pore prism shown in Fig. 1.170 (b) is a of angular dispersion and mean deviation.
right- angled prism, it turns light through 180°. Fig 1.170 (c) ^- — - —("u -"r)
, .sjiows two right-angled prisms, used, in binoculars. Figs. Dispersive power (co) —
.84 (fty-l)
^ l;17p .(d) and (e) illustrate image formation with and
without deviation;-in both the cases the image is laterally Dispersive power is a dimensionless number and
inverted. depends on the material of the prism.
Dispersion Combination of Prisms
According to Cauchys experimental relation, the Normally a prism deviates as well as disperses white
refractive index of a material has dependence on the light but with a combination of two prisms, deviation or
wavelength of light, and may be expressed as angular dispersion can be nullified.
For the combination of two prisms,
net mean deviation, 8^ = 81^ + ^2y
This is a polynomial , in - l/X^. . . =(n^-l)A-F(n;-l)Ar (1)
Normally only the first two terms are • net dispersive-power, D = Di. + D2'
taken. The constants A and B can be
'' ' • '= (nj-'n;)A +Cn;-'n;)A'- (2)
determined by plotting ,n (X) vs. l/ A.is
We have two cases :
the intercept oh the n(^) axis for l/X^ = 0
and B is the gradient. A graph of Violet Red' ' 1. Dispersion without
refractive index against wavelength is . Fig..l.l71 deviation : From.eqn. (l) .we.have.
shown in Figi 1.171. The refractive index' .• '1 • 5y. —0 ' *) . r
for violet light is greater than for red-light. The prism
deviates violet light the most and red the least. When white or A = A ... (3)
M'y-.l) , Fig. 1.173
light passes through a prism, due to different deviation of
component colours, white light disperses into its In this case net dispersive power, from eqn. (2), is
' components. This phenomenon is called dispersion of f (ny-l)
light. D = (Hy -n^)A + (n'^ -n[)i ^
(n;-l)
Dispersive Power
The average deviation is measured from the deviation of = (iny-l)A
• ' yellow colour, referred to as mean deviation (64). If 8^ Tly -1 (n'y-1)
and 5„ are the deviations of red and violet colours = Criy -l)ACco-(o')
respectively, the emerging spectrum is spread in an angle
5y -8^. This angle is called angular dispersion. = 5jy((O-O)0 ... (4)
[ GEOMETRICAL OPTICS
If 0' > 0, then D < 0, the dispersion is in opposite
direction to deviation from first prism. Such a combination
of prisms that produces dispersion without deviation is
called a direct vision prism.
2. Deviation without dispersion : From eqn. (2) Vn r2=C.^--V90''
we have D = 0

or A = -
... (5)
K -n;)
ny -n,
As 0 -
ny-1
From eqn. (3), A = 2C or C = A/2
, \
ny-n, . ^ 1 . A 1
and 0' = As sinC = —, sin— = —
n'y-1 n 2 n
A
-n, =0Cn^ -1) n = cosec—
Ftg. 1,174 2
and n'y - = (xy'in'y -1)
coCn^ -1)
Eqn. C5) reduced to A' = - (6)
0'(n^ -1)
i Arayoflight PQisincident at an angle i bhface AB ofa pHsni;
In this case net deviation is
ABC and it emerges grazingfaceAC. Ifn is therefractive indeo^
'5^ =Cn3, -l)A + (n;-1)A' •and A the angle of the prism, show that "I
= (n^-l)A + (n;-l) I^ fL {r smi
• • + cos A 1
(K-K) •n = U +
sin A
(n^ -1)
= <iny-l)A 1- A
(.Uy-lXn'^-n',)

Normally 0 >0'. so net deviation (5_y) is in the same rV90°

direction as deviation by first prism Such a


combination of,prisms; is calledan achromatic combination.

, , Fig, 1E.98
{•WTidt should^ be the minimum value ofre:frdctiveHhd6c'of a-
-pr^m; refracting angle A, so that there is no emergent ray' Soiutioni-From Snell's law atiface-AB,-
irrespective ofai^le of^ incidence ? , \ lsini = nsinri ' • ... (1)
sini sini
Solution: if the ray just emerges from face AC, n - [as A = ri +r2]
sinrj sin(A-r2)
e = 90° and r2 = C ... (1)
From Snell's law at face AB, we have
sini
...(2)
(sinA cosr2 - cosA sin r2)
lsini = nsinri ... (2)
and
From Snell's law at face AC,._ . .
A = Tj + r2
= Ti + C
nsinr2 = lsm90°
... (3)
From eqn. (2) n is minimum when is maximum, i.e., sm r-, = — I '4
...(3)
= C. In this case i = 90°.
. • 1
cosr2 = —(n^ -1)^^ ".:.(4)
n
92 OPTICS

Now we substitute eqns. (3) and (4) in eqn. (2)


sini
n -

sini4 - cosAOJn)

nsini
...(5)
(n^ - s i n A - cosA
which on solving yields
sini + cos A
...(6) Fig. 1E.100(a) , j
sinA

sini + cos A Solution: The given parameters are 5 = 30° and


n^-l =
sinA, A = 60°. Let us test whether the prism is in the position of
V2 minimum deviation.
^ (sini + cosA 1 . /30°+60°'\
or n = 1+ sin
sinA sin 45°
n =
. (60°\ sin 30°
99

A ray of light suffers minimum deviation through a prism of. =^x2 =V2
42
Refractive index -^2. What is the angle ofprism if the angle of
[incidence isdouble the angle ofjefraction within theprism ? As n = 42, the ray suffers minimum
deviation through the prism. Thus
Solution: in the case of minimum deviation,
r, = r2 = r = - = 30°
il = ^2) ~ ^2 ^ ^ 2
So ' 5 = 2i-A Inside the prism, the ray makes an Fig. 1E.100 (b)
and r = A/2 angle 60°with the faceAB, so it is parallel
According to problem, i = 2r = A to base.

So- •' 5„i„=2A-A = A


j 101
sin

Thus n -
- • '2 •
• /
'Consider a prism with refracting angle A and refractive index'
n. What is the critical angle of incidence for which the[
sin
emergent ray grazes out ?
.A A - A
2sm —cos—- /> V

.
1

.sinA 2 2.
. A . A ! ** / * \ '
sin — sm — ^
.2 2

A n '42 1
cos— = — =
2 2 2 42 Va

or A = 90°

*1^ 100 Fig. 1E.101

A ray of light undergoes a deviation of 30° when incident on Solution: The ray grazes face AC
jan equilateral prism of refractive index 42. What is the angle So, e = 90°, r2 =C (1)
isubtended^by the ray inside the prism with the base of the.
^prism ? and A = ri + r2 = ri + C
Also sinC = l/n .(2)
OPTICS 93

From Snell's law at face AB,


Isinij = nsinri (3)
sinii = nsin(A-C)
= n[sinAcosC -cosAsinC]
= n[sin AVl^-sin^ - cosAsinC
'1 - '2
- sin A - cosA
Fig. 1E.102 (b)
= sin~HVn^ -1 sin A- cos A]
Eqn. (5) gives
,ll02' 5 = 2i-A ... (8)
and A = 2'r ... (9)
A ray of light is incident at angle i on one face of a prism,
irefracting angle A Show that angle of deviation 6 is^ From eqn. (8), t =^ +
'5= sin'^(n sin rj)-f sin'^[n sin(A - rj -A ;
From eqn. (9),, r = A/2
Solution: in apqr, Sij^M +5^n
ri+r2+a = 180" (1) sini V 2
n =
In quadrilateral PQRA, smr
sm — ,
90°+a + 90"+A = 360° (2) .\2J.
A + a = 180° (3)
103

An isoscelesprism has one of the refracting surfaces silvered. A!


Toy of light is incident normally on the refracting face AB.j
'After two reflections the ray emergesfrom the base of the',
prism perpendicular to it. Find the angles of the prism. _l
Solution: The incident
ray passes without deviation
Fig. 1E.102 (a) from face AB. It suffers'
reflections at P and Q. From.
From eqns. (1) and (3). + r2 + a = A + a ' ; " Fig. IE.103, incident-ray and
or Ti +r2 = A ' . , ... (^) normal at Q are parallel;-
therefore ^ '
Angle of deviation,
a = 2Av.. (1) /
5 = (i-ri)+,(e-r2) ...(5) A"
Also ' - B
= (i + e)r^A -
2a + A=180° ' ...(2) r ^ Fig.1E.103
From Snell's law at faces AB and AC,
On solving eqns. (1) and (2), we get
Isini = rtsinri
A = 36°, a = 72°
i = sin"^(nsinri) -..(6)
n sin r2 = 1 sin e 104
. sine = nsin(A-ri) ...(7)
Now substituting eqns. (6) and (7) in (5), we have Two identical thin isosceles prisms angle A and refractive]
'index p.are placed with their bases touching each other. This\
5 = sin"^(nsinri)+ sin~^[nsin(A-ri)]-A
• The angle of deviation is minimum when the ray
l^stem ,can act as a crude converging lens. Draw a neatj
Idiagrdm showing the path of parallel incident rafs. ObtainI
travels symmetrically through the prism, i.e., \the focal length of the system. The height of incident is H: •1
u =io =i
r, = To = r
194 OPTICS

Solutioni The deviation produced by thin prism Angle a = 45°-30°=15^


The required angle = 2a
\k
~Y^s = 2 X15° = 30°

'

F.
iThe refracting angle of a prism is 90°. If y is the angle o/j
minimum deviation and [3 is the deviation of ray which enters
lat grazing incidence, prove that siny = sin^p andj
.cosy = p cosP,p is refractive index of material of the prism. ,
* f
Solution:
Ixsin 90°=psinyi
1+ sin^p = p' ....(1)
,5 = aL-lM IxsinP = pcosyi
For small angled prism
•' ' ' ' 6;-tan5'= y'
^=;cp-i)A
t.

or. / = , Fig. 1E.106


(p-DA

cosl - l+^sinf-
sin 45° LV2 [2) -V2
An isosceles prism of angle 120° has a refractive index V2.>
p = cos - + sin
'Two parallel monochromatic rays enter the prism parallel to\
'each other in air as shown in Fig. 1E.105. Find the angle,
ibetween the emerging rays. Squaring both sides
p,^ = 1-i-'siiiy ....(2)
SdllltioniThe angle of incidence on the inclined face From eqns. (1) and (2), ^ •
of the prism is 30°. If r is the angle of refraction, then by
Snell's law
sin^p = siny Proved
. , -

^30° From eqn. (2), . .

*
W
f - 30°^ '
'
cc ^•'
l-cos^"y =Cp^-iy^ =p'^^ +l-2p^^; ^
'
.'. cos^y = p^(2-p^) = p^[2- (l + sin^tp)],
120°A ' • = p^[l - sin^p] = p^.cos^p
2a
'Cosy = pcosp* . — • Proved

107
A/] d

.
jTTre refractive indices of the crown glass for violet and red;J
lUghts are 1.51 and 1.49 respectively and those of the flint'
Fig.1E.105 \glass are 1.77and 1.73respectively. Anarrow beam ofwhite\
llight is incident at a small angle of incident on the shown:
sin 30°
\combination of.thin prisms. . . , .^
1 'Find the value of a for which the mean deviation of the]
or sinr =
'fricident beam, is zero. Also calculate the magnitude -qfj
'dispersion-produced by the combination and state order ofi
r = 45° kolours in emerging light,from top to bottom. • t
GEOMETRICAL OPTICS

Flint glass

Fig. 1E.108

Solution: (a) From the geometry of the figure,


Crown glass 02 +03 = A ...(1)
Fig. 1E.107 (a) Maximum value of 03 is critical angle, when emergent
ray grazes surface AC. -' -
Solution: CG, p.t,=151; ^.^=1.49 From eqn. (1), ©2 is minimum.in this case. From Snell's
Hv =1.77 j P-R = 1.73 law,
- ' Flint glass sin0i = nsin02 ... (2)
I When 02 is minimum, 0^ is also minimum.
When the incident ray is at the minimum angle for
which the light ray emits the prism, the light emits along the
second face.

From Snell's law, sin0o = —'= —0.625


n 1.60
. . • , . . 03 =38.68° ' • . . ,+ + •, .
As from eqn'. (l), »
•• '02 =-60°-38.68°= 2l'.32°" ' '
from eqn. (2),
Crown glass , ^ 7 1•'> 'siu0i.-= n.sin02i= 1.60;sin,21.3,2°r^,0.58l'7**
^ Fig. 1E.107(b) •' '
Thus' -^ • ' eV ="35:6°''^ '• •
e = 0j -02 +03 = o' (b) From Snell's law at Q,
= Cp-y - 1)4-- l)a +(fXy - 1)2^^ 0" nsin03 = sin04 = sin'0i ...(3)
= (1.57)4- (1.57- l)a + (1.5 -1)2 = 6 ©1 = 84, given in the problem
a ='4°' - . Also 02 +03 = A = 60° ' ' ...(4)
5;a =5i-^52 +63 =6^ - Thus sinOj = risin(6O°-02)
- =(py -pR)4T(Py -PR)a + (pv -p,)2 = nsin60° cos02 -ncos60°sin02
= 0.02x4-0.04x4+0.02x2 = -0.04° From Snell's law at point P,
Magnitude is 0.04° but (-) ve sign .... sinOj = nsin02
Indicate spectrum inversed top colour is violet and sin0i
bottom is red. ' '
sin02 - ... (5)

sin 01
COS02 = = fl-
'Consider a prism of refracting angle A = 60°' refractive ind^,
in = li60 (a). What is the smallest angle of incidence 0fvr\ Hence sinOj = nsin60° 1-
sin 01
- cos60° sinOj ,
[which a ray can enter the leftface ofprism and exit t/ieWght-
face?' (b) What angle of incidence'^ is required for the,ray to\
jexit the prism with an identical angle 0/or its refraction'-? or (1 +cos60°)sinOj =sin60°-Jn^ -sin^Gj
l96 OPTICS}

On squaring both sides and solving for 0^, we get


^110
. n sin 60°
sin 02 = •
VCl + cos60°)^ + sm^60° jFig. IE.110 shows a triangular prism ofrefracting angle 90°.
1.60 sin 60° A ray of light incident atface ABat an angle 0 refracts at point
= 0.80 !Q with an angle of refraction 90°. (a) What is the refractive
-\/l + cos60°)^ +sin^ 60° \index of the prism in terms o/0 ?
Thus 01 = 53.1°

J109
'a prism ofangle 60° deviates a ray oflight through 40° for]
[two angles of incidence which differ by 10°. What is the]
irefractive index of the glass of theprism ? _ ;
Flg.1E.110
Solution: in Fig.
1E.109 shown, incident ray What is the maximum value that the refractive index can
PQ is deviated through 40°. \have ? What happens to the light at Q if the incident angle at
In accordance with principle •Q is (c) incased slightly and (df decreased slightly ?
of reversibility,', a ray
incident along SR wiU also Solution:- (a)-Let the ray be incident at an angle0i at
deviate through 40°. face AB. It refracts at an angle 02 and is incident at an angle
03 at faceAC.'Finaliy the raycomes out at an angle04 = 90°.
Hence, the two angles
of incidence for which the From figure, the nonhals at faces AB and AC make an
ray has same deviation are angle 90° with each other.
Fig. 1E.109
02 and 04. According to ^— Hence 83 = 9d°-02
condition of the problem,
sin03 =smC9d°-92) =cos02 =-\/l- sin^.02 ... (1)
02 -04 =ir ... (1)
From Shell's' law at face AC,
. Also 02 +04 =A + 5 = 100° ... (2)
On solving eqns. (1) and (2), we get • nsin03 -1

02 = 55°30' and 84 = 44°30' nyl-sin^02 -1 (2)


sin0T
sin ft. sin
sir 04
From Snell's law at face AB,
As n = = — ... (3)
sin02 sin03 IsinOj = nsin62
where 03 = 6O°-02 ... (4) ... - sin0i
sin 02 = (3)
sin55°30' sin44°30' n
hence n -
sin 02 sin(6O°-02) Froih'eqhs. (2) and,'(3),"we have
0.8241 0.7009
Sinn Vs .
2
1 .

cosn —Sinn
^2
-fW" ....(4)

On squaring eqn. (4) and'solving for n, we get


Dividing both sides in denominator by sinrj, we get
0.7009
n =^l +sin^0i
0.8241 =
V3I 1 sin n (b) The greatest possible value of sin^ 0i is 1, hence the
COSTi
2- 2 sin n greatest possible value of n is

—(VS'cot n -1) -
0.7009 "max = V2 = 1.41
2a.. . 0.8241 (c) For a given n, if 0i is increased, the angle of
, cotri=l'.559
refraction 02increases. - As' 83 = 9O°-02, the angle 83
decreases, i.e., the angle of incidence at face AC is less than
Ti = 3'2°40' the critical angle for" total reflection; hence light emerges
sin 55°30' 0.8241 into air.
Thus n - = 1.526
sin 32°40' 0.5400
(d) If the angle of incidence is decreased, the angle of
refraction 02 decreases. So'the angle 83 increases. The angle
feGEOfflETRICAl OPTICS _ "" 97

of incidence,at the second surface is.greater than the critical sin(60 + D^)/2 = 0.5619
angle; so light is;reflected at Q.
^ ^ =34.2
D„ = 8.4°
jThere is a luminous point at the base of an equilateral prism
lof refractive index^li = 1.5, Show that there is a direction'
Iwhich demarcates a brighter region from a darker region.
'Find this direction. • A parallel beam of lightfalls normally on thefirst face of a
prism of small angle a At the second face it is partly
transmitted and partly reflected,-the reflected beam striking
the first face^ again and emerging from it in a direction
making an angle of 6°30' with the reversed direction of the
incident beam. The refracted beam is found to have
iundergone a deviation of 1°15' from the original direction.
Calculate the refractive indexof the glassand the angle of the
Iprism.
(b)
Fig. 1E.111

Solution: The rays diverging from S fall equally on


faces-AC and BC. Those rays which emerge into air produce
uriifofm illumination [see Fig. lE.lll(a)]. The angles of
incidence of these rays lie from 0 to C. Those rays which are 5i = 1®15
incident at angles greater than C are totally reflected" from 52 = 6°30'
SC.toAC and vice versa. Those rays emerge into air and they
produce • greater illumination on one side and less
iiliimination oh'the other [Fig. IE.111(b)]. Let the direction Fig. 1E;i13
make angle i with the normal. Then
sini =)r-sin(A-C) Solution: We must first solve for the refractive
index, n, of the prism glass. Applying Snell's law to the
=> sini = p,(sin A cosC —cosA sinC) refraction at point C (see Fig. •1E.113}, we obtain
sinA |1--?- - cosA — nsin([)i = n'sin(|)2
= 11
11^ It But n' is-the refractive iiidex of air, which is 1. Hence,
smv= sinA - cosA nsin(|)i = sin(|)2 ••• (1)
Similarly, applying Snell's law to the refraction at point
= VirS^ - l.sih60° - cos 60° B,
sirii = 1.118X 0;8660- 0:5 = 0.9682-0.5 = 0.4682 nsin\|/= sin52 ..(2)
i = 28° Now, we must relate (|)i, (1)2 and \j/ to known quantities.
Note that ZHCP = ZDCE, since they are vertical angles.
Therefore,
ZHCP = ZDCE
A prism with refracting angle 60"provides the least deviatibn'
^ahgle 37" in'air.:Findtihe least deviation angle of that prism] <l>2 ~ *t'l
i^in water (ji ='4/3) I or 5i = 92 - 9i •" (3)
Noting that DC and BS are parallel,
Solution: We have
siri(60+37)/2 sin 48.5 0.7489 ¥ = 291 ...(4)
= 1.498
sin 60/2 sin 30 0.5000 We need one more equation relating any of Oi,02 and
Vtoa.
1 1.498
Now X X, = 1.1238 ZACD= 90°-a
all
But ZACD+9i=90°
1.1238 =
sin 30
Hence 90°-a + 9i = 90°
and 9i=oc ....(5)
[98
optics]
If all the angles ((i)i,(t>2.M'>6i,52) are small, we may
approximate the sine of an angle by the angle itself. Usinz
eqns. (1) and (2),
n(j)i ^ ^2 The easiest method of measuring the refracting angle of a
n\\f^^2
prism is to direct a parallel beam of light on to the angle
(6) . (vertex Ain figure) and measure the angular separation of
Taking the ratio of each equation in (6),
the beams reflectedfrom the two sides of the prism containing
the refracting angle. Show that this angular separation is
V 82 twice the angle of the prism. •
0
Hence,
82 . Solution: consider
ca three incoming rays, all parallel
II

and striking the prism at


1

Using (4) A
points A, B and C. Erect
normals to AB at A and B
or 82 = 2^2 '' . (7) and to AC at A and C.
Hence, solving (3) for.(t)i Designate the angles as
^1 - '1'2 ""8j in the diagram.
Since The rays striking at B
.. .... and C are reflected
according to the laws of
(8)
optics, as shown, and the
Solving the first equation of (6) for n angle between the
reflected rays is p. Bis the Fig.lE.ll4
point at which the - .
With eqns. (7) and (8), normals at B and C meet.
5o The sum of the angles of quadrilateral ABEC is 360®.
Since ZABE andZACE are each 90°, '
1-^ a + Y = 180° - CD
2 ^ • §2 In the quadrilateralBDCE,
Using the given data,
P + Y+ 01 +02 = 360° - (2)
n = - Since two oftheangles surrounding Aare right angles.
2(1^150
1-
(6°30')
a+ 01 +02 = 180® - (3)
1 Adding eqns. (1) and (3),
n =
1® 2a + Y+ 0i + 02 = 360® ... C4)
2(.lU
1- 4 Subtracting eqn. (2) from (4),

C6L)
or 2a = p
1
n =
^ 1- 5/13
Thus, the angle between the reflected beams, p, is twice
the refracting angle of the prism a.
13/2
13
n = = — = 1.625
13-5
Furthermore, from eqn. (5), The path ofa ray oflightpassing through an equilateral glass
a = (f>i
prism ABC is shown in Fig. IE. 115. The ray oflight is incident
But, equation (8) tells us that
on face BC at the critical angle for just total internal
reflection. The total angle of deviation after the refraction at
a=(])i =%-5j = 6°30'
-1°15'
face AC is 108°. Calculate the refractive index of the glass.
2 ^ 2
Hence a = 3°15'-l°15' = 2®
GEOMETRICAL OPTICS 99

+ 8.
sm

=
. A
sm —
2
. A +8.
sin :

^3 =
. A
sm —
2
A +81 . A + 8-:
,, ,,, sm ^ + sin :
Fig. 1E.115 1^1 +M-3 _ 2 2
1^2 . A + 8.
sm ;
Solution: in the figure shown,
r2=C 2o sm
• 2A + 82+83 cos — 83
^
Tj = 60 - r2 = 60 - C
A+8.
rg = 60 - r2 = 60 - C sin

^1=^3='" (say)
Since 81,82 and 83 are in A.E
ii =12 (say) ^
Net deviation, 5 = (i - r) + (180- 2r2) + (i -r) = 108' 282=81+83
or r2 + r - i = 36°
2 sm
A + 82 cos ~51-83
or C + 60-C-i = 36°
1^1+^3
or i = 24°, sin 24°= 0.40
1^2 • ^ •'"So
sm ~
From Snell's law, we have
sin24° = |i sinr 81-8:
= 2 cos
or 0.4 = ^sin(60°-C)

or 0.4 = ^l
V3 cosC^

r Multiplying and dividing by sin^^ ,we get
2 2|.i 4

,, ,,, O • 51+83^cos^^
2sm^^ 51-83i sin^
.81 + sin-i
.80
or 0.8 = 1-^-1 Ei±1£3_ 4 4__ 2 2
[^2 . 81 +83 . 8.
sm-!^ ~ sm-
or -1 = 1.08

or II = 1.447
117

J116 |7n an isosceles prism ofangle 45", it is found that when the
wangle of incidence is same as the prism angle, the emergent
The refractive index of the material of a prism has values Iray grazes the emergent sujface. Find the refractive index of,
M.i,|i2 and 1^3 respectively for light of three different \the material of the prism. For what angle of incidence, thel
wavelengths. If 82^,62 andB^O-re the corresponding angles of tangle of deviation will be minimum ?
minimum deviation for a given prism and these angles are in
arithmetical progression, prove that .A=45° " i
. Sj .83
.. sin-i- + sin~
M-i +t^3 _ 2' 2
\^2 sm

Solution: For minimum deviation,


4- 82 Fig. 1E.117
sm-

1^1 =
A Solution: As the ray of light grazes the second
sm
surface, r2 is the critical angle.
100 OPTICS

and for refraction at EG,


I.e., sin To -

nol 90-&- = 1x7,


r2 = (45='-ri)
and
__sm45° 1- Here, ) =9O-£-0i
sinri = ^ 2
: f BA
f 90-^
or r = n2 - Hi 90 T- (1)
sinr2 = sin(45°-ri) 2, l 2)
=-L[cosri - sinrj] For a biprism of angle 6 kept in air, similarly.
42
r =no|90-£l-l 90- C2)
"1 = 1-
\i 42 2\i^ 4^ From eqns. (1) and (2),
g^|3(n2-ni) +i8oo -1

or 1 = J_ V2n'-1' • 1'' "2 -1 r o —1


42 4^' ' 4^
l=©J119
or , 2=V4l^-1-1
or 2^1^-1 = 9 )A thin biprism (see Fig. 1E.119) of obtuse angle a = 178° isi
'placed at a distance I = 20 cmfrom a slit.\How many images:
2\i^ =10 \are formed and what is the separation between them ?]
= 5 or\i = 45 \Refractive index of the material\i = 1.6. ";
At minimum deviation, T • • •
45^
r, = To = = 22.5'=

^ sinii = (Vs) sin(22.5° )


Sinn
Fig. 1E.119
I'l = 58.8=
Solution: Two images are formed by the two thin
118
prisms-one above the axis and the othei below the axis by
the same distance. The refracting angle of each thin prism
A biprism with an angle p that appreciably dijfersfrom 180°\
isplaced into a vessel filled with a liquid having a refractive' =^ i(jt - a) where a is the obtuse angle in radian.
2 2
index or serves as one of the waUs of this vessel (see Fig.'
'lE.118). Calculate the anglebofan equivalent biprism in air., Then 6 (deviation of a ray) = Qi -1)
\The refractive index of the prism substance is n2- j
d
- = 18
2
90°-B/2>.
or d=21(u -1)i (tc - a
2

or d = (p -1) 1(tt - a)

Here d=(1.6-l)x0.2o|^ji -178x-^l


180j
= 0.6 X 0.20x71 X — = 0.004m = 4mm
Fig. 1E.118 90

Solution: Consider a ray falling normally on AB, for 120


refraction at CD and RG.
For refraction at DC, U glass prism whose cross-section is an isosceles triangle,
P =n2^i stands with its (horizontal) base in water, the angles that its I
90-^l [two equal sides make with the base are each 0.
GEOMETRICAL OPTICS 101

M C

V
N\
Water A
Fig.1E.120 (a)
Fig.1E.121
lAn incident ray of light, above and parallel to the water
\sujface and perpendicular to the prism's axis, is internally Solution: According to the initial condition, the
reflected at the glass-water interface and subsequently incident beam and the, beam that has passed through the
re-emerges into the air. Taking the refractive indices of glass
3 4'' ' '
prism are mutually perpendicular. Therefore, Z<^ = Za and
and water to be—and respectively,, show that 0 must be atj also Zy = Z^ (Fig. 1E.121). The,sum of the angles of the
2 3 I
quadrangle AKMN is 360°. Therefore, ZKMN = 90° and
least 25.9°. J beam KM is incident on to face BC at an angle of 45°. If we
Solution: At the point where it enters the prism, know the angles of triangle KBM, it is easy to find that
sin a
angle'of refraction is (j). p = 30°. In conformity with the law of refraction, - n

Snell's law applied to the initial entry into the glass gives sinp
/* \ Hence, sina = 0.5n and arcsin0.5n
= n„ sin ^ Since full internal reflection at an angle of 45° is
observed only when n >V2, the angle a is within
45°<a<90°.

Three fight-angledprisms of refractive indices


Water
are joined together so that thefaces of the middleprism are in'
contact each with one of the outside prisms. If the ray passes'.
Fig.1E.120 (b)
through the composite block undeviated, show that >
From geometry we determine the angle between the ILi +1x3-ill =1
incident ray and the normal to the glass-water interface at
the point where the ray meets the boundary to be 0 + (|).
For total internal reflection to occur this must exceed the
critical angle C= sin"^(n^^/n^). These two conditions can
be . combined using the formula sin(0 + <|)) =sin0cos^ Fig. 1E.122
-i-cos0sin ^ to eliminate ({) and obtain
n„ - n. > cos^0(n^ -i-l-2n^) Solution: We have

Substitution of the given values for the refractive indices r -t- r2 = 90' .. (1)
yields the stated result. ^3+^4= 90 .. (2)
and r5+r6 = 90 ..(3)
Since the emergent ray is parallel to incident ray. we

have
Thefaces ofprism ABCD made of glass with a refraction index,
n form dihedral angles: ZA = 90°, ZB = 75° ZC = 135° r7=90°-i ...(4)
and ZD = 60° (the Abbe prism). A beam of lightfalls on face Writing the Snell's equations of refraction,
AB and after complete internal reflectionfrom face BCescapes\ sini = p-i sinrj
through face AD. Find the angle of incidence a of the beam\
|ii sinr2 = 1I2 sinrg
onto face AB ifa beam that has passed through the prism is\
perpendicular to the incident beam. • [i2sinr4 =iJ.3smr5
and p 3 sin = sin r^
Using equation (1) to (4) in the above equations, we get
sini = Pi sin r -..(5)
!102 OPTICS

•. lO-i cosri = ^2 sinrs ...(6) til Refractive index of medium in which the object
tiacosrg =^3sinr5 ...(7) lies

^3Cosr5=cosi ..(8) n2 —> Refractive index of mediu n through which


(equation 5)^ + (equation?)^ - (equation 6)^ - (equation 8)^ refraction takes place
gives sin^i-+^| cos^ -jif cos^ Tj -|j.| cos^Tg N Normal to refracting surface, a radial line through
centre of curvature.

=!ii sin^Ti +|i3sin^r5 -}i|sin^r3 -cos^i The shaded area shows a medium with higher refractive
index. Due to refraction the ray bends toward the normal if
or it is entering in a medium of higher refractive index and
or
away from normal if it is entering in a medium with lower
= 1
refractive index. When refracted ray bends toward principal
SINGLE SURFACE REFRACTION EQUATION axis, it forms a real image on that axis. I
Consider a thin transparent spherical shell filled with • When more than one refracting surface is present the
water, a solid crystal ball. Refraction through one side of this subsequent surfaces are taken in ^the order in which
ball is an example of single surface refraction. The laws of they are encountered by the ligh|t. The positions of
refraction are "applicable when-rays' pass' through the images from successive refractive surfaces are
interface. ' - obtained by using single surface : efraction equation
at each surface.
The single surface refraction equation is
• While considering refraction or re lection of the light
^1 _ n-j - n i
from an object by two or more sur "aces in succession,
V u, R the image formed by first surface becomes object for
t

In Figs. 1.175 (a) to (f) the symbols have the following the second surface.
meaning : • Be careful while substituting for object and image
R -> Radius of curvature distance in the equation. For every refraction the
C —> Centre of curvature
origin of the cartesian system is i 1Iways taken at the
particular refracting surface under consideration.
0 ^ Position of object Distances are calctilated from the origin of
1 Position of image the particular surface under consideration
and not from any previous surface. The origin
is shifted after each refraction:' so numerical
I
values of
Real Real distances (and their, signs) i^ust be adjusted
according to new origin. If it is directed away from
the. principal axis, it cannot fo[m a real image;
how;ever, backward extensions of it and other
refracted'rays can form a virtual image. Figs, (a) and
(b) show real images and (c), (d), (e) and (O .show
virtual images. I -
• The single surface refraction equation is valid for
Virtual convex or concave and even aplan^e surface (in which
case R = co). We use cartesian sigr convention for all
Virtual
the distances in the equation.
• When the image is located to the right of the origin,
the image is real since the refracted rays really exist in
this region and ^converge to the image point. If the
•image is to the left" of the ori^n (the refracting
surface), the image is virtual, "^e refracted rays
never intersect at a virtual image point; they appear
to intersect there. A positive image distance indicates
Virtual, Virtual
a real image and a negative imagi distance means a
virtual image. The magnification for single surface
refraction is
_ 'n-iV
4
m -
Fig. 1.175
.GEOMETRICAL OPTICS 103

n The overall •magnification of the entire, system of ; .(c) When the object is very far from surface, u = oo; so
surfaces is the product of the magnifications we have
produced by individual surfaces. (3/2) (4/3)_0/2-4/3) '
r -+ . V . (oo) +5 •
V - +45 cm • • '

U very long glass rodof2 cm diameter has one end ground to The image is real and formed at second focus of
la convex spherical surface of radius 5 cm; its refractive index refracting surface.
\is 1.5. A point object is on the axis of the rod. The glass rod.
jand object are immersed in water, refractive index 4/3.'
\Characterize the image in each of the following cases: (a)'
\object is on the axis ofthe rod 20 cm from the surface; (b)^ A ring of radius 1 cm isplaced 1 m infront of a spherical glassi
Object is5 cmfrom thesurface, (c) Object isveryfar from the\ lbdiiof radius'25'cni with refractive index 1.50. Determine the]
.surface. Draw g ray diagram for each case. ' '^sitioriofthefinal image ofthe.ring anditsm(^mfication.^i
Solution: (a) From single surface refraction equation, Solution:. Light, rays from object are refracted
3 (4/3) _ (3/2-4/3) through the glass, ball twice; once at.surface Sj, from air to
2v (-20) (+5) glass, and second) at surface S2 from glass to air. We use
paraxial approximation, so that single surface refraction
where ni=4/3,. n2 = 3/2, 'r = +5, u = -20cm, equation- can be used.'
V = -45 cm
First Refraction Second Refraction
.Magnification, m=^ =C4/3)(-45) ^ air to glass glass to air
n2U (3/2) (-20) A .

The final image is formed 45 cm to the left of convex Origin for Origin for
refracting surface,' virtual, erect and ma^ified," see Fig. refraction refraction

1E.123. • •• , from Si from $2

(b).From single surface refraction equation,


M-u = -100 cm*
' ; 3 ' (4/3)_(3/2-4/3)"' . image
•2u . (-5) " ..-(+5)..,; • ' " Air

- V = -6.42cm
Magnification, 150 cm

• Jn^iv _ (4/3) (-6.42)


n2U (3/2)(-5) Fig. 1E.124

^ = +1.141 • ^ .
Refractlbh at First Surface
The image is virtual,' erect and magnified. Ui = 1, =1.5,1/ = -100cm, R = +25 cm
The radius of curvature is positive because centre of
curvature is to the right.
Substituting these values in single surface refraction
equation,
1.5 1 _ (1.5-1)
V (-100) ~ 25
On solving for u, we get
v= +150cm

' The image is located 150 cm to the right of the first


refracting surface. The magnification due to refraction at
first surface,
njv 1(150) -
mi = = -l
n,// 1.5(~100)
Fig. 1E.123
tl04 ^PTICS ]
Refraction M Second Surface- . The second image acts as:object for refraction-at curved
For refraction at. second surface the origin of the surface S^, with = 1.5, n2 =1. .. ._
cartesian coordinate system has to be shifted to the vertex of U3 =-r20- —
the second refracting surface. L 3.
The object distance for refraction at S2 is 46 _ 1r. •
cm,R = -10 cm
u' =+(150-50) = 100 cm 3

This is the virtual object for S2, the light rays converging 1.5 1-1.5 ^
to I^aie refracted at S2 before they can actually converge to (-46/3) (-10)
form the image. U3 = -20.9cm
(b) For the object at 20 chi
1.5 1 (1.5-1)
A transparent glass sphere of radius R = 10. cm and refractivei (-20) 10

lindex n = 1.5 has its one half silvered so that it acts like a'
M = ~—L = o
'Concave mirror. Find the position of the final image seen by\ 20 20
one eye to the leftof the objectand the ballfor an objectat (a)!
tij = 00
30 cmand (b) 20 cmjo the leftofthefront surface ofthe.ball. \
The rays are parallel to principal axis after refraction,
Solution: The light ray is refracted at surface The parallel rays are brought to the focus of the curved-
then reflected at S2 and finally refracted out through mirror. Hence the second image is at the focus, at
From single surface refraction equation, ' f = R/2 = 5 cm from the vertex of the mirror. The final-
Normal Reflection at mirror
image is obtained after refraction from 5^
Toward
normal
1 1.5 _1-L5
Vj +(20-5) ~ +10
.. Normal
1)3= 20 cm
The final image is at the vertex of the mirrored surface^
Final First On substituting the values into the single surface
image refraction refraction equation.
S21 Imageof reflection U'
<•2 n. Ho — Hi

%/ Away from V u R
/ normal 1.00 1.50 1.00-1.50
Normal
V 100 -25
Fig. 1E.125 Note the values of and n2.
Hi . Uo - Hi
Solving for u, we get u'= 29 cm
V u R The image is located 29 cm to the right of the second
refracting surface.The.final image is real because light after
(a) For the object at 30 cm
1.5 1 _ (1.5-1) refraction at S2. actually reaches this point. The
magnification, m2 at S2 is
(-30) ~ +10
n^v' (1.50) (29)
Uj = 90cm rrio — = 0.44
(1.00) (100)
The object distance for concave mirrored surface of-
sphere, Total magnification.
U2 = 90-20 = 70cm m total = mim-

/ = -10/2 = -5cm = (-1.00) (0.44)


„ . .111
From mirror equation, 1- = -0.44
^2 / The minus sign indicates that the final image is inverted.
J___l
Image size = m^^^^ (object size)
70 ~ 5
= -0.44(20)
.14
Vo = cm
= -0.88
^ 3
The final image is located below the optic axis.
1 GEOMETRiai OPTICS 105

•m Now substituting A and A from eqn."(4), we have


126
A+A =i
[For refraction at a spherical swface, the firstfocal length isi V u

idefined as the value ofu corresponding tov = '>=as shown in\


'Fig. lE.126(a). The second focal length /2 is defined as the\
Example
\value of Vwhen u = as shown in Fig. lE.126(b). ;
\A thin lens with refractive index n and tadius ofcurvature Rj
and R2 is located at the interface between two media with]
indicesof refraction n^ and n2 as shown in thefigure below, j

Fig. 1E.126

(a) Show that

(b) Show that the general relation between objectand imagel Fig. 1E.127
distance is ;
(a) If u and v are the object and image distances respectively]
A+&=i, " . I \and A and A the respective focal lengths, show that .
U V
" A,,,A._
Solution: We will apply single surface refraction
equation "for determining and /2. Find A £^<1A ih terms of ni,n2^ R^ and R2. ___
n2
(1) Solution: We will apply the general thin lens
T" R equation to obtain A and A-
n-t n-> - Ui
n - U] Ho - n
R
+ ... (1)
A Ri
v-l R. ^
1\2

n2R
A - (2) The focal lengths are obtained by substituting u = <» (for
rio Hi
A) and u = 00 (for A).
n-, - Hi n - rii no - n
and = + • ..(2)
R Ri R.
A
n^R
/i=- ... (3) fi=- ...(3)
Jlo Hi n - rij no - n
+
From equations (2) and (3), we have Rn R.

A _ R A R I2 n —Uj n2 —n
and ... (4) and ..(4)
"2 ~ rio rii A Ri R'

Il+A =o ... (5) A - ..(5)


"2 n-n-i no - n
+

A Ri R2
or ..(6)
A '•2- From eqn. (3),
• As Hi and n2 are alwa5rs positive and unequal, /j and A nju
(6)
A will have opposite sign and also unequal. The two u n - U] "2 - n
focal points lie on the opposite side of refracting R. Ro
surface.
From eqn. (5),
Further, from eqn. (1), we have
HiR A n2lv
= 1
(7)
V n - Uj n2 - n
v(n2-ni) u(n2-ni)
Ri Ro
RW
OPTICS I
Adding eqns. (6) and (7), ' — —.>11.^ .1,j

The image will be real if^2 is positive, i.e.,


A +A = C"2/^)-CnT/u) ••(8)
n-H]
C4n - 9) ,
"2 - n (4)
+
i?o
ClOn- 9)(n-2) ^
The equation (4) is satisfied if : ,
From eqn. CU right side of eqn.(8) becomes unity, so we
have Case (i) (4n _ 9) > 0 => n > 9/4
-and a0n-9)Cn-2)>0 0.9 < n < 2
A +A =1 There is no common solution.
Case (ii) 4n - 9<0 => n <9/4
128 L (lOn - 9) Cn - 2) < 0 n > 2, n < 0.9
;-Figiire shows a glass slab of refractive index n, with Wo Hence the result is 2 < n < 2.25
^curved ends ABC and DEF as shown in Fig. IE.128. Locate the
•final mage ofan object point Blocated atadistance 2Rfrom L ^ :i29
'^he vertex ofsurfaceABC. What is the condition to be satisfied A^ansparentsphere ofradius 20 cm and refractive index 1.6
ji/ the image is to be real ?
ISfixed ma hole of the partition separating the two media-A
' r .
Origin for first Origin for refraction
atsecond surface (refractive index n, =1.2) and B(refractive index n, =17)
refraction A Aluminous point object is placed 120 cmfrom the surface of
the sphere in medium A. Itis viewedfrom Din medium Bin a^
Ei/VR/2 direction normal to the sphere. Find the position and nature
ofthe image formed by the rays in the direction OD

03 = 1.7

Fig. 1E.128

Solution; For refraction at curved surface 5,


n_ 1 ^(n-1) 120 cm cm»:
^1 C-2R) R
n 2n-3
2R ...(1) Fig. 1E.129(a)
^1
The first image acts as object for refraction at second Solution; We will have to consider two refractions:
surtace S^. The origin of our cartesian coordinate system is
now at vertex of surface S^. Object distance for second first at surface 5;^ and the second at surface S2.
refraction, For refraction at surface :
"2 =-C3R-Uj) _ (1.6-1.2)
C-120) 20
2ni?.
3R-
(2n~3) Uj = +160cm

'4n~9'
2n~3
R the right oTA
For refraction at curved surface S2, at surface S^: The first image is
Object for refraction at second surface.
A_ " _ _ (l-n)
V2 C4n - 9) ...(2) "2 = +(160 - 40) cm
Ci?/2)
R 1-6 _ 1.6-1.7
:C2n-3)
V2 +(160-40) C-20)
On solving the above expression for V2, we get
C4n - 9) V2 - +204cm
Uo
-'2 =
...(3)
(10n-9)(n-2)
GEOIViETRICAL OPTICS 107 1

ni = 1.2 0, = 1.7
For refraction at third surface S3:
Norma
Toward 128
Normal
Uo = -i — + 8 I = -
7

Normals 15- 1 (1.5-1)


ji^Toward
""vNormal ^ (-128/7) (-9)
U3 = -16.5 cm
Norma For refiaction at fourth surface S4 :
U4 ^-(16.5 +9) =-25.5 cm
1 3/2 (1 - 3/2)
Normal = 0
(-25.5)

V4 =-17cm
Fig. 1E.129 (b)
The final image lies at 17 cm to the left of surface S4.
The final image is formed at 204 cm from the vertex and
on the right side.
i iE>:am,p.l.e

130 A glass sphere, refractive index 1.5 and radius 10 cm, has a.
spherical cavity of radius 5 cm concentric with it. A narrow]
A cubical block of glass, refractive index 1.5, has a spherical beam of parallel light is directed into the sphere. Locate the^
cavity of radius r = 9cm inside it as shown in Fig. 1E.130. A final image.
[Luminous point object O is at a distance of 18 cmjrom the Solution: We will have
cube (seefigure). What is the apparent position ofOas seen single surface refractions
Hi = 1 Hi = 1
<jrom A.
successively at the four surfaces
and S4. Do not forget
1 to shift origin to vertex of
ni = 1 n2= 1.5 ni = 1
respective surface.
Refraction at first
^ ni =1^ surface Sj : Light travels from
air to glass.
S3 84

0 1.5 1_ (1.5-1) Fig. 1E.131

Uj 00 (+10)
Uj =30cm
First image is object for the refraction at second surface.
•18 cm- •9 cm -H" 18 cm' e-9 cm *1
Refraction at second surface Sj: Light travels
Fig. 1E.130
from glass to air.
1 1.5 „ 1-1.5
Solution: We have to consider four refractions at V2 (+25) ~ (+5)
-54 respectively. At each refraction we will 1*2 = -25cm
apply single surface refraction equation. Refraction at third surface S3: Light travels from
For refraction at first surface : air to glass.
^ 1 1.5 1 _ (1.5-1)
= 0, Vi = -27cm
1^1 (-18) U3 (-35) (-5)
First image lies to the left of S^. V2 = —35/3 cm
For refiraction at second surface S 2* Refraction at fourth surface S4; Light travels from
1 . 3/2 (1-3/2) 72 glass to air.
=
cm
- (27 + 9) +9 7 1 1.5 1-1.5
~ -25cm
Note that origin of cartesian coordinate system lies at -(35/3 + 5) -10
vertex of surface S2. The object distance is (27 + 9) cm. The The final image is virtual formed at 25 cm to the left of
second image lies to left of S2. the vertex of surface S 4-
[108 OPTICS

E-S^^mple. 132 133!

A biconvex lens separates two media of refractive indices 1.3- A glass sphere of radius 2R, refractive index n has a spherical'
and 1.7. Therefractive index of the lens is 1.5 and the radii of cavity of radius R^ concentricwith it. A black spot on the inner
,curvatures of the two sides of the lens are rj = 10 cm and^ surface of the hollow sphere is viewedfrom the left as well as\
r2 - 60 cm. The medium of refractive index 1.3 extends to 78, right. Obtain the shift in position of the object. [
cmfrom the lens and that of refractive index extends to 34 cm
from the lens. A luminous object 0 is at a distance of 144 cm
from the lens. Find the position offinal imagefrom the lens. Origin for first
refraction

n, = 1.1 n2 =1.3 -I I n4 =1.7 n5=1.1


M= 1.5 Origin for second
refraction
0

C2 I 1 C,
Fig. 1E.133
K 66 cm * * 78 cm 34 cm —

Fig. 1E.132 (a)


Solutioni (i) viewer on the left of hollow
sphere : Single refraction takes place at surface S. From
Solutioni Once again we the single surface refraction equation we have
"3
have to apply single surface 1 n _ (1 - n)
refraction equation to the four (-2R)
surfaces 5^,52, S3 and S4. But we ^2 Ci which on solving for v yields
have learned the general thin lens
equation V V = -
2R

rio rii rio - n- Fig. 1E.132(b) n + lj


+
R. Image is on the right of refracting surface S.
which will take account of the two refraction at S2 and S3. Shift = Real depth - Apparent depth
For refraction at isurface Sj : 2R \ (n -1)
= R- •R
1.3 1.1 (1.3-1.1) n + lj (n + 1)
= 0
V (-66) (ii) When the viewer is on the right, two refractions take
V=-78cm place at surfaces Sj and S2.
Image lies to the left of the surface S^. This acts as an For refraction at surface Sj:
object for the lens. n 1 _ (n -1)
For refraction through lens: We apply the general ^ (-2R) " (-R)
thin lens equation. Object distance for lens. which on solving for Ujjdelds
u' = - (78 + 78) cm 2nR
=-

17 13 1.5-1.3 1.7-1.5 2n-l


+
V (-156) +10 (-60) The first lies to the left of Sj and acts as object for
v' - 340 cm refraction at the second surface. We have to shift the origin
For refraction at surface S 4. Object distance for of cartesian coordinate system to the vertex of S2. The object
distance for the second surface is
refraction at S^ is
2nR r4n-l
340-34= 306cm U2 = - + R R
2n-l ,2n-l
1^__17
= 0
v" (+306) J___ n 1-n
Vi 4n-l -2R
1.1x306 R
or V =
2n-l
1.7
= 198 cm to the right of S4
109
iGSSnMaioPTicr
2(4n-l)D A lens is a transparent object with two refracting
On solving for ^2 we get V2 - -1) surfaces whose central axes coincide; that common cenffe
axis of the lens is called the optical axis or principal axis. The
The minus sign shows that image is virtual and lies to function of a lens is to deviate light; itcan either converge or
the left of S2- diverge aparaUel beam of light. Depending on this property
~! ' Shift = Real depth - Apparent depth lenses maybe characterized into two categories: converging
•„ 2(4n-l)R^_Cnj^^ lens and diverging lens.
" C3n-1) (3n-l)
THIN LENSES
In a thin lens there are two closely spaced refracung
surfaces. The thickness of the lens is considered negligible
compared with the object distances, the image distances and
the radii of curvature of the two refracting surfaces.
Fig. 1.176 (a) shows a converging lens (convex (a)Aconverging lens (b)Adiverging lens
lens). Rays of light entering aconvex lens parallel to its axis ! Rg. 1.177 j
are converged at its focal point.
Athin lens may be considered as made-up of a large
number of small-angled prism placed together; the angle of
prism is zero at the. optical centre and it increases as we
61-,../ move away from the optical axis.
1 • Athin lens is alens in which the thickness of the lens is
92 small compared to the object, the image distance, or either
ofthe two radii of curvature ofthe lens.
Fig. 1.176 (a) ,J Derivation of the Thin Lens Equation
We will consider refraction at
Fig 1176 (b) shows adiverging lens (concave lens). the two surfaces and S2 (Fig-
Rays oflight entering adiverging lens paraM are 1.178)
Refraction at first
1 c::; surface SjZ Applying the single
surface refraction equation, we
2 •Axis!
Axis 4 get
3' _ ("2 (I)
V u Ri
Fig. 1.178
Fig. 1.176 (b) The magnification produced,
Fig. 1.176 (c) shows avariety of lenses. Lenses thicker m mi ... (2)
the middle are converging while lenses thinner mthe mid e "2"
The single
are diverging. Refiraction at second surface S2
surface refraction equation now is
Converging "3 - ^3 ~^2 ... (3)
Lenses v' u' R2

Note that the image formed after refraction at the first


Plano-convex Bi-convex Positive surface acts as an object for refraction at the second surface,
Meniscus i.e.,v = u'; eqn. (3) becomes
^ _ ng -na (4)
Diverging V V R-
^2
Lenses
The magnification produced,
...(5)
m-> =
Bi-concave
Negative n^v
Piano-concave Meniscus
Fig.1.,176i(c)_
hio OPTICS

Note the twp^terms encircled in eqns. (1) and (4) ; they • Focal length of a biconcave lens :
are same'.* If we add eqns. (1) and (4), we get
n-i n^ Ho - Hi rio -
1

4 ^'•2 1—1

Eqn. (6) is the most general form of the thin lens


equation. Total magnification is Air yGlassX Air
n-,v n-yv _ n^v'
"1' Fig. 1.180
m = rriimg = —^ x —— ...C7)
rioU jioV rinU

n If media are the same on both sides of the lens, then


_(-R0 (+R2)
"3 = ^he eqn. (6) becomes
/ \
n^ n Tin -n^ rin - n-
1 _ "2 "2 - "1
+
V u R
I "1 J .
Ri K2.
which on rearrangement gives • Focal length of a plano-convex lens :
1 1 ^2 - Ui
...(8)
rii R, R2 7

We have dropped prime v', the magnification


now becomes
V
m = — ...(9) Fig. 1..181
u

Focal point and focal length : When an object is r


Tin - n^
placed infinitelyfar away from the lens, the incident rays are
'•I } i-Ro)
parallel. The image point is the focal point of the lens, the
distance of the focal point from the lens is called the focal rin - n^ Tin -rin 1
length. U
From eqn. (8), on substituting u = /,u = 0°, we get • Focal length of a plano-concave lens :
_1 i -
00 / R^ /^\R2= R
V "1 V^i
- n^
...(10) I 1
1 C2
f~ \
n.
2 J 1Sh-i
Eqn. (10) is called lens maker's equation.
1

Fig. 1.182
The thin lens equation now reduces to i h-lh-»
/ \
t

(11) "2 ~ "1


V u f
I "1 J
V
and magnification m = — (12) Tin ~ rii Tin - n .
u

n Focal length of a biconvex lens V ^27


Air /—\ Air Note that focal length of a lens is same, irrespective of
the side on w hich the object is kept.
R,
• Eachlens has two focalpoints. The focalpoint located
from lens maker's equation is primary focal
point. Any ray incident on the lens parallel to the
optic axis (u = 00) after refraction from lens passes
Fig. 1.179 through or appears to pass through the primaryfocal
point as shown in Fig. 1.183.

C+RO (-Ko)
; GEO/VIETRICAL OPTICS 111

Light from object point f > 0 m lens, and, when the surfaces are of the same curvature, is at
at u = - oo m
the centre of the lens. For a thin lens the lateral
'
displacement is negligible, and so, for our work with lenses,
0: rays passing through the optical centre are considered to be
undeviated. Thus in graphical work it is legitimate to treat
: rays passing through the optical centre, taken as the
intersection of the lens position and the principal axis, as
continuing their path undisturbed.
Primary focal point
CONVERGING OR CONVEX LENSES '
f < 0 m

Light from object point


at u = - oo m

Fig. 1.183
(a) Double
Each lens also has a secondary focal point. Any ray (b) Piann Concavo-convex (Convex lens
passing through or appearing to pass through the secondary
convex
r. M^x or meniscus
converging with one surface)
concave
focal point; after refraction from lens, passes parallel to DIVERGING OR CONCAVE LENSES
principal axis as shown in Fig. 1.184.
Converging lens

Secondary
Primary focal point
focal point
F
(d) Double (e) Piano- (f) Convexo-concave (Concave lens '
concave concave or diverging with one convex
meniscus surface)
Fig. 1.186 !
Diverging iens

Ray Diagram for Thin Lenses


Three rays are employed to locate the position of an
image :
Primary focal
Secondary focal point F2 (1) A ray passing through the centre of lens, the origin
of our cartesian coordinate system, passes essentially
Fig. 1.184
straight through the lens. A lens behaves as a glass slab in
OPTICAL this region.
The second general rule for drawing (2) A ray from an object parallel to the principal axis
diagrams involves the use of the so-called passes through or appears to pass through the primary
optical centre of a lens. Consider a ray AB focus.

incident upon a converging lens at B (Fig. Ray(1


1.185) and being refracted to C, such that Primary
the inclination of BC to both surfaces is . focal point
F
the same. Then the path of the emergent
ray CD is, obviously, parallel to AB. Thus Ray (3)
the effect of the lens on the ray AB is Secondary
merely slightly to displace it laterally. It focal pint
F2 Ray (3) Image
can be shown geometrically that all rays
passing through the lens so that the Fig 1.187 (a) Converging lens
incident and emergent directions are Fig. 1.185
parallel must pass through a fixed point
0 in the lens. This point 0 is called the optical centre of the
Ray (2)
Solution: Let ±eobjectbe placed at adistances from
the-pole Pj of the sphere. Ifa real image is to be formed at
Ray (3) equidistant from the sphere, then the ray must pass^
symmetricaUy through the sphere, as shown in the figure-
: Primary——
; focal poinU-''*
Applying the equation at the first surface, we get
.t*' Image F2 . M-i _ P2 r Ui ^
Secondary or x = \ —— R
focal point Ray(1)
(„. ('j) 5[verging lens- ' Note that the real image is formed only when the
refractive index of the sphere is more than that of the
(3) Aray from the,object passing through or appearing surrounding, i.e.,p2 >Pi.
to pass through-the secondary focal .point .emerges from the.
lens parallel to its princip^ axis.
• U — 6© -

Image at F
u="2fiv
Image at,2F,-
real inverted real inverted,
^ Imageat.60 I ;A small object of height 0.5 cm is placed infront of a conv^
reai.inverted •' i
very small equal.- "enlarged / surface of glass (p =1.5) of radius of curvature 10 cm. Find
'• Real •
Ithe height ofthe image formed in glass.
• Real-, Virtual,
1 : Inverted- : . inverted". erect •
small ; . enlarged . u = i.5
1 •
enlarged
V ^•
2F ^
,

jOBJECT: Between Between - Between. .


and 2F 2F and F - F and 0 ,
[IMAGE: Between Between 30cm
i F and 2F-. Between
2F,and«'. Fig. 1E.135
ahd-object •
(a) Convex lens .
i« u — oc Solution: According to Cartesian sign convention
Image at F
! virtual erect
u =-30cm; R=+10cm; pj =l;.p2 =1,5
/ very small Applying equation, we get
1.5-1 1 ^1.5-1
Virtual
erect V -30 ~ +10
small or V= 90cm (real image)
Let hj be the hdght ofthe image, then
IOBJECT: For all positions infront of mirror
, IMAGE: Between F and optical centre A = ^ (1)(90)
p-2" (15) (-30)
(b) Concave lens
hi = -2ho =-2(0.5) = -l cm
Fig. 1.188 The negative sign shows that the image is inverted.
136

jAsphere of radius Rmade ofmaterial of refractive index\i..l gl^s sphere of index 1:5'and radius 40 cm has.'half its
mere would an object be placed so that a real imag^isl '^emispherical surface silvered. The point wh&e a parallel
formed atequidistantfrom the sphere ? j ]beam of light, coming-along a'didmeter, fihdithe focus (or
'appear to) after coming_outdf.sphere.

Fig. 1E.134
Fig. 1E.136(a)
GEOMETRICAL OPTICS 113

Solution: i? = 40cm 1 ^ 1.5-1 ^4/3-3/2 ^3


for refracting surface oo X 10 oo
^w=4/3
o
M-2 _ M-i _ ^2 "1^1
V X 10
H=3/2 . •
n = 120 cm X = -20 cm
Fig. 1E.137(b)

fej 138
iThe diagram shows an equiconvex lens. What should be the'
condition on the'refractive indices so that the lens become
[diverging ?
Fig. 1E.136 (b)
R1
For mirror (2)

i +l - i
V u f
1 1
- + Fig. 1E.138(a)
i; .40 (-20)
v = -40f3
Solution: -.lii - tL = +tiZlii
for refracting surface (1) ' V u R2
40 _ 200 ^3 1^1 _ 1^2.-1^1 I ^3-1^2
u = 80-
3 3 V 00 R (-i?)
1.5 1-1.5
1^3 _ M-2 ~M-1 _ O-'-S "^2) < 0
V -(200/3) -40
V ' R ' R
1 ^.4.5 _ 0.5
1 • '• M H3
1 1 4.5

V 80 200 ; ^ - • Li
V - -100cm

So distance of image, from centre mil be • Fig. 1E.138(b) ]


100 - 40 = 60 cm (left of the centre)
For diverging nature v < 0
137
R R

object'0' is kept in air in.jrontofa thin piano convexlens] 2[L2 <lli+ll3


jo/radius of cuiyature' 10 cm. Ifs refractive index is 3/2and' Focal Lengths for a Lens with Different Medium an
the- m^diurn towards right of plane surface is water o/ either Side
yefi-active index 4/3. What should be the distance ' x' of the'
For paraxial rays only.
lopjectso that the rays become parallel finally.

= 4/3

ng =3/2-\
Fig.1E.137 (a).

Fig. 1.189
Solution:.1^3 _ 1^2 -Hi , 1-13 ^1^2
=
V u Ri R^
[114 OPTICS

For first refractory surface, Transverse Magnification (m^):


l^f -1^1 rrif = mj X m2
.(1)
Vi u Rj mi transverse magnification for first surface
For second refractory surface, W.' •transverse magnification for second surface' '
/• \
^ ^ Hi El
mi = mo =
V C^i-t) R2 l^a v^i /
For thin lens thickness't' is negligible, Result-3: Transverse magnification
P-2
V R2
.C2) (mj = Hi ...(D)
1^2
Adding equations (1) and (2) we get, Longitudinal magnification (for very small "object,
placed along the principle axis)
.(A)
V u R,
*•1 , R^
^2 dv
^i=T~
First principal focus (u =/i;u = 00) du

1-^2 Cl^f-^2) Ei_Ei = 1^/_.EizE2


00 /j Rj - V u Ri R2
^2
Differentiating both sides w.r.f.-Ui' :
...(3)
fi 1^1 ^1 ^2 •• ••

' K ) du '• '


Second principal focus (u = == f2)
^ ^ |£l f V 2
/2 . ~ Ri . R2 du. P2 V"
a
a

^ 2^
...(E)
/2 ^2 Ri
4
Rr
^^2 • 1^2 V" y

Dividing equation (4) by (3), we get,


> , ,mi =
f
E^v
\ 2
fRa^l
Result-1: A= (B) .^2 "y
/a 1^2
^rtii _\^2 (mj)^2
Relation between first principal focus and second ...(F)
V-i
principal focus.
Since ratio is -ve, so first and second principal focus IMPGRTANT POINTS
1

always lie on the opposite sides of.lens.. 1. If nature' of medium on two sides of lens is same
h-'
From equation (3) and (4) we get. (ji'i = q.2 =115)1.1^5 refractive index of surrounding.
Equation (A) transforms to,
f --Hi. •f =Hi ' 1
k' • k
V ' u Ra
k = say
1 ^
R.
*•1 R-
^2 •i_A - -1 ...(G)
V u
^aj
u ku' V . kv. ^ Equation (3) transforms to,
+ ~— —lii _1
u V k[ V u Ri R.

^-^ = k from (A) Equation (4) transforms to, ' ^ '


V u
•" '1 a_i.V r'^' 1
fi+ — = -xk
/a 1
u V k
kRi R 2 y
/a
Result-1 /i = -A; \fi\=\f2\=f
Result-2: A +A =i ...(C)
First and second principal focal lengths are numerically
equal.
115
"qeoivietricaioptik
Double or Bi-Convex Lens
Result-2 1-1 =(^-1 i_il =-l =l 1 =2 ^
y " VM-s A^i ^2/ /a f
Ri Ro Ri -^2
Sign of'/' is according to sign of second principal focal _
1
lengtli. Equation (D) transforms t, +ve
Ri R2
V f-v f
Result-3 = - = —
u / f +u

Equation (E) transforms to, /2 Us. JUl ^2 Fig.1.191

If p, >p,,then/2 =>+ve (Converging nature)


Result'4 mj = — = mj
u If p, <p,, then/2 =>-ve (Diverging nature)
Ri = R2 ~ ^ ' (Equi-convex)
.Equation (B) transforms to, If
i.'' '•
Results
fl Us
Equation (C) transforms to, Piano-Convex Lens
i4+A.-=i 1-1 =Jl' /I
U V w U /2 Ri R2 Ri
2. Lensmaker's Formula 1^
•"•l=T^-r Rj R2
It relates the focal, length of the lens to the refractive /2 •' ;r2=
index p. of the lens material and the radii of curvature of the If pj >Ps, then/2 '=^+ve' • •
two surfaces. . Fig. 1.192
(Converging nature)
i/ =a^-iA
V'^l R
^2 If p, <pJthen/2 =f'-ve^ (Diverging nature)
Double or Bi-C'oncave Lfens
where Rj is the radius of curvature of the lens surface 1

1 1 '1' •1 1
first reached by the light, and Rz- is the radius of curvature of a = — —+- -

the second surface. _ , Ri T2 L'Ri •R2_


(i) Ifthe lens is immersed ima medium whose refractive 1 A •

index is not equal to unity, "then jxrin the above formula is A


-^1
Us J yRi R2J
replaced by p, = •
1^medium

(ii) For converging'lens secondary focal length is +ve


and for diverging lens secondary length is—ye.

Fig. 1.193

Ifp, >p,,then/2 =^>-76 _ (Divergingnature)


. Ifp/ <P5,then/2 ^+ve ' (Converging nature)
(Ri=r2=r) (Equi-concave lens)
.Fig. 1.190

3. Application and limitation: . , , ; A Us -Jv i? J-' -


The Lensmaker's formula is application for thin lenses Piano-Concave Lens ,
only. The value ofRi and R2 are to be put in accordance with -ve '
the Cartesian sign convention. Ri R, Ri
116 OPTICS

A Us . . V.^1 ^2 If^j >10-5,then/2 ^+ve (Conver^ng nature)


. If|x, >n,,then/2 =>-ve (Diverging nature)
Concave-Convex Lens
(•-'

Ri < R2 (numerically)
1 1

Ri
'p t 1 . 1
+ve
Ri ^1" 1

Fig. 1.194
\
-L
lf\ii >^„then/2 =>-ve
J0
(Diverging nature) A Ills Rn 2 j

If|ij <p.s,-then/2 :^+ve (Converging nature)


Convexo-Concave Lens
1
L = _Ji.+ ^
J?i. .R^

Ri > R2 (numerically)
1 1
— < — becomes + ve
Ri R2 Ri R2 Fig. 1.196

>^,,then/2 =>+ve (Converging nature)


If)x, >^l„then/2 ^-ve (Diverging nature)

Fig. 1.195

OPTIONAL

T he Factor'
s. / Reciprocal of the focal
Type of Lens Shape of Lens Nature of Lens
No.
!.
. 'V
---1
A AJ ^ ij
length in air or vacuum

CD Concavo-convex
ILl' -1,-1' Converging
" 'Qx-I)
i)-( y. y.

(2) Convexo-concave •v_i Diverging


V --Oi-D
y.

(3) Biconvex Converging


fi.il
ili U yJ U yJ
GEOMETRICAL OPTICS 117

C4).
'4. ».
Biconcave
-(1.1) • -di-l/i+il Diverging

! 11 U y) U y)

(5) Planoconvex 1 (h-D/R Converging

1
• d'D 1 R

to
(6) Planoconcave • . 1 -Oi-D/R • ' Diverging

!H
i
,R

C73 Symmetrical biconvex " 2 Converging

Symmetrical biconcave
11J R

2 Diverging
C8)

[1. R

Graph of u vs. v for a lens : According to lens (b) Concave lens :


equation, it is a hyperbola, as shown in Fig. 1.197.
(a) Convex lens :
1' - .f"-
V \\ !
♦ ;•««—1— Virtual object
' Reallmage
!
v : !
L.-
Real object • j t i ; 1
Real image \/ 2f i [
-U-r s.-s.-:Vtrtual object 0 i+f j+2f u u -> +«>
— y —-J- —
Real image Real object* • 1
Virtual image j... J ^ Virtual object
_ OO <- u -2f : -f/ 0 :f i2f -J u u
: /; Real image
; j -f I I !
i i
? 1 «• •

- , , "I ' ' - - . . - ^

Fig 1.198
Real object
Virtual image •^OO^
u = -2/ -L 0 "+/' +/ +2/ 1

Fig 1.197 . 2 2

_/ •''0 ' +f +2/ +o6 V = -2/ _/ _/ 0 ,+/ +00 —OO


-2/
u = —OO
-2/ -/
3 2 3
4

V = +/ +2/ +00 -
-/ _/• 0
u +/
"3l 2 3
OPTICS
ANALYSIS OF G^PHS
Concave Lense
Convex Lens

l_l/ 4
u < ORq
v>0 R, u>OVo
u >0R '
" ^ i

u<0 Rq
y>0 R
u>0 Vo
u>0 V,
u < ORq
u <0V

u<ORo
V < 0 V,- u >0 V
y<0 V

'
' J r
1, . 3
'
' 4
2F —t '
F . F 2F

Fig. 1.199

Re^on 1:,Object lies between optical centre and focus


real object, virtual:imagejw|>|ul.. - ' '• ' Fig. 1.200
V
m=- thus ImI>1image is enlarged is if^ 0': v<0thus'
m>0image is erect. As object moVes-frohi focus towards of aW.? °f areal
alens. Object is real image is virtual object infront
|u|<Iu|.
optical centre magnification goes on decreasing.' ~ |ml< 1image is smaller in size.u <0,u <Om >0image
Region 2i Objeh'lies hetWeeh ^anh'^ oblecfis real' IS erect. As object is moved away fi-om lens image size i.e.
image is also real |u(>14 - magnification goes on decreasing. '
|m|> 1image is enlarged v>0;u< 0m<0image is
inverted as object moves from F to 2F magnification centr^a^d focus.™ between optical
decreases and equals 1at'2F^' , ' , ' , Object IS virtual and image is real |v|>| u|;|m| >1image
IS enlarged u>, u>0, m>0image is erect.... . ^'
rP.l
real and image is also real. between.2F,
|ul<lul. " andjnfinity
' .y object
j i is towards F, image size goes on
increasing I. e., m increases. . •
l™3ge is"smaller'in:size^u'^ 0:u <'07h'< Oimairp
Region 3:, Virtual ^object' lies .between +F and +2F
objectis virtud and image is virtual|ul>lul.; Iml> limageis
enlarged u>C^v<0;m<0 image is inverted.
decr^s£''' magnification goes an
"SoiScA^ics
Region 4: Virtual object lies between 2F and infinity
object is virtual and image is virtual lul<luLlnil<l image is Optical
smaller insize u>0;i;<0;m<0 image is inverted.
Measurement of refractive index of Uquid by a
convex lens
Fig. 1.201 shows an equiconvex lens placed on aplane
mirror. An object pin is moved up and down. When the pin Fig 1E.I 39
lies atthe focus of the lens, there is no parallax between the
object and the image.
Solution: The rays of light diverge from the object;
itis areal object. Focal length of aconvex lens is positive.
1 ^ nz - Hi r 2^
f "i u,
(3/2)-ir 2;i_
1 lioj 10
The given parameters are
u = -30cm
Fig.1.201 f = 4-10cm

When aliquid whose refractive index is to be obtained is Substituting these values in thin lens equation, we have
placed between aplane mirror and aconvex lens, the object l_i = i
v.uf..
pin 0 has to be shifted downward-so that no parallax exists
between itand its image. The position of object Ofrom the 1 1 _
lens is now equal to the combined focal length of lem and V (-30) 10
liquid' combination. If / is the focal length of the liquid lens, which on solving gives v = 4-15 cm.
then the combined focal length is given by
:"i I ', 1 The image distance is positive, which implies that the
+ image is real. The lateral magnification is
F /o 7 _ u _ 15 _ _ 1
2
The liquid lens is plano-concave type as its lower surface
is the plane surface of the mirror and the upper surface is the , The image is half the size of the object; size of the image
curved surface of the convex lens. If R is the radius of is 0.50 cm. The negative sign with magnification shows
curvature of the convex lens, then inverted image.

/ • l^-R -=>
1 _ Qx.-l) ' 'If the pencil in the previous problem is kept at 6cmfrom the,
or
/ R *' ilens; locatk and characterize thejnm
Thus, Solution: From lens equation,

V f u 10 C-6) 15

' v = -15cm
pencil of height 1 cm is placed 30 cm. from eqmconv^ V —15 _ n r/-v
Lateral magnification, ^ =
hens, refractive index n=3/2, radius ofcurvaturefor both the, 1-4 . ^ '

[surfaces, R, =R2 =R''=10cm.Find^helocation'ofthe-image^


landdescribe its characteristics.^ The minus sign with image distances shows that the
image is located on the side .of .the object. The image
upright, because magnification is positive. ,
is
[120 0?j^]

•If in the previous example we use a diverging lens with a focal


'length 10.0 cm to form an image of the pencil kept 15 cm
dnfront of the lens, locate and characterize the image.

Fig. 1E.141

Solution: in accordance with cartesian sign


convention the given parameters are,
/ =-10 cm, u =-15 cm
From lens equation, we have
1
- • ' •V f u (-10) (-15)-, 6

^ u =-Scm '
- Lateral magnification, - . .
rh = —'= = 0.40
u -15 Fig.1E.142

The 'minus" sign, with image shows that the image is


located on the side of. the object. The magnification is The image produced by the first lens acts as an object for
positive and m < 1,'which shows that the image is upright the second lens; this image is,called the first image.'the first
and diminished. ' • image can lie between the lenses or-to the fight of the
second lens. In the first case it will be a real object and in the
second case it will be a virtual object for the second lens,
(a) For the image formed bythe first lens,
Apencilwith a height of5 mm isplaced^45cm to the leftof a 1 a- 1 . 1 'l
\converging lens offocal length 20 cm.A diverging lens dffocal 20 (-45) '36
'^length 15 cmisata distance 10 cmfrom thefirst lens.Whatis fi "i
\(a):the image distance Vijand (b) the height 11,of the image^ Vi - +36cm ' •
•produced by, thefirst lens ?r(c) What is thepbject distancefor. We thus find that the, first lens is attempting to form a
\the seconddens ?Find (df'the image distance v2 and(e) thex real image, 36 cm behindTt.
Iheght 12 ofthe image formed by the second lens. . ^ However, before .this image can be formed the rays are
intercepted by the second lens located 10 cm behind it, see
Sblutlon: The image formed by a converging lens
can-be real or virtual. It depends on, where the object is Fig. IE.142 (a). The point of convergence of these rays is a
virtual object for the second lens.. .
located relative to the focalpoint. If the objectis to the left of
the focal point as in this case, the image is real and formed (b) The height of the image,
on the right side of the lens. If on the other hand the object is 36
located between the focal point,and the lens, the image is = (5x10"^) = -4mm
'1 J -45
virtual, located on the left of the lens.
The minus sign indicates that the image is inverted.
121
GEOMETRICAL OPTICS __ _ _ ,
(c) The object distance for the second lens
= (36-10) cm = 26cm
to the right of second lens ; hence i;2 =+26cm •The vato of di"^d d^ for final rays to be parallel to the]
iprindple axis are: (focal lengths of the lenses are written on,
(d) From lens equation for the second lens, khe lenses) _
35.5
i '' 10cm 20cm ; -5cm
V2 A "2
V2= -35.5 cm
The negative sign shows that it is on the left of the
diverging lens (virtual image) ; 10cm
(e) The image height for the first lens is object height for
the second lens. Height of the image formed by the second i Fig.1E.144(a)
lens
-35.5 1(a) di =10 cm, d2 = 15cm
I2 - O2 ^l = (-4xl0^^) + 26
= 5.46 mm
(b) dj= 20cm, d2 =15 cm
U'
'(c) di = 30cm, dz = 15cm
The positive sign for 12 means that the final image has the i(d) Nojie_of_these^
same orientation as the first image.
Solution: (a), (b), (c)
143

Udtverging lens offocal length 10 cm is placed 10 cm mAontj


iofaplane mirror as sKown intheFig. 1E.143 (a).Lightfrom\
la veryfar away sourcefalls on the lens. What is the distance^
\offinal image ?
10 cm

20 cm

Fjg. 1E,144(b)
d2 = 20-5 = 15cmn
dj can take any value.
Fig. 1E.143 (a) ite .1145

Solution: •'Ati objectis placedatadistance of15 cmfrom aconvexlens of\


focal length 10 cm on the other side of the lens a conv^
•mirror is placed at itS'focus'such that the image formed by.
1 1 I,'
^combination coincides with the object itself Find the focal
dength of concave mirror. •
20 cm

10 io
Fig.1E.143 (b)
Flg.1E.145 (a)'
1 1
V u / Solution: For retracing of ray; ray must fall normally
1 1
on mirror i.e., towards the centre ofcurvature.
V -30 ^-10
= => u = -7.5
i; 10 30 30
The final image distance = 2.5 cm in front of the mirror.
fl22 OPTICS

f=10cm
I = ..-.(1)
471(60)^
Energy received by lens

15cm 10cm ^ 2f *^ ^2 =
4;t(20)'
_ 0.75^2
i o —

30cm

Fig. 1E.145 (b) From similar triangles


1 1 1
u = -15
^ = 25
A,
V u /
1 J__ 1 12 = 0.271
/ = 10 cm
V 15 ~ 10
'147
V = 30cm
T7 mirror
. I + l
— =l Apoint source of light S is placed on the axis of a lens offocal
For — — u = 2f
V u length 20 cm as shown, in theFig. IE.147. A screen isplaced
1 normal to the axis of lens at a distance xfrom it. Treat all
v^2f~ f rays as paraxial.

v = 2f 25 cm

Hence from the ray diagram


2/= 30-10
/ = 10 cm

'e 1146
f = 20 cm

'A point source of light is placed 60 cm away from screen.


Fig. 1E.147(a)
[Intensity detected at point Pisl. Now a diverging lens offocal
ilength 20 cm is placed 20 cm awayfrom S between S and P. (a) As X is increased from zero intensity continuously
'The lens transmits 75% of light incident on it. Find the new decreases
value of intensity at P. (b) As X is increased from zero intensityfirst increases then
' 20cm decreases
(c) Intensity at centre ofscreenfor x = 90cm and x = 110cm,
is same

(d) Radius ofbrightcircle obtained onscreen isequalto 1 cm


60cm
for X - 200 cm
Fig. 1E.146 (a)
1
Solution: (b, c, d) 1-1 =i =
Solution: u = -20,/=-20 V u f u C-25) 20

gives, ^ = -10
^ l^J^ 1 _ 1
11 = 100 cm
Let P = Power of source
V 20 25 ~ lOp
110 cm
90 cm

25cm 100cm

Fig. 1E.146 (b) Fig. 1E.147 (b)


GEOMETRICAL OPTICS 123

I From O to J intensity increases and then decreases at


X = 90 cm and 110 cm intensity is same.
(d) Radius at x = 200 cm is equal to radius of lens.
DISPLACEMENT METHOD
If a thin converging lens of focal length '/' is placed
between an object and a screen fixed at a distance D apart
and if D > 4/, then there are two positions of the lens at
which a sharp image of the object is formed on the screen.

H
0
LU'
Fig.1.203
-5
u \ V=(D-u)
0
(i) If the distance between two positions of the lens is x,
0
1 ^< X= 1/2 - Ui = -jDiP - 4/) from equation (2),
\ / - i.e., x^=D^-4Df
So, / = (D"-x")/4D ...C3)
Fig. 1.202
(ii) The image distance corresponding to two positions
of lens will be,
If the object is at a distance 'u' from the lens, the
distance of image from the lens v = (,D-u). So from lens
=D-Ui =D-Hd-^DW-V)]
formula,
=hD +VDCD-4/J]='u2
1_1^ r
V u f
and V2 =D~U2 =D--[D + VDCD-4/)].
1 1 1
we have.
(£>-u) ~u ~r =hD-V5(5T47j]=u,
I.e., -Du + Df = 0
i.e., for two positions of the lens object and image
So that u = —[D ±-jDiP ~ 4/)] ...(1) distance are interchangeable.
2
(hi) As X = U2-Ui
Now there are three possibilities:
and D = Ui+Vi = Ui +U2
(a) If D < 4/: u will be imaginary, so physically no
position of lens is possible. as Vi=U2^
(b) If D = 4/: In this situation u = D/2 = 2f. So only one i.e.. "1 =(U2) = (D-x)/2
position is possible and in this situation, and "2 = c^i) = (.D + xy/ 2
i, = D- u = 4/-2/ = 2/C= u) • • ' So the magnification" for two position of the lens will be
(c) If D > 4/: In this situation both the roots of equation respectively,
(1) will be'real. . , ///. h ^D + x
i.e., =^D-^D(D-4f) 0 k^D- X

h = £2 = (D-x
and : '...'(4)
and U2 =—[!> + ...(2) 0 "2 ^D+x
2 ,
And hence,
So if d > 4/, there are two position of lens at distance
and u 2 from the object for which real image is formed on the (a) X m2 = (/1/2
screen.
i.e., o = TiyT - ' 1' ' ...(5)
This method is called Displacement method' and is used 2
D + X V (D + x D + x
in laboratory to determine the focal length of convergent Cb) X

lens. In case of displacement method.


m2 I2 \D-x). {D-xJ {D-x
D + x] (D-x^ 4Dx
mi - mo =
D-x D + x D^-X-
1124 OPTICS

With the help of equation (3) it transforms to, P = Pl+P2


m-i — tTZo — —
/
-=i-
/ kfi
I f
X
I.e., / =
nil - TTi' p=tPi
i=l
( 't

(a) When Lenses are in Contact Note that the effective focal length or power is
Considertwo thin lenses with respectivefocal lengths /j independent of the order in which different lenses are
and /2. Ifu be the object distance, and u' the image distance arranged.
due to the first lens, then from lens equation, (b) When the lenses are separated from each
other coaxially: Determine the deviation produced by a
lens, is given by h 1
where, h is the height above the principal axis where the
light ray PQ strikes the lens. ,.

Fig.1.204

V
...(1)
A
Evidently, the'image V formed by tiie refraction at the
first lens behaves as an object for the second'lens. Let the Fig. 1.206
distance of the final image be v'. Again from lens equation,
Now, consider two thin lenses and •I'2'i'with
1 - i = A' ...(2) respectively focal, lengths fi and f2- and placed co-^ally
V A ,,
V
with separation of d. (Fig. 1.207) , , . -
Adding equations (1) and (2), we get
1-1=A+A ...(3)
" /l /2
If / be the focal length of the equivalent lens i.e.,
corresponding to the same object distance u, if/be the focal
len^h of a single lens, for which the image distance is also v,
then from lens equation,

V u f
' ...M
Fig. 1.207

A light ray PQ parallel to the common axis strikes the


lens Li at a height hj above the axis and gets deviated by an
angle a, such that
a =

Fig. 1.205
Similarly, for the second lens, the point of striking of
Comparing the R.H.S. of Eqns. (3) and (4), we get light rayQP is at a height h2 above the axis and its deviation
is j3.
1-A A
f'A^A
Similarly, if P be the effective optical power of the lens
Mf. •
system, and i\,P2 individual powers of the two lenses, Evidently, the total angle of deviation
then
GEOMETRICAL OPTICS 125

8 =a +p i.e., m = nil xm2'x

or, !lL =h+h ...(1)


If lens is cut parallel to principal axis, then radius of
f fl h curvature of none of the refractory surfaces changes, so focal
length remains unchanged. If lens is cut perpendicular to
where / is the effective focal length, of a single lens kept
principal axis then radius of curvature of one of the
in plane P which serves the purpose of the lens system.
refractory surfaces changes and so focal length changes.
0:to QP{r and RO-^T, we have
From similar triangles
Silvering of One of the Surfaces of Lens.
QOi OiT
Silvered lens is equivalent to mirror. Focal length of
OfT
equivalent mirror is given by.
hi h2
. .or.
fl fi-d J
_hi(fi-'
or
/i h ^3
1

Substituting for ^2 in Eq. (1), we get


h, _/zi , h^ih-d
Fig. 1.208
f fl /1/2
or i-+ ...(2)
n m
(proper sign has to be considered).
/ fl A /1/2
Here it is worthwhile mentioning that the values of F =>Focal length of equivalent mirror
and /2 should be substituted in terms of proper sign and the n => Number of refractions taldng place
sign of the resulting / automatically reveals,the nature of the m =>Number, of reflections taking place •
lens system.
fl => Focal length of lens in unsilvered condition '
Equivalent Focal Length of A Lens
Combination: f^^my " —•
R => Radius of curvature of silvered surface.
If'n number of lenses of focal lengths A, A, A,; A
are joined together then the equivalent focal length of the If P is - ve, then equivalent mirror is converging and if F
combination is: is +ve, then equivalent mirror is diverging.
1111 1 Case (I): Plane surface silvered:
— — 1 H +.
^ fl. f2 A A
-^ =01-1)
Sign Convention ^ Jl

Focal length of a'converging lens is taken as positive and ft — — ^ ~ 00

that of a diverging lens is taken as negative. In terms of 2'


power, the above expression may be written as A__ J_ 2(\i-iy
-0
P = Pi + Po + P'l +. .+p„ 1.209
fl fm R

Power of a lens is defined as the reciprocal of focal


R
length, where / is measured in metre. F = - l^ =
2(^1-1)
1
P =
/ Case (II): Curve surface silvered:

= (p-l) '1'
-1
The unit of power is dioptre ID = Im"
When several lenses or mirrors 'are used co-axially, the Jl u
image formation is considered one after another in steps. R
fm=-
The image formed by the lens facing the object serves as an
object for the next lens or mirrorl the image formed by the 1 _ 2 !• "2(p-l) 12*
second lens (or mirror) acts as an object for the third, and so
F" . R ' R-.
on. The total magnification in such situations will be given
by, R Fig. 1.210
P = -
A
i A h 2p
m = —= —X —X.
0 0 h
1126

Case (in): Surface mirror and system is


dipped in alcohol: Y =^2-Vi 1-1.
R oo R •
r * "'"water

/• fl f2 /s R R. R ' ' '


/f1f fa
ALCOHOL 1 / =
R

(1^1 -1^2)
Wi
Concept: If lens offocal length '/' is cut into two equal
parts bya horizontalplane AB (Fig. 1.212), then as none ofp.,
Ffg.1.211 Ri and J?2 ^dl change, the jbcal length of each part will be
equal to that of original lens.
1
h -1 — + (Glass lens in alcohol)
fl Hai

1 . 1
(Water lens in alcohol)
R3
1 1
— + (Glass lens in alcohol)
/s •^M-ai > ^^3 ^4 /
1*
•--h
. 2

1'^- Fig. 1.212 (a)' '


f

F Jl fm) However in this situation as light transmitting, area of


r X --'f ^>2 each part becomes half of initial, so.intensity will be reduced
— + — + —
. .p-rr.
1/1---/2" •/3 to half
Equivalent focal length F in surrounding alcohol. I However if the same lens is cut into two equal parts hya i
^vertical plane CD [Fig. 1.212 (b)] then'radius ofcurvature of
148 \one of the refractory su^aces changes..and so focal lengthy
i_i changes. , ^ ^ \
[rifie adjacent Fig. 1E.148 shows athin planO'Convex lens of I i ^ C . -
fefractive index[Li and a thin plano-concave lens ofrefractive
\index \i2, both having same radius of curvature Rof their^
\cuTy,e4 surfaces. The.thinlenspfrejfactjve.indexii^ has radiusl
\ofcurvature R of both its surfaces, fliis lens isjo placed in,
petween the plano-convex and plano-concave lenses that the,
Iplane sujfaces are parallel'to each othkr. What is the focal]
dength of the combination. ' ^ ^ ' '•

, Rg. 1.212(b) ,

Thefocal length of each part will becomes,

] Fig:'lE.148
'
;
lf =ai=i)fi-Tl=M=±
U ~J R V
^1
i.e., /'= 2/ (Here original lens is considered 05;
Solution: —= -i)fl +l ]= equi-convex lens) ' , ^•
/i A- Rj R i.e., focal length of each part will be double of initiaV-
,value. In this situation as the light transmitting area of each),
-A = 0^3-1) -1^11
R R
=0 'part of lens remains equal to initial, intensity and aperture^
J2
\will not change. 1
127

Two Thin Lenses Separated by a Distance


If a lens offocal length' f is divided into two equal pam Aco-axial system of two thin convex lenses of focal
asshown inFig. 1.212 (c) and each part has afocal length f length '/' each separated by a distance 'd\ In case of two
then as,
thin lenses separated by a distance 'd', we have,
^ 1-2 ^
F fi fi /1/2
For /l =/,/2=/
1 =1 1-A
f'/""/ /'
/• . So
i.e., F =
2/-d
Fig. 1.212(c)
(a) If d<f,F will be positive and </, so the system
i.1 +1 i.e., f'=2f will behave as converging lens of focal length < /.
. / /' /' (b) If d=f,F =f, i.e., the system will behave as
i. e., each part will have focal length 2f. converging lens offocal length / .
Now if these part are put in contact as in Fig.1.212 (c) Cc) If f<d<2f,F will be positive and >/, so the
the resultant focal length of the combination will be, system will behave as converging lens of focal length >/.
1 =J_ +i i e., F=/(=initial value) (d) If d=2/,F will be infinite i.e., the system will
F 2/ 2/ behave as a plane glass plate of infinite focal length.
If a lens offocal length is cut in two equal parts as (e) If d> 2/,F will because negative i.e., the system
.shown in Fig. 1.212 (d) [i], each part will have focal length will behave as diverging lens.
Now ifthese parts are put in contact as shown in Fig. Concept of Image Forming at Object itself;
'1.212'(d)[iiJ, the resultant focal length will be. In some unique situations the image is formed at the
same point as the object. For a lens, if we apply the lens
formula we get,
1-1 = 1
V u f
1 1 _ 1
Fig. 1.212(d) or "7
u u f

i =i + / = (//3 which is not possible.


F f f
On the other hand, for the case of refraction atross a
Angle of deviation of ray when it passes single curved surface, the formula is _,
through a lens. [i 2_ M-i _ Qt 2~1^1^ ,
V u R • • '
which reduces to
h
b7"\
P
1 \ V u R
0
The solution for which is
u = R. Therefore the object and
image coincide when the object is
' Fig. 1.213 placed at the center of curvature
CFig. 1.214).
a=hfi-- Similarly, in the case of
Fig.1.214

mirrors, the formula is


or.
1+1=2
-7 V u R
128 OPTICS

which simplifies to
^ '150
1 1_2
u u R
•A convexlens offocal length 20 cm is placed 10 cm injfqnt'of
and the solution once
la convex mirror of radius of curvature 15 cm. Where should a
again is u = R. Therefore for
•point object be placed in front of the lens so that it images.qn
mirrors and refraction across a
]foitself2^ ' „ _
single curved surface, we can
say that object and image will Solution: The convex lens and the convex mirror are
coincide only when the object shown in Fig. 1E.150. The combination behaves.dike a
Fig.1.215
is kept at the centre of
concave mirror
curvature.

In problems in optics, we
will usually have a train of
optical ' elements with .the
stipulation that the image is'
formed on the object itself. In
such cases," there will have to i X 10 cm " •15 cm
be a mirror at the end of the
optical train and the rays haye ' • I Fig.1E£l50
to be incident normally oh the Let the distance of the object from the lens be x.
mirror in order to retrace their paths. Three kinds of mirrors
are possible. For .the ray to retrace is path it should be incident
normally on-the convex.mirror, or,m other words the rays
should pass, through.the center of curvature of the mirror.
From'.the diagram we see .that.for the lens
•A glass slab of thiclmess 3 cm and refractive-index 1.5 is, .u = -x,f = +20 cni> V = +10 + 15 = +25 cm
placed in front of a toncdve mirror 'of focal lerigtH 20 cni.' From the,lens equation.we get
Where should a point object beplaced'ifit is to image on to- /i^l^l.'' J L =l_
\itself ? The 'glass slab and the concave mirror are shown in' V u f 25 -X 20,
or x = 100cm

'Solution: Let the distance of the object from the


mirror be x' We know that the slab simply shifts the object!
The shift being equal to
l' [A convex lens offocal length 10 cm is placed 30 cm infront o/|
s = t 1- = 1 cm
>a'second convex,lens also.of the same, focal length. A plane!
\mirror is placed after the two lenses. Where should]a point]
The direction of shift is towards the concave mirror, •object beplaced infront of thefirst lensso that it imageson to\
the apparent distance of the object from the mirror is titself? ' " „i
x-1.

;0 O'

Stab--<

! - .r.Fig.*j1E'.149 : .Fig. 1 E.I 51

If the'rhys'afe to'retrace'their paths, the object should


Solution: The convex lensesand the plane mirror are
appear tb^Ve'atfthex'eriter'bFcurvature ofthe mirror. shown in figure.'
x-l = 2/ = 40cm The combination behave like a concave mirror.
or X = 41 cm from the mirror.
GEOMETRICAL OPTICS 129

"Let the distance of the object from the first lens be x. (b)Plane-MitTor method
: For the ray to retrace its path it should be incident : A convex lens is placed on a
nonnally on the plane mirror. plane mirror. An object pin O is
- .From the diagram we see that for lens L2, moved along the principal axis of
the lens. At a certain position,
From the lens equation we get the image coincides with the
1-1 = 1 or u = -10cm object. The distance of object pin
V u f 0 from the lens is the focal length
From the diagram we see that for lens of the lens. When an object is TtrnnnnrTOnfttnTi cmTtflTTTTOmmTm
placed at the focus of a lens, the
u = 30-10 = 20 cm, / =+10 cm, u =-jc Fig.1.219
refracted rays become parallel to
From the lens equation we get the principal axis. There parallel
j L =A ' rays are incident on the mirror normally, therefore .they
V u ,f 20 -X 10 retrace their path and converge at the focus.
, or X = 20 cm (c) Measurement of focal length of a concave
lens : Since a'cbhcave lens does not form a real image of a
%mei;hods,F:Or determining focal real object, therefore, a convex lens is used to form a real
;. aLENGTH OF A CONVEX LENS iniage as iri-the case with the convex mirror.
' <(a)-.Graphic^,]|iethod : The convex lens forms real
•and inverted images^ of an object placed between infinity
.and focus.
From-lens equation, using cartesian sign convention, we
•.have'
u = -x, v = +y, / = +/
L-i
+y -X +/ Fig. 1.220

' 11' In Fig'. 1.220 a convex lens Lj forms a real image of an


' If a^graph is.plotted between — and —, it represents a
+v object 0. When a concave.leris I2 isplaced in'between 7;^ and
-u
•.straight'line with.slope equal to -1, as shown in Fig. " Li, '.then the 'convergence of "the beam, coming from the
1.217.:The,mtercej5t;with the horizontal axis is -y and that, ' convex lehYs'shifted to position"/^.instead of 7i. .
-, The point acts as virmal object to the lens L2 and it
•with .the-yertical;axis is —. If a line OPis-drawn at angle 45°, forms ayealimage 72 ofij,'whose distance from 73 isy, then
, accOrding'to Gqrt^idn Sigh'Convention, •
-at intersects -the line AB at the point P. The coordinates of the'
. 'Y 1 1 ' ' ''' ^ ^ • •' ii'= -i-x,' u = +y
point Pare, ,—
, By lusing the"'lens,formula
!•

V " /
1 -1 1.
we get = —
+y +x /
xy
or /-•
•y-x.

Thus, by measuring x.and y, the focal length of the


concave lens can be easily Calculated.'.
Fig. 1.217 Fig.-1.218
Note: The focai iength of thexonvex lens used in this method
Altematiyely, a graph.between u and.-u can be plotted - he greater than the focal length of the concave
'Jens, otherwise, the real image by the concave lens will
as shown in Fig. 1.218. A Jihe drawn, at angle 45° from the ^""riot-be;forrhed. • "
origin intersects the curve at the point P whose coordinates
are (-2/,+ 2/). By measuring the coordinates o^'this point,
the. focal leiigth of the. muror can also be measured.
[130 OPTICS

Overall magnification,
152
m = m^ X m2 X m^
A biconcave lens fj = -25 cm is placed 20 cm infront of a = (+0.625)(- 0.205)(0.641)
Iconcave mirror f2 = 5 cm as shown in Fig. IE.152. A 2 cm = -0.0822
'.)
]high object is placed IS cm to the left of the lens. Using the Final image is virtual and inverted.
lens and mirror equations and a ray diagram, find the-
location of the three images : (a) the imageformed by the lens' 153
las the rays travel to the right, (b) the imageformed after the
trays reflectfrom the mirror, and (c) thefinal image after the^ A converging lens, fj = 12 cm,is separated by50.0 cmfrom ai
leftward travelling rays once again pass through the lens. > diverging lens, fz = -10.0 cm. Anobject 1.0 cmhigh isplaced'
25.0 cm to the left of the first lens. Find the position of the'
final image and its height. Complete the ray diagram and,
characterize the image. _ „ , _ I
Solution: The object 0 lies outside the first^focal
point Fi of the first lens, so this lens produces a real image I.
Fig. iE.152 The light rays that strike the second lens diverge from this
real image just as if I were an actual object. Hence the image
Solution: (a) From lens equation, made by the first lens acts as an objectfof the second lehs.
— =—i—, V-- 9.375 We first find the positiori of the first image. Frbm"lehs
u C-15) (-25)' equation, • . (n"

Magnification, mj = —=
-9.375^ = +0.625
i f
= —,v = +
1 ^ 300
cm
u . -15 J . - (-25) ' 12 • .13

The first image is a virtual, erect, diminished and The first image I is cm to the right of the first lens.
formed.9.375 cm to the left of the lens. '
(b) The first image acts as an object for ^e mirror; .•The magnification is
object distance for the mirror is (20+9.375) cm. v ' f 300/13
mi = - = = -0.923
From mirror ^uatibn, - - u .. -25 y -
-1 - • 1/ ' ^•
v' - -6.026cm Hence the height of the first image = (-0.923^(1)
(-29.375)/ (-5)
9
•'• = -"o'.92'3cm"
' -6.026 300^
Magnification, m, = - — The first intake is 50 - Icm to the left of the second
V-it-' . ^9.375J
= -0.205 lens; hence the object distance, for the second lens is
, 350 ^ .
The second image is'real'(infront of the mirror), u = cm. " ' V -
13 •' ^
inverted diminished, .formed at 6.026 cm irifront^of the
mirror. •> . • From lens "equation,-for the secdridTens,
(c) The second image is an object for second refraction .. 1 = +^,n' =-7.29cm
at the lens when the rays return. n'' (-350/13) -10'
Object distance =.+(20 r 6.026) cm , The final image is 7.29 cm to the left of the second lens
, ^ 1. • ,1 - ly;.'. •. r.-' " and 42.71 cm to the right of the first lens. The magnification
u" (+13.974) ~ 25 produced by the second lens is
f .• • > I J . N , ,

v"=+'8.96 '• ' ' "-7.29


m-) = = 0.270
Note the sign of focal length of lens and object distance. -350/13
Height of the second image = C0i270)(0.923) = 0.250
Magnification, nio = — = I = 0.6.41, , .
• h'--'' l:l'3.974j Overall magnification m = m^ x m2
Final image is 8.96 cm to the fight of the lens. ' • • • =(0.270)(-0.923) =-0.250
j )

Jh^final image.is virtual and-jnyerted.


, if.''
131
rGEpiii^^
154

Aconcave mirror has radius ofcurvature R = 30 cmApencil


[lies perpendicular to the axis of the mirror at each of the\
following object distances: (a) 40 cm (b) 30 cm (c) 15 cm (d)
\5 cm. Find the image of the pencil, magnification of eachj 30cm
—15cm

[image and show tt_^aphically. i


Fig. 1E.155 (a)
1,
Solution: - The focal length of mirror is
/ = R/2 = 15cm. \lf the image is formed at coordinates j^lO cm,- —cmj ,i
'From mirror equation,
1
[assuming paraxial ray approximation, determine : .
(a)' i+ ^ V = -24cm
;(a) the radius of curvature of the mirror,
u (-40)' (-15)
\(b)^e-refractiv£indexofthepmm. _
The minus sign shows that the image is infront of the
mirror,.real. Solution: A thin angle prism produces a deviation
•^24
= -0.(
5= (n_l)A, toward the-base., of the prism. The object
u 1-40 appears to be at0' due to deviation as shown in Fig. 1E.150
(b). ^
As m < 1, the image is dminished in size; minus sign ., 00' = To = C15)5
shows that the image is inverted. ,K
' 1 1 •' 1 -,u = -30cm = (15j(ii-l)
(b) •+ ISO,
V (-30) (-15)
The image is real, formed infront of the mirror.
The image isformed at (30 -10) cm from the mirror, at
V
As m= -l, the object and the image are equal," minus
sign shows that the image is inverted.
1 -
(c) -+ ^
n (-15) (-15)
•, V =
,
,'t ~ t ' I'* > •••
Inthis case outgoing rays are, parallel, corresponding to
an infinite image distance. ^ l*-10cm-*4 - •
_v, rjj .CO, •15 cm—
t. - ' = -oo or (+'o°)
"""""""(-15) —r 30 cm

(d) i+ ^ u = +7.5
v (-5) (-15)
Magnification, ~ ^-23^>P1''
^^7 ' • '•
The plus sign shows that thq image is behind the mirror,
virtual. It is a general observation that an object placed
inside the focal point of a concave mirror produces avirtual
• •pr =f6o' =|(n-U^ =a^-i)^
image. The rayof light, after reflection fr'om the mirror, passes
m
' v\ ( +7.5
= +T5 through the prism and is again deviated by 5, so_that the final
-5 image is formed at J where v'. v" . '
As m>1, the image is magnified; plus si^ shows that it J/' = (15-;1P)5 •
is erect. ' . •' - . ,, I :=,(^-l)-^cah^^ ^ ^ •
.155 y-coordinate of |he final image '
. , -PI = -(PI' + I/'X v; '
A thin prism, refracting angle ^ = ^
infront of aconcave mi&or. The object is kept at ih^ ori^n o/j
[the coordinate^smn
t132 OPTICS

1.5-1
= -01-1)
12 0.3 1

Hence, R = 0.3
^ 4/3 1 (3/2)-l , (4/3)-(3/2)
So we have — = — +— , ,,
or ^1 = 1.5 u 0.9 0.3 0.3 ';
as 11-1 or ti = 1.2m
V u R
The image formed is real, to the right, of the lens and
2
+ • behind the mirror. It is a virtual object at a distance 0.4'm.for
(-20) (-30) R the mirror. Now the mirror forms an image infront of it.- This
R = -24cm image is an object for the lens system, with
= 4/3, n2 = 3(2, ^3 = 1, R = +0.3 m, u = - 0.4 m
1561 jh "2-"]
u' R R
'A thin equiconvex lens ofglass of refractive index n^ = 3/2, 4/3 ^(3/2)-(4/3) ^l-(3/2)
and offocal length 0.3 m in air is-sealed into an opening af
(-0.4) 0.3 (0.3)
loneendofatankfiUedwithwater, n^ = 4l3. On the opposite,
of the lens a mirror isplaced inside the tank on the tank waW v' = 90 cm
perpendicular to the lens axis as shown in the Fig. lE.lSl.i
The separation between the lens and the'mirrof is 0.8 m. A.
smallobject isplaced outside the tank infrontof the lens at aj
distance 0.9 mjrom the lens along its axis. Find the position 'A quarter cylinder.of radius R and refractive index I.S aIs,
(relative to the lens) of the image of the objectformed by the placed on a table. A point object P is kept at a distance mR>
system. 'from it. Find the value of mfor whicha rayfrom P will emerge'
0.9 m 0.8 m
parallel to the table as shown in Fig. 1E.157.

n, = 4/3 !'Normal
Normal
n, = 3/2

N-mR >1**1 h«-,mR >!• >1


Water K K

(a)
Fig. 1E.157

Fig. 1E.156
' SolutljOn:'' We will consider'two reffactions'at plane
Solution: Let us trace the path-of light from the Surface 3, and curved surface 82- Refraction at Sj,' '•
object. First it is refracted at surface (air to glass), next
V ;:y
refraction takes place af surface $2 (glass to water), then Vi (-mR)
reflection at the mirror. Where does the light go after being . y = - nmR
reflected from the mirror ? It is reflected back and refracted The image is virtual and on left of surface S^. It acts as
through surfaces S2 and once again in the order of their an object for refraction at the curved surface
appearance. First we will apply general thin lens equation
for a lens. The given parameters are Refraction at S2:
u = -0.9m,"ni = 1, 02 = 3/2 = -4/3 1 " _ 1 ~"
^n-, -n^ Mr, - n- i>2- -(nmR + R) (—R)
!h. +
' V uu R ' 'R ' ' n , 1 - n
or

But R is not' given f we will determine it ffdm the lens (nmR + R) (-R)
maker's formula ; "' ' ' • ' • • > •• - ' as emergent' rays are parallel to' principal axis u2 = ««•
Un — n^ On solving for m, we get
Ro
' 1• _ 4
m -
n^-n (3/2)^-(3/2) 3
GEOMETRICAL OPTICS 133

+
(-20) (.-x)
20x
-A thin biconvex lens of refractive index 3/2 is placed on a V = -
horizontal plane mirror as shown in Fig.lE.158. The space X + 20
{between the lens and the mirror is filled with a liquid of A virtual image is formed due to first refraction at the
refractive index 4/3. It is found that when a point object is lens. This image is an object for reflection from the concave
placed 15 cm above the lens .on its principal axis, the object mirror. Object distance is
^coincides with its own image. On repeating with another 20x \ f25x + l00^
liquid,- the object and the image again coincide at a distance 5 +
20+ x x + 20 J
\25 'cm from the lens. Calculate the refractive index of the'
iiquid. From mirror equation.
1 (x + 20) 1
—+
1 nummWmuiirffmmimii v' 25X + 100, " -10
Fig.JE.158 _ x + 20 10x + 200-25x-100
-L = - i
v' 10 25X + 100 250(x + 4)
Solution: The light retraces its path if it is incident
normally on a mirror. The rays after refraction through the 50(x + 4)
V = —•
•lens and the liquid are parallel. We will apply the general (3x-20)
thin lens equation with parameters, This image is formed to the left of the mirror.
ni =1, n2 = 3/2, n^ - 4/3, u = -15 cm and v.= ~ Object distance for second refraction through concave
n 1 _ (n2-"i) ^3 ~ lens.
u R R 50(x + 4)
W = - 5-
(3/2)-1 [(4/3)-(3/2)] (3X-20).
~ (-15) R R
We assumed that second image lies between lens and
On solving for R we get R = 10cm. Similarly, when mirror.
second liquid is filled, we have The final image is produced at the object itself; hence
" ' "3 " 1 (3/2)-1 v" = +x
oo (-25) 10 .10.
From lens equation, ,
On solving for we get, = 1.6 1 . 1 • 1

[5-^
I- . 3x- 20
Adiver^ng lens, focal length f =.,^0 cm, is separated by 5 cm\ On solving for x, we get
'.from a converging-mirror, focal fength .f2 =10, cm. where'' 25x^ - 1400x - 6000 = 0
Ishould an object be placed so that a real image is formed at
^ ,\x^-56x-240-p'
khe object itself ? \ ' 1
. (x-60)(x +4) = 0
Hence x = 60 cm.

The object must be placed at 60 cm to the left of the


diverging lens.

•70cm •H-25 cm-H


N 40 cm
A parallel beam of light is incident normally on the flat:
Fig. 1E.159 • suiface of a hemisphere of radius R = 6 cm and refractive
index 1.56. Assuming paraxial ray approximation,
Solution: Let the object be placed,at a distance x to (a) determine the point at which the beam^ is focussed as
the left of the lens. • ^• - ... measured along the axis jrom the curved surface;
From the lens equation. \(b) determine the new focal length measured from the flat
surface if the rays are incident at the curved surface.
134
OPTICS

fa) the image formed by lens as rays travel to the right/(b)i


khe image formed after rays reflect from mirror and (c) the
final image after leftward travelling rays once again pdssl
[through lens. Complete the ray diagram and characterize^the
dmage. ^ ;j 1' i
Fig.1E.160 (a) Solution: (a) From lens equation,
111— y = +20 cm
Solution: Rays passwithout deviation at flat surfaces. V (-20) 10
From single surface refraction equation for surface S2, we +20
have Magnification, = —= = -l
u ^-20
1 n (1- n)
V 00 (.-R)
Image is real and inverted, same size as object. • '''
R (b) The first image acts as object 'for concave mirror.
V -
n-l 1.56-1 Object distance for mirror is (40 - 20) cm. ' ''
From mirror equation,
^ =10.7 cm
0.56 1 1
, v' = -12 cm
This focal point as rays travel from left. v' (-20) (-7.5)
From single surface refraction equation at Si, we have
n _ 1 _ (n~l), Magnification, m2 ~ ~ ~
V 00 R
nR The second image is 12 cm to the left of the mirror, real,
V =
erect (that is reinverted).
n-l
(c) The second image acts as objects for the lens. The
object distance for second refraction at the lens.

' • ' ' Si' ' S2


! K-20 cm 40 cm
; _ _ Fig.1E.160(b) , ,
Fig. 1E.161 (a) ^
This is the first image, it acts as an object for refraction
at«the plane-slirface. ' ''u" = +28cm
••Object disfahc^ fofrefr'actioh at S2l''' - From lens equation, ' ] I
> i . 1
"/'i • \ 1, ; • '•, •.u''-i=vl5.6cm * •

n ^ 1. . (n-l) u" (+28) . -10

So we have = 0
Note the sign convention for / and,u.
v' +,(R/n-l) .
Magnification, mo =—'= I_1^ = -0.556
V -
' u" I -fas
n (n -1) The final image is real, f ^7'+
6 inverted and lies 15.6 cm to Ray of light (Fi positive)
- 6.9cm
1.56(1.56-1) the left of the lens.

Hence focal point is at 6.9 cm.from the plane surface. Overall magnification,
m = mj X m2 X m3 Ray of light (Fg negative)
161 = (-1) X(- 0.6) X(-0.556) '";
Fig. 1E.161 (b) ^
= - 0.333--. .
'A equiconvex lens, fi =10 cm,-is placed 40 cm infront of a-
.concave, mirror, fz = 7.50 cm, shown in Fig. 1E.161. Aw
object 2 cm high is placed 20^cm to the left ofthe lens. Find the\.
location of three images : i
GEOMETRICAL OPTICS 135

Image is virtual, erect and magnified.


(b) The first image acts as an object for the convex
•A. diMerging lens (fj = 20 cm) is separated by 25 cm from a. mirror. Object distance for the mirror, u' = (20 + 5) = 25 cm.
'concave mirror (f2 = 20 cm). An object is placed 70 cm to the From mirror equation,
left of the lens. Find the focal length of the lens if the image' i 1 _ 1 , ^75
V = H— cm
coincides with the object. v' (-25)"+15^ 8
(+75/8) 3
Solution: The light refracts through the lens, reflects Magnification, m2 =
at the mirror and finally passes once again through the lens. (-25) 8
The final image will coincide wi± the object if it retraces its Image is virtual (to the right of the mirror), erect and
path after reflection from the mirror, i.e.the ray strikes the diminished.
mirror normally. The normal rays at the mirror after being (c) The object distance for second reft"action at the lens
extended must pass through the centre of curvature of the _ 75 ^ 115
mirror. Thus, the object for second refraction at the lens is at 8 8
C and its image is at I, see Fig. IE.162. From lens equation,
1 1 . _ 1
v" (+115/8) -20'
460
V = — = +51.1 cm
9
+460/9
Magnification, m3 =
70 cm k-25 cm-H I 115/8 J 9
40 cm
Overall magnification.
Fig.1E.162 m = mi X m2 X m3

From figure, PC - (40 - 25) = 15 cm = (2)x^ X


9
From lens equation, — — = —
(-15) (^70) / Hence size of image = —X 2 jcm = 5.33cm
/ - -19.1cm .. . . 13
Final image is to the right of the lens at a distance 51.1
1163 cm firom the lens, real, erect and magnified. .

A biconvex lens, fj = 20 cm, isplaced5 cm infront ofa convex


:mirror, f2 = 15 cm. An object df length 2 cm is placed at a
.distance 10 cm from the lens.,'Find the location of three A luminous object and a screen are at a fixed distance D
images: (a) the image formed by the lens qs the rays travel to apart, (a) Show that a converging- lens of focal length f
fhe right, (b) the image formed after the rays'reflectfrom the placed between object and screen willform a real image on the
mirror and (c) the final image <ifter the-lejtward travelling screenfor two lens positions that are separated by a distance
'rays once again pass through the lens.' ' - '
d = ^D(.D-4f)
(b) Show that
(D-dY gives the ratio of the two image sizes'
KD + dJ
for these two positions of the lens.
,(c) If the heights of two images are hj and h2respectively, the
height of the object is h =
(d) If the distance between two positions of the lens is d,
; Fig. 1E.163 -" • '•
'D^-d^
f =
Solution: (a) From lens equation, 4D

I'"' 1 1 — '' d
— , V = -20cm .and f =
V (-10) +20 , rhi —m2

Magnification, m^ = ^ =+2 where mf and m2 are magnifications in the two positions of


' (-10) the leTis.
136 OPTICS

Solution: - Let the object distance be x; then the Hence rrii - mo = —


image distance isD~x. /
From lens equation, —h—-— = —
X D~x f or / =
mi - m-

On algebraic rearrangement we get


x^ -Dx + Df = 0 [v;^;«gnn>j3jje 11651
On solving for x, we get
A thin plano-convex lens of focal length f is split into two
D-VD(D-4/) halves. One of the halves is shifted along the optical axis. Thel
Xn =
[separation between the objectand imageplanes is 1.8 m. The
magnification of the image formed by one of the halflens^ is,
Xo =
_D +^DW-4n '2. Find thefocal length of the lens andseparation between the;
'two halves. Draw the ray diagram for image formation.
The distance between the two object positions is
d = X2 - Xi = - 4f)
(b) If the object is at u = Xj,
D-Xi
mi = — = -
0 1.8m-

Fig. 1E.165(a)
Now, Xi = - (D - d), where d = ^D(D - /).
2
_D-CD-d)/2 _(D + d] Solution: splitting of a lens in two parts does not
So, , mi = affect the position of the image. Each half forms an image at
(Z) + d)/2 'yD-dJ the same position but of reduced intensity. The previous
Similarly, when the object is at X2, the magnification is problem shows that for a fixed object and screen there are
two positions of the lens for which the image is formed at
X2 = —(D + d)
2 the same position. Let the object and image distances for the
I.2 —^2
two lenses be u,v and u',v' respectively. In accordance with
mo = — = principle of reversibility of light,
0
First lens Second lens
^ " _ D~(_D + d)/2,_D-d : Screen

(D + d)/2 ~D + d .
The ratio of magnifications is i .
m-2 _ jP-dyXD + d).^ 'D~4
mi Id + d)/(D - d) .D + d
' v^
(c) As mi = -d- = ^
-• O 'Ui -

and m2=^=^
O U2
- . Fig. 1 E.I 65(b)

u' = v and v' = u


mi Xm2 = •^1^2 -= -Lx-^
^1 ^2 = l
—-L V, ^ Hence u-\-d + v'=u + d-\-u = D
O'
D-d
or u =
Hence
(d) As d=VD(i'-4/) • D + d
Also V=D-u=
d^ -b.^;;-4D/ /'/ ,
V D + d , ..
As m = ^ = -—- = 2 (given),
- c-'4r,.-r. .• u D-d

~-'l) + a' 'D-d'-- 4Dd — = 0.6m


Also -1712 = 3 3
'D-d D+ d ' -d'
Hence li =i(D-d) =0.6m
2
GEOMETRICAL OPTICS 137

and v = —iD + d) = 1.2m Thus two inverted images are formed whose tips are at
2 /j and I2 respectively.
From lens equation, —- ^ Thus /1/2 = 2y + A = (2 X 0.1) + 0.1 = 0.3 cm
1.2 -(0.6) /
167
/ = 0.4m
Consider an equiconvex lens with one ofits surfaces mirrored,
(a) Determine the focal length in terms of refractive index, n;
and radius of curvature, R. fb) Consider a planO'Cdhvex lens
fApoiht object 0 is placed at a distance of 0.3 mfrom a convex;
of refractive index n and radius R. (c) Determine its.focal
\lens'(focal length 0.2 m) cut into two halves each ofwhich is length (i) if its plane surface is mirrored^ (ii) if its curved
^displaced by 0.0005 m as shown in Fig. 1E.166. Find the\
Iposition of the image. If more than one image isformed, findl surface is mirrored.
\their number and the distance between them.

I i15 A..i 2x6.05 cm II


•30 cm
•V'fH
' h
Fig. 1E.167
Fig. 1E.166 (a)
Solution: (a) Consider a concave mirror (same radius
Solution: As explained in the previous problem, of curvature as lens) separated by x from the lens. Let an
each'half lens will form an image in the same plane. The object be placed at a distance u from the lens.
optic axes of the lenses are displaced, « _1_
From lens equation, ...(1)
From lens equation, Ui (-u) I
= —, u = 60m The first image will act as object for mirror. Object
u (-30) 20' distance for mirror is Vj - x. From mirror equation,
From similar triangles 01^12and OP1P2, we have 1 1 1
+-— ...(2)
hh _ u + v (Ui-^x) fn V2
' -^-1^2 The second image will be an object for second refiraction
90 >. through the lens. The object distance"= U2 -'x.' .
or 'h^-2 - —(2x 0.05) = O.'Scm-
30 . • •• - -
From lens equation, —- ^" =A ...(3)
Thus the two images are 0.3 cm apart. •,,r, ,u_-(u2—x) /i
where v is final image distance. "
(Image Adding eqns. (1), (2) and (3), we get
•/ height) s
11111
- + - = — + — + • X + •
" " /l /m fl _Vi(.Vi -x) V2 (.V2 -x)
111
If we set X = 0, we get —+ —= —
V u F
u = 30 cm V = 60 cm
1
where i = _L Jl + —
Fig. 1E.166 (b F fl^frr
fl
• This result can be generalised; the optical power of a
Alternately, imagine two arrows (see figure) that act as composite system of lenses or mirrors is the sum of
objects for the lens. the optical powers of individual components,
•r; • ' m = -^ =• ———
Magnification, (+60) = - 2- n The optical power of the lens has" been considered
® u (-30) twice because light .travels twice through the lens
Image of height of arrows is before the final image is formed.
X = 2x(0,05) = 0.10cm
'1 _ 2(n-i)' J^^2
fl R ^ f^ R
138 OPTICS

1 2(n-l) , 2_2(2n-l)
Thus — = 0 [silvered surface is plane]
F ' R R R fm
R
F = Power of composite optical system,
2(2n-l)
1 = 1. A 1
(b) When the plane surface of a plano-convex lens is R fl^fm^fl
mirrored,
1 _(n-l) 1 _ 1 = 2x -hO
0.24
• fi R 'fm -
•D
as radius of curvature of a plane mirror is infinite 0.12

Hence
1 ^ 2(n-1) -hO The focal length of the equivalent concave mirror is> ;
F R
F = 0.12m = 12cm
R
F = The parallel rays will converge at focus of composite
2(n-l)
system as shown in Fig. lE.lOSfa).
(c) When the cuiyed surface is mirrored, •''' *
From mirror equation,
1 _(n-l) 2
1
fi R fn R
1 +1 = ,V=-12cm
V ~ (-12)
1 _ 2(n-l) 2^^
Note that first image after refraction is virtual, second
F R R R
image/asfter reflectionis virtual and final image/gis infront
F = A of composite system, hence real. n,"'

I I '

• In case Cb) the composite optical system will behave 169


as a concave mirror of focal length
R
F = - jThe diameter ofaperture ofaplano-convex lens is 6cmdnd its?
(2p-l) fraximum thickness is 3mm. If the velocity of light in the]
and in case (c) f = -A Imaterial of lens is 2x10® m/s, calculate the focal length of.
2n [the lens. , ^ j

168 Solution: Given thickness of the lens at the centre


t = 0.3cm.' If R be the radius' of the spherical surface of the
A plano-convex lens, refractive index n = 1.5, radius o/ lens, then
curvature R = 12.'cm, has the plane surface silvered, (a) At C2F-0t =
what distance frorh the leris will parallel rays incident on the - or 2Rt-t^ =yl.
convexface converge ? (b) Determine theposition of imagefor
an object at a distance 20 cmfrom the lem. As t'<<R, '''2Rt~y^' "•
/ i
1
*
i

b '3' 1 li '3 '1 1 I2 •


2R-t
i

1
(a) (b) 1

Fig. IE.I68 ' 1

^ig. ^E.169
Solution:- From thin lens'equation,
-2
and y. (3)'
R = =jl5cm
, • , \i2 oo .,. / .2t 2x0.3.
If |i is.the refractive index of material' of the lens, then
^ ' '3x10® ;'J
2x10®
GEOfVlETRiCAL OPTICS

Now by lens maker's formula, we have Solution: For a convex lens forming real image, we
1 ^ have
y=C^L~l) 1
J +
+V -u f V u f
rj_ Here u + v = l (fixed)
U5 00^
••11 o
/ - 30 cm. + - = -=su^-ul + lf = 0
l-u u f
1 . 1

An observer holds in front of himself a thin symmetrical Similarly, we have


'converging lens; a is the numerical value of the radius of
Icurvature of each face. He sees two images of his eyes, onei - + - ^ = -^v^ -vl + lf = 0
,erect'and the other inverted. Explain the formation of these V l-v f
'images and assuming the refractive index of glass to be 1.50, I . 1
prove that he will see two erect images if the distance of the'
Hens is less than 0.25 a.
Since u and v, cannot be equal, we take
Solution: First image seen due to "
reflection at first surface of the lens
since'thin surface behave as convex I 1
mirror so the image seen is erect. i
Second image seen due to reflection at
"'--2-2^
I 1
second surface of the lens. Since the = . ••• •

lense is behave disk a lens sivered at Fig. 1E.170 '


I 1
back surface work as a concave mirror ' '
and if the objection placed within the focus and pole then
the image will be erect or if distance of eye is more than 2F
then enverted image is seen. Now, mi = ^ = —L = -e—^
For reflection from see surface
2 2 , , : . . I • • '
= +4 =2(n-i),-+- =-?c2ji-i) •.
' • a a a ' L+l^l2_4lf

nio —
'^2 V2 _:2. .2' . . . . V.
F=-~= —
P 4p-2
/.V....- - \ . - . - • >2" 2^ ' ' •
To see erect image distance between eye and image
should be less the effective focal length F. We'see tliat,'-''mi; m2 =1' " ' ' \ '
=> |u| < |f| . - • • =>
hi' h'2' '• •
-±.-± = 1
a \ h h
u <
4[i-2 h= •h2 = -J2x'8'^_ V16
a h = 4mm .- •
u <
Q
—^ = - = 0.25a
6-2 4
4X--2
2

u < 0.25a i . • . (
[A converging lens and a converging mirror are placed withl
Itheir principal axis coinciding. Their separation equals 40cm.|
jApoint source Sis placed on the principal axis ata, distance of
lA thin convergent lens is placed 'between an object and a' •.12cmfrom the lens as shown in thefi^re: It isfound thatthej
•^screen whose positions are fixed. Tfiere are two positions o/j ifinal beam comes out parallel'to^the principal axis. Focal
Ithe lens at which the sharp image of the object is formed on; •length of the lens equals 15cm. Find the focal length of the\
'the screen. Find the transverse dimensions of the object if at imirror. ; !
[one position the transverse dimension of the image' is
.hi = 8mm and at the other /12 = 2mm.
OPTICS

Fig. 1E.172 (a)

Fig.1E.173
Solutioni Refraction from lens:
u = X
12 cm
f-j=15 cm V = ?

R = 15 cm

Pi = 1 (air)
3
40 cm 1^2 = -
2
Fig.jlE.172 (b) [£2 Pi _ P2 "M-i 3 ^1 _ 1
1_1 V u R 2V X ^ 2x15

/l V u

2V ~ X 30
- 15 cm, u - -12 cm]
15 V 12 For accord refraction
1 Uj = -60cm u = V
15 12 "i; V = ?
Reflection from mirror: R = - 25cm
Ui for mirror U2 - -100cm 3 '
if finally rays coming parallel to principal axis then the
object should be at focus between mirror and lens. Then 4
image created by mirror will be "' = 3
V2 = - 25 cm from the pole of mirror , , 4_3
' 1 ' 1
^ 3^3 2 3 _ 1
for mirror , —; = —+ —
u V ar 2V -25 2V ~ 150
J^_ 1 +u ' " ' • '1- 1 '"i » I f

^ . —— + — = X = 25 cm
/, ~ 100 25 " •100 - - . V
X *50
30 '' 15(7 ' '

/m =-20cm To coincide the final image to the object the reflected


fay must retrace the path'
J173 t!l> ^ V = 00

—il-r ^ 174
•A thin bi-convex lens made up of glass of refractive index —is •
2
placed in front of a plane mirror. The space between the lens, A convex lens is held directly above a coin lying on a table and
4 forms an image of it After the lens has been moved vertically
jOnd the mirror is filled with water of refractive index —.
a distance equal to its focal length, it forms another image of\
The radii of curvatures of the lens are Rj = 15 cm and the coin equal in size to the previous image. If the diameter of
'R2 = 25 cm Apoint object is placed at distance x jrom the the coin is 16mm, what is the diameter in millimeters ofthe^
surface whose curyature is Rj. The distance x is greater than' image? ' • •
,the separation between the lens and the mirror. Find the value]
lofxso that image must coincide with.the object. Solutibh:'' Magnitude of magnification remains
unchanged ^ ^.
Solution: For first refraction
GEOMETRICAL OPTICS 141

Solution: 1-^ =1 V = m^u

O'
V 25 f
1 1 „ 1 1
_L-i
xo Xo+f or — ...CD
v - 25mi 25 - / 25mi 25 ~ /
(a) Similarly, in second case
Fjg._1^75 1
...C2)
/ / 40 m2 40 /
m = - +.

/ + C-^o)| / + (-^o-/)| f~XQ Xf nil ~ 4m2 -.(3)


±Xo =/-Xo From eq. (1), (2) and (3) we get
either / = 0 or Xo = f/2 m2 = 1, mi = 4, / = 20
f image
m - = 2 = i«^J 177
f-f!2
t^image = 2 X = 2 X16 = 32mm '•A lens isheld directly above a coin lying on a table andforms',
an image of it. After the lens has been moved vertically at
175 distance equal to its focal length, it forms another image of
the coin equal in size to previous image. If the diameter of the.
'i4n equiconvex air lens (R^ =R2 = 10cm) is made in an coin is 4.0cm. What is the diameter of the image?
lextended glass medium having refractive index CM- = 3/2).!
{Find the refractive index of the material to befilled in, so that] Solution: object distance = 2u. Focal length = / and
the power of the lens changes without change in its image distance = v
[magnitude. _ _ _ _ _ ' By lens formula ^ i-i+1
f V U f V u
Solution: when no matter was filled, it behaves like l^u-f
an air lens V =
V Uf u-f
1
V f
Ui \—1
R2 So, mi = - =
u u- f
...(1)

'1 ^ '2
—+1 If lens has been moved by / vertically down, then
V^i .3
Object distance (u') = -(u - f). Focal length = /and
f = -15cm, divergent lens of .focal length 15cm Image distance = v'
in opposite case it will behave as- convergent lens of ''1^1-- -1--- :
15anfocal leng^. , ^-'r v'^\u-fj
Let |i^, be the refiractive index of the, material to be u-f^f
fiUed. /Cu-/)
1^1
-1
/("-/)
JJi RjJ 1,3 jUO 10, ("-2/)
2 So, ...(2)
m^ —
'Hm = 2 (u-/) (u-2/)
15 1,~3 10 10
mi = - m < ...(3)
176: ./ /
(u-/) Cu;-2/)
When an object is placed at a distance of 25 cmfrom a concave, u - 2/ = —u + /.
mirror, the magnification is m^. The object is moved 15 cm'
2u = 3/ ' • •
farhter away with respect to the earlier position,'and the,
3 .
magnificationbecomes m2.ifmi/m2 = 4^nd thefocal length ...(4)
of the mirror. (Assume image is real mi,m2 are numerical', "=2^
values) !
OPTICS

Put u in (1) from (4) coordinates of image formed by lower lens = (135,-0.75)
v = 3f ...(5) Magnification due to mirror = =—
m = - = 2 15 3
u
.*. Coordinates of final image = (110, - 0.5)
I = 2x4= 8cm

b-—-- - r I jgffJ179
j 178
An equi-convex lens offocal length F in air is cut into two;
'A lens havingfocal length 30 cm is cut along a plane paralleh \halves along the plane perpendicular to the optic axis. One of,
•to the principal axis of the lens at a distance 5mm above{ the halves is placed between a fixed object and a 'screen',
lOptical centre 'A' and upper part of lens is shifted by 5mm\ separated by distance of 90 cm. Two images are formed on the>
•from thex-axis as shown is thefigure. Apoint objectis placed] screen for two dijferent positions of the lens- -with'
'on the principal axis of the mirror at origin 0 (0,0). Find the- magnification 2 and 1/2 respectively. Find the value-pf F.
'number of im^es formed just after the reflection from the' What would be the value of the focal length if the lens is]
•mirror and write their coordinates. i immersed in water ? Refractive index of the materialpf the]
lensjs 1.5 and that ^ water is li^ = 413. '
f=30cm f=30cm Solution: Let focal length of each part be -• , =
=> f = 2F
•A
!0
5mm — = 2=>v-^=2u^
; p.o)-. 5mm "i

Ui + 2ui = D
90cm 30cm => "i = •b'/S i' '
i - Fig. 1E.178' U2 1 • -'
' n.2 2
Soiutioni Image formed by upper lens also, : V2+U2 = D
1 1 1 . ,
— =^Ui=45cm • as U2 =Ui+ 30
(-90) 30 ^ UV.+ 30
^ H tUi +30 = 15
This will act as an object for mirror' > 2
I •• I '-I ' => ' .3U-, + 90 = 2D ' • - . - -
— + ^ ='-^ = -10 cm . '• I
i;2 15 > 30 ,• - => D = 90 cm (in the above discussion, we used
coordinates of,image, (110,-O)/. . .. the absolute values of the image + object distances)
Image formed directly by,mirror. ,,, , , Applying ' ...
1 L =zl ' ' 2 vi;^_r .
V 120 30 ,• f • ^
-3 _ 1
1-Zl J_ - • Ui^=?- =—=36x:m ' "
u ~ 30 120 120 ~ 40 3 .. 3 . >•
u = - 40cm Vi = D-Ui = 90- 30 = 60cm
coordinates (80,0), , ^ 1-1 1 _ l + 2_ 1
Image formed by lower lens; ^ • / 60 (-30) ~ '60 ~ 20
1 => -' / = 20 cm
= — => u. = 45 cm
"-30" . ,
f = //2 = 10cm
This acts as ah objects from mirror. ' In water, the focal length is
1 ' "l ' / \

—^ = -10cm 1
iL - ^X
1 -7 1
.-<l-5-i 30
7" /
• • • 45 1
Magnification due to lens = —
90 2 - = (p-l)
R. 40
GEOMETRICAL OPTICS 143

1 ^
(1.5-1) ...(1)
Ri R2 40

1.5x3 ^ Jl 1 A hollow equiconvex lens made of a very thin glass sheet has\

L^i A
...(2) one ofits curved swfaces silvered. It converges aparallel beamj
of light at a distance of 0.2 m infront of it. Where will itj
From eqn. (1) and (2) ^ converge the same rays iffilled with water, refractive indexl
4.5
-1
40 ."w=4/3?___ I
Solution: As glass is thin, we can ignore its
/water =4x40 = 160cm refractive effects. The lens behaves as an air lens with a glass
boundary.
1
-^ =(1-1) = 0
A concave mirror of focal length = 20 cm is taken for some fi (-R)
,e:icperiment. An object is placed on its axis at a distance of If R is the radius of curvature of the curved surface,,
'30cmfrom the mirror. Where shoulda plane mirror beplaced^ J_ 2 R
,so that the image of the above point isformed at a height of fm =
fm R
'20 cmfrom the principle axis and at the centre of curvature of
the mirror. Consider'one refraction from concave mirror and', With air inside, the system will behave as a concave
,one reflection from plane mirror. mirror of focal length.R/2. For parallel incoming rays, from
mirror equation we have '
Solution: ~1.'+ —1 = —1 => w
V, = 60 cm
Vi 30 20 =1. f =_o.2m
(-0.2) — fj-'"'
The first image of the object is formed at a distance of
60 cm from the pole P, on the same side of the object. Hence, R - -2/^ - 0.4m -
If a plane mirror is kept at C, perpendicular to the When water is filled in the lens.
principle axis, after reflection from the plane mirror, the
-1 = —D
image is formed at a distance of 20 cm infront of the mirror 0.4 (rO,4), \ 0.6.-
i. e. at the focus of the spherical mirror
Power of composite system,
1 1
+ —
•.'fl-'..-., .rr
-• •- -.5 .
0 F = 2x
0.6 ^-0.4 '0.6
^ 20cm
Hence, F = -0.12fn = -12crh - ' ' ''
30cm
For parallel rays we have ' • '" •' '
; Fig. 1E.180(a)_ _ ;
= -

When the plane mirror will Be rotated by 45° along the V -12 '

axis of the lens the reflected rays and hence the final image ' -i> = -12cm
will be rotated by angle 90°, and hence, the image will be
The final image is formed at 12 cm infront of the lens.
rotated by 90°
Illustration 13: Figure shows a mirrored lens place at
.•. The final image will be formed as shown in the IE.180 (b)
10 cm below water. Find position of object such' that its
, I
'X
•s,
image concider with the object.
t
20 cm
>u
' 20 cm
* . r *0
'• \p
, h2cm
-^ * E

' 20 cm C 0 l2.*F 1 Fg=3/2'


R=30cm.

Fig.1.221
f \

1 Fig. 1E.180 (b)


In this situation if after refraction through water then
refraction at curved surface of the lens the light is incident
144 OPTICS

normally on the mirrored curved surface light will retrace its


path and according to principle of reversibility of light the
final image will coincide with the object.

30cm

Position 2 Position 1'

Fig.1.222

We solve the problem after the light reflects from mirror.


Let image forms kt v after event 1. • •

> h
14cm', .
event 2 40 cm 20 cm
R = 20cm

24

levent 1,

The are two cases possible


(i) If final image is formed at. centre of curvature of
Fig.1,223
mirror.

(ii) If final- image is formed at pole of mirror.


--3-
3 2 Due to presence of slab the image of lens shifts itowards
3v 2x30

•' u = 24 cm
-30 . •
right by,t|^l-—j =2cm. Thus required distance is 102 cm
Let image be formed at v' or 82 cm.. . •

•••.' I--'-'' 4
v' 3x12- . .

or - 36 - 9cm
v = — . '-- • [ The cony^ suiface of a thin concavo-convex lens ofglass of
4 • ' rrejrac0e .index, 1.5 has ,a'zrddius of curvature 20„ cmi The\
Illustration 14: [concave surfcice, has a radius of curvature 60 cm. .3^e convex
[Side is silvered and placed' on a horizontal surface [see Fig.

o. " ' /V • \ \lE.182(d)]. •' •


0 I
.

40 cm \/\ / 20xni
. 'V*
t
• ,J
f = 20cm ^i = 3/2 R = 20cm
''.r,.- t = 6cm ..
r,«
r < Fig.r1424^,
F
.»"<-• U.J
I • • r-{ . -r.
-Lz frr>m lone

a glass (a) Where should^ a pin.byplaced on.the optic axissuch that


nurror imagey'fqi^ed atthe>dme place ? ____
is formed-at pbtject iteelf.'What is/are.separation of object^',
from mirror.- ^ ' ' . ' • ' •
145
l^pTRICAl OPTICS __ _
For the refraction at the upper surface of the lens,
\(b) Ifthe concave part is filled with water of refractive indexl = ? X2 = 20, R = +60
^4/3,find the distance through which the pin should be moved, ni=1.0, n2=l'S,
iso that the image ofthepin^ain coincides with the pin ?_ J (X2 =+20 ensures that the rays fall on the silvered face
normally).
./Solution: (a) The optical arrangement is equivalent 1.5 1.0 1.5-1.0 1.0 . 1.5 0.5 _ 3.0
HrMorH
to a- -concave mirror of focal length F given by 20 Xi +60 X-, 20 60 60
1= +-L where A is the focal length of the lens
or - 15 cm
F h L h
without silvering and f^is the focal length of the ^tror. Method 3: We use lens maker's formula and the
. 1 1 1
equation — •
0 .
R = 2f
/ ^2 ^1
The given optical arrangement can be visualised as a
--F convex lens of focal length 60 cm and a concave mirror of
T
f
R
focal length 10 cm kept in contact as shown in the fi^re.
R = 60 cm
R = 20 cm

Rg. 1E.182 (b)


2F = 20 cmi

R = 60 cm

1+20, +60 60 R = 20 cm ^R = 20cm

/g.T,60cm Fig.'lE.182 (d)


and 20/2 = 10cm
iJi+i+i=A If the rays'fallnonrialiy on the mirror after refraction
'F. .-.OO/ 10' '60 60
through, the lens," they will retrace backwards and meet at
• the pqiht' of the 'pin again.
.F = —= 7.5cm
or
81
'• •' 'For the. lens,
• Xi =?
For the. image to be formed at the place of the object,
X WR = 2F = 7.5 X2 = 15 cni" X2 =+20 (for normal incidence on the mirror)
' Ho til ^2 - Hi / =- 60 (using cartesian-coordinate sign convention)
.Method 2 : We use the.relation = — t -R - _ . . .
^1
: -60 +20 Xi
For the ;object_and the image tq coincide, the rays fall
normally ohthe' reflecUng surface';•i:e., on tlie silvered face or x^ = 15 cm
of the lens. (b) When the concave part is filled with water of
0'9 refractive index-4/3,the optical arrangement is equivalent to
a-concave mirror of.focal length F such that
1 1 1 1 ^ 1 ^
1 F— +
F fw fg frn fg Jw
2F = 20 cmi

where
L (3 ~J 180
or /^= 180 cm
(Rays fall.normally and fg = 60cm (calculated earlier)
.liet,X2 = 20 crn "/,/ v' oh tfte mirror)
1 1 + + —= 26

. ,Fig.1E.182 (c) F 180^60^10 60 180 180


^ 180
F = cm
Then the rays. r,en:ace backward and meet at the object .or
26
point again (opticai^.reversibility).
146 OPTICS

1 Sf) 180
Now, Xi=R = 2F = — x2 = — = 13.85cm A .
^ 26 13
Q
Aj: = 15.0-13.85 = 1.15cm
0.6 cm
Method 2: We use the equation = ' . . I
X2 R

For refraction at the interface 1' (air-water) •30 cm- •20 cm-
4/3 1 _ 4/3-1 Fig. 1E.183
... CD
X2

The image of interface '1' is the object for the interface Solution: The image formed by the lens acts as an
object for the mirror.
1.5 1 _ 1.5-4/3 From lens equation,
+20 +60 111
= —, i; = 60cm
u (-20) 15
or = — = 13.85
^ 26 - Image is formed 60 cm to the left of the lens.
V (60) _
Ax: = 15.0-13.85 = 1.15cm Magnincation, m = — = = -3
® u (-20)
Method 3: Using lens maker's formula and the relation
1_ . Height of image = (-3)(1.2)'= -3.6cm
F X2 Xj ' The image formed is real and inverted.
= 180 cm (using lens maker's formula) According to' sign convention,- all the distances
longitudinally and laterally are measured from the optical
/„ = 60 cm (using lens maker's formula) centre or pole of the refracting or reflecting surface.
® . t ^ ' t / . .
• ^1- • 1 — -
•1
= (for the water lens) ...(D The first image acts as a virtual object for the mirrpr.
-180 X2 Xi

From mirror equation, 1+1
— —,='1
— ' '
and —3^ =— — (for the glass lens) ...(2) ' ' • y' u' f
-60 +20 Xi - • 111
_ +
(The image by the water lens is object for the glass lens .v' (+30) (-30)
and if the image by the glass lens is at +20, then the rays will
t)' = -15 cm
fall normally on the mirror)
Adding equations (1) and (2) Magnification m = = 0.5
^ • u' (30)
+
-180 -60 20
Note that A^B^ is object for the mirror.
1 fin
One part (BBj = 0.6cm)lies above the principal axis and
or Xi = — = 13.85 cm, • the other part (A^B = 3ctn,),lies below the principal axis.
13^
Height of partBjB' (i.e., above principal axis.)
Ax: = 15.0-13.85 = 1.15cm
= (0.6>x'(0.5) = 0.3 cm, '
183 Height of part B2A' (i.e., below pripcipal axis)
= 3x (0.5) = 1.5cm
A convex lens offocal length 15 cm and a concave mirror of
focal length 30 cm are kept with their optic axes PQ and RS If we consider the .pole of the mirror as origin of our
parallel but separated in the vertical direction by 0.6 cm as coordinate system, the coordinates of B' and A' are
shown. The distance between the lens and mirror is 30 cm. Ah ^ respectively (15 cm, 0.^ cm) and (IS^cm, -1.5 cm).
[upright object AB of height 1.2 cm is placed on the optic axis. Total height of image A'B' = (1.5 + 0.3) = 1.8 cm
PQ of the lens at a distance 20 cm from the lens and the'
reflection from the mirror, find the distance of A'B' from the.
pole of the mirror, and obtain its'magnification. Also locate,
.the positions of A' and B' w.r.t. the optic axis RS.
147
GEOMETRICAL OPTICS

I.= - A=
A
& 184 MJ CA+A"^^

Two thin convex lenses of focal lengths fi and f^ o.re^ Thus y-coordinate of tip of J2
separated by ahorizontal distance d(d </i and d< f2) and] = A-I2 = ^ 1-
A
their centres are displaced by a vertical separation Aasshoww (/i+A-d)J
in Fig. 1E.184. Taking the origin of coordinates 0 at the' (A-^)A
centre offirst lens, what would be the x and y-coordinates of
the focal point of this lens system for a parallel beam of rays\ (A+A-d)
coming from ^ _ _ i
^ 185

\A thin biconvex lens offocal length f =25cm forms a reaV^


image of a real object on ascreen. The object is separated by 5
•cm from the lens. The screen is drawn closer to the lens by a
'•distance 18 cm. Through what distance must the object be]
\shifted so that its image is again fomed on the screen ?_ ^
Fig..1E.184(a) i Solution: From lens equation,
1-1=1
Solution: The paralld rays will be focussed at the V u f
focal point of the first lens. The first image lies at I, at a fu
distance /j, from the origin. This image will act as an u = ... (1)
object for,refraction thrpugh the second lens. The object f-v
distance for the second lens, u = (/i Td)- Let the screen be shifted by Au and the corresponding
shift in object position be AuV >• • ' •
Ray passing through From eqn. (1), we have
optic axis of second lens
Ray passing through /(u-Au) • -- '
focus of second lens U + ^U = -— — . • •
f -(v —Av)

i.-.^_Optlcaxis of • ' ' " f(v - Av) A'


Au=.
"V. second lens / - (u - Au) / - V
.1^
Optic axis -f^Av
of first lens
(v-Av-fjiv-f)
-1
f^Av •"Au

Fig.1E.184 (b)
{v-f-f. A.C^-A.
f^Av , Au
T-f
From lens equation, —- 7 iv-ff (v-f)
V •' +C/i " d) '/2
f^Av =•-0.5 mm
V = (475)2
(/i+/2-d)
We have.neglected higher orders of Au. •
Hence, the x-coordinate of final image^2 is •
x = a + 'U—
'd+ACA-r.d)
-•—=
. :':V: ,

d(/i -ci) + AA \(a) Fig:iE.T86 (a) shows the-opticalaxis ofa lens, thepoint:
C/i+A-d)
•source of light A and its virtual iniage,A\'Trace 'the rays to'
•find the position of the lens and of its focuses.'What type 0/-
Imagine an arrow tip isat /j; its image from lens is the dens is it ? . ^ ;
final image. ,(b) Solve the problem similar to the previous one using Fig. '.
V I2 I2 ^1E.186 (b). .... _. . -i
Magnification, m = - = —
u 0 A
148 OPTICS

oA' oA to the lens as it passes through the centre of curvature of the


lens.
oA oA'

(a) (b) 'Fig. IE.188 (a) shows a luminescent point and its image
Fig. 1E.186 produced by a lens with an optical axis NjN2- Find the
position of the lens and its foci. •
Solution: (a) The paths of the rays are shown in Fig.
1E.186 (c). First draw the ray AA' until it intersects with the
principal optical axis and find the centre of the lens C. Since
the virtual image is magnified, the lens is convex. Draw the
ray AB parallel to the principal optical axis. It is refracted by ! > Fig.1E.188(a) I
the lens ,so that it passes through its focus and its
continuation passes through the virtual image. The ray A'B Solution: Case (i). If A is the source and B is the
intersects the principal optical axis at point the focus of the image, then the lens will be convergent. The position of the
lens.
optical centre of the lens 0 and its foci F can be found by
construction as shown in Fig. 1E.188 (b) (i). Object and-
image lie on a ray passing through the optical centre. A fay
'Ab,
parallel to the principal axis must pass through the focus of
the lens.
Case (ii). If B is the source and A is the image," the lens
is divergent. The respective construction is illustrated in Fig.
1E.188 (b) (ii). ^ '

(d)
Fig. 1E.186 j ,

Cb) The paths of the rays are shown in Fig. 1E.186 (d).

Case (0 Case (ii) -


Fig. 1E.188 b)
Fig. 1E.187 (a) shows ray AB that has passed through ai
idiyergent lens. Construct thepath of the ray up to the lens ifj
[the position ofits foci F is known. |
'Aconvexo-convex lens has a focal length offj =10 cm.Onedf\
',the lens surfaces having a radius ofcurvature ofR = 10cmis\
Icoated with silver. Consfrucf the image ofthe object produced]
•by the given optical system and determine the position ofthe\
•image ifthe object is at adistan^ ofu = 15 cmfrom the lens. [
Fig. 1E.187(a) , ' ^ '
Solution: The image A'"B"' obtained in the system is
Solution: The path of the ray is shown in Fig. 1E.187 shown in Fig.lE.189. Fj and F2 are the foci of the lens and
(b). Let us continue AB up the mirror, and A'B' is the image produced by the lens if its
to its intersection with the surface is not coated with silver.
focal plane of lens NN. The The image A'B"" produced by a concave mirror can be
beam of parallel rays after plotted, taking' into , account that ray BO, after passing
refraction in the lens so through the lens and being reflected- from the mirror
travels that .• the surface, will travel along OB"; here ZBOA = ZB"OA. Ray BC
continuations of the rays emerges from the lens parallel to the optical axis of the
should intersect at F'. Ray system and after reflection passes through F2.
F'O is not refracted as it Fig. 1E.187 (b)
passes through the optical ------ -
centre. Thus, ray CA passing to point A is parallel to F'O up
149
GEOWETRICAL OPTICS
Solution: Two cases are possible :
(a) The mirror is at adistance of d=/ +Rfrom the lens.

z
CM

Fig. 1E.190 (a)

The path of the beam parallel to the optical axis of the


Fig. 1E.189 system and the image of object AB are illustrated in Fig.
1E.190 (a). Image A'B' (direct and real) is obtained to full
The rays reflected from the mirror are refracted in the scale with the object in any position.
lens once more and produce image AV. Point B'" lies at the
intersection of rays OB" and CD. Ray OB" passes through the A
optical centre of the lens after reflection and is thus not
refracted. Ray CD can be plotted as follows. After the first
refraction in the lens and reflection, ray EC will travel in the
direction of and will be refracted in the lens once more.
Ray OD parallel to CF2 is drawn through the optical centre 0
until it intersects the focal plane of the lens. The sought ray
is obtained by connecting C and D.
Since the rays are refracted in the lens twice, focal — d—H

length / ofthe system can be found from Fig. 1E.190{b)

/ /i h
(b) The mirror is at adistance of d=/ =Rfrom the lens
[Fig. 1E.190 (b)]
where A = - is the focal length ofthe mirror.
2
The image of object A'B', also full scale, will be inversed
and virtual with object in any position.
/1/2 = 2.5 cm
/ =
h + 2/2 j E>?cimp!^ 191
Therefore, the distance v to image A'"B"' can be found
from the formula
A source of light is located at double focal length from a
convergent lens. The focal length of the lens is f =30 cm. At
what distance from the lens should aflat mirror be placed so
that the rays reflected from the mirror are parallel after
Hence, = 3 cm passing through the lens for the second time ?
Solution: The
mirror should be placed
between the focus and a •
2f
point lying on the
At what distance from a convexo-convex lens with a focal double focal length. The
length off metre.should aconcave spherical mirror with a path of the rays is
radius of curvature of R=1 metre be placed for a beam shown in Fig. 1E.191. Fig.1E.191
incident on the lens parallel to the major optical oxfr ofthe
system to leave the lens, remainingparallel to the optical axis,
after being reflected from the mirror? Find the image of the
object produced by the given optical system.
150
OPTICS 1
192 The position of the image is given by
Apoint object is located ata distance of100 mfrom ascreen. ^ = - f + at, at time t ...C2)
^ lens offocal length 23 cm mounted on amovablefrictionles's O

stand is kept between the source and the screen. The stand is .. The corresponding position ofthe object is given bv
attached to a spring of natural length 50 cm and spring (at time 0 o
constant 800 N/m as shown. Mass ofthe stand with lens is 2 i-l =lori = 11 •
kg. How much impulse Pshould be imparted to the stand so' V u f u v{t) f
that areal image of the object isformed on the screen after a-
^fixed time gap. Also find this time gap. (Neglect width of the 1 du .>.1.
1 dv
stand).
dt dt
Solution: Let the distance of the lens from the du __ u
J u i-

object be /when areal image is formed on the screen Then


- • 1 ^
or
dt v {a = ~(a) ... (3)

2
100-I -I 23 du
'dt~
a ... (.4)
On solving, we get 1= (50± I0V2) cm.
Now, if the lens performs
Screen 194
SHM and a real image is formed
after a fixed time gap, then this
time gap must be one-fourth of •A convex lens offocal lengthf is mounted on astand of total'
the time period. . p—
,mass m, which is connected through a spring with a fixed
f«:-50 cm- surface. A point object is placed on the optical axis at a
Phase difference between -100 cm .distance 3ffrom the pole. If the amplitude ofvibration ofthei
the two positions of real image lens is a' find the distance between the two extreme ends of
must • be, jc/2 As the two Fig. 1E.192
the vibration of the image.
posirions.are symmetrically located about the origin, phase
difference of any ofthese portions from ofi^n must be 7u/4. Solution: 1-1
V u
... CD
10V2cm = Asin- • A = 20cm
4 v-u = x;v = u + Xputting in (1)
To achieve this velocity at the mean position. 1 1_ 1
A
K u + x_ u /
Va=Aa) = AJ—
'm 1 -I-
- =l +l = 'i±l l«—3f-
B .
Required impulse p = mvQ= = 8kgm/s. u'+x U f uf
uf
Lfc Jiia; 193 u + x =
U+f Fig. 1E.194

Apoint object moves along the principal axis of a convex lens' X -


_ uf
—~-u = u /-•
•of focal length f such that its image, also.formed on the u+f L"+/
-1

principal axis at adistance ^f(at t=0) moves away from ~w


sJ r X -
Ithe lens with a uniform velocity, a Find the velocity of the u+f
poinyource as a function of time t. Differentiating w.r.t. x,
Solution:' Applying the 5x —
-•2u
du
equation for image formation-by " +/ (u+ff
a lens, we get
--2u(u + f)+u^' u^ + 2uf
V
1
u f
1^1
...CD
I1
. Fjg. 1EA93
(u + ff
>2
Su-= -

.
(u + f)
f2
du

-1 du
' f u+f
~f
3
' (.u+ff
u = -4/ du = a «|u|=|3/I
IGEOfllETRlCALOPTicS
/ 3
'(a) Find the coordinates of the final image formed by the
dx = -1 a - —
4
a system of lenses taking 0 as the origin of coordinate axes. -
-3/ + /
(b) Draw the ray diagram. _ ___ . .
Amplitude = 16x1= - a

195

Ia convex lens is held 45 cm above the bottom of an empty •2f-


tank The image ofapoint atthe bottom of the tank isformed Fig. 1E.196(a)^ _ J _
136 cm above the lens. Now aliquid is poured into the tank to
\a depth of40 cm. It is found that the distance of the image of Solution:The image will be formed atthe focus of the
'the same point on the bottom of the tank is 48 cm above the> convex lens after refraction through the convex lens (at J).
Jens. Find the refractive index of the liquid.
Now for the concave lens, I acts as an object.
Since the distance is always taken from the pole on the
!
36 cm
48 cm i
principal axis.
If itcan be assumed that the point Jis the tip of an object
erected on the principal axis of the concave lens, ^e image
of point f will be the tip of the final image (let it be J)
Distance of object = u =/cosa ,
40/m^
45 cm \ , 1 1_1
40 cm -
Applying lens fprmula, ~ ^ ~

Fig. 1E.195

Solution: From the given conditiqni •/ •


y = _ 45 cm, b = 36 cm
+ or'/ = 20cm _ Fig.1E.;^6(b)
/ 36 45
1 1 _A
Now let |ibe the refractive index of the liquid. Then in or
V -/cos a -/
the present case,.
y^ 5+ 12 1; =+48cm; / =20cm i=zi^1 1+
1+—^
/ cosa
-/ cosa.
or =.0.029 V -
l-+cosa , . ,
20 48 3^ , 40
- 5 + —
, , fcosa, , 1
\i- distance 01 -1 '— ^
Now
1 + cosa cosa.
. • f » t i - ^
or 5+12 =34.29 So x-coordinate of
/
r = 00'-07' = 2/-
— = 29.28 1 + cosa

. 2+2cosa-l I ^ l + 2cosa^
/ [1 = 1.365 ' 1+cosa 1 + cosa j
y-coordinate = 0 •
f/l + 2cosa j ^
!ln"the given figure there are two thin lenses of same focal So coordinates of image are 1/1 q+cosa"J
[length f arranged with their principal axes inclined at an
I

\anzle a The separation between the optical centres of the We can verify the result by putting a = 0.
denses is 2f. Apoint object lies on the principal cljos of the Coordinates after putting a = 0
convex lens at ajarge distance to the left of the convexjens.
Similarly w'e can determine d2.
jth
^ , 7
This result is right ifthe principal axes of both the lenses "2 —3--tA = -A = 0.7 mm
4 4
j'

coincide, then the image will be formed (finally) atx =^ Two real imagds Sj and S^ will act as two coherent
he
2' sources separated by a distance
197 L d = di+d2 - 2.7 mm
InFig 1E.197, Sis a monochromatic point source emittingl The distance of screen from S1S2, D=2-1 =1metre.
Mht of javelength X=500 nm It is placed at a distance} Point C is at a height x =d^ ~~ =0.65mm, firom
-.b - 2mfrom ascreen £. Athin lens offocal length f =16 cml
.15 cut into two identical half. They areplaced atadistances 2o\ perpendicular bisector of,SiS2.
xrn qnd 80 cm from S. The part Lj is shifted A=0.40 mm! ^ the path difference of the rays from S, ari^ 5,
iwhile I2 IS shifted 3A transverse to the line SC The sous reaching atCis zero, hence awhite spot will be formed at C
,betweenthe line SC and the lens parts arefilled by an opaque The white spot is observed at C, i.e., the shifting of'-ffinge .
'material as shown in the figure. Calculate : pattern is equal to x = 0.65 mm. Hence the slab should be
(i) where the image ofS willform,
(li) the refractive index pofa transparent sheet of thicknessl tilr^ns" emerging from the lower part of
t - Jmm to be placed in the path of the rays emergingfroml As shifting x=4):—
iOne of the parts ^^hite spot is formed atpoint C. I d

0.65 X10"^ = (H-l)x2xl6-^xi


-L,A I 2.7x10"^ .
L which givesp = 188
r

-b-

Fig. 1E.197 (a)


Aparallel beam of light falls jfoni vacuum on a surfacel
Solution; Both the lenses will form images of 5 enclosing amedium with refractive ind^^ n. Find the shape of
From lens equation, that surface.x(yX if the beam is brought into focus di'tUe
pointFatadistancef from thecrestO. What is the mmkmum
radius of a beam that canstill be focused-?
V u f
y
For lens L^, I- = L-± Vi - 80cm P(x.y)
Vi 16 20' T A

For lens L2, V2 =20cm f


T 0
V2 16. 80' -

Magnification, m =
-u

ForL 80-
mi = = -4 Fig. 1E.I 98
-20

h' Solution: The optical path for OFray must be equal to


di,
the optical path for APF ray.
Henc4' nf-~kyif^'xy+y^ '
20 cm i t-
(n^ - Dx^ +nV^ - 2nCn -1)^ =0
i.

•80 cm «— D - 1m-

_FigJE.197 (b) X =
+2n(n -1)/±74n^Cn -1) - 4(n2 _ :
Thus, the light rays converge above the principal axis of - - 2(n2-i)
ii ataheight4A,i;e., 5A above thelineSC. nf (n + lfy^
X =

Hence, dj = 5A = 5 x 0.4o =-2nim (.n + 1) Cn-1)2/2


2xl6yi
is the required equation 1-16V
For real value of x, ;•
2..2
(n + 1) y
-1+16V? =32(Xi -X) =32(8yf -X)
-<1
2 £2 X = -Lm = 3.16cm (whichis unique)
Xn-irf 32
(n-1)
y^f
(n + 1)
Hence, the position ofthe image will be 3.16 cm ifrom 0.
(n-1) -4
l-s 1200
So/.i" y max =/
(n + 1)
'Consider Fig. 1E.200 shown a hemispherical cavity ofradius-
199 'r 15 curved outfrom asphere =1.5) of radius 2R such that
the principal'axis of the cavity coincides with that of the
1pointobject is kept ata distance d=lmfrom aparabolic\ value sphere. One side of the sphere is silvered as shown. Find the
•eflectingrsuvface x=Sy'. An equiconvex lens is kept at a, \itself. ofXfor which the image ofan object at 0isformed at 0'
iistancej s=80cm from the parabolic surfa<x (see^
Fig.IE.199). The focal length of the lens is 20 cm. Find the}
position of the image after reflection from thejwface. •
' vl"" ' '•
^ A
Object

(b)
Flg.1E.200_ _
Fig. 1E.199 (a)
Solution: Refraction at plane surface (at A),
sblutibn: The only action of'convex Iras is to ma^e^ • " / ^ u = -(2R + x)
he ray after passing through the lens parallel td the x-axis.^
L.et the ray be incident at a point PCx^,yi)- , .j
1 \i. = 0
V —(2R + x)
- (2R + x)
or V = i = -C2fi

For refraction at B, image acts as an object.


X.: 2R + X
u = PI.^=-\R +
^i-l
Fig.1E.199 (b) R 1
2R + XM -R
R +
= tana =

V =
(1)
a = 9O°-0 andp = 29 , ^ ^^,,, .
The reflected ray is^passing through F(X,0) an^ ^as^ ^•R^ -(2R + x + jiR'),
slope-tanp. . - -r > Final image is formed at 0, when the rays are incident
normally on the silvered surface,-i.e.-, the object is placed at
(xi-X)">-'^"- r-,- . , - - ,
2tan0
= - tan 20 = - ••The image formed by the hemispherical surface of the
i-tan^0 cavity acts as an object for-the silvered surface, the image
_yi 2 cot a should be formed by the hemispherical surface atCi-
(Xi-X)' 1-cot^a i.e., v' = -(R +x)' -'^3)
[154
,OPTICS
From eqns. (1) and (2), we get
men liquid of refractive index pis filled to the right c
2;c +9Rx -8R^ = 0 (puttingfx =1.5) this lens, the first surface of the lens (radius of cu^^hire =
10 cm) forms the image at the object only. Consildering th^
or X= +Vl45R^ -9K +12R refraction at the second surface,
4 4
Q _p-1.5
or (••• same area oo
~ -10 10
4
p = 3

'A Parallel beam oflightfalh successively on athin convex Zens!


.0/focal length 40 cm and then on athin convex lens offocal' 'Anequiconvex lens offocal length 20 cm and refractive inde:
^length 10cm as shown inFig. 1E.201 (a). , •^512, IS cut into two equalparts and onepartL2 is silvered am
•placed as shown. Another concave-convex lens L (radii Wen
Screen
Screen And 20 cm refractive index =3l2)isplacedwith its principa
'O^ parallel to that of as shown in. the figure.^A poini
^object IS placed at a distance of 80 cm from the lens on its
50 cm 40 cm-» I*—50 cm— -40 crrr*
principal axis as shown in thefigure. Find the position of the
(b) fnal image.
Fig. 1E.201
^ 93.33 cm %
jnFig. 1E.201 (b) the second lens is an equiconcave lens of
i/ocaUengtft^JO cm and made of material of refractive indexl H—80 cm—i
•1 \ *
.1.5. In both the cases, the second lens has an aperture equal to "t'J
i/
\(a) Compare the area illuminated by the beam oflight on the . • L, /
screen wJpch passes through the second lens in the two cases. i I Fig.1E.202 (a) ,
Now a liquid of refractive index is filled to the right of the Solution: Due to refraction from the first'le'ns, th
|5econd lens in case Bsuch that the area illuminated in both image will be at 80 cm right and on the principal axis of th
\the cases is the same. • •\ ' lens (since its focal length is 40 cm).
6f the liquid. I A silvered lens wiU behave as an effective mirror o
power p = 2P^ +
Solution; in case (A), the incident parallel beam
emerges as a parallel beam. = 2l- 1
Aj = jr(l)^= Tucm^ fl fm fl R
In case (B), Let x be the radius of the area illuminated
Then • , • • 1

I I -

1 . I

"2
• N—80 cm 93.33 cm

! Flg^1E.202 (b)
For R (using lens formula).
10 cm

X r i? = 20 cm

A:
40 -20 20
^2 7-4^.(9)^,.= SlTucm^
- 1 20
Thus,
A.
- ^ = 81
=-p =-y =-6.67
Now image due to first lens is at/, which is in the plam
passmg through the centre of the effective miiror. Hence
due to reflection from the effective mirror, position of tht
"geoivietricaloptics_
imkg^ will be at the same position as but 4mm below the
principal axis of the lens L,. Again, due to refraction from
thelens, position of the image is above the object at4mm.
Hence, is the image due to refraction through lens
Ljand 12 is the image due to reflection through L2, and Jg is Fig.1E:203(c)
final image due to refraction through lens ii- For the lens L2, u = - 60, / = +15
203 1
w -60 15

Aconvex lens offocal length 15 cm is spUt into two halves and u = +20cm
,the two halves are placed at a separation of120 cm. Betweem u 20 _ -1
'the -two halves of the'convex lens a plane mirror is placed u' -60 3
'horizontally and at a distance of 4 mm below the principal
icDds^ofthe lens halves. -An object of length 2mm is placed at a Negative magnification shows that the AC of
'distance of20 cmfrom one half lens as shown in Fig. 1E.203. A'C by the lens is inverted as compared to A^C
(a):s: ; A'T'" =-. A'C = 2mm
: ' rr f = 15 cm > 3

Li 0"C"' —1
-1 => 0"C'" = —O'C" 2
= - mm
A O'C" 3 33
2mmt 1
20 cm
14 mm Final image is at a distance of 20 cm behind the
rnTTUUlUUUiUUUUUlUUlUUUmiinnUU second half lens and at a distance of (2/3) nim above the
• 120 cm- principal axis. The size of the image"is 2uhm'and is inverted
Ii , Fig. 1E.203(a) _ ' • as compared to the given object.
•'(b) Ray diagram.for the formation of iniag^. ^•
(a) Find the position"arid size of the final image.
,(b) Trace the path of tJierifys forming the irn^^ .,
^ ' . . * » T' A A"! i
A
• Solution', (aj For refraction at lens Li, "
A
'' ' ju u f
A-r'
V —20 +15
, Fig. 1E.203(d)
= 60 cm
V 60 _ o .
Magnification, ^ &4
Hence, the imageis inverted. •^ Vwo identical plano-convex lenses Lj (\ij-1.4) andj
AC = 2mm„AB = BC = i2fS)mm 1-2 fM-2 =1-5) of radii of curvature R=20 cm are placed os^
A'C''=—X 2= 6mm 'shown infig. 1E.204 [a(i)].
i t
,,..20 j^4.5 mm i)
A'B' = 4mm, B'C' = 2mm Li/ Principal axis ' \
The image is formed by the first half lens as shown in [ai of iens Li ! ;

Fig. 1E.203 (b).; i• A'

-C" t :s-.
/.'A / Lz A
A A't. B'

(i)
B' C
Fig. 1E.2b4 (a)
C A"
Fig.1E.203(b) ..

The image fonned by the plane mirror is A' C as shown


in Fig. 1E.203 (c)., .. •
[156
qPTitsl
\(a) Find the position ofthe image ofthe parallel beam oflight
.relative to the common principal axis.
(b) Now the second lens is shifted vertically downward by a
,small distance 4.5 mm and the extended parts of and L, Near a prism, a convex lens offocal length 12 cm and: an
.are blackened as shown in Fig.lE.204 [a(ii)]. Find the new object 0 have been placed as shown in the figure. The lens
•position of the image of the parallel beam of light relative to I aperture has a radius 1 cm. The refractive index of/the]
fhe principal CD^ ofL-^. material of the prism is Vs. The object is placed such that its
image formed by the lens is virtual. The rays of light, after-
Solution: (a) Focal lengths of lenses L, and L, are getting refracted through the lens, pass through the prism and'
respectively given by fall onface PR of theprism. !
mat is the greatest distancefrom the lens at which the object^
fi u
•/i = 50cm shmld be placed such that all the rays which are getting'
refracted by the lens andfalling on theface PR oftheprism-get'
/2 = 40 cm
totally mternally r^ected ? It is given.-that.
/2
The equivalent focal length / of the combination is
ZQPR =-^ +sin ^ 1 t/ic l&us is thin.
given by
i - 1. 1 r 200 .

\
7 - "T + — cm
f fl f 2 9 - . .
Hence, the image of the parallel beam is formed on the
common principal axis at a distance of22.22 cm from the
combination on the right side; ' •• •, > A
(b) Image formed by ij is ata distance of 50 cm' behind ° \lV
the lens. This image lies.on.the principal.axis of I,and will 90° ' ' \
act as an object for I2. n \
Q R

7K- Fig. 1E.205 (a)


CO
Solution: The critical ray is shown. The lens forms the
0
0

image of 0 at I. From the figure we see


r + C =P
••(1)
Critical angle for the prism is
HgJE.2q4{b) r- = sm
• -1'• —1
C
ForL2, objectdistance, u = +50cm Vs
/2 = +40cm and P = ~ + sm~^^
200 .6 Vs
V cm
V u _f 9 From eqn. (1), we get
Magnification caused by I2, r/ - ^
™ 4 .~ 6
m = — = —
sini
u 9 Now = V3
For i2, object./i is at a distance of 4.5 mm above its smr

principal axis.
Hence, distance of image /2 of the object (virtual) 12 is •\c/
at adistance - x4.5 =2mm above the principal axis ofI2 p?\ ' ' ••
" / 1 cm \
[ ••• height of,image = mx height of object].
I
Hence, final image is at a distance of22.22 cm behind
the combination at adistance of 2.5 mm below the principal
axis of Ij. , ' • f y ^
Q

Fig. 1E.205 (b)


"rGioiviETRlCAL OPTICS 157

Hence, sini = Vs-sin —=> i = 60® => V = 18cm to the right of the convex lens.
6 (b) If the mirror is rotated by 1°, the reflected ray rotates
, Image distance t; = 1* cot 60° = cm by 2°. The virtual object for the lens formed by the reflection
V3 from the mirror is displaced by :
.Now i_l = i Ay, - 50x -^cm
V u f u 12 ^ 180
-12 The magnification due to the refraction at the two
u =cm
surfaces of the lens is
I + 12V3
' u
12 m. — —I — Jl Jl
l"l = cm
.113/ \^2l i.^1
I + I2V3
V 18 / 40 IS
JL
1^1 (^3) 30

'•A thin equiconvex lens of glass C}j. = 1.5) having a focal length' The displacement of the final image is
\of 30 cm in air isplaced at a distance of 10 cmfrom a plane 18
— X 50 X
27C 7C
cm = — cm
^mirror, which, in turn, is placed with its plane perpendicular 30 180 3
\to the optic axis of the lens. Water C|x = 4/3) fills the space
'between the /ens and the mirror. A parallel beam of light is,
\incident on to the lens parallel to the principal axis.
(a) Find the position'of the final image w.r.t. the optical^ 'A hollow sphere of glass of inner and outer radii R and 2R\
centre of the lens. •'respectively has a small mark on its inner,surface. This mark\
;(b) Ifthe mirror is rotated by 1°, asshown in thefigure, find' 'is observed from a point outside the sphere such-that-the,
centre of the^sphere^ lies in between.^ Prove that the mark will
the displacement of the image.
'appear'nearer than it really is,'by'a distance ^ ,where'
Solutioni (a) The focal length of the glass lens is 30 (3^1-1).
cm. ^R is the radius of the inner surface.
Jl 2(11-1)
Solution: Refraction at surface 2,
••1 R-
•'^2 R
1£+ 1
The radius of curvature, R = 30 cm. u 2R • -J?
The object distance, u = °°.
or Cu-1) + T
Now we apply Gauss's law at surface and 82-
3 -(2LI-1)
-1
^_1_ 2 ... (1)
Vi u 30
or
4 3
3 2 _ -1/6 -...(2)
V v^ -30

=> u = 60cm
After reflection from the ' 10 cm

mirror, the light rays appear to pig. IE.2O6


converge to a point 40 cm to , ~ ~
the right of the convex lens. This serves as a virtual object for
, .Fig.1E.207
the lens : u =+40cm
3 4 1 1; For surface 1
.2_-_-^ = A ^ R +
2pR 4p-l
Uj +40 30 , u = — R
2U-1
3 _1 ' 1 ^p(2m--1)_1-^i
- 1 2 _ 2 V (4ji - 1)R -2R
V Ui -30 •
158 OPTICS

I The rays reflected from the rear face will be deflectediby-


R 2 (4^1-1) an angle 0 determined from the equations : M*'
sma sin(a + 0)
1 4ji-l-4p.+[i + 4p.^-2|j. = n and = n
sinp .sin(2a-p)
R 2(4^1-1)
When the angles are small, 0 = 2a(n-l). Fon.this
I 3^-1 reason the second image will-'>be at a distance
R 2(4ji-l), d2 = 2a (n -1)/ from the source. The total distance between
2R(4tr-l)
the images is d = dj + da = 2an/.
or V = - d
(3^1-1) Hence, n = as in the first case. r ;-
2a/ -
Distance between the final image and object,
9)^. 3 —8(.i + 2
(311 -D- 3^1-1 (3^1-1)
A concave mirror has the form of a hemisphere with a, rcidiusi
\ofR = 55cm. Athin layer ofan unknown transparent liquid^
[is poured into this mirror, and it was jbund that the given,
A glass wedge with a small angle of refraction a is placed at a 'Optical system produces, with the source in a certain position,
"certain distancefrom a convergent lens with a focal lengthf • •two real images, one of which coincides with the source and
ione surface of the wedge being perpendicular to the optical the other is at a distance of I = 36cm from it. Find the
joxis of the lens. A point source of light is on the other side of fefraction index n of.the
..
liquid.
^ ^ ^
\the lens iri its focus'. The rays reflected from the wedge-
'produce, after refraction in the lens, two images of the source
Solution: Since the image that coincides with the
source is formed owing to reflection" from the part of the
'displaced with respect to each other by d. Find the refraction'
mirror not covered by the liquid, the source is ob\dously
'index of the wedge glass. ' • • • ,
arranged at centre O of the hemisphere. Let us firid the
SoiutiOTir Two cases are possible-: position of the other image (point 'A in Fig. 1E'209).
(1) The optical axis'ofthe lens is perpendicular to the According to the law of refraction, ' ' '
front face'of Ae wedge. The fays reflected from the front sin a a' , . sind) (b
= n = — and - —^ = n = —
face pass through the lens and produce an image of the point sinp p sin0 0
source that coincides with the source itself. The rays As can be seen 'from 'the drawing, -0 = p + 2y, where
reflected from the rear face will,be deflected by an angle ^ 7 = a - p is the angle of incidence of the refracted ray on the
sin({)
[Fig. 1E.208 (a)] determined by^the equality - n. mirror and (R -1 - h) tand = (R- /ijtaria.. ^
sin 2a
Since the .angles are small, ^ = 2an.
The" second image of the source will be obtained at a
distance "d from the first image, namely, d = ^ '= f- 2an.
. . .

• • t

t » "c"

• (a) Fig. 1E.209


F!g. 1E.208_ .... i
Neglecting h as compared with R, we ca,n find from this
Hence, n = system of equations that
2af 2R-1
n = = 1.6
(2) The optical axis of the lens is perpendicular to the 2(R-0
rear face of the wedge. The rays reflected from the front face
will be deflected by an angle 2a [ Fig. IE.208 (b)] and
produce an image at a distance of dj = 2a/ from the source.
GEOMETRICAL OPTICS

210
J211J,
. narrow parallel beam of light rays is incident on a^ Find the position of the main planes of atransparent spherei^
used^alenj._ . '
ransparent sphere with aradius Rand arefraction ttidexn m\
he direction of one of the diameters. At what distance ffrom, Solution: Let
he centre of the sphere will^e rays_beJocmsed ?__ __J us extend ray BF until it
intersects the
Solution: Two — continuation of the ray
ases are possible: the s incident on the sphere
Dcus is outside the parallel to the optical
phere, and inside it. j axis (Fig. IE.211). It
.et us consider the | can easily be seen that
irst case. The path of j section DO that Flg.lE.2il
he ray. incident on the ' connects
connects the
me point
puun of
ui . ' . u 1
iphere dt an angle i is intersection with the centre of the sphere forms aright angles
jhowniriFig. 1E.210. ' with the direction of the incident ray. Triangle ODF is aright
Bearing in mind | Fig.lE.2i0 one, since-
that the angles i and r ' " ,„ R n 2i (n -1) _ I,;
OFB = - ^
are small; we have, in accordance with the condition of the 2 n-1 ri
problem,
BC-Rsma = Rs\n(.2r-i) For this reason the main planes of sphere-MN coincide
and pass, thrpugh'Its centre. . . . , / >'
= RC2r-0 = —C2-n) . r '
- • — n 212
The ficus obviously li^s^ outside the sphere when .h;<:2
and on,the surface ofthe sphere if n = 2. , Acam^rawith an objecdveofS dioptresfocal powerMitsedtoj
The distance
'itake photographs of an object lying on the bottom offl-pondl
BC 1.2 mdeep.What is the distance of thefilm from the objective. ,
CF = BC cotp s lens ? The lens is.placed close to the "
P
2i(n-l) , • Solution:' First one
and" P^2(i'-r) =
should estimate the apparent
• depth of the pond di(Fig.
as can easily be found from Fig. 1E.205. 1E.212). - ' '
The sought distance is di =lcotp, d = lcpta
_ tana
and .^.=-
d

tanp
When n>2, the path of the,ray is as shown in the For smah angles
igure. The sought distance is ;; tana = sina,^
/' = OF = CF - R dsiii'a'-' d
So di^=
As can be seen from the figure. sinp' n
Fig. 1E.212
AC Ri
CF - AC cotCi - r) = -
I - r i - r
From the thin lens formula,
R we obtain —+- =-• Therefore. the
di d' f .
required.distance
Hence, f =
n-1
between the objective lens and the film is d' =dfjid-nff
An objectAB is atadistance ofa =36 cmfrom a lens with al
focal length of f =30 cm. Aflat mirror turned through 45°: Aconverging bundle of light rays in the shape ofacohewith
mthrespect to the optical axis ofthe lens isplaced behind itaV tfte vertex angle of40^falls on acircular diaphragm of20 cm
:a distance of1= 1metre (Fig. 1E.213) diameter. Alens with afocal power of5dioptres isfixed.in the
'diaphragm.y\/hat will be the new cone angle ?
Solution; The path of -
the rays is shown in Fig.lE.214.
To find Xand h one should make a v..
h
use ofthe similarity oftriangles.
0 c Mr..'-" F
We pass the ray OiV||AB
through the centre of the lens
until it intersects wi± the focal
Fig. 1E.214
plane. In this case the ray AN is,,
the continuation of the rayiWA after its refraction in.the lens
The points Dand Bare conjugate; for instance^ Bis the
virtual image of point £i. Hence, denoting OB =aj, GO =
we obtain +^ = But a,=Rcota. a,'=«cotp,
Where Ris the semidiameter of the lens and we. obtain
tanj3 = tana+.5,

•The martian year is k = I.9 tipies longer than Earth's. A


collecting lens on Earth produces an image of Sun. on Earth
•fuith diameter^ =4mm. Find the diameter of Sun's imaze'
!
(b) •
I
'
I
produced by the same; lens on Mars.'Assume planets', orbiotl
Fig. 1E.213 i
<are circular. ' ' '

At what distance Hfrom the optical axis should the bottom of, Solution: GM^M, _M,v
a tray with water be placed to obtain a sharp image of the
object on the bottom ?The thickness of the water layer in the ' . r-

r = distance between'Sun and Earth


•trayis_d = 20cni
Sun
-

1
\

absence of the mirror, the image


AB of object ylB._[Fig.. lE.213Cb)] will be. obtained af a (^
distance of b=~~ =180cm from the lens.' After being
di( ^ .
!

AB and will be atadistance ofH' =6-1 =80 cm from position


the
optical axis. The layer ofwater with afthickness ofdwill shift (a)

the image over adistance ofH -^=^'dfl-~\ where n'is n is


Sun

, ' - ^ . .r" I n/ •
the refraction index of water. ( ^
Hence,
H- H'-i- df l-i )=85cm ^2^
(b)
Fig. 1E.215
! GEOMHRICAL OPTICS
161

V ~
GM.
V oc
For small longitudinal dimensions of the body
(x « a - /) the longitudinal magnification isa =
T (b) For a = 2/ the lateral magnification is p =1and the
T = 27tr oc (r)
, .»2
,...(1)
V longitudinal ihagnification is a =—3^— where r is the
1-r//
for Martin radius of the ball.
...(2)
As can be seen, the longitudinal dimension ofthe image
1.9oc|1^2 is greater than the lateral dimension, so the ball will look
like an elongated rotational ellipsoid.
Earth
f
A = !^ [At present the best sprinters run 100 min 10 s. What is the
di f ;appropriate exposure in making snapshots, if the blurring on.
From eqn. (A) and (B) jthe negative must not exceed x=0.5 mm 7The snapshots aret
\made at a distance of d=6m, the focal power of the
=^20 dioptres.
di f
2/3 Solution:- The ratio of the velocity of the ray across
dz the negative to the velocity of the runner is the same as the
>.r'J '^^[1.9 ratio of the dimensions of the image to that of the object.
= 4x 0.652 - 2.6 mm Denoting the lateral magnification byp, we obtain
a d' f
216 'neg = pV = V — = 1}— =
d ^d'
(a) Compare the longitudinal and the lateral magnifications since when the focal power ofthe objective is high,,the
of a thin lens. Consider the case of small longitudinal image wiU.lie practically in the focal plane. The exposure
dimensions of the object time is found by dividing the blurring ofthe image by the
velocity obtained : .
(b) Aball is'placed ata distance double thefocal lengthfrom
the lens on its axb. What wiU^eth^ of its image ? \ T =
_ ^d
neg V
Solution: (a) The lateral magnification is
. p=i:=5:=^
h a a~f
The longitudinal magnification isa = x'/x. To calculate Landscape shots are made, with a camera having an objectivei
it, write the thin lens formula in the form offocal power 7.7 dioptres. The camera isfocussed on objectsl
1.1],
12 m awayfrom it. It is desired to obtain, a sujficiently cleari
image of objects within a distance of 3m infront and!
a-x a' + x' /' behind-their blurring on the negative should not exceed 0.2\
hence a' + x' =
f(.a-x) \mm. What should be the setting (Le. diameter) of thel
a~f~x Idiaphragm ?What will be the aperture of the objective atthis
[Setting ?
But a' = of/(a -/); therefore

X' =
/(a-x) of Solution: a schematic diagram of the path of the
a~f-x a-f Ca-/)(a-/-x) rays is shown inFig. 1E.218. Suppose the image ofa pointA-
on the negative is sharp.
Hence the longitudinal magnification is
Then the image on the negative of point B which is
_
closer to the lens will be a small dot of width x. Let the
(a-/)(a-/-x) distance from pointA to the lens be a, and the distance from
point B to the lens be b.
_P'(a-/)_
a-f-x l-x/(a-/)
OPTICS
162

Condition of Achromatism
In case of two thin lenses in contact.
dF d/i d/2
y B' ,2 r2 r2
F fi fi F' fi fi
l2

The combination will be free from chromatic aberration


if dF = 0

i.e..
Then a' - and b' = Obviously -(J) =^ 0 /i' fi
a-/ h-f
which in the light of eqn. (5) reduces to
where ^ is the diameter of the diaphragm.
Q^i/i • ^2/2 =_ 0,
^ •
i.e., ^tOi + ^COo = 0«
Hence ^ = _ xh' = bxia-f) = —
bx(a(f)-l)
^ fi fi fi fi
n Note that for an achromatic combination two lenses
CHROMATIC ABERRATION must be of different material, for
Consider a parallel 1 I n
beam of white light
incident on a thin
convex lens as shown in or 1.0
F
Fig. 1.226. Since blue
light gets refracted more or F = 00

than red light, the point The combination will behave as a glass slab.
at which the blue light O Dispersive powers co^ and CO2 are positive numbers,
would focus is nearer Fig.1.226 the condition for achromatism will hold when and
the lens than the point /2 have opposite sign. Thatmeans if one ofthe lenses
at which the red light would focus. Thus, the image will is convex, the other must be concave.
appear to be coloured. Removal of Chromatic Aberration of a Separated
The focal length of a thin lens is given by Doublet

...(1) Let us consider two thin lenses \ ' -f- f .


R.
V'M R< J
^2 of focal lengths / and /' and j
If a change of n by 5n (the change of n is due to the
separated by a distance t (see Fig. |
1.227). The focal length of the i
change in-the wavelen^ of the light) results in a change of combination F would be
/ by 5/ then we obtain by differentiating Eq. (1)
5n 1 i = ...d) ^
F f r F i
/2 [r. RU n-lf Fig. 1.227
The focal length ofthefirst lens ^
5n
I.e., 8/ = -/ ...(2) would be given by
n-1
_1_
which represents the chromatic aberration of a thin lens. y=(n-l) R.
...(2)
IfnV and represent the refractive indices forthe blueand .^1
red colours respectively, then With a similar expression for 1//'. If A/ and An represent
the changes in the focal length and in the refractive index
/r " /v - / ...(3) due to a change AX in the wavelength, then by
n —1
differentiating eqn. (2), we obtain .
would represent the chromatic aberration.
From the definition of dispersive power, -4 =Au Rt (n-1)/
An

2J
5ri
CO =
n-1
Thus, differentiating Eq. (1), we obtain
A/' t Af Af
Thus 5/ =-co/ f f. f 2
r
An An' An' An
+
(n-1)/ (n'-l)/' /(n'-l)/' /'(n-1)/
163
["^flUETRICAL OPTICS
V2
\2
\

X =
\l2^!h^^^ 1-1^
/ r ff IC2. h^ + Xn
where, as before, co and co' represent the dispersive V2
powers. Consequently, for0 the combination to have the same
V2
focal length for blue and red colours we should have
P-2
tC(b +to') _TCO ' ^to' X
0
or x = —X(
^0 y

or
ff' f
^ co/' +co'/
1f .
V2
5^'
co + co'
X ^
Hi
If both the lenses are made of the same material, then
•0 V-2 Xq'
CO = to' and the above equation simplifies to
2^
M-2 1 +
= —X
2 •
-fit- 2Xf . P-2
implying that the chromatic aberration is very small if the
distance between the two lenses is equal to the meanof the For paraxial ray approximation, h 0
focal lengths. This is indeed the case for the Huygens'
eyepiece. • ir ~
2\
(

to
1-^1
Thus, 1 +

1
X = x
2x1 .

point object is placed infront ofaplane refracting surface at Y1.2


,a distance Xofrom it Ifa light rayfrom the objects meets the 2_
x-x
=—
-
Irefracting surface ata height has shown in Fig. JE.219, find 2^Pa il2j
the spherical aberration produced b^the^plane surface.
v•^ F2 *OPTICAL INSTRUMENTS
h Human Eye; Corrective Lenses
The human eye behaves like an optical instrument. It
has a diaphragm, called iris, that adjusts automatically, to
Fig.1E.219 (a)
control the amount of light entering the eye. The eye
produces an inverted real image of objects on the retina. The
Solution: Using Snell's law.
brain then performs inversion of the image to produce the
image that we perceive in its proper orientation. The retina
of eye plays the role of a camera film. It has two types of
cells ; rods which detect low levels of light and cones
which detect colour and bright light conditions in night.
h
The light isrefracted at thefront surface ofthe cornea.
The ciliary muscles accomplish fine adjustment for
focussing at different distances. Ciliary muscles adjust the
curvature of the lens, thus changing of the focal length.
To focus on a distant object the muscles are relaxed and
Fig. 1E.219 (b) the lens is thin [Fig. 1.228Ca)], the parallel rays focus at the
focal point (on the retina). To focus on a near object, the
sina = 1I2 sinp muscles contract, the eye lens thickens, thus shortening the
X = h cot p =
focal length so that the image can be focussed on the retina.
Also,.
smp Myopia (Nearsightedness)
Amyopic eye can focus only on nearby objects, there is
sin^ a inability to see distant objects clearly. Even with a relaxed
or X -
Pi sina^ -te) ciliary muscles the focal length of the eye lens is not large
h enough to image distant object on the retina. For a myopic
From the figure, sina = eye the image of distant object is located between the eye
^lh^ +x^
* Note that this article is not IIT-JEE syllabus
[164 ^ .
OPTICS
lens and the retina. Acorrective lens makes the final image -From the lens equation
of the lens and optical system appear on retina.
1111 1
The farthest object that can be seen clearly by a myopic
person is called far point for a normal person the far
V
f -dr
u
•'near f -d

point is taken as infinity. Here d is the convenient distance of object.'


A corrective lens will place the As d < dneai-. the focal length is positive. Thus
image of distant objects at the far Myopic eye" converging lens is used to correct for hyperopia.
point of the eye. The image of the Object at Visual angle : The
lens becomes an object for the Visual angle
s =-oo m angle subtended by an
eye. If the object lies within the far object at the eye is called
point, it will be clearly visible. The visual angle. How large an
corrective lens creates a virtual object appears depends on image;
Object at
image of a distant object that acts s = - oo m
the size of the image it
as a virtual object for the eye. makes on the retina. V^Tien Fig.1.230
Applying the lens equation an object is viewed at a - -
shorter distance, the image on the retina is greater, so the
1-1 = 1 (1) Fig. 1.228 object appears larger.
V u f'
u = - oo, V = -d
From the figure. tan0-6 = —
t/ar X

Negative sign implies that the image is virtual. An optical instrument improves the visual angle ofanobject.
Thus 1 Astronomical Telescope
~d^far oo / (i) Refracting astronomical telescope: In Fig.
1.231 shown the objective forms a real, inverted, diminished
or
f = -dfar image ofadistant object inits focal plane. The eyepiece then
Hence a diverging lens (/ < 0) whose focal length is
equal in magnitude to the far point is needed to correct forms a virtual, erect and magnified image of the
myopia. Opthalmolegists quote the focal length in dioptre intermediate image formed by the objective.The
(dp). intermediate image lies between the focus and the optical
centre of the eyepiece.
1dioptre = , Magnification of a telescope (M)
/ (in metre)
_ Visual angle subtended byfinal image
Hyperopta (Farsightedness) Visual angle subtended by object at unaided eye
Hyperopia refers to the inability to focus on nearby
objects although distant objects are seen clearly. Even the
simple task of reading can only be accomplished by placing a tana A'S'/C^A'
the book far from the eye. In this case the eye produces an
image of a nearby object behind the retina. CzA' u. ...(1)
The closest distance at which Case (1) : Final image at least distance ofdistinct vision
the eye can focus clearly is called Hyperopic eye
(D = 25 cm).
the near point. A normal eye H-25
has a near point of 25 cm.
Let us denote the near point u„ (Uo<V)
Objective
dnearj is the closest distance Eyepiece
from the unaided eye that can be
seen clearly. The corrective lens
creates a virtual image of the
reading book at this
virtual image then becomes a
virtual object for the eye. For the
eye the virtual object is far away Fig. 1.229
and the image produced by the
eye is sharp on the retina. V. = D

Fig. 1.231
GEOlVlETRiCAL OPTICS _ _
From the lens equation,
(ii) Reflecring astronomical telescope :
(Newtonian reflecting telescope)
J_
-D C-Uj +fe
In this telescope objective is a concave or parabolic
mirror, the image formation is by reflection. Fig. 1-234
shows its image formation.
or
U^ feDfA D
Thus, .A = - ...(2)
"e /A ^ Focus

In this formula only absolute value of foJe ^


substituted.
Case (ii) : Final image at infinity (normal use)
^ "" C— '

a —A"
Fo
•iV

•i B"* ; FigM.234 ....


I
I ' FigM.232 _! The formula for magnification is same as eqns.(l), (2)
and (3).
In this case the eye is relaxed (unaccommodated), hence
referred to as normal iise. Final image at D, M=-^|^1+^
In this case an
intermediate image is
Final image at infinity, M = -
formed at the focus of
the eyepiece.
In areflecting telescope, absorption of light is negligible,
From eqn. (1), we therefore abright image is formed. Lenses oflarge aperture
have are difficult to mould, because during solidification ot hot
M = -
fo glass distortion occurs. Due to large aperture of areflecting
Fig.J(.233 telescope, its resolving power is greater in comparison to a
fe
Length oftelescope in this case, refracting telescope. Reflecting telescope is free of chromatic
aberration and spherical aberration.
L = fo+ fe
Terrestrial Telescope
In this case the emerging light from the telescope is It is similar to a refracting telescope, •having an
parallel to the axis of the telescope, then additional convex lens at a distance (A+2/) from the
Apertureof objective _ D
7^ " Apertureof eyepiece d objective. This lens erects the final image. The intermediate
image A'B' from the objective is at a distance 2/ from the
Hence magnification, M = =
erecting lens that inverts it to A''B" which is now at 2/from
Je the erecting lens. Now A'R" is an object for the eyepiece that
• In order to increase the magnification, focal length of forms avirtual, erect and magnified image. The formula for
objective should be larger and eyepiece smaller. magnification is the same as obtained earlier accept with a
positive sign.
• Minus sign with magnification shows that final image
is inverted. ^ M = + —
a When final image is formed at least distance of
distinct vision, magnification obtained is greater and Case (i) When the final image is at least distance of
length oftelescope is smaller. distinct vision.
Objective
Eyepiece

I
Inverting
lens u <f

Fig. 1.237
Fig. 1.235
Case (iO : Final image at infinity.
Magnification,"

Length oftelescope, i = /<, +4/ +


and u = fe^
' fe+D: -
the final image is formed at infinity.
Fig. 1.238

Magnification,

Length of telescope, L=f^~f^


Inverting SIMPLE MICROSCOPE
Objective lens Asimple magnifying glass is aconvex lens of short focal
length. Amagnifying glass.allows to place the object closer
Fig. 1.236

Magnification, •M =^ hj
fe
Length of telescope, l =f^+^f +f H

Galilean Telescope N-v= D to


Unaided eye
In this case the eyepiece is aconcave lens, kept in such a With a lens
way that the rays from the objective converge at the first (a)
focus or away from it. The first image A'B'is inverted and it
acts as a virtual object for the eyepiece that forms an
inverted image of A'B'. The final image A"B'' is erect and
magnified. Formula for magnification can be obtained by
substimting in place of /, in the formula for
magnification obtained earlier. Thus, Fig.1.239

—^
M = ^f0 (T1_ fe Fig. 1.239Cb) shows that the object is placed atthe focal
"e fe i point orjustwithin itso that a ^h^tual image isformed. Ifthe
Case (i) ;Final image at least distance of distinct vision eye is relaxed, the final image is formed at infinity.
(D).. Magnification ofa microscope is defined as ' ' '
Magnification, M ~ Visual anglesubtended by final image
fe I D Maximum visual angle ofobject subtended
Length oftelescope, L= f^ -u (where Ug > /J at unaided eye
_ P ,
a.
GEOMETRICAL OPTICS
i (1) (3) (4)
From figure, Object placed Primary Secondary
V u just outside focal point focal point
1secondary of objective of objective
Eyepiece
and a. ifccal point lens
!of objective Objective
lu„ = -f„ Jens
Thus
ih/D) u
Case(i) : Final image atleast distance ofdistinct vision.
From lens equation, we have
i_i = l Secondary
V u f focal point
of objective
J. (2)
-D C-u) f
(6) B"
Enlarged inverted real
or 1= i.i' j Final inverted image, imageformed by objective
u 1/ D yirtuai, greatly magnified
— D

m = - = d •Fig. 1.240 (a)


or
u / D "7
A'B'.( D
In this case image distance vis minimum, so uwill also AB IeA'
36 minimum, hence magnification is maximum. Eye is in A'B' +v
naximum strain position. As - -t^and EA' = ~u,
AB -u e
Case (ii) : Final image at infinity.
V -D
Asn--~, hence u=/ then M =
-ue \

In this case magnification obtained is least. In this u. \ u e

position the eye is relaxed, unstrained. ^ and length of microscope, L = + Ug


n If the eye is at a distance x from the eyepiece, the
magnification in case (i), M=1+-^— and mcase Case (i) : Final image at least distance of distinct vision
(D). From lens equation, we have
D^x J_ =
(ii),
f " -D -u, /e
• If higher wavelength is used, foc^ len^ of the lens or ±=±+i
increases, resulting in reduced magnification. fe D
COMPOUND MICROSCOPE
It is a two lens system, both the lenses objective and or -^ =(1 +^
u, •fe
iyepiece are convex lenses of short focal length and small
iperture. Due to small aperture, spherical aberration is
nullified.
or M=-—— = [l+y
The object Is placed between F^ and 2Fg. The objective L = v„ +u, =v„ +
feD
forms a real, inverted, magnified, image A'B' is formed and

between 2F and oo, atu,. Intermediate image AB is a real


object for the eyepiece and lies between the eyepiece and its In this case is minimum because is minimum,
focus. The final image A^B" is erect (relative to A'BO and therefore maximum magnification is obtained.
magnified. Anglesubtended by final imageA^B" Case (ii) : Final image at infinity.
Magnifying power = j^g^ij^um visual angle at unaided eye From lens equation,
J_
_ p_ tanp _fA'B'^ /f— -00 C-Ug) fe
a tana VM'J/vD
or Ug ~ fe
^oPTicTj
M = -^ £ iel=|MP|x0„ =40x|:|xl0"^ =0.3684rad
3.8

i.e., |e|= 0.368x —^21°


IT

222i

An astronomical telescope consisting of an objective offocal]


length 60 cm and eyepiece offocal length 3cm isfocussed on!
the moon so that thefinal image isformed atleast distance of
,distinct vision, i.e., 25 cm from the eyepiece. Assuming the'
.angular diameter ofmoon as (1/2)° at the objective, calculate'
Fig. 1.240 (b)
(a) angular size and (b) linear size of image seen through the-
telescope. !
and
^ = '^0 +fe (maximum) Solution: As final image is at least distance of
In this case is maximum therefore magnification is distinct vision,
minimum.
\MP\ = fo 1+A - 60 1+ —
3 1
= 22.4
220 fe D 3 25.
Now as by definition MP =(0/0^ ),so the angular size of
An astronomical telescope has an angular magnification of image,
magnitude 5for distant objects. The separation between the
objective and eyepiece is 36 cm and thefinal image isformed e = MPx0, =22.4x "l
at infinity. Determine the focal length of objective and-
eyepiece. • 71
= ll-2°=-^xll.2 = 0.2rad
Solution: in case of astronomical telescope if object loU

and if I IS the size of final image which is at least


and final image both are at infinity,
distance of distinct vision,
= -(/o//e) and l = f,+f. 0 = CV25),
So here //J =- 5and /,+/,= 36
Solving these for and /„ we get i.e., I = 25 X0 = 25 x 0.2 = 5 cm

/o = 30cm and/g = 6cm 223


0'2^lr> AGalilean telescope consists ofan objective offocal length 121
diameter of the moon is 3.5x10^ km and its distance^ cm and an eyepiece offocal length 4 cm. What should be thef
•from the earth is 3.8xlO'km. It is seen through atelescope: separation of the two lenses when the virtual image of a\
havingfoca.1 lengths ofobjective and eyepiece as 4 mand 10' distant object is formed at a distance of 24 cm from thei
cm respectively. Calculate (a) magnif^ng power of telescope' eyepiece? What is the magnifyingpower ofthe telescope under
•(b) length of telescope tube and (c) angular size of image of 'this condition ? ;
moon. ^ J.
Solution: As the object is distant, U. u=-oo so ^
Solution: For normal adjustments, 111.
- - — = T - v = /„=i2cm
(a) [mp|-A- 4x100_ Jo

fe 10 i.e., the objective will form the image at its focus


(b) L=f^ +f^ = 400 +10 = 410 cm = 4.10 m which is at a distance of 12 cm from O.
(c) As the angle subtended by moon on the Now as the eyepiece of focal length -4 cm forms the
objective of telescope. image / at a distance 24 cm from it,
3.5x10^ 3.5 J 24
X10"^ rad -24 u, -4'
i.e., Ug = — = 4.8cm
3.8x10^ 3.8
and as IMP| =((0/0^) (, the angular size of final image. i.e., the distance offrom the eye lens M is 4.8 cm. So
the length of tube.
169
GEOIVIETRICAL OPTICS
f„= 12 cm Now as in case of lenses in contact.
l =J_ +Ju+ ori=—+ —withi =^+';^+""
F A A F f^ F F A A
So if one of the lenses is removed, the focal length of the
remaining lens system,
= = i.e., F' = 2.5cm
r F A 2 10
Fig.1E.223 _
This lens will form the image of same object at a
L = OA -M = 12- 4.8 = 7.2cm distance v2 such that
Now by definition, A L =J- i.e., U2=15cm
^ tane (AB/£A) _ OA V2 -3 2.5
MP =
%ane, (AB/OA) EA So to refocus the image, the eyepiece must be moved by
fn 12 10 or the same distance through which the image formed by the
So,
U.
objective has shifted, i.e., 15-6-9 cm away from the
objective.
i.e., the image is erect, virtual and is at a distance of 2261
24-7.2 = 16.8cm infront ofthe objective.
The focal lengths of the objective and the eyepiece of a
compound microscope are 2.0 cm and 3.0 cm respectively. the^
'The aperture ofthe largest telescope in the world is 5m. If the 'distance between the objective and the eyepiece is 15.0 cm.-
separation between the moon and the earth is 4xJO fcm and^ Thefinal imageformed by the eyepiece is at infinity. Findthe,
•the wavelength of visible light is 5000A calculate (aT 'distance of object and image produced by the objective, from^
'^resolving power of telescope and (b) minimum separation the objective lens. .
^between objects on the surface of moon so that they are just Solution: As the final image is at infinity, the
resolved. ^ - — - • distance of intermediate image from eye lens u, will be
Solution: (a) definition, given by .
D •xlO' i.e., Ug = -A--3cm
RP =
1.22X 1.22x5x10 ^ 1-22 —00 u. A

(bj As limit of resolution A0 =(1/RP) and if d is the and as the distance between the lenses is 15.0 cm, the
distance between objects on the surface of moon which is at distance of intermediate image (formed by objective) from
the objective will be
a distance r from the telescope, A0 = (d/r), ^ = L_u,-L-Arl5-3 = 12cm
1 d . 1 t
So, and ifu is the distance ofobject from objective,
RP r PP
^^(4xl0^yxl0"xl.22 ^ =i i.e.,u =-2.4cm
12 u 2
10^
So the object is at a distance of 2.4 cm infront of the
•e. 1225 objective.

Acompound microscope is used to enlarge an object kept at a.


•distance 0.03 mfrom its objective which consists of sev^al (a) An astronomical telescope consists of an eyepiece having
'convex lenses in contact and hasfocal length 0.02 m.
•'of focal length 0.1 mis removed fi-om the objective, find outj •two lenses offocal lengths 5cm and 4cm. The gap t between
the distance by which the eyepiece of the microscope must be^ the eyepiece lenses is adjusted such that the lon^tudmah
'moved to refocus the image. . -
'rspherical aberration caused by itis minimum. The telescope is.
'•adjusted for final image at infinity. Determine the',
Solution: if initially the objective forms the image ^magnification of the telescope.
at distance Vi,
1 —=-, i.e., Vi = 6m
Vi -3 2
• 100 cm

j Objective Eyepiece
Fig. 1E.227 (a)

(b) /In astronomical telescope is being used in normj


adjustment (i.e. thefinal image isformed at infinity). If the]
length ofthe telescope is reduced by~th ofthefocal length o/i, Fig. 1E^27 (c)

the eyepiece, then thefinal image isfound to beformed at the> In the second case u. = ~
nearpoint ofthe ^e, the least distance ofdistinct vision is 25\
cm and the angular magnification of the telescope in f/ie! v.=-D = -25
normal use is 20 Determine the angular magnification afterl Using 111
changing the tube length. Also draw the ray diagrams in both e "e Je
=2.5cm
CClS^Sm >

In normal adjustment, | |=^


Solution; (a) The least spherical aberration is due to fe
separated doublet. The separation of t must be equal to the where =+20,/, =2.5
difference of focal length of lenses.
=> /o =2.5x20 = 50cm
^ - fi~f2 = 5- 4= lcm
Now the focal length of the eyepiece is given by When the final image is formed at near point,
i _ 1 , 1 1
fe A f2 /1/2 20
•fe = (20/8) cm
For final image at infinity,
=2ofl +||l =20 (1.1) =22
M=A = 100
= 40
228
fe (20/8)
(b) When the final image is formed at infinity, the first In a compound microscope the objective and the eyepiece
unage is formed at the foci of both the lenses and tube [havefocal lengths of0.95 cm and 5cm respectively and are
length =fojje- l^ept at adutance of 20 cm. The last image is formed at a
distance of25 cmfrom the eyepiece. Calculate the position of
r
ft
fPf .If^t9Pj^fbejotal magnification.
H
»-— ( Fe.F,
Solution: As the final image is at 25 cm inffont of
the eyepiece,
r
_1 1
-25 u, 5
I.e., . 25
u„ = -

and so. -25


=-^
= 6 ...(1)
_^'g-jE.2^(b) I u, (-25/6)
When the final image is at least distance of distinct Now for the objective,
vision, the first image will be lying between focus F and U= =20-(25/6) = (95/6)
optical centre E of the eyelens. So if the object is at a distance ufrom the objective
— =

95 u 0.95 '
GEOMETRICAL OPTICS 171

95
i.e., u = cm Solid angle subtended by a spherical surface at its centre
94 IS

i.e., the object is at a distance (95/94) cm infront of the (47cr^)cos0


CO =
field lens.

Also, ...(2) = 47t steradian '


u (-95/94) Any closed surface subtends 47t steradian at any inside
So-total magnification, point and zero at outside point.
"94" PHOTOMETRY
M = m X m„ = - x(6) = -94
In this branch of optics we study the factors that are
i.e., the final image is inverted, virtual and 94 responsible for the sensation of brightness, light emitting
times that of object. capacity of a source.
Basic definitions and principles
Remark : In this example as final image is at least 1. Radiant flux (R) : The total energy radiated by a
distance of distinct vision D (= 25 cm), source per sec is called radiant flux, measured in watt.
ma = 1+ R. . 25 ^ 2. Luminous flux (d) : Total light energy emitted by a
L source per sec. Its unit is lumen (Im). In the visible light all
So MP = mxm0=mxmg= Linear magnification the colours and wavelength do not produce the same
sensation at our eye. Only the wavelengths between 4000A
Solid Angle to 7800A produce visual sensation at our eye, out of which
Figure shows a differential area element ds at a green-yellow light (^ = 5550A) gives maximum sensation.
position vector r from point O, that subtends solid angle dco Radiant flux expresses radiated energy of all the
at O. wavelengths whereas luminous flux expresses capacity to
generate total brightness.
Area ds
Unit of luminous flux, lumen, is arbitrary, defined as : If
Area ds cos 6
monochromatic light of wavelength 5550 A has radiant flux
watt, then luminous flux is one lumen.
685

3. Luminous intensity (L) : Luminous flux per unit


solid angle.
Ad
I =
Fig. 1.241 Aco
It represents light energy emitted by a source per sec per
, ds-f ds . . unit solid angle. SI unit is lumen per steradian, expressed as
dco = —— = — n • r candela (cd).
For an isotropic point source.
dscosG
dco =
47tr'
CO = = 47c steradian
as n • f = COS0.

Here vector d s has magnitude ds L=i =


CO 47C
and direction outward normal
or , = 47ti
represented by unit vector h. Angle
between f and n is 0. Total solid angle ds = ds n Luminous flux = 47c x (luminous intensity)
subtended by a finite area can be 4. Illuminance (J) : The luminous flux incident per
obtained from unit area of a surface.
r ds COS0
Fig. 1.242 Illuminance (/) = —
As

Its unit is lux (Ix).


[172"

/ r
D xmwmmm

Fig. 1£.229 (a)


Point source

Line source Parallel beam


47ir2
Solution: By
'^ 2^ the law of reflection,
Normal area does not the spot of light AB on
change with distance the wall will remain
hence 1is independent
stationary and hence
of r
the velocity of the spot
Fig. 1.243
along the wall is zero.
The size of the
Relation between luminous intensity and
illuminance (Lambert's cosine law)
mirror is always half Fig. 1E.229 (b)
the size of the spot as
Illuminance at an angle this is seen from the similar triangles S'AB and S'NM.
0 on an area ds is
Here area of AB is four times the area of MN. Hence
1=^ intensity of spot of light is one-fourth of the illumination of
ds
the mirror. At a large distance illumination of the mirror is
and L = zero because of distance and again when the mirror is
dco closest to the wall, the illumination is zero on account of
oblique incidence of light. Hence in some intermediate
Fig. 1.244
ds position illumination is maximum.
For an isotropic point source, Let d be the distance of the normal to the mirror from S
ds' ds COS0 along the plane. Then
dco - -

Pcos0
r r I (illumination of the mirror) =

1 =
Icos0 cV?
where P = illuminating power
This law is called Lambert's cosine law. Px Px
I =
For a given source and plane of
illumination L and h = const,
Formula for illumination is J =
_ Pcos0^
or rcos0 = h = const.

or dZ _^(d^ + - X- 3/2(d^ +
Fig.1.245 dx id^+x^)^
or
' ='4 When I is maximum,
or I oc COS^ 0 — =0^ - 3x^(d' + x^)V2 ^ 0
dx
and
=> d^+x^-3x^ = 0

d^ = 2x^ => X = A.
4^
Aflat mirror M is arranged parallel to a wall and lightfrom a
^point source S on the wall is reflected back to the wall and.
forms a spot of light. With what velocity will the spot move
along the wall if the mirror is brought up to the wall with a At what distance should the postsfor street lamps be installed
'velocity v ? How will the dimensions of the spot of light and its so that the illumination on the ground at the point lying
]illumination change ? 'halfway between two posts is not less than 4125 be ? The
height of the posts ish = 12 m. The luminous intensity of the\
lamps is I = 300 cd. Assume that a noticeable illumination is
provided only by the two lamps on either side.

GEOiWETRlCAL OPTICS
Solution: The illuminance near the edge of the
Solution: The illumination provided by one lamp / cosa _
tableisE= — Ih where h is the elevation
on the ground at a distance a =- will be
F. Ih of the lamp above the centre of the table. Let us find the
maximum of the function obtained. The condition for a
maximum is

-h^ = 27.5 m = 0
Hence,
dh

231 We have -|h(r^ + 2h =0


[Duringfitting jobs in asubway an electric lamp is secured at, _3h2Multiplying
+
by + we obtain
=0, so h=r/ V2. From this we can then
\the top point of the tunnel A[Fig.lE.23J (a)l What is t e find the illuminance at the edge and at the centre of the
Iratio between the illuminations produced by the lamp at the table.
ilowest point Band those at the point Clying at the level with
Ithe horizontal cross-section of the tunnel ? The luminous
Hntensity of the lamp is £he same in all directions. ^
Apoint light source of 10 cd luminous intensity
icen^tre ofaconcave mirror with aradius ofcurvature of40 cm
'.and of20 cm diameter. It illuminates ascreen at adistance of
[2 mfrom the source. What is the maximum illuminance ofthe
screen ? How will the illuminance change, if the mirror is.
iwithdrawn? _ _ '

Fig. 1E.231 (a)


Solution: The maximum illuminance will be at a
point on the principal axis of the optical system (Fig.
1E.233). In this case of a
Solution: The illumination at the : sufficiently narrow beam • " " ' Screen
point B is of light, the role of the
I
^0 = concave mirror is to
4R' double the luminous flux
and at the point C, falling on the screen.
I cosa Therefore, if the mirror is
~ ' ' ng.1E.231(b) withdrawn,
illuminance will decrease Fig. 1E.233
From triangle ABC [Fig. 1E.231 (b)], by one-half.
r = AC = 2Rcosp
where Pis the angle at the vertex A. Therefore
Epcosa
^ cos^ p Ulens of diameter Dand afocal length f projects asmall
'.object place at agreat distance from the lens on to ascr^n^
When a =13 =45°, E, =B„^/2 Show that the illuminance of the image on the screen will be,
proportivnal to the luminance and to the aperture ratio ofthe
'lens. (The aperture ratio is the square ofthe ratio of the lens ^
diameter tojts^ocal length) =
Around table of radius r is illuminated by a lamp of Solution: ifthe object is removed far from the lens,
luminous intensity (candle power) Ihanging "boveils centre
What should be the height of the lamp above its image lies practically in the focal plane. T e
kluminance of the table fringe to be maximum ?^ magnification is P=h'/h =d'/d =//d.
value ?What is the illuminance at the centre of the table m, The illuminance of the image is equal to the lununous
these conditions ? . .- .- - flux (J) =in divided by the area of the image S - nh /4.
Noting that the solid angle is Q= .where Dis the
diameter of the lens, we obtain
[174
OPTICS

The sunrays which reach the earth suiface bring in 'every'


it-
••-••-..v.--- minute an energy approximately equal to 1.94 cal per 1 cm^\
- d-
of the terrestrial surface, (with perpendicular incidence).
Fig. 1E.234
'Determine the total amount of energy received by the entire'
terresPial surface. Whatfraction in this of the total energy of
£ =A = light em^sion from the Sun ? What planet receives morel
47td^h'^ from the Sun—the Earth of Jupiter ? The distance
. D^Id^ I
from the Earth to the Sun is R, =I.5 xlO'km, the distance]
to Jupiter R2 IS 5.20 times larger than to the Earth the-
radimofthe Earth is 6.3 xio'km and the radius ofJupiter
41 IS 11.14 times of the Earth.
But B = is the brightness of the object and
nh-
Solution; 0=2.4x10^=kcal, 0.441 xlO"' of all the
R=Tip =^ is its luminance. We have E= where energy of light emission from the Sun; the total amount of
is the focal power of the lens. Note that we have by'j^iten'""'' received
/' The amount of energy received by the Earth is
neglected the light losses in the lens. Qi = TiTi^qj
BM« 235 where nr^^ is the cross-sectional area of the Earth.
The ratio of Qto the total energy of emission £ is
Ascreen u1mawayfrom alight source. Adiverging lens of
5dioptre focal power is placed between the source and the
E 4n 4r2
.s^een so that the position of the light source coincides with
dhat of the virtual focus. How will the illuminance of the
0" optical axis ofthe system change ? he Sun. The ratio off the energies received by themakes
Earthwith
and
Jupiter is
Solution:. According to the sign convention
Qj_ ^ ^
dltenird''fr
distance d from the virtual^ image are bothlensnegative
and the Q2 0)2 r2^R2
Without^the lens the luminous flux is distributed over an
rea Sq - 7zDq/4. With the lens the same flux is distributed
over an area S = tzD^/^ (Pig. IE.235).
In constant conditions of illumination a certain object isl
photo^aphed first from a large distance and then from al
small distance. ,
How will the iUuminations of the photographic plates in the
camera differ in these two cases? Which of the two requires a^
longer exposure ?

"'""'"^tion of a plate in a camera


Fig. 1E.235
on
or, the ratio
T of the area of the object being photographed
through to
the lens and
Therefore f =\ But Da=^emdD= 'I'CT-d-d') the area ofIts image on the plate. pneu to
^0 D d ' The quantity of light Qpassing through the lens is
where^(j) is^the lens diameter. In our problem d=-f, so proportion^ to the solid angle which the lens makes with
~j ^ =y. i-e., d' =I. We have area rof the
th aperture of the lens Sproportional to the
and is inversely
D
proportional to the square of the distance a, ffom Ae

D
=

2L + f
and camera to the object [Fig 1E.237 (a)] orQ ~A. The ratio of
^0 (.2L +fy
the linear dimensions of the object to those Uiof the image is
I GEO/VIETRICAL OPTICS 175

equal to the ratio of the distances of the object and the presence of the mirror is equivalent to the appearance of a
image to the camera lens [Fig. 1E.237 (h)] new source (with the same luminous intensity) arranged at.
a distance from the screen three times greater than that of
the first source. For this reason the illumination should
increase by one-ninth of the previous illumination, i.e.,
= 2.5lx
(b)
Fig. 1E.237 (b) The concave mirror is so arranged that the source is
in its focus. The rays reflected from the mirror travel in a
Therefore, the area of the object o ^ is related to the area parallel beam. The illumination along the axis of the beam
of the image a 2 as the square of the distances of the object of parallel rays is everywhere the same and equal to the
and the image from the lens, i.e.. illumination created by the point source at the point of the
mirror closest to it. The total illumination at the centre of the
£1 _ "1
C-y screen is equal to the sum of the illuminations produced by
the source at the centre of the screen and reflected by the
Comparing the results obtained, we find the rays :
illumination of the image
Ef, = 2x 2.251x = 4.5lx
Qoi
a? 4
1 r—

i.e.,the illumination of the image in the camera is


inversely proportional to the square of the distance from the = - \
= 1 F 0
lens to the image. 1
The image of a distant object is closer to the lens than z
^ 2 '
that of an object located nearby. Hence, the illumination of
the image of a distant object will always be greater than that
Fig. 1E.238 |
of the image of a near object. In the first case (near object), a
longer.exposure is required than in the second case. (c) The virtual image of the point source in the convex
mirror is at a distance of 2.5 r from the screen (r is the
distance from the screen to the source). The luminous flux (j)
•A point source of light placed at a distance from a screen emitted by the virtual source is equal to that of the real
icreates an illumination of 2.25 heat the centre of the screen. ' source incident on the mirror :

•How will this illumination change if on the other side of the JjCOj = 72*^2
source and at the same distance from it we place: Since the solid angle ©i of the flux incident on the
,(a) An infinite flat mirror parallel to the screen ? mirror from source S (Fig. 1E.238) is one-fourth of the solid
i(b) A concavemirror whose centre coincides with the centre of. angle ©2 Itiside which the rays from the virtual source Sj
\the screen ? ' , propagate, the luminous intensity 72of the virtual source is
one-fourth of the intensity of source S. For this reason the
[(c) A convex mirror with the same radius of curvature as the[ virtual source creates at the centre of the screen an
kojicave m i r r o r _ i illumination of 4x (2.5)^ = 25 times smaller that the real
Solution: (a) The rays reflected from the flat mirror source. Hence E^ = 2.34 Ix.
increase the illumination at the centre of the screen. The
selective Problems
•••• •

1. When the Sun is either 4. One face of a slab of


Apparent
rising or setting and Position of
thickness t is painted white /Clear
appears to be just on Sun and a small hole scraped /surface
the horizon, it.is infact clear at point P serves as a
From the Sun
below the horizon. source of diverging rays
Assuming that the when the slab is illuminated ^Painted
Earth Atmosphere
atmosphere has from below. Ray PBB' surface
uniform density and Fig. 1.1 strikes the clear face at a
Fig. 1.4
hence uniform index critical angle and is totally
of refraction n, and extends to a height h above the reflected, as are rays PCC. On the painted surface
surface of Earth at which point it abruptly stops, show there appears a dark circle of diameter d, surrounded
that its apparent position is an angle 5 above the true by an illuminated region or halo. Derive an expression
position, and 8 is given by for n in terms of measured quantities d and t.
8 = sin ^
nR
- sin
-1 R [Ans. n=[l+C4t/d)2]i/2]
.R + hJ U +h 5. A djmamite • , " . -
2. A mountaineer on a peak observes a rainbow in a explosion at S Geophones
'Sources Detector 0;
valley, caused by water droplets in the air 8 km away. surves as a source

The valley is 2 km below the mountain peak and of sound on the Unconsolidated
Region
entirely flat. What fraction of the complete circular arc surface of Earth.
of the rainbow is visible to him ? The velocity of
' White sound in an
Rock Layer^
unconsolidated
White region immediately Fig. 1.5
below the Earth's
surface is much lower than the sound velocity 1^2 in the
rock layer below. Two rays reach the detector SRD
(direct reflection) and SABD that encounters a hard
rock layer at a critical angle, (a) What is the time taken
for the path SABD. (b) Determine the condition under
which the critically refracted ray reaches D before the
^ _ Fig. 1.2 directly reflected ray.
[Ans. 62.2%] y Vj +^2
[Ans. (a) t = — + — cos C, (b) X > 2d
3. A large transparent cube, refractive index n = 1.59, has v., u, V2-Vy
a small air bubble below one surface. When a coin of 6. An optical fibre has a cylindrical
diameter 1.90 cm is placed directly over the bubble on cross-section of diameter d and index
the outside surface of the cube, one cannot see the of refraction n bent sharply. What is
bubble by looking down into the cube. What is the the smallest radius of curvature at a
range of the possible depths of the air bubble beneath short bent section for which the total
the surface ? internal reflection will be assured for
[Ans. 1.08 cm<d < 1.17cm] light initially travelling parallel to the Fig. 1.6
axis of the fibre ?
[Ans. (Ri +R2)^ + Mi +^2)-(Ci +^2)/coCiC2]^]
177

7. A layer of oil of refractive index Uq floats on water of


refractive indexn.^.Abeam of light is directedupward COS! ^^ I=)a,cosl ^
from a source in the water. Show that the critical angle
12. In Fig. 1.12 shown the light beam strikes surface 2 at
for totalinternal reflection is independent of Rq, even
though, for angles close to C (critical angle), the total
the critical angle. Determine the angle ofincidence i^.
internal reflection always takes place at the oil-air
Surface 1
surface.
8. A prism having a 120° apex angle is made of glass for
which n = 1.53. (a) Is there any range of angles of Surface 2

incidence ijfor light rays striking one face of the prism


that will result in total reflection at the second face ?
(b) If so, what is the value of that corresponds to the
critical angle for total reflection ? Fig. 1.12
[Ans. (a) yes, for 40.81 v<i-^ <90' (b) no value of
exists] [Ans. 27.5°]
9. Three plane' mirrors are 13. A transparent
Mirrored
arranged so that they are cylinder' of radius Incident ray '•
Surface
mutually perpendicular as R = 2m has- a
shown in Fig.1.9. Let the line mirrored "surface on
of intersection of the mirrors its right half as
be the coordinate axes, (a) A shown in-figure. A
Emergent
ray OP represented by vector . light. ray -travelling- ray
Fig.M.9 in air is incident" on
^1 = is reflected the left'-surface of Fig. 1:13
from the mirror in the-x -y the- cylinder -1 The
plane. What are the'components of the' reflected' incidentTight.fay-and-eniergent ray.are -parallel- and
' ^

vector? (b) If 13 is now'reflected at'the j-z plane, what d = 2 m.' • Determine " the' "refractive index of the
material.-"-
* - ^
are the components of-thereflected vector r3? (c) If 14.-AlightTaytmvellingin' - -
Emergent ray
is reflected at the z-x plane; what are the components air is incident 'oh one •

of the reflected vector (d) Is there any significant face of a right angled
prism .with a refractive
difference for an incident ray that is reflected from the index n = 1.5 as shown'
mirrors in some other order ?
in the figure and the
[Ans.- (a) r2 =Ui.yi.-Zi). (b) ^ =(-^i,yi,-Zi). ray follows the path -
shown in Fig. 1.14. If' Fig. 1.14
(c) = (-Xi,-yi,-2i) (d) No ]
0 = 60° and the-bMe of
10.'A material having- an Incident the prism'is mirrored, what is the angle (j) made by the
index of refraction n is ray emergent ray with the'normal to the righfface of the
surrounded by a vacuum prism ?
and is in the shape of-a 15. "An- optical fibre of a glass
quarter circle of radius R. Emergent' core (refractive index ni)is
A light parallel to base of ray
surrounded by a cladding
the material is incident (refractive index 03 < Hj).
Ftg. 1.10
from the left at a distance Suppose a beam of light
of L above the base and enierges out of the material at enters the fibre from air at an
the angle 0. Determine an expression for 0:' • angle 0 with the fibre axis-as
Fig. 1.15
[Ans. 0-= sin
-1
shown in the figure, "(a)
.R Show" that the greatest' - •]
11. A ray of light is refiracted through'a sphere'(refractive possible value of 0 for which a-ray'can be propagated
index p) in such a way that it passes" through the down the fibre is given by
extremities of two radii which make an angle p with -1

each other. Prove that if a is the deviation of the ray


= sin a/"? ~"2- •
caused by its passage through the sphere. (b) In an optical fibre different rays travel different
paths along the fibre, leading to different travel times.
This causes a light pulse to spread out as it travels
IMS _ orilcs^
along the fibre, resulting in information loss. The over both pa±s. However, the light reflected from
delay time should be minimized in designing a fibre. falls on the scale at S^, and the light reflected from S2.
Consider a ray that travels a distance L along fibre axis Explain qualitatively why (b) the light beams are
and another that is reflected at the critical angle as it
• travels to the same destination as the first. Show that
the difference At in the times of arrival is given by
At = -^2)
C 112

16. A light fay enters a


rectangular glass slab at an
angle = 45° and emerges at Source
Position 1

an angle 12 = 76°, as shown in


Fig. 1.16. Ca) Determine the Position 2
refractive index of refiraction
for the slab, (b) If the light ray >
enters the plastic at a point I = 50 cm from the bottom
edge, how long does it take the light ray to travel
through the slab ? •
[Ans. (a) 1.20, (b) 3:40 ns] • ' '
17. Fig. 1.17 shows an ' • -• --.
arrangement of Fig. 1.18 -
•V-'
mirrors forming a
range finder for a' displaced from Sq and (c) the two displacements are
camera. Mirror Mj
not same, (d) Let S^ be the distance'between Sq and
is partially silvered
Si .- Neglecting the difference between the speeds of
• and •• fixed -in,
light in air and c, the speed in vacuum, show that
position while M2 4Rcod ' - '•
is totally silvered c =

and rotates about


5i
a vertical axis. (e) Let 82 be the distance between Sq and S2, show
Looking into Mp that
you would see two = 1-h ^-1.
images and by Fig.1.17
.5i, .
rotating M2, they,
which is independent of co and c.
are made to exactly overlap. Since d, is known
accurately and the angle through which the second 19. A fish' is at a depth d under water. Take the index of
mirror is turned can be easily measured,, a scale can be
refraction of water at, 4/3. Sho>v that when the fish is
computed that displays the distance-1 directly on the viewed at an angle of refraction i, the apparent depth
of the fish is
camera. Write,an expression fqr^L in terms of (j) and d, . .Sdcosi. , . ,
taking the tilt of the fixed'mirror to. be 45°.' h =
[ Ans. L = d/tan 2q ] ' ' • ' '1 I1j -j74^9co5^ • '
18. Fig. 1.18 shows an arrangement to determine the 20. If light travelling in a medium for which the index of
speed of light in air to that in water. When mirror M , refraction is rijis incident at an angle 0 on the surface
which can be rotates,'"is'"at rest in^pbsition 1, light ' "'ofa mediuln' bfindex n'2'so that the angle between the
travels from the source through a'narrow'slit to M ,' reflected arid refracted rays is'pj show that
112
then to M-^ then back to M', arid finally'to-a-'ground tanS =
rii - n2 cosp
glass scale where ari image of slit is cast at Sq. When
M*is 'at rest iii ppsitiori 2, the light reflected from - 21. A concave mirror forms the real image of a point
M*passes through a'water filled tube, is reflected at source lying on the optical axis,at a distance of 50 cm
M2, passes back through the tube to M*, and again from the mirror. The focal lengA of the mirror is 25
cm. The mirror is cut in two and, its halves are drawn a
falls on Sq. (a). What happens when M*. is neither in
distance of 1 cm apart in a direction perpendicular to
position 1 nor in position 2 ? When M* is rotating • the optical axis (Fig. 1.21). How will the images
rapidly with angular speed co, light travels alternately' formed by the halves of the mirror be arranged ?
f geometrical OPTICS '179
arrangement of the object, the mirror and the screen)
for the mirror to reproduce an image of the entire
object ? The diameter of the mirror is d = 10 cm.

i Fig.1,21
[Ans. At a distance of 50 cm from the mirror and 2 cm from
each other. ]
22. A thin flat glass plate Fig. 1.24
is placed infront .of a
[Ans. hj = 5 cm]
convex mirror. At
what distance b from
25. Photographs of the ground are. taken from an aircraft
the plate (Fig. 1.22) flying at an altitude, of 2,000 m by a camera with a
should a point source .focal length of 50 cm. (a) What will be the scale of the
of light S be placed so photographs ? Howwill.the scale change if the aircraft
that its iriiage flies at an altitude of 1,000 m?, (b) The size of the film
produced by the rays in the camera in the.previous problem is 18 x 18 cm.
Fig.1.22
reflected from the "
What area can be photographed by this camera at any
one time ?
front surface of the plate coincides with the image
[Ans. (a) 1:4000 and 1:2000,. -
formed by the rays reflected from the mirror ? The
focal length of the mirror is F = 20 cm and the distance (b) Si-0.52km^S = 0.13km2] - .
from, the plate to the, miijor a = 5 cm. How can the 26. A convergent lens -forms on a'screen" an'image of a
coincidence, of the, images he-established by direct lamp magnified to twice its normal size-. After the lens
observation ? , >. . .i - ,has been mpved 36 cm closer to'the screen; it gives an
[Ans. b=v2aF4^'= 15cm] image diminished by a factbr-bf two. Find' the focal
length of the"lens, i
23. The focal length'of a
[,Ans. / = 24cm] • •
• concaye , mirror . can
27. What are tKe'smallest details of an object that can be
roughly be determined
by ±e following observed-separatelywith the' naked eye-at a distance
of 2 km ? The minimum angleofvision of eyeis b = !'•
method: place a needle
[Ans. 60 cm'] ^•
A at a distance d from
28. A thin convexo-convex lens is placed on'a flat mirror.
the mirror (Fig. 1.23),
then place a flat mirror
"Where should a point source of light be arranged so
P at a distance a from"
'thatMfs 'imagbipfoduced-'by this sykem' is real and
'coincides wth''the source'itself?'
the concave ' mirror
Fig.1.23 [Ans. At tHe'p'rihcipal Focus"bf the lens ] • ' ' '
and a second needle B
at a distance' b.from.the flat mirror. Move the mirror P
29. 'An optical system'consists of "a convergent lens with a
to match the ^virtual'images A and B of both needles
focal length'of'30-cm'and a^flat niirrof placed at a
distance b = 15cm from the lens. Determine the
formed by the concave and flat mirrors. Knowing the
position of the image formed by this system if an
values of a, b and d corresponding to the coincidence of
object is.at.a distance.af =; 15cm infront"of the lens.
the images, deterpiine the focd^length of-fhe mirror."
•?lpt-the path of the rays.in'±is case: ,
Can these images.Be otiserved by theeye-at^ the same' . [Ans. At'^69 cmpn.the same side.as'the object].
time ?
-dib-a) . . ,30. Plotijhe^iimage pf an, pbject-dn an-optical system
[Ans. F = .consisting of-a convergent, lens and a flat mirror
'i?-Ca'+d)'
24. A screen S is placed^a'distance fr = 5 cm'froni'a-circular'
arranged in the'focal"plane' of the lens.' The object is
infront of the, lens| and bbtween ^the Jpcus and the
convex rriirfbr as shown in Fig. 1:24'. Ah object KP of
doiible focal length pif the lens. What will be the size of
height h = 3'cm is arranged a distance a = 5cm from
the image if the object is posifipned' arbitrarily ?
the screen. Where must an observer position himself to [Ahs. The image will Be real and will lie between the lens
see the image of the entire object ? What are the .and the focus ] . • - . . ,
maximum'dimensions of the object (with.the given
OPTICS.
i180
31. Determine the position of the image produced by an
optical system consisting of a concave mirror with a [Ans. Each section of the lens produces a full image
irrespective ofthe other sections. Therefore, there will be no
focal length of 10 cm and a convergent lens with a strips on the photograph, and the image will simply be less
focal length of 20 cm. The distance from the mirror to bright.]
the lens is 30 cm and from the lens to the object 40 cm. 39. The layered lens shown inFig. 1.39 (a) is made of two
Plot the image.
kinds of glass. What image will be produced by this
[Ans. 100 cm]
lens with a point source arranged on the optical axis ?
32. A convergent and a diverging lenses having focal Disregard the reflection of light on the boundary
lengths of30 and 10cm, respectively, are arranged at between layers.
a distance of 20 cm from each other. Where should a
source oflight be placed for this system to emit a beam
of parallel rays ?
[Ans. The source should be at infinity]
33. Plot the image of an object formed by a system oftwo
convergent lenses. The focal length ofthe first lens is9
0
cm and of the second 15 cm. The second lens is in the S2
focal plane of the first lens.The object is at a distance 3

of 36 cm from the first lens. Calculate the distance Uj


of the image from the second lens.
[Ans. 2.5 cm]
34. A piano-parallel plate is cut as (a) (b)
Fig. 1.39
shown in Fig. 1.34 and the, lenses
thus obtained are slightly drawn [Ans. Any section of the lens gives an image identical in
apart. How will a beam of parallel".' shape to that produced by the entire lens. The layered lens
• ' ' ' rays change after"passing through •. can be therefore regarded as two lenses with different focal
this system if the beam is incidents' lengths, but with a common optical centre. Accordingly this
• lens will produce two images: at point and at point S2
' •'• (a)' from ,the'' side •of' the' 'convex [Fig. 1.39 (b)]. The image,of the source will be surrounded
lens? (b) from the side of the . Fig.1.34
by abright halo with a diameter ofab or, respectively, cd on a
concave lens ? How will the screen arranged perpendicular to the optical axis at point Si
behaviour of the beam depend Ion the distance
or S2.] , - , .
between the lenses ? •' .40. Aconverging beamofrays isincident onto a divergent
35. The distance between two point sources of light is lens so that the continuations of all the rays intersect
I = 24 cm. Where should a convergent lens with a focal at a point lying on the optical axis of the lens at a
length of/ = 9cm' be placed between them to obtain distance of b = 15 cm from it. Find the focal length of
the images of both sources at the same point ? the.lens in two cases-: (a) Afterbeing refracted in the
[Ans. 6 cm from one source and 18 cm from the other] lens, the rays are assembled at a point at a distance of
36. Athin convergent lens produces the image of a certain Ui = 60 cm from-the lens, (b) The continuations of the
object on a screen. The height of the image is hi- refracted rays intersect at a point at a distance of
Without changing the'distance between the object and 02 = 60cm infront of the lens.
-the screen, the lens is shifted, andlt is found that the . / [Ans. (a) 20 cm, (b) 12 cm]', - •-
height ofthe second sharp image is'h2: T)etermme the 41. The distance between an electric lamp and a screen is
height of the object H. d = 1 metre. In what positions of a convergent lens
[Ans. H = ^hih2] with a focal length of/ = 21cm will the image of the
37. What is the radius K of a concave spherical'mirror at a / • ' lamp filament/be sharp? .Can.an image be obtained if
distance of a = 2 metre from the face of a'man if he . the focal.length is,/' = -26cm'? ' n r . •
sees in it his image that is one and ahalf times greater [Ans.70 cm and 30 cm from the lamp, no]
than on a flat mirror placed at the same distance from 42. Find by construction the optical centre of a lens and its
the face ? ' ' •' • •• main foci on the given optical axis N1N2 if the
[Ans. R"=6m] " positions of the source S and the image S' are known
38. A man wishing to get a picture of a zebra [Fig. 1.42 (a)]
photographed a' white donkey after fitting a glass with
black streaks on" to the objective of his camera. What
will be on the photograph ?
GEOMETRICAL OPTICS

the image of the object to remain distinct ? The


refraction index of the glass of the plate is n = 1.8.
•s
[Ans. The screen should be shifted by 60 cm]
47. An object is at a distance of d = 2.5 cm from the surface
•S' of a glass sphere with a radius of R = 10 cm. Find the
position of the image produced by the sphere. The
(a) (b) refraction index of the glass is n = 1.5.
•Fig. 1.42
[Ans. Virtual image at 15 cm infront of the sphere]
[Ans. -The centre of the lens 0 is the point of intersection of
straight lines SS' and N1N2. The foci can easily be found by
48. A spherical flask is made of glass with a refraction
index n. The thickness of the flask walls AR is much
constructing the rays parallel to the major optical axis [Fig.
1.42 Cb)] less than its radius R.Taking this flask as an optical
43. The position of the optical axis N1N2, the path of ray system and considering only the rays close to the
AB incident upon a lens and the refracted ray BC are straight line passing through the centre of the sphere,
known [Fig. 1.43 (a)]. Find by construction the determine the position of the foci and the main planes
position of the main foci of the lens. of the system.
R'
[Ans. -]
2(n-l)4R'
49. A beam of light is incident on a spherical drop of water
at an angle i. Find the angle 0 through which the beam
is deflected from the initial direction after a single
reflection from the internal surface of the drop.
[Ans. 0 = Tc + 2i -'4 sin
-ifsini
I n
50. A glass sphere (n = 1.50) with a-radiiis^of 15.0 cm has
• (a) (b) ^ a tiny air bubble-^S.OO cm above its centre. The sphere
. Fig. 1.43
is viewed looking down .along the, extended.radius
[Ans. Point6, which is the optical centre of the lens, can be containing the bubble. 'What is the apparent depth of
found by 'dropping perpendicular,BO onto straight line N1N2 the bubble below .the surface of the sphere ?
[Fig. 1.43 (b)]. Let us draw ah'auxiliary optical axis DO [ Ans.8.57 cm] • • - . , ,
parallel to ray AB'and extend straight line BC' until it 51 A hollow glass sphere of radius 5.00 cm is filled with
intersects DO at point E lying in the focal plane. Let tis drop a glycerine. When shaken, bits of white plastic simulate
• ^ perperidiclilar from E onto N1JV2 to find point F; one of the
snow falling on a small figure of Santa Glaus inside the
' 'main foci of the lens. By using the property of reversibility of
'the ray, we can find'the other main focus Fj] sphere. If the Santa figure is 4.26 cm from the outside
surface of the sphere, how close to the surface does he
'44.' A parallel beam of rays is incident on a convergent lens
appear to be to someone looking in at him ? (Assume
• with a focal length of 40 cm. Where should a divergent
the sphere has •the same index of refraction as
lens with a focal length of 15 cm be placed for the
glycerine.)
' beam of rays to remain parallel after, passing through
[Ans. 3.98 cm] , , , ,
the two lenses ? • " • ' '
52 One end of a long glass,rod (n =.1.50) is formed into a
45. A parallel beam of light'ds incident' on a system
•.'convex surface of radius •,6.00 cm.. An object is
consisting of three thin"lenses v\nth'a common-optical
positioned in air along the axis of the rod. Find the
axis. The'focal'lengths of the-"lenses are'equal' to
image positions corresponding to object distances of
/i=+10cm, /2=-20 cm, and /g =+9 cm,
(a) 20.0 cm, (b) 10.0 cm and (c) 3.00 cm from the end
respectively. The distance between the first and the
, -of the rod.:,.,
second lenses is "I'S^cni and "between the second and
.'a l' '.[Ans. (a)-45 cm, (b).-9,0-cm,- (c)_-6 cm] ,
the third 5 cm. Find the position of the'point at which
. the beam -converges' when it leaves the-system of ' 53 ,A: goldfish'is swimming inside a-spherical plastic bowl
• I I' 1 ofIwater,'With an-index of refraction of 1.33. If the
lenses. ""' • • ' ' •' ' ' ^
[Ans. The'rays will travel in a parallel beaih.l • '
goldfish is 10.0 cm from the wall of the 15.0 radius
bowl, where does it appear to an, observer outside the
46. A lens with a focal length of / = 30 cm produces on a
bowl-?
screen a sharp image of an object thalt is at a distance
[Ans..Inside the bowl at,-9.01;cm] •.
of a = 40 cm from the lens. A plane parallel plate with
a thickness of d = 9 cm is placed between the lens and
54 A goldfish inside a spherical, bowl sees-a Cheshire cat
through the side of the bowl. If the bowl has a
the object perpendicular to the optical axis of the lens.
diameter of 25.0 cm and the cat's eye is 10.0 cm from
Through what distance should the screen be shifted for
[1'82 OPTICS

the bowl, where is the image of the cat's eye as seen by [Ans. (a) 20 cm to the right of second lens; M = - 6 (b)
the goldfish ? Ignore the bowl's thin layer of glass. inverted (c) 6.67 cm to the right of second lens, M--2,
[Ans. - 18.1 cm] inverted]
55. A novelty aquarium is built so that one glass face 59. The object in Fig. 1.59 is Lens Object Mirror
(n = 1.50) is actually a plano-convex lens. The convex midway between the lens
face with radius of curvature 10.0 cm is on the outer and the mirror. The
side of the aquarium, which is filled with water mirror's radius of
(n = 1.33). To a viewer on the outside, what is the curvature is 20.0 cm and
apparent length of a 1.00 cm long tetra swimming the lens has a focal length
•25.0 cm-
parallel to and 25.0 cm ft:om the inside face of the of -16.7 cm. Considering
aquarium ? only the light that leaves Fig. 1.59
[Hint: When two- different media of refractive indices the object and travels first
and Hg are on opposite sides of a lens of index Hj, the lens- toward the mirror, locate the final image formed by
maker's formula becomes this system. Is this image real or virtual? Is it upright or
a-h. [13 _ inverted ? What is the overall magnification ?
u V R- [Ans. 25.3 cm to the right of the mirror, virtual, upright,
56. Two converging lenses with a common optic axis and enlarged 8.05 times]
the secondary focal point of the first coincident with 60. A converging lens
the primary focal point of the second is known as an (/i = 120cm) is separated 4
afocal arrangement. A pair of afocal lenses can be used by 50.0 cm from a
as a beam expander to change the diameter of a diverging lens
collimated beam of light entering along the optic axis. if2 = -10.0 cm) as shown |
If the lenses have" focaT lengths'/j and and the in Fig. 1.60. An object 1.0 25 cm 50 cm

entering beam has diameter find the expression for cm high is placed 25.0 cm Fig.1.60
the diameter D of the exiting beam. to the left of the first lens.
tAns'. D=c/2/:/i-)d]';[ Find the position of the 'final image' and its
57. ^A biconcave lens. , ' ' ' height.Complete a ray diagram and'characterize the
(fi= -2'5.o'cm)is^ ' V , ' imagel • ' i • ,. ..
placed•' 20.0 cm' "t [Ans. - 7.29 cm, to the' left of the second lehs,'m =-0.25,
infroht: of " a
virtual, inverted, ^2 = C>.250 cm] " ' • .
61. The lens and mirror in
concave ' mirror Object Lens Mirror
Fig. 1.61 have focal
(A = 5.00cm) as
shown in ' Fig.
1.57. A 2.00 cm "
Fig. 1.57
lengths of -1-80.0 cm and
- 50.0 cm, respectively. JL
high object is placed 15.0 cm to the left of the lens. An object is placed 1.00
Using the lens arid mirror equations and a ray m to the left of the lens,
[<-1.00 m 1.00 m-H
diagram, find the location of three images : (a) the as shown. Locate'' the
final imagej which' is Fig.1.61
image formed by the lens as rays travel to the right, (b)
the image formed after rays reflect from the mirror, formed by light that has gone through the" lens twice.
•and .(c) the final image'-after the leftward .travelling State whether the image is upright or-inverted and
rays once.again-pass.through-the le'ns. Complete the determine the overall magnification, 'c'
ray diagram and characterize each imagcand-object. [Ans. 160 cm to, the left of-the lens ; inverted, m = -0.8]
[Ans. - 9.375 cm, + 0.625 cm, virtual erect ; 6.026 cm, 62. Fig. 1.62 shows a ray, parallel to the axis but distant
-0.205, real, inverted; - 8.96 cm, 0.641 ; final image is 8.96 from it, falling on a concave spherical mirror of radius
iCm to the right of the lens, ^ p;6822, vi^al,,enlarged] . R at P. The centre of curvature of the mirror is atC, and
58. An object 2.00,cm high is placed 40.0 cm.to-the left of the three angles'(ji are equal. The reflected ray misses
a converging lens with a focal length of SO.OiCm. A the focus F. The distances x and' 'y are known,
diverginglens with a focallength of -2p.0-cm,isplaced respectively, as the longitudinal arid transverse
110 cm to the .right .of.jthe converging ,lens, (a) spherical aberrationr "•
Determine, the-final position and magnification of the
final image.'(b) Is the image upright or inverted ? (c)
Repeat parts ^(aj.'md. (b) for the case in which the
second lens is a converging lens with a focal length of
-I- 20.0 cm.
GEO/ViETRICAL OPTICS 183

What type of geometric figure is this ? (c) Determine


the area of the image.
66. A biconvex lens = 10.0cm) is placed 40.0 cm
infront of a concave mirror if2 = 7.50 cm) as shown in
Fig. 1.66. An object 2.00 cm high is placed 20.0 cm to
the left of the lens. Using the lens and mirror equations
and a ray diagram, find the locations of three images:
• (a) the image formed by the lens as rays travel to the
right, Cb) the image formed after the rays reflect from
Fig.1.62 the mirror, and (c) the final image after the leftward
travelling rays once again pass through the lens.
(a) Show that x = — Complete the ray diagram and characterize each
2 image and object.

(b) Show that y = x tan 2(j) = — -1 tan 26


2 ^C0S(1)
63. The spherical
Directrix
aberration shown can
be eliminated by using a
a mirror of ay^ •20 cm- 40 cm-
paraboloidal form. ,

The figure shows a Axis. , Focus Fig.1.66


cross section of such a , [Ans. 26cm,-1, real, .inverted; +12 cm, -0.6, real, erect
mirror. Shown also is (that is reinverted);, 15.6 cm, -0.556, re^, inverted;.overall
the geometrical magnification'0T333.]
definition of' -.a •> F|g.'1.63 'j 67. A spherometer is a device use'd to measure the radius
.parabola ,as-the. locus ^^ . i" . ^ . of curvature R of spherical surfaces. It consists oif three
of points equidistant froni a point called the focus and legs^^positioned in an equilateral triangle with sides of
a line called- the directrix. Show that 'all- the rays length y. A cenwe probe is raised or lowered by means
parallel to the axis are r.eflected-so as to pass through ofia micrometer drive. -The spherometer is first placed
the focus, regardless of their (distance from the ^axis. on. a flat surface and the zero position of the
[Hint. You rtiay find it easier to carry out the proof in terms micrometer noted. The instrument is then placed on a
of parallel wave' fi-onts, using Huygen's construction], ' . spherical surface and the micrometer is adjusted until
64. According to,the geometrical (definition, an.ellipse is the centre probe just touches the surface. The
the, locus of, all points for" which, the sum of the displacement z of the micrometer between the two
distances from two points called the foci is a constant. reading's is known as the sagitta. (a) Make a simple
Using this definition, show that a mirror in the form of argument to show that y^+(z-R)^ =R^ so that
an ellipse,'with a point object, at one .focus, will 2.^ + y^
produce an image.at .^e other focus. , , o-. ,- R =
. 22
65. Fig.T.65 shows a ,i'('"•..v
-(b)''/A .student- records the- following micrometer
thin glass. ''"'A' -positions'.in millimetre ':,flat surface'11.222, convex
(n =1.50),_,..,A ; I ' *20.0crn' *i'\ face '8.'716, concave face 12.473," when using a
converging .lens
F F spherometer with y given by the manufacturer as 50.0
for which the
mm. The student subsequently measures the focal
radii of curvature 1 J - . !
.A , . - -l' ^ . ' /
length'of the lens to be 186 crh. What is the index of
are =15^.(}cm , _ ci _
^refraction'of' the'material from which the lens is
and. , 1•I-W 'fcohslructed ? - -
cm. To the 1^ of. : ^ TAlis:(b)'1.538] ' I- I
the lens is a cube Fig 1'65 ' 1 > 68. Two rays travelling parallel to the principal axis strike
with a face area '• - - " " - --
-a'"large plano-convex lens having a refractive index of
of 100 cm^. The base of the cube is on the axis of the
1.60 '(Fig: 1.68). If the convex face is spherical, a ray
lens, and the right face is 20.0 cm to the left of the
lens, (a) Determine the focal length of the lens, (b)
near the edge does not pass through the focal point
Draw the image of the square face formed by the lens. (spherical aberration occurs) . If this face has a radius
of curvature of 20.0 cm and the two rays are
184 OPTICS

hi = 0.500 cm and h2 = 12.0 cm from the principal *73. Sources of light and $2 of equal intensity are
axis, find the difference in the positions where they arranged at the vertices of an isosceles right triangle
cross the principal axis. (Fig. 1.73). How should a small plate A be positioned
for its illumination to be maximum ? The sides of the
triangle ASj = AS2 = a
A

Fig.1.73
Fig.1.68
[Ans. 21.3 cm]
[Ans. Er = 42—-]
or
69. A parallel beam of light enters' a glass hemisphere '74, An attempt to use a photometer to measure the
perpendicular to the flat face, as shown in Fig. 1.69. luminous intensity of a certain source of light failed
The radius is R = 6.00 cm and the index of refraction is since the luminous intensity was very high and the
n = 1.560 . Determine the point at which the beam is illumination of the photometer fields with the aid of a
' focussed. (Assume paraxial rays). standard source could not be equalized even when the
source being investigated was placed on the very edge
of the photometer bench. Then a third source was
employed with a luminous intensity lower than that of
the one being investigated. At a distance of =10 cm
from the photometer the standard source produced
the same illumination of the fields as the third one that
was placed at a distance of r2 = 50 cm. After that, the
• V' ' Fig. 1.69 ='•' '• " standard source''was replaced by the one being,
[Ans. 0.1,07 mto die right ofthe vertex ofthe hemispherical investigated and equal illuminations were obtained at
face'.] ' ' ' distances froni the photometer r3'"='40cm (source
70. A biconvex thin lens is made of glass having index of being investigated) and "= 10 cm (auxiliary source).
refraction n. The lens has surfaces of equal curvature Find how many times the luminous intensity of the
R. The lens is made into a mirror by silvering one of source being investigated is greater than that of the
the surfaces. Parallel light is incident on the lens from standard source.
the clear (unsilvereH) side. Show that the focal length [Ans. p = 400/0]
of the lens/mirror is '75, The ray of a searchlight falls on the wall of a house and
produces a bright spot with a radius of r = 40 cm. How
' /= 2(2n^ -1) •. many times will the illumination of the wall of a
'71 ,'A'. round hall' with a "diameter of D = 30 metre is
remote house be smaller if the radius' of the spot on it
is 2-metre ? ' ' ' '
'illuminated by a lanip secured in the centre "of the
[Ans. The illumination will be 25 times Smaller]
de'iling. Find the height Hof the hall if the minimum
illumination of the wall is double that of the floor. '76, A lamp with a luminous intensity 'of I = 100 cd is
[Ans. /i = 7.5 m ] fastened to the ceiling of a room. Determine the total
"72 ,A lamp rated at f^ = lOOcd hangs above the middle of luminous flux falling onto all the walls and the floor of
'the fdom. •
'a "round" table with'aMiamkef of p"= 3'nietre at a [Ans. $ = 628 Im ] '
height of H = 2 metre, "it is replaced by a lamp with
12 = 25 cd and the distance to the table is changed so '77, What'part of the energy radiated by the Sun reaches
that the illumination "of the middle of the table the-Earth ? The radius of the Earth is,'6,400 km and the
remains as before. How'will the illumination of the average distance from the' Earth of the Sun is
edge of the table" change? 149,000;00d km.
[Ans. =3] . ; ' [Ans. The earth receives ^ of the total energy of
2.25x10^
the Sun]

Problem based on photometry


LGEOWETRICAL OPTICS 185

-'•78. A hot glowing wire is placed on the axis of a hollow 86. Two men, a far-sighted and a short-sighted one, see
cylinder with a radius R. The length of the wire is objects through their spectacles as a man with normal
much greater than the height of the cylinder. How eyesight. When the far-sighted man accidentally puts
many times will the illumination of the internal on the spectacles of his short-sighted friend, he found
surface of the cylinder change if its radius is R2 that he could see distinctly only infinitely far objects.
(assume that R2 < Rj) ? At what minimum distance a can the short-sighted
[Ans. ^ =-^ ] man read small type if he wears the spectacles of the
£2 Ri far-sighted man ?
*79. At what height should a lamp be hung above the [Ans. 12.5 cm]
centre of a round table to obtain the maximum 87. An object is examined by a naked eye from a diistance
illumination on its edges ? D. What is the angular magnification if the same object
[Ans. /1-O.7IR] is viewed through a magnifying glass held at a
80. An equiconvex lens of crown glass and an equiconcave distance r fromthe eye and so arranged that the image
lens of flint glass make an achromatic system. The is at a distance Lfrom the eye ? The focal length of the
radius of curvature of the convex lens is 0.54 m. If the lens is /. Consider the cases :(!)£ = «>, (2) L = D.
, focal length of the combination for mean colour is 1.50 [Ans. +
m and the refractive indices for the crown glass are
fXjj = 1.53 and |Xy = 1.55, find the dispersive power of 88. The objective is taken out of a telescope adjusted to
flint glass. infinity and replaced by a diaphragm with a diameter
[Ans. cOf = 0.053 D. A screen shows a real image of the diaphragm
81. Two lenses, one made of crown glass and the other of having a diameter d at a certain distance from the
flint glass, are to be combined so that the combination eyepiece. What was the magnification of the
is achromatic for the blue and red light and acts as a telescope?. . '
convex'lens of focal length 35 cm. Calculate the focal [Aiis.§]
length of the components if for : d . . . -

Crown glass jiy = 1.5175 and Qig -pg) = 0.00856 89. The double-lens objective of a photographic camera is
Flint glass |iy = 1.6214 and ( j i g = 0.01722 made of a divergent lens with a focal length of
[Ans. = -23.8,cm, = 14.17 cm] fi = 5 cm installed at a distance of I = 45 cm from the
82., State whether the-fqllpwing statement is true or false film.Where should a convergent lens with a focal
,. ^ giving reason in brief; 'A parallel beam of white light length of /2 = 8cm be placed to obtain a sharp image
• is incident on a combination of a concave and a convex of remote objects on the film ?
, lens, both of the same material. Their focal lengths are [Ans. d = -——__!±A h 4/2.
15 cm and 30 cm respectively, for the mean 2 2 1 + /1
wavelength in white light. On the other side of the lens 90. Verify the claim in the text that, in order to invert the
system, one sees coloured patterns with violet colour image of a celestial telescope using a converging lens
at the outer edge." of focal length /, the length of the telescope must be
83. A telescope objective of focal length 60 cm is made of lengthened by at least 4/.
two thin lenses, one of crown glass of refractive index 91. The main focal length of the,objective of a microscope
1.52 and other of flint glass of refractive index 1.66. is /obj ,= 3mm and of the eyepiece = 5cm. An
One surface of the flint glass is plane. Calculate the ' ' .object is at a distance of a = 3.1mm from the
radii of curvature of both the lenses which'form die objective. Find the magnification ofthe microscope for
achromatic doublet if dispersive powers of crown and a normal eye.
flint glass are 0.0151 and 0.0302 respectively. [Ans. 180 times]
[Ans. Crown glass: 25.74 cmand 39.6cm; Flint glass: 39.6 92. The Yerkes refracting telescope has a 1.00 m diameter
m and <»] ^ ^ _. , . •
objective lens with a focal length of 20.0 m. Assume
84. Why does a swimmer see only hazy contours of objects * ' ' that it is used with an eyepiece that-has a focal length
when he opens his eyes under water, while they are of. 2.50 cm. (a) Determine the magnification of the
distinctlyvisible if he is using a mask ? . , ' planet Mars as seen through this telescope, (b) Are the
85. A short-sighted man, the accommodation of whose eye Martian polarcaps seenfight side up or upside down ?
is between ai=12cm and 02 = 60 cm, wears [Ans. (a) - 80.0, (b) upside down]
spectacles through which he distinctly sees remote 93. Astronomers often take photographs with the
objects. Determine the minimum distance 03 at which objective lens or the mirror of a telescope alone, with
the man can read a book through his spectacles. out an eyepiece, (a) Show that the image size h' for
[Ans. 15 cm]
this telescope is given by h' = fh/if - p), where h is
186
OPTICS I :
the object size, / is the objective focal length, and p is 97. Galileo's telescope [he called it a cannocchiale ('eye-
the object distance, (b) Simplify the expression in part tube')] is sketched in Fig. 1.97. As in the telescope the
(a) for the case in which the object distance is much second focus of the objective coincides with the first
greater than the objective focal •length, (c) The focus of the ocular. However, the ocular is a diverging
"wingspan" ofthe International Space Station is 108.6 lens. Drawa ray diagram for the Galilean telescope. Is
m, the overall width of its solar-panel the image erect or inverted ? real or virtual ?
Objective Ocular
configuration.Find the width oftheimage formed bya
telescope objective of focal length 4.00 m when the 1*2 obj
^1 ocu
station is orbiting at an altitude of 407 km.
94. Galileo devised a sample terrestrial telescope that
produces an upright image. It consists ofa converging
objective lens and a diverging eyepiece at opposite •u

ends of the telescope tube. Fordistant objects, the tube Fig. 1.97 , •
length is the objective focal length less the absolute V

valueofthe eyepiece focal length, (a) Does the userof 98. A nearsighted person cannot see objects clearly.
beyond 25.0 cm (her far point). If she has no
the telescope see a real or the virtual image ? Cb) astigmatism and contact lenses are prescribed for her,
Where is the final image ? (c) If a telescope is to be
constructed with a tube 10.0 cm long and a
what power and type of lens are required to correct
her vision ?
magnification of3.00, what arethe focal lengths ofthe
[Ans. 4 dioptres ; a diverginglens]
objective and the eyepiece ?
[Ans. (a) virtual, (b) at infinity] 99. Show that, if you place the object so that the imageis
95. A compound microscope has an objective of focal at your near point instead of at infinity, the angular
magnification is increased to the value )
length 0.300 cm and an eyepiece of focal length 2.50
cm. If an object is 3.40 mm from the objective, what is a = —+ 1 '
the magnification ? /.
[Hint : Use the lens equation for the objective] 100.A fisherman .tying flies desires a magnifier that
[Ans. - 75.0] produces an angular magnification of2.00 when used
96. The distance between the eyepiece and the objective with eyes in the relaxed (unaccommodated) state.
lens in a certain compound microscope is 23.0cm. The What focal length lens should be used ? How close to
focal length ofthe eyepiece is2.50 cm, andthat ofthe the fly does the sportsman's'eyes-have to be ?
objective 0.400 cm. What is the overall magnification [Ans. 12.5 cm]
of the microscope ?
[Ans. - 575]
GEpnRiai OPTICS

Problems

1.' For a prism,kept in air it is found that for an angle,of (a) 15km , (b) 15km , . . ;^
incidence 60°, the angle of refraction 'A', angle of (c) 15m V - f (d) IS.m
deviation '5' and angle of emergence' e' become equal. 5. Four identical mirrors are made to stand vertically to
-Then the.refractive index of the prism is : form a square arrangement.as shown in a top view. A
^.(a) 1.73 ' (b) 115 / ray starts from the mid point M of mirror AD and-^er
' (c) 15 (d) 133 , . . two reflectibhs reaches corner D.'Then, akgle 6 mdst
2. Aparallelbeam of light is incidenton a prismof prism be:'- ' • ^' ,w ^- v.- , ,,
angle A as shown figure. Find the angle 0 between the
]two reflected' beams R\ and i?2 faces as
I i \ - 5 '
' shown.

iI: • ^. r

§
^mmmy//////D
t
Ca) 2A ^ (b) 3A (a) tan^Ho.75) (bj cot-Ho.75)
(c) 4A (d) none of these (c) sin"^(0.75) Cd) cos-no.75)
3. Two thin prisms of dispersive power co and co' are 6. A small object is placed at the bottom of a beaker
combined such that they neither produce average 4
deviation nor do they produce dispersion. If angle of containing a liquid of refractive index ~ to a depth of
3 •
one of the prisms is 2° and the refractive index is 15 for 20 cm. A pin is placed horizontally over the beaker and
yellow light, 1.49 for red light and 1.51 for violetlight, adjusted until there is no parallaxbetween the image
find to upto two digits after decimal: of the pin formed by reflection at the liquid surface
(a) 0.01 (b) 0.02 and the image of the object. The height of the pin
(c) 0.03 (d) 0.04 above the liquid surface is :
4. A television picture is built up by scanning successive (a) 20cm (b) 15cm
horizontal lines across the screen. It takes about 50 (c) 10 cm (d) 30 cm
microseconds to scan the screen width. Ghostingrefers 7. A parallel beam of light passes parallel to the axis and
to a weak horizontally displaced repetition of the main ...falls on one face of a thin convex lens of focal length /
picture. These occur" vvh'en , the TV . displays afi,. Vafid after two internal .reflections emerges from the
additional delayed signal, due to reflection fromCah second face and forms a real image. Find the distance
object such as a hill. If the ghost picture is displaced by of the image from the lens if p. is the refractive index of
10% of the screen width, what is the additional path the lens? .
length of the reflected signal ? . (a) /ai-lV3p-l (b) (p-l)//C3^t-l)
Cc) (3|i-l)//ai-l) (dj/Oi-l)
OPTICS
188

8. The figure shows the path of a ray of light as it passes


through three different materials with refractive
indices ni,n2 and ng. The figure is drawn to scale.
What can we conclude concerning the indices of these (c)
three materials ?

12. Figure shows graph of deviation 8 versus angle of


incidence for a light ray striking a prism. Angle of
prism is :
st
30'

(a) ^3 > n2 > Til (b) Ho < Rt < Tl'


(c) no < n, < n. (d) Hi < < Ti2
9. It is desired to produce an achromatic combination of 15° 60°

thin lenses having an effective focal length / in air. The (a) 30= (b) 45^
two lenses to be used for the combination must be (c) 60= (d) 75=
have their dispersive powers coi and CO2 and focal 13. A thin lens of refractive index 1.5, and focal length in
length for yellow as fi and /2 respectively then : (An air 30 cm is placed inside, a large container containing
achromatic combination means ^at the combination two immiscible liquids as shown below. If an object is
' - must, have same focal length for dl the colours) placed at an infinite distance close to principle axis
CO
(a)/i = 1- / Cb) f2 =; •1-^ f (common intersection line of the layers), the distance
C01
>

Ms. between images : '.'' •


|i,i=1.2 1^2=1-5
(c) l +
1-h (d) % = CO 2j
/

16. A certain large plane mirror has its area normal along
+ i direction. A particular point P.on t mirror has its
r\
initial position vector as Si. The mirror starts rotating ji3=2.5 t»3=2-5,
in a counter clockwise direction in the x-y plane
without changing its orientation such that the position
of point P at a times is 'given, as (a) 25 cm (b) 45 cm
Tp = 5{cbs(27it)i +sin(2jtt)'j}"^ • (c) 65 cm (d) 85 cm
14. The left end of a long glass rod 6 cm in diameter has a
. Acertainobjectlies,in the x-y plane at the point (10,0). convex hemispherical surface 3.00cm radius. The
.Now choose^the correct statement : - ' refractive index.of glass is 1.61 Thisiro'd is immersed in
. . ' (a) x-coordinate'ofthe,imageatt = —secis207iunits. a liquid: An object'90.0 cm from the"-vertex of the left
4
• end of the rod and on its axis from'the image at a point
(b) Maximum speed of the image is IOtt units.
1.6 inside the rod. What is the refractive index of the
(c) Maximum acceleration ofthe iniage is40n^ units.
liquid ?
(d) Relative speed of the image w.r.it. 'mirror is 2071 (a) 1.22 ,(b) 1.32
units.
(c) 1.42 ' (d) 1.52
11. Choose the correct ray diagram of a thin equi-convex
15. A thin prism with angle 4° made of glass of
lens which is cut as shown in the figure:
refractive index 1.54 is cornbined with another thin
prism P2 made of glass' of refractive index 1.72 to
produce no deviation. The angle of prism P2 is.
(a) 3= (b) 2.6°
(c) 4® (d) 5.33=
i GEOMETRICAL OPTICS 189

16. AB is incident wave front on mirror R and mirror S (a) 15 (b) 18


whereas PQ and P'Q' is reflected wave front from (c) 24 (d) 30
mirror R and mirror S respectively. 20. The power of a lens having refractive index 1.25 is +3
diopters. When placed in a liquid its power is -2
P'
diopters. The refractive index of the liquid is :
(a) 1.2 (b) 1.4
Mirror R Mirrors CO 1.5 (d) 1.6
21. The coordinates of the image of point object P formed
Q'
by a concave mirror of radius of curvature 20 cm
Choose the correct statement : (consider paraxial rays only as shown in the figure is :
(a) mirror A is concave and mirror B is convex
(b) mirror A is plane and mirror B is convex '20 cm 20\/3R'
(c) mirror A is plane and mirror B is concave
(d) mirror A is convex and mirror B is plane
17.. Variation of magnification .m
(m) produced by a thin
convex lens versus distance (a) (16cm,-4%/3cm) (b) C16cm,+ 4V3cm)
• (v) of image from pole of the (c) (30cm,-10^/3cm) (d) C30cm,-10V3cm)
• • lens is shown in the graph. 22. A point abject O is placed ' ^ ' " — " ""
,:,oWhich of the following in front' of a concave t= 30 cm

- statements is not correct ? iP mirror of focal' length Glass slab

r (a) Focal length of the lens is equal to intercept on 20 cm on the principal


v-axis 5 cm.5 cm
axis of the concave mirror
(b) Focal length of the lens is equal to inverse of the
at a distance^of 5 cm fi-om
slope of the line.; '
the pole as shown in the
(c) Magijitude of intercept on m-axis is equal to unity.
(d) Magnitude of intercept on u-axis is equal to unity. figure. A glass slab is also placed in front of the mirror,
perpendicular to this principal axis.'The'image formed
18. A ball hits a rigid horizontal surface with angle a
by perpendicular to this principle axis. The image
incidence 45®. If angle of reflection is 60° then the
formed by the system for first reflection and then
factor by which vertical component of the velocity of
refraction is : . , ,
the ball changes during collision by :
(a) virtual behind the mirror .-
1
(a) VS (b) real between the mirror and the. slab ,
"75 (c) virtual between the mirror and the slab
Cd) A (d) can't be decided by above data
- Vs 23. A real image of a luminous object" is formed by a
19. A convex lenSi of focal length 40.cm is held coaxially convex lens at the bottom of' an empty beaker. The
above a. concave >mirror of focal-length .18cm. A beaker-is filled to-a-depth of-^cm. with a liquid of
luminous-point object placed d cm above the lens on its refractive index 1.4. Then in order to get the image
axis gives rise to final image coincident with itself. again at the bottom of the beaker, how much should
Then d is equal to : , , , the beaker be move'd vertically?
O

d cm

if= 40 cm

12 cm

f= 18 cm; (Assume that lens and object remain fixed)


190 OPTICS

(a) 2cm upward (b) 2.8cm upward (a) 5/ and I (b) 5/ and 3/
(c) 2.8 cm downward (d) 2cm downward (c) 91 and I (d) 91 and 3/ ,
24. A ray of light passes through and equilateral glass 29. Monochromatic lights of wavelengths 420 nm and
prism in such a manner that the angle of incidence is 540 nm are incident simultaneously and normally on a
equal to the angle of emergence and each of these double slit apparatus with slit separation of
angles is equal to (3/4) of the angle of prism. The angle 0.0756mm and screen is at a distance of Im. The total
of deviation is : number of dark fringes due to both wavelengths on
(a) 45° (b) 70° the screen is :
(c) 39° ' (d) 30° (a) 360 (b) 280
25. A double convex lens is made of glass which has its (c) 19 (d) 38
refractive index 1.55 for violet rays and 1.50 for red 30. A point source 0 kept at distance
rays. If the focal length for violet rays is 20 cm, the of 2R form pole P of concave Hi=2\f3
focal length for red ray will be: interface having radius of
(a) 9 cm ' (b) 18 cm curvature R emits a
(c) 20 cm (d) 22 cm unidirectional beam at an angle of
26. The adjacent figure shows a thin 30° to the principle axis PO. The
plano-convex'lens of refractive index |ii distance of the image of point source 0 from pole P is :
and ' a thin plano-concave lens of (a) 2R/3 (b) R
refractive index V2J bbth having same (c) 2R/Vs (d) none of these
radius of curvature R of their curved'" 31. If the critical angle for total intemal'reflection from a
surfaces:.The.thin concavo- convex lens medium to vacuum' is 30° the velocity of light in the
of. refra'ctiom index P3 has" radius of medium is : . y
curvature R of. both its surfaces. The (a) 3x10® m/s (b) 1.5x10® m/s
qoncavo-convex lens is so placed in between the (c) 6xl0®m/s ,(d) Vs X10® m/s
plano-convex arid plano-concave lenses, that the plane
,32. A diyer looldng up throughcthe wateriit a lake sees the
surfaces -are parallel .to each other, the. focal lengthof
outside world- contained in a circular, horizon. If the
the combination is : '
R ' ' ' '' 'R' ' refractive index-of water is,(4/3),and,the eye of the
(a) ihy diver is situated 12v cm below the surface, what is the
(Hi+1^2+1^3-1) O-Li'+1^2+^3) radius of the horizontal circle : ' ,
R '"' R"
(c). (d) (a) 12x3xV5 (b) 12x (4/3)
(Pl -IU2) (^1 -tt2 -^3.-3) (c) 12 X(3/^^) , (d) 12 X^ X3
27. An equi-convex lens has power D. It is cut into two 33. Interference fringes of yellow light^of wavelength
symmetrical halves by a plane ^containing the principal 6000 A are formed by Billet split lenses. The distance
axis and then two pieces are joined as shown in figure.
from source to lens is 24 cm:The focal length of lens is
The power of the combination will he:',
15cm. The lens halves-are separated by 0.06mm. The
distance of source to screen is 200 cm. The fringe
width (in mm) is : •
; (Symmetric situation)
f= 15 cm

jA
6000A[ ] • i .
Screen

0.06 mm

(a) P (b) 2P .
.24 cm
(c) f/2 (d)Zero ...
200 cm"
28. Two,, coherent.-rnonochromatic light beams of
intensities / and 41 are superposed, the maximum and (a) 6.1 (bj 7.1
minimum possible intensities in the resulting beam (c) 5.1 (d) 8.1
are:
GEOMETRICAL OPTICS 11

34. A ray of light is incident on a boundary between two (d) Both visible light waves and microwaves can be
substances as shown. The substances have different refracted by glass
indices of refraction. Which incident ray could not 40. A thin converging lens forms a real image of an
have produced the outgoing ray ? object located far away from the lens as shown in the
figure. The image is located at a distance 41 and has
Outgoing' height h. A diverging lens of focal length I is placed 21
ray.
from lens . Another converging lens of focal length 21
is placed 3Z from lens . The height of final image thus
formed is :

(a) A
Cc)C
(a) h
35. An object is moving towards a converging lens on its
axis. The image is also found to be moving towards the (c) 4h (d) 2h •, .
lens. Then, the object distance 'u' must satisfy : 41. A rod of length Llies along the axis of a concave mirror
(a)2/<u<4/' • '(b)/<u<2/ ' of focal length /. The near end of the rod is at a
(c) u > 4/ ,, (d) u < / distance I > / from the mirror. Its image will have a
36. For a glass prism of refractive index Vs, the angle of length :
minimpm deviation is equal to the angle of the prism. - Lr , iV' ' ^1
The angle of the prism is nearly : (a) (b)
a-fX2L~f)
(a) 80° (b)70° . '
• 2Lf.,. • 2L^'f
(c) 60^' ' ' ' '^ (d) ^50° (c) (d)
1 I I s ' 5 V « -I
' (I - /)(-2L - /) ' •- - {L- /)C2L -7)
37. Two plane ihoriochromatic coherent waves produce .
- i
interference pattern'(of alternately bright and dark-, 42. tWo,convex lenses offocal length
bands) 'on 'a screeh: When the angle 0 between the two / and 4/are arranged to have
beanls'is decrMsed the fringe width.- their foci at the same point as
(a) incfe'ases" ' - ' - ' " ' ' shown in the figure. A beam of
(b) is unaffected'-"' • ' parallel light of diameter 1cm
(c) decreases - ' • - falls on to the first lens. The beam exiting .the second
(d) increases arid' then decreases lens is : '

38: An object was placed upright 25 cm in front'of a (a), diverging .....


converging lens with a focal length of 2'O'cm/A concave ' (b)'converging'' - \
mirror with a focal, length"of 15 cm was placed 120 cm '(c) parallel'of size 0.25 cm • - ^ '
(d) parallel of size 4 cm
behind-the'len^.-Which of. these describes-the final
'image'ri'-... 43. The ratio of angle of minimum deviation produced by
(a) real, enlarged .. ii(^ a thin prism (ji = 3/2) in air to that in liquid of
(b) virtual, upright refractive index 9/7 is :
(c) virtual, inverted (a) 1/3 (b) 3 .
(d) inverted, diminished , (c) 1/4 • " ' (d) 4 '- ,
39. In comparing the properties of visible light waves to 44. A ray of light is incident on a face of equilateral
microwaves; 'whicfr'of' the' following statements is triangle at an incident angle 40°. At this angle,
false ? ' minimum deviation occurs. The value of the deviation
(a) Visible light waves travel, at the same speed in is :

glass as do microwaves " (a) 60° (b)10°


(b) Visible light waves have a higher frequency in (c) 20° (d)40°
glass than do microwaves 45. A ray of light is incident on' a right angle prism of
(c) Visible light waves travel at the same speed in •. refractive index Vs as shovm in the figure. The rayof
vacuum as do microwaves
light is reflected and refracted at the surface AC of the
192 OPTICS

prism. The angle between the reflected and refracted (d) Yes, the image is larger and farther away than the
rays at the surface is ; object
' a" ' 50. Three coherent, equal intensity light rays arrive at
point P on a screen to . produce an interference
minimum of zero intensity. If any two of the rays are
blocked, the intensity of the light at P is . What is the
intensity of the light at P if only one of the rays is
blocked ?
(a) 0 (b)/i/2
(c) h (d) 2fi
51. A direct-vision prism is made out of three prisms, each
with a refracting angle of 60°, attached to each other
(a) 90° (b) 120° as shown in the figure. Light of a certain wavelength is
(c) 150° (d) 45° incident on the first prism. The angle of incidence is
46. A small amount.of transparent liquid is- 30° and the ray leaves the third prism parallel to the
poured "over the shiny convex base "of a direction of incidence. The refractive index of the glass
vessel as.shown. The ma^itiide of"focal of the first and third prisms is 1.5. Find the refractive
length of the equivalent mirror thus formed is : index ofthe material ofthe middle prism. (Vb = 2.45)
(a) less than that of convex hase of the vessel
- (b) ..more-than that of convex base of-the Vessel
i(c) equal' to that oTconvex base of the-vessel
(d),independent ofthe refractive index of the liquid
47. -A converging'lens forms, an image of ^ah''object on a (a) "1.95 (b)2.96
. screen."The image is real and has'twice the.size of the (c)'-2:84 (d) 3.95
- object.Jf the .positions of.thescreen'and the "object are '52. ' The 'curve' Of angle 'of incidence
interchanged, leaving the lens in its original position, versus 'angle of deviation shown
1what is the new image size on the screen ? "has been plotted forprism. The
• '(a) Twice, the object size value of refractive index of the
G)) Same as the object size • prisni used is :
(c) Half the object size 40° 60° i.
(d) Can't say as it depends on the focal length of the
lens
(a) V3 (b) V2
"48. What.should be the .value of distance d so that final
r ^ --=
(c) ^ (d)A
'image is formed on the "object itself.' (focal lengths of
V2 V3
the"lenses are written on -theTenses).
' ".iOicm r20 cm'
53. In the given curve'of above question 52. Find the value
of angle in degrees is :
(a) 40° (b) 60°
(c) 70° (d) 90°
10 cm 54. An object is placed infront of a converging lens at a
distance equal to twice the
(a) 10cm .••-(b)- 20 cm focalTerigth fi of the lens. On • '
(c)".5cm ' (d) none of these • "the bthief side of the lens is a >
49. The •passengeriiside-view mirror on an "a!utomobile concave mirror'of focal length •t-
often has^Ae--notationr''"Gt^^^ seen in mirror are /2 separated from the lens by ;
closer thanlthey rappear"l~Is'the image really farther a" distance '2(/i'-i-/2). Light j
away jhan-the-o"bject ? from the object passes '
(a) Yes,:thein\age-is smaller and farther away than the rightward' through the lens, "
object •reflects from the mirror, passes leftward through the
(b) No, the image is smaller and closer than the object lens and forms a final image of the object.
(c) No, the image is larger and closer than the object
-GEOMETRICAL OPTICS 193
(a) The distance between the lens and the final image 59. Two identical thin plano-convex lenses of refractive
is equal to 2/^ index n are silvered, one on the plane side and the
Cb) The distance between the lens and the final image other on the convex side. The ratio of their focal
is equal to 2(/l + f2^
lengths is :
(c) The final image is real, inverted and of same size
n „. n-1
as that of the object (a) (b)
n-1
(d) The final image is real, erect and of same
n-l-1
size as that of the object (c) (d) n
55. A ray of light enters into a thick glass
60. A vessel of depth d is half filled with a liquid of.
slab from air as shown in figure. The
refractive index pj and the other half is filled with a
refractive index of the glass slab
liquid of refractive index p 2.The apparent depth of the
varies with the depth x form the
vessel, when looked at normally from air is :
topmost surface of the slab as
1
|i = V3—where Xis in meters. (a) -!-• (b) d(pi +\i2)
Vs >1 P-lj
Slab is of sufficiently large thickness. Find the (c) ^(pi 4-P.2) J_ +J_
(d)d
maximum depth reached by the ray inside the slab. M-i M-2
(a) 3 m (b) 1.5 m 61. A ray incident at a point.at an angle of incidence of 60®
(c) l.Gm (d) 0.5 m
enters, a glass sphere of p = Vs and. is reflected and
56. The diagram shows a concavo-convex lens. What is refracted at the farther .surface of the sphere. The
the condition on the refractive indices so that the lens angle between the reflected and refracted rays at this
is diverging ? surface is :
III K.V (a) 50® (b) 90®
(c) 60® (d) 40®
1 2R / R
, The,refractive Index 62. An equilateral prism deviates a ray through 45® for
;of the lens is 1^2 two angles of incidence differing by 20®. The angles of
. (a) 2^13 <|Li 4-112 (b) 2^L3 > Pi 4-^12 incidence are ; :
(c) P3 > 2(pi -P2) Cd) none of these (a) 60® and 40® (b) 50® and 70®
57. A convexlens of focal length 20cm and another plano (c) 62® 30' and 42® 30' (d) 60° 30' and 40° 30'
convex lens of focal length .40cm are placed co-axially 63. Three right angled prisms of refractive indices are
(see figure). The plano-convex lens is silvered on fitted together so that the faces of the middle prisms
plane surface. What should be the distance d (in cm) are in-contact with one of the outside prisms. If the ray
so .that final image of the object 'O' is formed on 0 .passes through the composite block and emerges
.itself : . without any deyiation then :
(a) pf 4-p2+p2 =1
(c) p?-pi+pi=l (d)Pi-KP2-P3 =1
0
64. Solar rays are incident at .45® on the surface of water
•10 cm' 4
p = - . What is the length of the shadow of a pole of
; f^ + 20cm 3
(a). 10 (b) 15 length 1.2m erected at the bottom of the pond if the
(c) 20 (d) 25 pple is vertical assuming that 0.2m of the pole is above
58. An ..object is placed at a distance of 12cm from a the water surface ?
converging lens on its principal axis and a virtual (a) Im (b) 0.75 m
image of certain size is formed. Now the lens is moved (c) 0.825 m (d) 1.2m
4cm.,away-from the object and a real image of the 65. The image produced by a concave mirror is one
same size as ,that of the (previous) virtual image is quarter the size of object. If the object is moved 5 cm
formed. The fo.caLlength in cm of the lens is : closer to the mirror, the image will only be half the size
(a) 14 (b) 16 of the object. The focal length of mirror is :
(c) 18 (d) 20 (a) /= 5.0cm (b) / = 2.5cm
(c) / = 7.5cm (d)/ = 10cm
OPTICS
66. The- observer. 'O' sees the distance/ AB as infinitely
large. If refractive index of liquid is ^and that of glass Z . i\
is|.i2, thennj/|i2\is : . . (a) X'
(b)
iV.

(a) 2 - (b)i/2 (c) ix-

^ ' .(d) none of these


1
67. A man is standing on'a straight road in a summer
noon. The refractive index of the air changes with 70. Seeta and Geeta were asked to draw the trajectory of a
light ray near, earth's surface when refractive index is
height as|x = |x Q 1 + y
due to variation of the
SCmetre) decreasing with increase in height. Figure shows the
vertical temperature where Po is. the refractive index respective ray diagrams :
ofthe air atroad surface and y height. Eyes ofthe man
are at height of 1.5m above .the road. .What is the Vertically up; [
apparently visible length .of'the road. Vertically up i
Earth's surface
2 Seeta
(a) (b) Earth's surface
V5 Geeta
(a) Seeta is correct
(d) (b) Geeta is correct
Vs V5
(c) Both are incorrect
68. An insect of negligible mass is sitting on a block of
mass M, tied with a spring of force constant K. The (d) No conclusion can be drawn
block performs simple harmonic motion with 71. A thin plate of transparent '
amplitude Ainfront ofa plane mirror placed as shown. plastic is embedded in a thick i glass plastic, glassj
The maximum speed of insect relative to its image will slab of glass. The index of 1.50-
be : refraction of the glass is 1.40-

n = l,50: the index of 1.30-

refraction of the plate changes 1.20-

as shown in the diagram. A


beam of light passes through glass and strikes the
surface of the plastic plate. What maximum angle of
incidence enables the beam to pass through the plate
(a) 63° (b) 43°
V2 VM (c) 53° (d) 73°
(c) AV2j— (d) A, — 72. Aperson is looking at the flat'surface of transparent
hemi sphere ofrefractive index p = ^/2. Half ofthe flat
69. Acoordinate axis as shown below is kept in front of a surface is coloured black-and halfof curved surface is
converging lens at a distance 2/ from it.Where / is the marked at 6 points equally spaced as shovra in the
focal length of the lens. What is the approximate shape figure. If the person sees only through the flat surface,
of the image as seen by an observer on the other side he can see ;
ofthe lens at a distance 3/,from it. Assume thatx-axis
is the principal axis of the lens •
r

aA -XI

LZ
(a) all the points (b) Y and I only
(c) Y,I and B (d) 0, Y and G
GEOMETRICAL OPTICS
195
73. A plane mirror is placed in y-z plane facing towards 77. The refractive index of the medium- within a certain
negative A:-axis. Themirror is moving parallel toy-axis region, x > 0, y > 0, changes with y. A thin light ray
with a speed of 2cm/s. A point object P is moving traveling in thex-direction strikes the medium at right
infront of the mirror with a velocity (Tcm/s)i angles and moves through the medium along a
A

+(1cm/s)j. Then which of the following statements is circular arc. The maximum angular size of the arc is
incorrect ? (The material having greatest refractive index is
(a) The velocity of image is (-i + j) cm/s diamond with refractive index n = 2.5)
(b) The velocity of image with respect to mirror is y "'
-(i + j)cm/s
(c) The velocity of image with respect to object is
(-2i) cm/s
(d) Tjie unit vector is the direction of reflected ray is Ca)90° (b) 30^
(i + j)
(c) sin-^l^ (d) cos
74. A beam of the light incident vertically on a glass 25 1,25
hemisphere of radius R and refractive index V2 lying 78. A concave mirror has the form of a hemisphere of
with its plane side on a table. The axis of beam radius R = 20cm. A thin layer of an unknown
coincides with the vertical axis passing through the transparent liquid is poured into this mirror and it was
centre of base of the hemisphere and the cross found that the given optical system wi± the source in
R
sectional radius is The luminous spot formed on a certain position produces two real images, one of
V2 which (formed by direct reflection) coincides with the
the table is :
source and the other is at a distance = 8 cm from it.
(a) R Cb) A The refractive index p, of the liquid is :
V2 Ca)Z .
Cc)-^ (d) (^^3 + l)R
CVS-fl)
75. A ray of light, initially horizontal, passes through a
cylinder of glass of radius R, as shown. Assume that 79. The lateral magnifications of the lens with an object
'^air ~ 1. Based on the geometry of the ray's path as
located at two different positions U-^ and U2 are and
shown, find index of refraction of the glass, 7712 respectively. Then the focal length of the lens is :
(a) / = Vmim2 (u2-"i) (b) "2-"i

horizontal ray ("2 -"1)


(c) (d) ("2 -"i)
(^2)^^ -(mi)"^
80. In a Young's double slit experiment X= 500nm,
-- Honzontal
d = 1 mm and D = lm. The minimum distance from
the central maximum for which the intensity is half of
the maximum intensity is :
= 1.00 (a) 2x10^-4"m (b) 1.25xl0-4m
(c) 4x10"^ m (d) 25 X10-4 m
(a) 273 Cb) 1.73
(c) 295 (d) 3.73 81. A bird is flying 3 m above the surface of water. To a fish
underwater, •the height of the bird from the water
76. Surfaces of a thin equi-convex glass lens (|i = 1.5) have
radius of curvature R. Paraxial rays are incident on it. surface appears to be [Take = 4/3]
(a) 2 m (b) 3.33 m
If the final image is formed at a distance — from pole (c) 4m (d) 4/3m
13 ,
of the lens after n internal reflection. Then n is : 82. In the above question 81, if the birdis diving vertically
(a) 2 (b) 3 down with speed = 6 m/s, his apparent velocity as
(c) 4 (d) infinity seen by a stationary fish underwater is :
(a) 8 m/s (b) 6 m/s
(c) 12m/s (d) 4 m/s
OPTICS

83. A plane mirror .approaches a stationary person with 88. Two diverging lenses are kept as shown in figure. The
some acceleration 'a'. The acceleration of his image, final image formed will be :
as seen by the person, will be :
(a) a • (b) 2a
O di
(c) a/2 (d) none of these
84. Aray oflight is passing from air into glass. Ifthe angle
of incidence, with respect to the normal to the (a) virtual for any value of and d2
interface, is increased : (b) real for any value of d^ and dj
(a) Total internal reflection willoccurwhen the angle (c) virtual or real depends on dj and d2 only
of incidence,equals the critical angle. (d) virtual or real depends on d^ and dj and also on
(b) Total internal reflectionwill occur when the angle the focal lengths of the lens
of incidence is less than the critical angle. 89. A diverging lens of focal length fi is placed infront of
(c) Total internal reflection will occur when the angle and coaxially with a concave mirror of focal length /2.
of incidence is greater than the critical angle. Their separation is d. A parallelbeam of light incident
(d) Angle of refraction angle will increase but there on the lens returns as a parallel beam from the
will be no total internal reflection.
arrangement:
85. A convex mirror has a radius of curvature of 20 cm. An (a) The beam diameters of the incident and reflected
object is placed at such a distance from the mirror that beams will not be same.
the size of the image is exactly half that of the object. (b) d = 2|/2l-lAl
The object distance must be : (c) If the entire arrangement is immersed in water,
(a) 20 cm Cb) 30 cm the conditions will remain unaltered.
(c) 10 cm Cd) 40 cm (d) None of the above.
86. An air bubble is inside water. The refractive index of 90. Sunbeam makes an angle 0 = 40° with the surface of
water is 4/3. At what distance from the air bubble the Earth. At what angle to the horizontal should we
shoulda point be placedso as form a real imageat the place a flat mirror so that a ray of sunlight after
same distance from the bubble ? reflection falls on the bottom of a deep well ?
(a) 2R (a) 40° Cb) 50°
(b) 3Jl (c) 80° (d) 65°
(c) 4R 91. An object of height h = 5cm is located at a distance
(d) the air bubble cannot form a real image of real a = 12 cm from a concave mirror with focal length
object 10 cm. Find the height of the image,
87. Apoint sourceis situated at a distance Xq < f from the (a) 10cm (b) 15 cm
pole of convex mirror of focal length /. At time t = 0, (c) 20 cm (d) 25 cm
the point source starts moving away from the mirror 92. A diver immersed in water sees the sun at an angle
with constant velocity. Which of the graph below 8 = 53° to the surface of water. What is the real
represents variation of image distance \V\ with the angularposition of the sun above the horizontal ? Take
distance x between the pole of the mirror and the 4
refractive index of water as —.
source ? 3
(a) 37° (b) 45°

(c) sin~^ (d) sin'


.25 16

(a) 93. Monochromatic light rays parallel


to x-axis strike a convex lens AB. If
the lens oscillates such that AB tilts
upto a small angle 0 (in radian) on
|V1 '1V| either side of y-axis, then
amplitude of oscillation of image
will (/ = focal length of the lens) :
(C)- (d)
GEOMETRICAL OPTICS 197

(a) /sec6 (b) /sec^B 98. A ray of light is incident on a plane mirror, along the
(d)Z5! ^ A A

direction given by, A = 2t - 3j + 4j. Find the unit


A.

4
vector along the reflected ray. Take normal to mirror
94. A beam of convergent light ^ A A

along the direction of B = 3i - 6j + 2k.


converges to a point 0.5m in
front of the mirror after -94i + 237j + 68k . -94i + 68j- 237^
(a) (b) ^
reflection at a convex mirror 49V^
but in the absence of the 3i + 6j - 2fe
mirror the beam converges to (c) (d) None of these
a point 0.2 m behind the mirror. The radius of
99. The co-ordinates of the image of point object P formed
curvature of the mirror is :
by a concave mirror of radius of curvature 20 cm
(a) 20 cm (b) 50 cm
(consider paraxial rays only) as shown in the figure is:
(c) 66.67 cm (d) 28.57 cm
'"yi""
95. A quality control engineer working at the Da^nnci p. •
Glass Company has invented a new method to (40cm, 3cm)
measure the thickness of glass panes. By directing a ••x

laser beam at an incident angle of 45° w.r.t. the glass,


he notices that not one but two parallel beams are
reflected. He measures the (a) (13.33 cm,-1cm) (b) (13.33 cm,-I-1cm)
distance between the (c) (-13.33 cm, + 1cm) (d) (-13.33 cm,-1cm)
reflected beams with an
CZW
100. A spherical mirror is polished on both sides. When the
automated camera system, Camera convex side is used as a mirror the image is erect with
as shown. (The camera is magnification 1/4. What is the magnification when the
pointed directly at the (45^
Glass' concave side is used as a mirror, the object remaining
Laser
reflected beams). Assuming the same distance from the mirror ?
that the index of refraction
Ca) -1
of glass is V2, and the distance between the reflected -4
.beams is measured to be 1 mm, how thick is the glass ? (d)+-
(a) -4= mm (b) J-mm
V6 V3 101. Rear window of a car has a size of 120 x 45cm^. The
driver sits at a distance L = 2m from the rear window.
(c) J-mm (d) None of these
What should be the minimum size of flat mirror,
96. The man, whose height is I = 1.75 m is standing at a hanging at a distance of 0.5 m in front of the driver so
distance / = 6m from a vertical pillar of height that he has best view of road situation behind the car?

h = 7 m. At what distance from the man should we put (a) (24cmx9cm) (b) (30cmx 11.25cm)
a small flat mirror on the ground horizontally to see in (c) (40cmxl5cm) (d) (20cmx7.5cm)
it the image of the top of the pillar ? 102. From the base of a hollow cone of height h with a
(a) 4.8m (b) 1.2m small angle at the top, a small ring was cut off and
(c) 5.2m (d) 4.2m placed in front of a parallel beam of light (the figure is
97. Refracting angle is the prism (j) (see exaggerated for clarity). At what distance x will rays of
figure) is 30°. The angle of incidence of light reflected from it intersect the axis of the cone.
the beam on the face of the prism ,Vi ,

a = 37°. Determine the angle of


deviation 5 (in degree) of ray the from the original
direction after passing through the prism. The •X

refractive index of the material of the prism n = 1.2. (a) h Cb)-


Round off the answer to the nearest integer :
(a) 30° (b) 37° A
(c)
(c) 7° (d) 0 A
198 OPTICS

103. A large convex'spherical mirror in an amusement park the bottom is . Take n for water = -, Round off the
is facing a plane mirror 10 m away. A body of height 3
Im standing midway between the two sees himself answer to nearest integer,
twice as tall as in plane mirror as in spherical one. In
other words, the angles subtended at the observer by
the image in plane mirror is twice the angle subtended
by the image in the spherical mirror. The focal length rElS.Om.
of the convex mirror is :
(a) 1.5 m (b) 3.5 m •
(c) 2.5 m (d) 2.0 m
104. The drawing shows a top view of a (a)12.1 (b) 10.1
square room. One wall is missing and (c)11.5 (d) 12.9
the other three are each mirrors. 108. A bright point S is on the principal optical axis of a
Target-[gi
From point P in the center of the open concave mirror of radius R = 40 cm at d = 30 cm from
y
side, a laser is fired, with the intent of its pole. At what distance (in cm) in front of the
hitting a small target located at the concave mirror should a plane mirror be placed so that
center of one wall. Identify vector in whose direction after two reflections, the rays converge back at point S.
the laser can be fired and score a hit, assuming that the (a) 30 (b) 35
light does not strike any mirror more than one: (c) 40 (d) 45

(a)-.j (b) i + j
109. A point object is moving velocity 6i+j + 2fe (m/s) in
front of a plane mirror whose reflecting, side is
x2-plane. If the mirror also moves with a velocity
Cc) -i + j (d)-i.i
j - j (m/s), what is the speed of the image in m/s
105, Which of these actions will move the real image point (a) 7 (b) 5
P' farther from the boundary ? (c) 9 (d)6
110. A beam of light of width t is incident on an air-water
boundary at an angle of incidence 45°. The width of
the beam in water is (Refractive index of water = p.):
(a) (p-l)C (b) pt
(1) Decrease the index of refraction n
(2) Increase the distance S (c) (d)
p p
(3) Decrease the radius of curvature R
111. A system of coordinates is drawn in a medium whose
(a) 1,2,3 (b) 1 only
(c) 2 and 3. only (d) 2 only refractive index varies as p = -, where
106. You are required to design a little dentist's mirror to be
fixed at the end a shaft for use in someone's mouth.
The image should be erect and when held at 1.5 cm
from a tooth, it should produce an image twice the size
of the tooth.
(a) The mirror is convex | /] = 3 cm
(b) The mirror is concave with | /1 = 3 cm
(c) The image of this tooth is 3 cm behind the mirror
(d) The image of this to this 3 cm in front of the mirror
107. A spotlight on a boat is 2.5m above the water, and the 0<y < 1. A ray of light is incident at origin at an angle
light strikes the water at a point that is 8.0 m 60° with j-axis as shown in the figure. At point P, the
horizontally displaced from the spotlight (see the ray becomes parallel to x-axis. The value of H is :
figure). The depth of the water is 4.0m.The distance f 9 V2
d (in m), which locates the point where the light strikes (a) -1

(c) {(V3)-1}^2 (d){(V3) + l}V2


GEOMETRICAL OPTICS 199

112. Light incident on a surface separating two media is


(a)AJ^ Cb)
partly reflected' arid partly refracted as shown in 2
figure. Pi, and P2 ^he respective,indices of the two A, k
media and C is the critical angle for the interface. (d)
2 Vm
Which of the equation is correct ? -
115 A medium bounded by the planes x = 0,x = d,y = b,
•y = -b,z = c and z =0 has refractive index
p(x,z) = Az^ -Bx + 2. A ray of light travelling along
positive z-axis enters the medium B at point P

Then the optical path introduced by the medium
(when the light ray emerges out) will be :
Ac^
(a) (b) 4Ac'

Ba
(a) sini = - (b) tani - — (c) 2c (d)
1^2
(c) sinC = tani •Cd) sinC = sini ne. A'coin is placed at the bottom
of an empty hemispherical
113. A ray of light travels r
bowl qf-radius R and is just not
from a. medium of ;
visible to an eye looking over Coin
• refractive index 'p to
the edge. When the basin is
air. ' Its angle of,
filled with water the whole of the coin is just visible to
. r incidence in • the
an eye in the same position. The diameter of the coin,
. medium.is 0, measured
if p is refractive index of water is :
from the normal to the
boundary and its angle

.^
of deviation is 8 with
the angle of refraction . Agraph is plotted between 5 (af (d,|?
and 0 which is shown in the figure. Choose the correct
117 A submarine is 300 m horizontally out from the shore
relation from the following :
and 100m beneath the surface of the water. A laser

61 - 5i beam is sent from the submarine, so that it strikes the


surface of the water at a point 200m from the shore. If
(c) sin (j) = —
3 . 2 '
(d) / = -;^-sin:^ - the outgoing beam from the surface of the water just
' 2 v JT
strikes the top of building standing vertically at shore,
114. An ,insect of negligible mass is sitting on .a block of then the height of the building is [p,^ = 4/3] :
mass M, tied with a spring of force constant k. The (a) 300m . (b) 140 m
block performs simple harmonic motion with (c) 240 m. (d) 170.7 m
amplitude A in front of a plane mirror placed as 118 A planoconvex • lens is ROC=20cm Observer
shown. The maximum speed of insect relative to its placed on a paper on which pir ^1=3/2
image will be flower is drawn. How far
xjmMmnM ~ ~ above its actual position
Paper
does the flower appear to ! t=!20cm
an observer looking down
from top ?
(a) 10cm (b)15cm
(c) 50 cm (d) None of these
119 A parallel beam of monochromatic light is incident on
a plane glass surface. In the glass, the beam :
(a) remains parallel but the beam width increases
(b) remains parallel but the beam width decreases
200 OPTICS'

(c) remains parallel and the beam width also remains 125. Ram is looking at his face in a mirror kept 10 cm away
the same and he finds that his image is erect and magnified
(d) becomes converging (m = 1.8). If he holds the mirror 50cm away :
120. A point object is -moving with velocity (a) He cannot see the image because reflected rays
falling on his eyes are converging
V 0 = 2 i - 3j + 4k in front of a moving plane mirror (b) He sees a magnified and erect image
whose normal is along x-axis. The mirror is moving (c) He sees a diminished and inverted image
with velocity VM=i-4j + 3ic. Find the velocity (d) He sees a magnified and inverted image

vector of the image : • ^ 126. A glass hemisphere of radius R


(a) 5j
(c) -4j + 2k
(b)-3j-i-4lc
(d) 2i- 3j-t-2fe
and material having refractive
index 1.5 is silvered on its flat 1 2R
face as shown in the figure. A
121. A clock hung on a wall has marks instead of numbers small object of height h is
on its dial. On the opposite wall there is mirror, and located at a distance 2R from surface of hemisphere.
the image of the clock in the mirror is read, and it The final image will form :
indicates the time as 8:20. What is the time in the (a) at a distance of R from silvered surface, on right
clock ? side
(a) 3:40 (b) 4:40 (b) on the hemispherical surface
Cc) 5:20 (d)4;20 (c) at the hemispherical surface
122. A beam is incident-parallel on the prism as shown in (d) at a distance of 2R from the silvered surface on left
side
the figure below :
Find the angle between the emerging rays (Given 127. A point object 0 is placed at a distance 2/ from a thin
convex lens of focal length / as shown in fi^re. The
\x = 1.66):
lens then starts rotating about an axis perpendicular to
the plane of paper, (object remains fixed and OA
remains in the same plane), the speed of image when
object is rotated by 2° is :

(a) 180° (b) 140°


(c) 40° (d) 320°
123. If a convergent beani of light passes through a
N-2f
diverging lens, the result :
(a) may be a conversing beam (a) m/s along AB (b) —m/s along BA
(b) may be a diverging beam 90 90

(c) may be parallel beam (c)


2fK(£t along AB (d) ra/s along BA
(d) all of the above 90 90

124. A point source of light is placed in front of a plane 128.A light ray is incident ton face AB of prism ABC as
mirror as shown in the figure. Determine the length of shown in figure. The second prism is kept in such a
reflected patch of light on the screen Z : manner that emergent ray from prism ABC is falling
S-- normally on face A'B' of prism A'B'C. The net
deviation produced by the optical system of the two
prisms is :
A" " "
A-

(a) I (b) 21
, , 31
(a) 48° (b) 112=
(c) 32° (d) 18°
GEOMETRICAL OPTICS 201

129. Consider a system of two thin lenses as shown in f=50cm fo—25cm

figure. An object of height 1 cm is placed at 40 cm from


convex lens. Mark the correct option related to final
image formed by the two-lens-system : -*»—v ,
i f,=2bcm fn=2bcm A B

25cm

O
(a) always diverging
,H >1* M j Cb) initially parallel and then diverging
i .40cm _ 8crri. „ J
(a) Final image is formed at 32 cm on right of concave (c) always parallel
(d) initially converging and then parallel
lens and is —^ cm in size
2.2 133. Two plane mirrors are arranged as r
(b) Final image is formed at 32 cm on left side of shown in figure. A ray has been '
convex lens and is 1cm is size incident on Mj at an angle of 40®,
(c) Final image is formed at 14.5 cm on the left side of find the deviation produced in it
concave lens and is cm is size after three reflections :
2.2
(a) 40° clockwise
(dj None of the above (b) 40° anti-clockwise
130. A point object P moves along line AB passing through (c) 140° clockwise
centre of curvature of concave mirror as shown in (d) 140° anti-clockwise
figure. The object velocity is constant and is 134. A convex spherical refracting surface with radius R
approaching the centre of curvature of mirror. For the
separates a medium having refractive index —from air.
instant shown in figure the image is formed at 1. At this 2
instant, the image" is having velocity along : As an object is moved towards the surface from far
away ftrom the surface along the central axis, its
image:
(a) changes from real to virtual when it is at a
distance R from the surface
(b) changes from virtual to real- when it is at a
distance R from the surface
(a) vi (b) V2 .(c) changes from real to virtual when it is a distance
from the surface
(c) V3 (d)V4 3
131. A point source of light S is place (d) changes from virtual to real when it is at a
2R
in front of a perfectly reflecting distance — from the surface
mirror as shown in the figure. Z 3

is a screen. The intensity at the 135. A real object is placed in front of a convex mirror
centre of screen is found to be I. (fixed). The object is moving towards the mirror, if Uq
If the mirror is removed, then the intensity at the is the speed of the object, Vj be speed of image, then:
centre of screen would be : (a) Vj = Vq, always (b) Vj > Vq, always
(c) Vj > Vq, initially and then Vq > Vj
(a) I (b)
9 (d) Vj < Vq, initially and then Vj > Vq
. , 91 136. Figure shows a thin converging lens for which the
Cd) 21
focal length is 5.00 cm. The lens is in front of a concave
132. The two lenses shown are illuminated by a beam of spherical mirror of radius R = 30 cm. If the lens and
parallel light from the left. LensBis then moved slowly mirror are 20.0 cm apart and an object is placed 15 cm
toward lens A. The beam emerging ft-om lens B is : to the left of the lens, determine the approximate
distance of the final image from the lens (in cm) :
202 OPTICS

(a) 5 (b) 15
(c) 10 (d) 12.5
141. An object ABof length 2 cm is kept in front of concave
mirror of radius of curvature 20 cm as shown in figure.
The upper half of mirror is exposed to air and lower
(a) 5.3 cm (b) 4.6 cm
half is exposed to a medium having refractive index
(c) dlcm (d) 12.7 cm 1.5. For this situation mark out the correct statement
(s) about the image of AB formed by mirror :
137. A ray enters a glass slab at an angle a from air and the
refractive index varies along the slab as ).i = jig - 40cm

where t is the distance measured along the normal, jig


and k are positive constants. If the glass slab is --1cm-:-:-:
sufficiently thick, how far along the normal will the
ray go before it reflects back ?
lig - sin9 lig + sin0 (a) The separation between x-coordinates of image of
(a) (b) ends A and B is zero
(b) The separation between x-coordinates of image of
jig - sin0 |ig + sin0
(c) (d) ends A and B is 2 cm
k ' ' V fc (c) The separation between x-coordinates of image of
2
138. Find the net deviation produced in the incident ray for ends A and B is —cm
the optical instrument shown in figure below. (Take 3
refractive index of the prism material as 2) : (d) The imageformed is perpendicular to the principal
axis of mirror
142. On the surface of a liquid reservoir of depth 3m is a
circular raft of radius 6 m. The surface is illuminated
by light incident from all the direction. Deterniine the
radius (in m) of the full shadow of the raft at the
(a) 66® clockwise (b) 66° anti-clockwise
bottom of the reservoir. The refractive index of the
(c) 54® anticlockwise (d) 54° clockwise liquid 1.25 :
139. A thin lens of material having refractive index p = 1.5 (a) Im (b) 2m
and focal length of 20 cm when placed in air is taken (c) 3m (d) 2.5m
and two mediums of dift^erent refractive indicies 143. A 2 cm diameter coin rests flat on the bottom of a bowl
III =1.2 and |i2 = 2.5 are there covering upper and
lower halves of the lens, respectively as shown in in which the water is 20 cm deep =- ]. If the coin
V 3^
figure. If an object is placed on the principal axis, then
is viewed directly from above, what is its apparent
its two images will form one after refraction from
diameter ?
upper part and other after refraction from lower part.
Consider the object to be at oo, the separation between
(a) 2cm ' (b) 1.5cm
two images formed would be :
(c) 2.67cm (d) 1.69cm
! " n=1.5 "f
144. The second lens in this optical instrument can not :

Mi=2.5 [ii=2.5

(a) 15 cm (b) 40 cm Lensi Lens2

(c) 25 cm (d) 65 cm (a) Cause the light rays to focus closer than they
140. The object is at distance 45 cm from the screen. With would with the first lens acting along
(b) Cause the light rays to focus farther away than
lens we obtain a small image of the object on the
they would with the first lens acting alone
screen. By moving the lens, we receive a different
(c) Cause the light beam to. diverge after refraction
image on the screen, whose size is 4 time greater than from it
the first. What is the focal length (in cm) of the lens? (d) Make the beam parallel
^GEOMETRICAL OPTICS 203

145. The reflecting surface ' y


(vertical)
2x = y2 (j)=slri"
represented by the equation 373
2x = y^ as shown in figure. A <{^^^rizontal)" (a)
ray travelling horizontal
becomes vertical after
reflection. The co-ordinates of the point of incidence
can be :
Ca) (1/2,1) (b) (1,1/2)
(c) (1/2,1/2) (d) (1/2, ±1)
146. A fish sees the smiling face /-\ — 1 (b)
. of a scuba diver =\_y vJ
through a bubble of air Diver Bubble Fish i
between them, as shown,
compared to the face of ±e diver, the image seen
by the fish will be:
(a) Smaller and erect (c)
(b) Smaller and inverted
(c) Larger and erect
(d) Can be either of above depending on the distance
of the diver (d) None of these
147. Ram sees his friend Shyam through a thick glass 151. At what angle with the normal (in degree) should a
slab (n = 1.5). Shyam is at a distance of 100 cm light beam fall on the surface of material with
from him, but he seems to be at a distance 98 cm refractive index p -1.732, if the angle of refraction
from himself. What is the thickness of the slab ? was half the angle of incidence ?
(a) 2 cm (b) 4 cm (a) 30° (b) 60°
(c) 6 cm (d) 5 cm (c) 45° (c) 90°
148. When an object is kept a distance of 30 cm from a 152. A light ray is normally incident on a right angled prism
concave mirror, the image is formed at a distance of of refractive index 2.1 as shown. How many total
10cm.If the object is moved with a speed of 9m/s, the internal reflections can the ray undergo inside the
speed with which image moves is : prism before coming out of the prism. Consider
(a) lOcm/s (b) Im/s possible cases :
(c) 9 m/s (d) 5 cm
149. A luminous object is kept is at a distance D from a
screen. In order to obtain the image of the same size as
the object, the focal length of a convex lens to be
inserted between them must be :
(a) D (b) Djl
(c) 2D (d) D/4 (a) 1 (b) 2
150. A light ray in water strikes the water surface at an (c) 3 (d) 4
angle of 60° to the normal. The path of the ray after 153. Figure shows converging rays on a glass slab placed in
striking is : air. 0 is point of intersection of incident rays and I2
are image formed after refraction at and S2 surface
respectively. Mark the correct option (s) :
air
\ ;• j "" " S2 i
watern^a,er=..3-: " • 'incident ray
204 OPTICS

(a) For refraction at $2, Ii is real object and J2 is real (c) Sign of acceleration of object is positive and image
image negative
(b) For refraction at Sj, 0 is real object and is its real (d) Sign of acceleration ofobject is negative and
image image positive
(c) For refraction at surface 5^,0 is virtual object and 155. Two objects are placed on the principal axis of a thin
is real image converging lens. One is 10cm from the lens and the
(d) For refraction at surface S2,li is virtual object and other is on the other side of the lens at a distance of
J2 is real image 40 cm from the lens. The image of both objects are in
154. A particle is projected above a plane mirror. Taking up the same plane. What is the focal length of the lens (in
as positive and down as negative. Mark the correct cm) ?
options : (a) 14 (b) 15
(a) Image is moving upwards during upward motion (c) 16 (d) 20
of object
(b) Image is moving downward during upward
motion of object
^GEOMETRICAL OPTICS 205

ANSWERS

Level-1: Only One Alternative is Correctl

1. Ca) 2. Ca) 3. Cd) 4. Cb) 5. Cb) 6. Cb) 7. Ca) 8. Cc)

9. Cb) 10. Cc) 11. Cc) 12. Cb) 13. Cc) 14. Cd) 15. Ca) 16. Cc)

17. W). 18. Cb) 19. Ca) 20. Cc) 21. Ca) 22. Cc) 23. Cd) 24 Cd)

25. Cd) 26. Cc) 27. Cd) 28. Cc) 29. Cd) 30. . Cb) 31. Cb) 32. Cd)

33. (0 34. Ca) 35. Cd) 36. Cc) 37. Ca) 38. Ca) 39. Ca) 40. Cd)

41. (a) 42. Cd) 43. Cb) 44. Cc) 45. Ca) 46. Ca) 47. Cc) 48. Ca)

49. Cb) 50. Cc) 51. Ca) 52. Ca) 53. Cd)- 54. Cc) 55. ' Cb) 56. Cb)

57. 'Cc) 58. Ca) 59. Ca) 60. Ca) 61. Cb) 62. Cc) 63. Cb) 64. Cc)

65. . Cb) 66. Ca) 67. Ca) 68. Cc) 69. Ca) 70. Cb) 71. Cc) 72. Cc)

73. Cd) 74. Cc) 75. Cb) 76. Cc) 77. Cd) 78. Cc) 79. Cd) 80. Cb)

81. Cc) 82. Ca) 83. Cb) 84. Cd) 85. Cc) 86. Cd) 87. Cd) 88. Ca)

89. Cb) 90. Cd) 91. Cd) 92. •Ca) 93. Cd) 94. Cc) 95. Cc) 96. Ca)

97. Cc) 98. Ca) 99. Cd) 100. Cb) 101. Cd) 102. Cb) 103. Cc) 104. Cd)

105. Cc) 106. Cd) 107. Ca) 108. Cd) 109. Ca) 110. Cc) 111. Ca) 112. Ca)

113. Cb) 114. Cb) 115. Ca) 116. Ca) 117. Cd) 118. Ca) 119. Ca) 120. Cb)

121. Cb) 122. Cc) 123. Cd) 124. Cc) 125. Cc) 126. Cd) 127. Cb) 128. Ca)

129. Co) 130. Cb) 131. Co) 132. Cb) 133. Cc) 134. Cc) 135. Ca) 136. Ca)

137. Cc) 138. Cd) 139. Cd) 140. Cc) 141. Cc) 142. Cb) 143. Ca) 144. Cc)

145. Cd) 146. Ca) 147. Cc) 148. Cb) 149. Cd) 150. Cc) 151. Cb) 152. Cb)

153. Cd) 154. Cd) 155. Cc)


B Level-1: Only One Alternative is Correct] PinP
( 1

1. (a) 8 = e = A 20

b = i + e-A P- 20 cm
z-z'i' 'i :•
6 = i = 60°
^ y N-l
e = 60°
A = 60° 60° I Position where both I
Ixsin 60° = |isin30° theimages coincide I

—x2 = V3
2 8. (c) From 1^^ interface rig <
From 2^'^
2^^^ interface > n2
3. (d) For dispersion without average deviation but since 03 < 01 hence n3 >
(.ny-l')A = -(ny-l)A' So n3 > rij > ^2
for no dispersion
=^ <1 [L^ <
A(5y-8R) = -C5V-8'R)A'
(n^ -1) Aco = - in'y - l)A'(o'
^ 00 = (o'[.-. -l)Aa) =-(n^ - 1)A']
because 8^ - 5^ = co8 -©
n„-n„
_ "y 151-149 0.02
0) - = 0.04
-1 1.5-1 0.5

4. (b) 50x10"® X3x10® X—


10
= 150x10"® xlO^ 9. (b) 1 = 1. 1. ...(1)
= 1500 m
f ///2
= 1.5km Now, =(|x^-|ij
6. (b) I, is image of pin which is formed at h below Jv Jr V^l ^2
20
surface, also apparent depth of object = — _i i_
+ 0^2v -ii2r)
20
X A fi
Thus -h = = 15 cm Thus must be zero for achromatic combination.
^3
I.e., ^CO, + _A
CO 2
= o^ ...(2)
A fi
Solving eqn. (1) and (2)
GEOMETRICAL OPTICS 207

1+1 1 j
f ^ f2 = 1-:^ / —= — =>a = 15cm
(0- COi -24 d 40

12. (b) 8 = i + e->l A = 45' 20. (c) y=(p-l) = 3


13. (c) For upper portion :
/i = ( / J = X 20.= 40 cm 3= (1.25-1)1 — ...(1)
Ii2 - 1 Ri
Ki R,
K2J
1.2
and -2 =
1.25

yRi R2
1 ...C2)
3_ 0.2^
—0.5u = 3.75 — 3u
2 1.25-p ^ ^
^ p = 3.75/25 = 1.5
21. (a) Use the concept of pole for the paraxial rays.
7 cm
For lower portion, —- (A) 23. (d) = 1.4
El - 1
1^3
1.5-1
X 20 = -25 cm
1.5
-1
2.5
X
Object is at infinity, images will form at focal 7 cm'
points. \f"' • "

Ax

Hence the distance between two images will


X = 5 cm
= 40 +1-25|= 65cm
Ax = 2 cm
15. (a) Deviation produced by one is cancelled by the 1 1 1 1
other. 26. (c) — = — + — + —
p A h A
Use 6 = A Cli -1)
17. (d) For lens, = ~
=(Pi-i)ri+i 00 i<
+ (P3 -1)
R
i
R
V u f
+(P2 -1) 1
Magnification, m = - —= —-1 R
" /
Graph between m and v will be a straight line _El 1 _m^2 ~ 1 _!^i ~M-2 - R
R R R
with -1 intercept on m-axis and slope 1^1-^2
tan 9 = 1//. '2n-l'
29. (d) dsine =
Putting m = 0, it gives n = / 2 id
(b), (c) and (a) are correct. =» o -1 < —
2n 2d
19. (a) The lens forms the image on the centre of X
curvature of mirror, so that the rays will retrace d 1 0.0756x10"^ 1
their path i. e. => n<- + - = ^ + - = 180.5
^ 2 420x10"^ 2
V = -24 (for lens) 2n-l 9 27
2m-1 7 21
n 5 14

m 4 11
24 cm
36 cm 5 + (n-l)9<180
30. (b) Pi sini = P2 siur
2V3 sin 30° = V3 sin r => r = 90°
33. (c) u = -24

V =
uf
u+f
OPTICS

-24x15 124x15 3-1


,..v =''^ = 40
-24 + 15 8 y 60
D = 200^ 24-40 = 136cm y = 30cm
40 Final magnification = (-4) x (-3) x (-1/2) = - 6
-0.03 -24 Final image is real, inverted and enlarged.
40 39. (a) \x varies with X which is different for different
R. = — X 0.03 = 0.05 mm
^ 24 waves.
(j
d = 0.05 x 2 + 0.06 = 0.16 x 10"^ m Velocity in a medium V = —
^ ,^00 X10"^? x.X36,x,10"^ h
P=
d ~ 0.16x10
-3
which is different for different media
p = 51xl0^.m = 5.1mm Frequeiicy of a wave remain same in different
media.
35. Cd).^ =4''"
"dt u- dt
41. (a) For the near end of the rod,

Since, direction, of, object velocity, and .image


1 =1 +1 ^ --
f u V u-f L-f
velocity, is always same, thus, image should lie in
object side i. e. u < f For far end of the rod,
A + D' 1 11 111 21-f
H
sin
/ u+L Vi f 2L"(/)(2L)
36. (c) \i - gives
A (/)(2L)
sin Ui =
2L-f
2cos(A/2) = ^ or cos(A/2) = 1.732J2 = V3/2 Length of image = v-Vi =
If (2L)(/)
or A = 60=^ L-f (2L-/)
_L/C2L-/)-2L/a-/)
37. (a) p = —when 9 is small
0 a-fX2L-f)
38. (a) _ 2L^f - Lf^ - 2LV + 2L/^ _ ;Lf^
a-/)C2L-/) (I-/)C,2L-/)

+1 42. (d) This is.solved by sketching a-ray.diagram and


using.similar ..triangles.
'final 25
120cm
20cm
30 cm

60 cm .60 cm
11 is first image of lens and object for mirror and
/2 is image of mirror and object for second _ ;f, ;
refraction at lens.
43. (b) 5 =,angle -of minimum deviation =, (jr,-1) A
For refraction at lens
112. 1-1 1 _ 1 Sair = -1)^ ^and = (i^ig - 1)A
V (-25) 20 V ~ 20 25 100
[(3/2)-1]
V = 100cm
(lI4. ^1) (3/2) - 1
For mirror (9/7)
1
l_ = zl ^ ,(1/2) _ 1.28-3
V -20 15 [(21/18)-1] 2 3
1 1
V = - 60 cm 44. (c) i = 40
V 60 ^ A = 60°
For final refraction at lens (ray moving leftward) for Smin =i = e
1 1_ = 1_ Kin =(i + e)-A
V 60 20
= 80 - 60°
= 20°
^MHRICALOPTICS 209

For the surface AC, 60+60


sm sm
A ~2
\^ = = V3
(60
30°\ sm •
— sm —
l2 u
53. (d) When angle of incidence is e = 40"=
(from reversibility of ray)
1
also 5 = 70°

'iof 70° = ii+40°-A


ii,= 9b°

—1 / 60y\^
'B ^2 q
54. (c)

V3siii30°= sinr
=> ,r = 60°
^clearly.angle^betWeen ray-2 and 3 is 90'=
•;47^'(c) •vJu^-2 ' ,55. (b) (1) sin 60° = Vs - ^ x\sin 90°
V3
•'nneW-,"/v = l/2
Vs
— = V3 - - = x
1
2 S
,o 56. (b) 41 = +1^3-^2
^ / : -2R . , • -R

+li3:-|^2 <0
According to principle of.reversibility of light - R •2-:
image, size will-, become half. ': ,2^3-(pl +H2) >0
Mirrbr is, convex in> nature •and produces
diminished iinage>at-closer distance but due to ^7, (c) _-The silvered, lens canbe replaced by.a, mirror of
. smaU'size/it;appears di'staiit. , focal length given as ' '
K;50'./(c) Tliey,niust be at phase difference of •27c/3.,from'
I . =oneanothertyhen twb.inteifere, phhsprydllbs':
a\ . , .

0 .

•^•5'10 cm5
f = 20 cm f = 40 cni

' thus^result will.be still


• 51.:(;a) .2r ^='60? ,=>. - r =-30° ' \ffn fl
1 9
^ = O^JlFj^ =-20
Fm 40 ^
For Ions V=
u+f
-10x20
1) - = -20
sih3O®="1.5sin0 -10 + 20
l5 sih(6D - 6) = p. sin 30° »V
. 1.5sin.6O°cos0-l-.5cos6O°sin.0 =psin30°
r
52;v(a) 8 = i-+e-A ' 7d .2
^> 1'1 ' '/ L
dihin ~ 60° when i = e —20 cm \j *- 20cm
60°=2i-A = 2(6b°)-A • " 40 cm
A = 60°
210

So this position has to be centre of curvature of r5+r6 = 90° ...(3)


mirror in order for the ray to retrace its path Since the emergent ray is parallel to incident ray,
so d = 40-20 = 20 cm. r7=90°-i ...C4)
58. (a) -l_l
- - =
= -i sini = Pi sinr^ pj sinr2 = ^2 sinrs
V u f !i2sinr4 ^Pssinrg
Pa sinrg = sinry
V /
m =
V
— =
/
u u+f
f / Using equations (1) to (4) in the above
-12 + / -16 + / equations
m is same in both the cases sini = Pi sinrj ...(5)
/ = 14cm p 1 cos Tj = p 2 sin Ta ...C6)
p2Cosr3 ^Pasinrs ...(7)
Pa cosr4 = cosi ...(8)
Carrying out the manipulation
612 (equation-5)^ - (equation 6)^ +
h2 = <3/(2^12) (equation - (equation 8)^
60. (a)
612
]'/ ^2 Ml hi-= d/(2^i)i wegetl +p^=p?+p^ . .V
A :hi
O

0—Initial object position - • 64. (c) In the figure AB. is pole". The shadow formed of
1— Final image position the pole is'ihdicated as.BR = BS + SR
61, (b) The path of ray AB after refraction at opposite BS = QC=AC '• '- " ,
face and reflection at opposite face as shown in 'siri45°' 4 3 ••
= — o r sin r = —=
figure. We have to find ZDCE sinr .-..S- , ' 4v^
,-SR-pQStanr-ECtanr = Itanr

tanr =
1.6

By snell's laW-(sm6b?/sinri)-='V3
or sinTj = sin6Q°/V3'-= 1/2.
Tj = 30° r and. hence ra = 30°
Considering refraction atC(sinra/sinia) = l/Vs
or sini2 = a/3/2
12 = 60° SB = 0.2
ZECD = 180°-(i2 + ra) SR = 0.625
= 180°-90° =90' RB = 0.825. metre
62. (c) 5= +^2 - A
65. Cb)
45° = 11+12-60°
1^+12=105°
ii -ia = 20° xo'rS-f
or '•2ii =125° - Solving / = 2.5 • "
ii = 62°3Cy, ia = 42°30' / 66. (a) B must be appearing at'ihfinity
63. (b) The figure displays the given problem. ^2 _ M-i _ 2"Hi'
rj + ra = 90° ...(1) oo -2R -R
r, + r. = 90° ...(2)
JEOMETRICAL OPTICS 211

Solving — - 2 1- sm90°= ^/2sinr


^2 : r = 45°
67. (a) 73. (d) The velocity of image,
v' = v = 2(u-n)n
h= 1.5m!
- (i +j) - 2[^(i +j)- ij(-i) =(-i +j) cm/s
Virtually visible length
- Actual length
2 cm/s

•M-osin 90° = ^0 sinG


3
2
sinG = —
• 3
Velocity of image with respect to mirror,
f cotG = — „ A. ...
. 2 V = f - Urn = (-i + j) - 2j = (-i-j) cm/s
h
cotG = . , . - . ; . •' velocity of image with respect to object
• b ' - ;
" , -h''-• '2 3 ' =(-iH-.j)-(i.+ j) = -2i,<m/s
1P.—-TT = 1.5 X — - m , Unit ve^toj". in. the. direction of reflected ray
• cotG-. '45 Vs '•
68. (c) 1 +j). • ^ / •

74, (c) Let Gbe angle of incidence and a be the angle of


refraction for'the exfreirie rays
j £ coA
i At equilibriunu i • i.- "s/2M' ^2
jposiUon ' ' TtoB ^ motion ofjmage
V^'=v42 = 42WA
69. (a) -Since image .is read arid-inverted.
70. (b). As i. the rayLbends-away from normal.
71. (c) j

de'creasing
Q.|"o>Q .

When-^r varies-cbntiiiuosly G^90' . \ t , sina- '• ^2 sma


cin w ' /n

To pass through plate b < 90° In AOPQ, Z OOP = 90°+G - a


• 3 •
—sinG = 1.2sinb Applying sine rule
. R •r "
for 0max (t)=w::- sin(90°+G-'a)' sina
3 2.4
-siri6 = 1.2 => sinG i= GV53°"
2 V ' 3
R sin 30°
72. (c) By Snell's law . Ta = sin-30° =
sm(90°+45°-30° ) cos 45'
R •

1 - .V3 1 ,i" ; '(Vs+i)


'.S:
V2 2 42 2
, OPTICS
212

75. (b) nCy) > no


horizontal ray 30°'^^ The material with the
;(R-y)
greatest known
refractive index is
diamond but even the
Horizontal refractive index of this
material does not reach
the value = 2.5. To find the maximum
nai,^=1^00 J angular size of arc, the refractive index changes
from no = 1 to n^^ax = 2.5.
Ixsin60'' = iasin30°
^ = VS = 1.73
sina=-^C90°-a) =cos"^f
76. (c) The rays will first get refracted, then n-times -1 1
reflected and finally again refracted. Maximum angular size of arc is cos
2.5

using ^ - Hil =1^2—!£iL fQj- flj-st; reft^action, we 78. (c) Formed by direct reflection, real
^ V u R-
have
image coincides with object,
then u = R = 20cm, 11
qi-i)
image is at a distance v =^R±l
R
Let F be the focal length of
V; = R system formed
12 1
« . 1 1 2 • F /lens /mirror
For first reflection, —+ — = — --
V u R 201-1) ; 2 ^ 2^1 _ 1_^-1
1 ^(^i-l) -2_ .1 _-(3ti-l) R R R V u
Ui nR R |iR
[L =
For second reflection, , (R ± 0 RJ 2
1 ^(3^1-1) _-2 1 -(5^1-1) for
V2 [lR R HR
Similarly after reflections,.
'^ 2(R + 0 2x28 56 7 '•
.(2(;i + l))X-l This is not possible
C2R - 0 _ . 32 _ 4
~'7~^ For V = R~l, \x =
2(R-0 2x12 3
'7'' I I.' '•ho Ui -
Finally using Hi- _ tiL = 79. (d) For lens
V V R
2^
[ll'=W,\i2 = 1. = -^n ^1 "1!
^_^'2n+l)^-l].^. 1-V V Uj /
R R m^ = — =
V-,
,/^1'' "1 +/
R
and m-, -
/
20m + p.+ 1)
"2 +/
R 3
Given v e = —; u = ,— J. J_ = *^"2 -"1) ("2 -"1)
^ 13 2
R R_ =» (3n +1) = 13 => n = 4. TTli mo
ffl2 / (m2)"^-(mi)'v-l
13 80. (b) Intensity of central maxima
2l -n + - - l 2 >,^12
2 2 . =(2Ao)' =4A^ =4Jo
77. (d) By Snell's law nsin a = constant Intensity at distance x from the central maxima
At y = 0, a = 90°, n = Hq At n = n(y) is half of the maximum intensity if
(1)_4/,
n(y) sina = Uq sin 90® I. =4JoCos^^ =
(R-J) 2 (b 1 d) jt J, ^
"o = nCy)(sina) = n(y) R if cos ^ = -=^>- = — =^(b = —
fGIOMETRICAL OPTICS 213

^ 2nd n ' XD 500x10"^ xl 91. (d) V=


uf
^ ;c = — => X = — - = -60cm
XD 2 Ad 4x10' u-f
= L25xlO'^m
— = — => hj = -25 cm
5 u -
B

V
92. (a) Isin0 =|sin37° ..

, 81- (c) h sin0 = - x - = -


3 5 5
\ i<53'
fish 3i,e=
V 25.. 5
sft(ap) = l^ = -x3 = 4m
93. (d) Along principal axis
82. Ca) V,p =|x6 =8in/s.
O ,
shift =—£— /.
. COS0
83. (b)
. .= /
1 - COS0
131 > !> ^
COS0 •2

/T 4-X /02
4-x amplitude =
2x 4 •
O ' '' 112
94. ic) - +— =-
-0.5 0.2 r

l =l;r =0.667m
r 2,
Since in the same time in which mirror moves by 95. (c)
X, the iniage moves by 2x. So same relation will
carry in velocity as well as acceleration.
84. (d) Since light is going from rarer to denser medium
hence no TIR will occur.
Hence option (d) is correct. ••
85.-(c) / =10cm '- m - ^ smi
r = 30
' " • /-" smr

m =
1

j-AB '^'42, '
•---2 : - ; -2~10-ii so CD = —;BD = = , — mm
"10-U'= 20 => u = -10 - • - - S ,2 -v^tanSO" ,V2

88. (a)
ftz: (a)
96. /• \ 6-x =.•—^
1.75 => x>= 4.8m'
X -,7,- '
^^+41r.2/2;d = 2|A|-|/i|
i" u 97. (c) Isin37° = -sin0
3

sinri = —/Ti ='30''h " •'

^ _ r2 =0°
e = 0°"':
6 = i + e-A = 37°+0-36°=7'
90. (d) i = 65- =» e =-25' 1 L .1 i.
=> Angle of mirror' "! -

=> 40°+25''=65? -
; .• -
214 OPTICS

Z'
3i - 6j + 2k 2i - 3j + 4k y 9:1^ JL.
98. (a) 3 0.45 3 1.2
//
y = 7.5 cm ^ X = 20 cm
3i - 6j + 2fe

2i - 3j + 4k 6 + 18 + 8
-2 (3i - 6j + 2&) s 9O°-0
7xV^x7 102. (b)
98i - 147j + 196k - (1921 - 384j + 128k) ,r:10-
^/^x49
0 - (90®-e) = 20-90°
-94i + 237j + 68lc
49V^ tan(20 - 90° ) = -
r
1
99. (d) = 2tan0 -r
- cot 20 = — =>
V 40 -10
1 - tan' X

A.
u ~ 10 ^ 2x
h r h
1 _ -4+1 _ -3 — = — => X = —

V 40 40'
_ -40 103. (c) — = 6, =0
10
• 3

u = -40;/ = -io 02
-40X-10, -40 1m
V = cm
u~f -40 + 10 +5f
j+5+f
-40^1 5cm 5cm 5cm 5 5cm'
L
!}l = fti _ 5/
3 -40 3
1 5+ /
=> hi = -1cm
/
100. (b)
00 =
5+ / /, .::ai
. . 5/ 25 +10/ . 2
D T 1' C I. -
5+ /
2/= 2.5 + /
/^2.5m
104. (d) tan0= ?^ ="^ "-^' '
2x
f —u —R 3k
2xy - X- -yx'
x-Y;
3xy = X 2
X

^=3
^ X 2x
-XX + - J
=> 4K = K + 2x=>x = —
105.(0
V —S R
n _ n-1^1
101. (d) 45cm
v~ R S
n n-1- - 1 - K
u > 0 ^ S >
R S n-1
GEOMETRICAL OPTICS 215

dT ^ —4- 110. (c) i4B is the width of the beam in air and CD is the
V width of the beam in water of refractive index jo..
1 = 1- J_
1-1 By law of refractive |4, = ^
V R riR nS R nS smr V2sin,
106. (d) m = 2 =
f ^ / or smr -
f-u /-C-1.5) V2^
2/ + 3 = / From the figure
^ ^ uf _ -1.5 X-3 _ 1.5 X3 = CD
3 cm sin(90°-r) =
u-f -1.5 + 3 1.5 AC
CD
107. (a) tana =A =1^=-3.2 \a
cosr =
2.5 5 AC
d- AB
tanp = K, AC
= sin 45'^

16 4.0 AB = t AC =
Isma = 1X , = —sinQ
3 sin 45^

12 CD = AC cosr
sinp =
f cosr =tV2|l—1
12 sin45® ]j 2\L^
= --2
Vl^ 4
48
•d = 8+- = 12.Vm 72^1 |X'
111. (a) Apply Snell's law between the'points O and P, we
108. (d) For n mirror 1 .
have
u = -30
ill = l 2x sin 60°= (sin 90°) x
V u f f = -20 (1 + H^)
1_JL___J_
u 30 ~ 20 2x—= l x
2 (l + H^)
1 ^_1 l_
V 30 20 (1 +h2) = -^, H =
i; = -60cm - 73
So x = 15cm
il2.:(a). - tani
R=40

P2 1
Pi sinC
I£2
tani = til
1^1
^2
smi =

VmAmI
So distance between two mirrors = 45 cm
113.(b)
f

109. (a) Uji = 6i + 2ic


! air
^01 ='^M-L =~C^01
=Vmx =-i'o± =+2i-j'=3j ' ' - !

= 6i- 3j + 2k
Ci) ^
u, =V36+9 + 4 = 7m/s

0 < C
216

5 —({) —0
E

1 OJ '

II
!
O
0/
A\ 116. (a)
^
y "h
*
Fig^(i) _
(ii) y 0

6 = -- c
2

• . 5 = -
. ' • 2 ygjij)
.
As-show in-fig. (i) when the hemisphere was
. j
empty the ray from the extreme end C of the coin
CA passes just above the eye E. •. -•
• • re.",".'" -
When the hemisphere is full of liquid of
i' '
refractive index the ray DA from the ne^ end
(iii) of the coin after refracdon just readies thd eye
and hence the whole coin in visible.
T, a 1 sini DM/DA '
From fig. (ii), — =
Q>c p sinr CM/CA
5"=7u-28 .(H-a)'
|X = = sin~^(|Ltsin83
sin0 ^ .
' (.R+'ay\
(1) 0 < C:5 = sin~^{|i sin0)-0—^nonlinear in the
lim. 0^ C, (from: left),-
^|(.R-hf +iR+af
lim 5, =--c'' 1 _ (R^a)^|(R-hf'+(R~d)^
0-»O" 2 ^ .
(2) 0>C:5 = 7c-^sin-^- linearly 5 decreasing
(R +a)V(R-hf+(i?-p),V.-
as 0 inereasirig
From-fig;.(iii),^a^ = h(2R-/i)y, - •
\ -1's 1
lim §2 = 71 - 2C "i O ' .
=2hK- •
Q-*0'
\

6o = 25
I,I
p (R +a)72R(R-d)' ' W+a '
[Discontinuily at 6 = G LCL,?^ RHL], y'-'-'R-h' • ' • 2" "R+'u"'
114. (b) The maximum velocity of the insect is AJ— • ••p2'-RVd'.'°^ •V'R-a
VM
p^R-p^a = R + a
Its component pe^endicular to the ^mirror is a(p2+l) = R(p2-l)
A.— sin60®. ." j'>-1 . -i >7
'M . . _ Fig. (in)

Thus, maximum relative speed =


-y 'V^ry. :y\; • 2 Mm
Diameter of coin = 2d =
115. (a) The ray-^will always be normal, its x and y
: y(ii'-Hp,
coordinates remains same
. I? C- " 4^4 , /
ASL^-f J(Pi-l)cte=y-,,_ •
0 ^
LGEOMHRICAI^PTICS 21

117. (d) Situation is shown dearly in the figure 122. (c)


sin 25°

From Snell's law, smr 1.66


sin6 — :?
i_JL ! 1

dn0 2 i •
4/3 • ^5" .

4
sin92 = *-200m-» • •

3V2 ^23(r)Xte •

200
tan02 = ; Shore
300m
(h -100) i
J\
I *
•" '-vi/
/z =^ +100 \
tan ©2
f 200V2 r = 45= So, 0 = 40'
+ 100 m = 170.7 m'"

118. (a) For refraction at the spherical surface,


123. (d)
3 '
^ li'i = li" = -20cm,u = ?,R = 20cm
I'n':: 2 ,j / '
1^2 •
'i=" v. <.u ^ ' R- • 124. (c) Ray diagram is as
i • 3 ,%!'1-1.5 , . shown in the
f *

w.-2xC-20j'-' 20 " ' figure below ' 1- \ \ ?] L •

SO PM '
. Ol—\">
• v/
Here, — =
.

1 X'
=> v = -10cni • - n Xm
! ' L
OP RM - : > 1 -X \

119. ia) The situation is shown dearly in figure


RM = 2L
X
A / X
Q, •

ST QM L •
Also — =
TQ SN < D < - Kl H

SN = -
....2

RS = RM-SN = —'
2

125. (c) 1.8 = /


We have,'"' • . i >r ' ' '
i 'I '•i' ' '.'/+10 .-j-
X = PQ cos i, - • y = PQ cosV-.' •
1.8/+ 18 = /
So, ' y ^
18 = -0.8/ •' • -
i.e./.inside the glass^ therbeam reihaihs parallel
but increases iri-width.~'" V ~ / =-22.5 cm
^ %i ^
' ' ' ' In second case, u = -50-' ' - ' '
120. (b) . Using vi+v*o =^2vm [Pe -careful that this
=> object is beyond C. ' ' ''
/ ' equation is valid orily along^the normal i.e., in
present case along >:-axis] image is inverted and diminished
^ Ui = 2x1-2 = 0 126. (d) Here, three optical phenomena take place-first
refraction, then reflection, and finally refraction.
So, the velocity vector of image is
For refraction at 1®^
vi=0i-3j + 4fe

121. (b)
Clock Image
15
V
OPTICSi

i; = oo So, 8 = 32°+16° = 48°- clockwise


L e.y rays after refraction get parallel to axis and 129. (c) Let image formed by 1^* lens be at distance
strike the mirror normally, get retraced back and from convex lens.
the final image is formed at the same place Ill
where the object is and of the same size. Image Now, —'
-40 20
would be real.
=> Uj = 40cm
127. (b) As the lens rotates by 6, the OA also rotates by 0
as shown in figure. Primed letters in figure are Size of intermediate image is, — =^ =-^5.
corresponding to rotated position of lens. ho u -40
yL' L
= -1 cm :
- ve sign tells that image formed is inverted.
0^
>A
\^^C9 We can find this image distance directly also, as
object distance is 2f.
This image formed by l" lens would be treated
as object for 2"^ lens, let final image forms at a
The object distance along the OA in rotated distance v from concave lens.
position of lens is u' = -2/cos0. 1 1 1 ' ' i\" r
= —r u =-14.5 cm '
From lens formula, V 32 -10
1 1 1 -ve sign tells that image is formed on left side of
V -2/ COS0 / concave lens.

2/COS0 The magnification produced by 2"*^ lens is,


v'.= as 0 is small so v' comes out to be
2COS0-1 V - 14.5
TTlj = — =
+ ve. m 32
So, the image distance along initial OA is, •• 14 5
So, size of final image is h = —^ x 1 = — cm
1
2/ 32 2.2
u = u'sec0 =
2COS0-1 So, final image is virtual, erect and diminished.
Velocity of image is, - 130. (b) As the object approaches C, image is also
dv 2/
X (-2(0 sin0) approachingC, so velocity of image'is alongV2.
dt (2cos0-l)^
where 0 = cot

For 0®= 1®= — rad, sin0 and cos0 - 1


90 ' ' 90

4/co X
90 4/itco
image m/s along BA. Aliter : Let us draw the ray diagram/ at two very close
(2-1)' 90
instants,',^amohg ohe'of which is the given
128.(a) Here we have drawn the ray diagram with aU instant. It si clear from the ray diagram that as
angles. The values of all angles can be found by object moves from P,to Pj, the' image moves from
using concepts of prism theory, refraction and I to and hence velocity of image, when object
basic geometry.
is at P is along V2-
131. (c) Let the power of light source be P, then intensity
at any point on the screen is due to light rays
directly received from source and that due to
' after reflection from the mirror.
P P '
I =
Deviation produced by 1" prism is, Ana' An X (3a)'
5i = 53° + 53° - 74° = 32° clockwise P
Deviation produced by 2"^^ prism is, 1^1
Ana' 9
82 = 53°- 37°= 16°- clockwise
6E0METRICAII0PTICS m
219

ti' =-12.5cm
4na' 1 _ 2 _ 1
When mirror is taken away, v' ~ 12.5 ~ -15 •
I - -P - 9/
^ 47ca^ -10 v' 25 15
1 _ 12-10
v" 150
y' = 75 cm
132.(b) 17". =-95 cm
1 ^ ,
Jl a.
Initial position Later position 95 5

i' '' Initially, the object for lensB (the image formed = 5.3 cm
, by lens A) is at its focus, so rays are parallel. 137. (c) Ast is increasing, p is decreasing, so refraction is
' As the separation between A and B decreases, taking place from denser to rarer medium as
the object for B is lying always from focus and light is proceeding in the medium.
: hence, the rays get diverged. , Using 'continuous varying refractive^ index, let
133. (c) 5i = 71 - 2 X4p° = 100°- clockwise the ray reflect back after' travelling t = to along
§2 =,It - 2,x 40° = 140° - anticlockwise
the normal, then af this' location," angle of
7Z '
refraction is—. So
2 •

1X siriG = (po - kto) Xsin— .


2

83 = It - 2 X20° = 140° - clockwise tn


_ |Po - Sine

Deviation (Net) = 140°- clockwise ' k
138. (d) Deviatioii produced by mirror is
Sj = 71 - 2 X60° = 60°; clockwise
i34: (c) = Deviation produced by,prism is;.-,\^'
• V .1, -U R.-
62 = (2-1) X 6°= 6°; anticlockwise
••

V R • u So, net deviation,, ^


2.5uR 6 = 5i +62 ,
V =
.1.5u~R ....... = (60°-p°); clockwise ~ 54°; clodcwise
I v = 00 at 1.5ii-R = '0/_ '' 139. (d) Focal length ^61* upper half is ' ''
! - 2R
! =^. u- —
' 1.5-^ 1
: 3 , • . . . ,/i,= f. '.i
Jair 20 = 40 cm
1 ^. /•-. • < 'J- ^ / . • r ' ♦ >
' -" For "u Ovirtual image^':"r,".%
-MVi
a.2
i . , , . .;.tj , . • . '
' i 2R ' t Focal lerigth for lower half is.
, •' i£'> '—,'u 0,Veal'image' '
' 3
So, image is changing from real to virtual at lt-1 1.5-1.
_2R /2 - fair 1.5
X 20 - -25 cm
,F/F2-1
^3;'' ' ' ' ' ' 2.5
135. (a) As object moves" from'infinity to pole of convex If the object is at infinity, two images will form at
mirror, .image moves from focus to pole, so, corresponding focused.
I Vj < VqI always. • •• ' So, the required separation is
136. (a) 1-J- =l ,. X= |/i| t I/2I = 40i+-25 i= 65,cm'"~''"'-C
u -15 5
140. (c) -
1 = 1 - Jl V ^
V 5 15 '' , 4/j2 ' •
-ho .
1; = 7.5 cm
220 OPTICS

(45-x) => 2x = l
2 -X 1
=> x = —=>y = ±l
=> x = 90-2x 2

X = 30cm 146. (a) The bubble acts as a diverging lens.


=> Image is virtual, erect and diminished
30 15 /
147. (c) NS = 100-98 = 2cm = t|l--
/ = 10 cm n

141. (c) As focal length of mirror is independent of 0^


2 = t t - 6cm
surrounding medium, so the image of ends A 1.5
and B formed by concave mirror are having the 148. (b) Vj =-m 2 Vq
same x-coordinates and image is perpendicular
to PA. 10
X9 = Im/s
For y-coordinates, :30

1+-=- r 149.(d) r
V u f j A'

111 !
V 40 -10
1 ^ J_-J_ B'
V 40 10 |hi| = ho => m = ±1
^0 object is at 2/ and image is 2/ from lens
V - cm
3
From magnification formula D=4/=>/ =£
.4
A'B' = -cm 150. (c) 1

3
From magnification formula
A'B' = - cm
3
142. (b) lsin90°= 1.25sin0
0 = 53° - sin 60° = ^ =A >1=sin 90°
3 3 2 VS •
— = tan0 x = 4
Ray will undergo TIR
r = 6-4= 2m 151. (b) lsini = V3sin—
2
143. (a) Dimension parallel to
surface does not change 2 cos i = Vs
1 S i
cos— = ^
2 2

144. (c) - = 30°


, 1

iThe second lens is diverging i = 60°

. The second lens is diverging 152. (b) j


145. (d) At the point of incidence slope should be ±1
y^=2x r-
y =4^ i
''^=72.^ =^ =1
dx 2Vx
30° 30
30° 30
4^ = 1
-40/
154. (d) Uq is decreasing => Oq is -ve V2 =
-40 + /
=> Qj is in opposite direction =>aj is +ve
In both situations, sign convention is opposite
155. (c) In one case image is virtual (u -10cm)
Vz --Vi
In another case image is real (u = - 40cm) -10/ _ 40/
uf -10/ • •
Vl =
u +f -10 + /
''^"-10 +/ -40 +/
/ = l6cm

'i'. . - - I
,. • •• j -

.'M
Problems

6V6l (2) &&.XMie:.M^miative islimeriii&m&st:


In displacement method, the distance between object 25 cm
and screen.is-96cm. The ratio of length of two images
fo^rned by a convex lens placed between them is 4.84.
(a) Ratio of the length of object to" the length of
.shorter image is 11/5 • . ,
(b) Distance between the two positions of the lens is
36cm ,•
(g). Focal length,of the lens is 22.5 cm f=20 cm
(d) .Distance of the lens from the shorter image is
(a) As X is increased from zero intensity continuously
30 cm ,
decreases
A luminous point object is placed at 0, whose image is (b) As Xis increased from zero intensity first increases
formed at las shown in figure. Line AB is the optical then decreases
axis. Which of the following statement is/are correct? (c) Intensity at centre of screen for x= 90 cm and
vO" X = 110cm is same
(d) Radius of bright circle obtained on screen is equal
to 1 cm for X = 200 cm
A ray of light from a denser medium strikes the plane
boundary of a rarer medium at an angle of incidence 1.
The angle of refraction is r. If the reflected and the
(a) If a lens is used to obtain the image, then it must*' refracted rays are mutually perpendicular the critical
be a diverging lens and its optical centre will be angle of the medium is :
the intersection point of line AB and 01. (a) sin~^(coti) (b) sin~^(tan/)
(b)- Ifradens is used to obtain the-image, then it must- Cc) tari ^(sinJ) (d)"hh~^"(fahr)
be a converging leris and its optical centre will be
A plano-convex lens Qx = 1.5) of focal length 20 cm has
the intersection point of line AB and 01.
its plane side silvered :
Cc) If a mirror is used to obtain the image then the
(a) The radius of curvature of its curved surface is half
mirror must be concave and object and image
subtend equal angles at the pole of the mirror,
that of a surface of equi-convex lens of focal length
20 cm made of same material.
(d) J is a real image.
Cb) An object placed at 15 cm on the axis on the
A point source of light S is placed on the axis of a lens
convex side gives rise to an image at a distance of
of focal length 20 cm as shown. A screen is placed
30 cm from it.
normal to the axis of lens at a distance x from it. Treat
(c) An object placed at a distance of 20 cm on the axis
all rays as paraxial :
on the convex side gives rise to an image at 40 cm
from it.
(d) It acts as a convex mirror.
^gIoMETRICAL OPTICS 223
A beam of white light is incident on a combinations of (c) / oc 16A; (d) 7 «13A;
thin prism with equal angles. In the out going beam
9. Two transparent plane parallel plates are stacked face
the rainbow colours are observed as shown, p stands
for refractive indexand0for angleof dispersion. Then: - to - face. The plate whose thickness ti = 4.5 cm has a
3
refractive index = - while the other plate whose
! White
highl thickness t2 = 2cm has a refractive index tin = —. A
3
narrow light beam is incident on the first plate at an
angle i = 37° to the normal. The lateral shifts of the
(a)^i<ii2 (b)p2=lLi beam emerging from the second plate into air space
(c) 01 >02 (d) Pi >P2 due to I plate only, due to II plate only, and due to both
7. A lens made from a material of refractive index 1.5 the plates are di,d2 and d respectively, then :
behaves as a converging lens in air. When placed in (a) di -0.5cm (b) d^ -1129cm
liquid of refractive index 8/5, it will : (c) d-1.5 cm Cd)d2~2cm
(a) still behave as a converging lens 10. A plano-convex lens of focal length 20cm has its place
(b) behave as a diverging lens side silvered :
(c) have its focal length increased (a) The radius of curvature of its curved surface- of
(d) have its focal length decreased plano-convex lens is equal to half of fa'dius of
8. The slit 1 of a Young's double slit experiment is winder curvature of a surface of equi-convex lens of focal
than slit 2, so that the light -from slits are given as length 20cm ' ' -.
(b) A object placed at 15 cm on the axis on the convex
Aj = Aq sin cot and A2 = SAq sin cot + — . Then side of silvered plano-convex lens gives rise' to an
3j image at a distance of 30 cm from it
resultant amplitude and intensity at a point where the (c) An object placed at a distance of 20 cm on the axis
path difference them is zero, is A and t respectively, on the convex side of silvered plano-convex lens
' then : ' ' ' gives rise to an image at 40 cm from it
(a) A= y^Ao (b) A= 4Ao (d) Silvered plano-convex lens acts as a' concave
mirror of focal length 10 cm

\ •

, w I I

ANSWERS
i'l » > I ' ^ ' .

, J / ' ' •

1. (a, b, dj- 2. (b, c, d;) , • ^ 3. cb; c, d) 4. . Ca, d) 5. . (a, b) • 6. (c, d) i


I ; <
1
7. (b, c) 8. Ca,d) 9. Cb, c) 10. (a, c, d)

) I '
OETICS

• .1,,. Solutions

111 1
Level-2: More than One Alternative is/are]Gorrect{ => - = - ^ = —=>y = 100cm
V 20 25 100
from O to I intensity increases and then decreases at
X = 90cm and 110cm intensity is same ,(d) Radius.at.
1. (a, b, d)
X = 200 cm is equal to radius of lens.
D = 96
4. (a„d)
Given ^ =4.84 PQ = Incident- ray QS =
h Reflected ray
Let I2 = 4.84 a and Ii = a
0 = ^|T^ = 2.2a QJ? = Refracted ray
r Required
• J ratio
• = 2.2a-= 11 ZSQR-180-(r + r)
(a)
a 5 = 90°
v^n i + r = 90° r = 90°-i
(b) .(1)
•u 5 sini. , sini •
u. = = tani-
v + u = 96 .(2) "^sinr . sin(90-^i) '
5u 1

11 + — = 96 If C is the critical angle |i =


11 sinG
16v
= 96 tani =
~n sinC.
=> u = 66 and u = 30,
sinC = = cot I
Distance between twp position , • tani.

= u-u,= 66-30 = 36cm', - !-i.(cotiv)<;


C = sin"
1
— —
i "1
1
;= •30+ 66 - ,
—pV-'—> 5.' (a, b> .
(c)
/ 66 30 , 30x66, •R
1 A 20 = ...CD
30x66 330 • F R 20 ^•(H-l)
f - •• • = = 20.625 cm
96 16
If equiconvex lens of / = 20 -
u = 30

3. (b, c, d)
then i =Qj, -1)—
20 R
From,eq.-(l) and ,(2) we conclude
(-25),
R = 40(n-1) ...(2)
110 cm that option ,(a) is correct.
90 cm- After^ silvering
Feq = -10 cm (combination will behave like a concave
mirror)
Cb) , . . u = -15
1 111
11 =
•100 cm
v^\L ,F, e q.
V 15 10
GEOMETRICAL OPTICS
225
111
—= =>v = -30cm 9. (b, c)
V 15 10
cosi
So (b) is correct. Shift d = 1- ti sini
:(c) u = -20 nf -sin^i
v = -20
COSl
So (c) is wrong 1- t2 sini
Alternative: V"! - sisin^i
For planoconvex 1 = 37^ Hi =3/2 ti = 4.5 cm
J. "2 ~ f2 = 2cm
R. R 2J
0.8
d = 1- -xO.6
= oo J?2 = Jl
V(3/2)2-0.6^ 2
r = 20(^-1)
0.8
For equiconvex —=(M-i)fA +i + 1- 2x0.6
20" [r R 7(^3)^-0.6^
R = 40(ji-l)
6. (c, d) = 1.129+ 0.39 = 1.5cm (d = dj + da)
5i +62 > 0 10. (a, c, d)
Pi >P2
For plano-convex lens ~ =Si—12
•J- 0> 0 / R
01 - 02 > 0 2 1
For- equi-convex lens —= (p -1) —
7. (b, c) / R' ]
R'
JL -1) R = .^ (choice (a) is correct)
2
fa L^l ^2
Let F be the focal length of silvered
[fa = fqcallength of lens in air]
plano-convex lens
1 1 (iP-g -1) 1
— = -1), 1 _ 2(p-l) ^1 _ 2(p-l)
Ul " 1) fa F R 00 R
3/2
-1 -1 Giveni = ± =Si^
v 8/5 1. a-b 1 1 J_ / 20 R

•3-1 fa 1 fa s'/. P = 10 cm (it acts as concave mirror) choice (d) is


I? correct

/L-=-8/a; negative sign indicate that lens ,is i +i-1


diverging. u V F

8. (a, id)
.Ifthe amplitude due to.itwo individual sources at point If.u =15cm, ~ =- -= ^ ^ =J-
P is Aq and 3Aq thep the,resultant amplitude at P, will V 10 15 30 30
he ; ^ p = 30

A=Ja^ +{3Aof 2(AoX3Ao)cos- =^/13Ao If u=20cm, - =— ^ =—^|, =20


••V 3 V 10 20 20

Resultant intens/ty, I « isaJ (choice c is correct)


OPTICS
226

Problems

Comprehertsmn Bas^d PpoMisrns


evel

f!
- t/ %Jl Vs- jm" '

A concave mirror of radius of curvature 20 cm is shown in Parallax refers to the different views that you see from two
the figure. A circular disc of diameter1cm is placed on the different positions. Try this experiment. Hold the index
principle axis of mirror with its plane perpendicular to the finger of your left hand vertical, 20 cm infront of you. Hold
principal axis at a distance 15cm from the pole of the the index finger of your right hand vertical, 40 cm infront of
mirror. The radius of disc increasing according to the law you. Now close your left eye and, using just your right eye,
r = (0.5 + O.lf) cm/sec. move the two finger sideways until they line up. Now close
your right eye and open the left. The closer finger has
'jumped' to the right of the further finger. Repeat a few
times. Compared to a distant background, both fingers
have both jumped to the right, but the closer one jumps
farther. If you measure the angles ±rough which they jump
and the distance between your eyes, you can work out how
1." The image formed by the mirror willbe in the shape of a: far away the fingers are. For distant objects, the distance
(a) circular disc between our viewing positions must be greater than the
(b) elliptical disc with major axis horizontal distance between your eyes. Fortunately for astronomers,
(c) elliptical disc with major axis vertical the Earth shifts our telescopes round the sum, so we can get
(d) distorted disc
a separation equal to the diameter of the orbit of the Earth
(16 light minutes) if we wait six months, as shown in this
2. In the above question, the area of image of the disc at diagram.
t = 1 second is :
(a) 1.2tc cm^ Earth

(b) 1.44Tccm^ now

Sun O
(c) 1.527t cm^
(d) none of these
3. What will be the rate at which the horizontal radius of
image wiU be changing :
(a) 0.2 cm/sec increasing
(b) 0.2 cm/sec decreasing Sun O OA OB

(c) 0.4 cm/sec increasing


(d) 0.4 cm/sec decreasing
Earth six months ago
; GEOMETRICAL OPTICS
227
In this sketch, which is not to scale, imagine an observer
looking at objects Aand B, standing at the pole ofthe Earth I#

with his head towards us. Now he sees object A to be to the-


right of B. Six months ago, he saw it to be to the left of B. A balloon is moving upwards with a speed of lOm/s at a
Now most stars are so far away from us that we cannot distance 5m infront ofa plane mirror oflength 2m asshown
observe any relative motion in thisway. However, for close in figure. Asource S is placed sjmmetrically before plane
stars it is possible. The next sketch shows the path oflight mirror. The plane mirror is placed at a height H = 15m
from a close object and from a ve^ distant star. above the surface of ground.
5m ""Ot
Earth light from a very distant star 2m v
• now *S
SunO
0.5m

light from same star, six monthsago H=15m

Earth six months ago


From trigonometry,
D - i?/tan0 = R/0
where we have used the small angle approximation for 0 The maximum time during which a man inside the
measured in radians. A parsec is defined as the distance to balloon can see the image of the source in the mirror
an object that 'moves' (w.r.t. to the distant stars) by an is:
angle of 1 second (1/3600 of a degree) when the Earth (a) 2.2sec (b) 1.7sec
moves by the mean radius of its orbit. In terms of this (c) 4.6 sec (d) 0.5sec
sketch, if0 =one second, D= 1parsec. Now all stars except^ When the balloon is at a height of 17m from ground,
the sun are more than one parsec distant, so to measure' the man drops himself from the balloon, the greatest
their distance by parallax, we need to be able to resolve distance over which he can see the image of source in
angles of about 1 second or better. plane mirror during free fall and corresponding time
duration are (g = 10m/s^) respectively :
1. In the first experiment if we have two point objects A (a) 22m, 2.2 s (b) 17m, 2.84s
and B in place of your left and right fingers (c) 12 m, 1.7 s (d) 2m, 2s
respectively then : If the balloon is moving upwards with a speed of
(a) your two eyes and A and B will lie on a circle 20ra/s, the man drops himself from the balloon at a
(b) your two eyes and A and B will always lie in a height of 17 m from ground, the time duration for
plane which he will see the image of source in mirror is :
(c) your two eye and A will form a triangle area of (a) 2.2 m (b) 1.7 m
which is be equal to the area of the triangle
formed by A, B and your left eye (c) 1 s (d) 2.84 s
(d) none of the above
2. In above if we have three objects A,B and C such that
you see only A andB from your left eye and A andC
from your right eye. Angle formed by AB on your left A convex lens sof focal length 40 cm is held at a distance
eye equals that formed by AC on your righteye. Then 12cm coaxially above a concave mirror of focal length
AB will : 18 cm.
(a) always equal to AC
(b) be equal to AC if your two eye and A form an
isosceles triangle
(c) be equal to AC if your two eye andAform a right
angled triangle
1
f2=40cm
(d) none of the above
3. Parallex method is useful only if the star whose
distance is to be measure should be closer to the earth
T
12 cm
and the reference star should be :
(a) very close (b) very far away
fi=18cm
(c) any other star (d) only pole star
OPTICS
[228'
1. Aluminous point objectplaced d cm above the lens on
its axis gives rise to an image coincident with itself,
then the value of d is equal to : 'The figure shows a constant deviation prism ABCD. The
(a) 15 cm (b) 30 cm iincident ray isPQ and the emergent ray isST. Although it is
(c) 24 cm (d) 18 cm !made-up of one piece of glass it is equivalent to two
2. In the above problem, if the convex lens is replaced by '30°-60°-90° prisms and one 45°-45°-90° prism. The
a concave lens of focal length 60 cm, it gives rise to an langle is the angle ofincidence onface AB. The path of
image coincident with itself. Then the value of d is : Ithe ray inside the prism is indicated in the figure.
(a) 20 cm (b) 30 cm ;For this prism, |i = 2sin6i. _
(c) 40 cm (d) 60 cm R
3. In the above problem, if the convexlens is replaced by
a glass plate of thickness 6cm, refractive index H= -
and gives rise to an image coincident with itself, then
the value of d is equal to :
(a) 20 cm (b) 30 cm
(c) 40 cm (d) 60 cm

ij W.
^ C v*- W ^V- ;
1. The ratio of is
02
Light from a laser is directed at a semi-circular glass block.
The light passes undeviated through the block and on to a •(a) 1 (b)l
screen, forming a spot as shown.
M V2
Laser
2. The total deviation of the incident ray when it emerges
out of the prism is :
(a) 90° Cb) 60°
(c) 30° (d) 45°
The semi-circular block is rotated about the point P. The 3. The angle of refraction on the face AB is :
spot of light on the screen is seen to move downwards. i (a) 30° Cb) 45°
•V\^en the spot reaches point B, it disappears. _ . . (c) 15° (d) 40°

1. Which of the following diagram shows the position of «y **• jr

the semi-circular block when the spot is at point B.


!A ray of light goes from point A in a medium where the
'speed of light is to a point B in a medium where the
(a) •speed of light, is ^2 as shown in the figure. The path ofthe
rays in the two shown in figure..

(d) none of these

b 1^2
2. In a particular experiment, the distance PA is 120 cm
and distance AB is 90 cm. Calculate the refractive
index of the glass of the block :

I3 ® 74
(d) none of these
GEOMETRiai OPTICS 229

1. The time taken for the light to go from the point A to


the point B in the figure is :
T .• asini • ' • , bsinr
— (b) Figure shows a simple version of a zoom lens. The
converging lens has focal length fi, and the diverging lens
Uoasini aseci bsecr
(c) (d) + has focal length /a =—l/gj. The two lenses areseparated by
Vjbsmr a variable distance d that is always less than. /j. Also, the
2. The slope of (i - r) curve is : magnitude of the focal length of the diverging lens satisfies
-b cos^ i acosr
the inequality ]/2|> (/i -d). To determine the effective
(a) (b) focal length of the combination lens, consider a bundle of
acos^ r b cosi
parallel rays of radius entering the converging lens. It can
asin^ r -atani
be shown that the radius of the ray bundle decreases to
(c) (d)
b sin^ i btanr
~ at the point, that it enters the diverging lens.
•^1 ^
Final image is formed a distance ^ ^ to
A parallel beam of light is incident on a transparent • (lAh/i+d)
hemisphere of RI2 as shown in figure. 0 is the centre of the right of the diverging lens. If the rays that emerge from
the diverging lens and reach the final image point are
answer of the following questions : extended backward to the left of the diverging lens, they
will eventually expand to the original radius Tq at same
Air

RI=l/\45'
jpoint Q. The distance from the final image I' to the point Q
is the effective focal length / of the lens combination; if the
combination were replaced by a single lens of focal length /
—m-2 [placed atQ, parallel rays would still be brought to a focus at
!j'. The effective focal length is given by / = /1I/2I
1. For the situation described above, mark out the correct
j. ClAi-A+d)
lAssume that' /i=12.0cm, /2 = -18.0cm, and the
statement (s) :
[separation dis adjustable 0 and 3.0 cm
(a) The central ray of the beam suffers, total internal
reflection at the plane surface of hemisphere.
(b) As we move up or down from central ray of beam,
chances of TIR at plane surface is more
(c) As we move down from central ray of beam,
chances of TIR at plane surface is more
(d) Both (a) and Cb) are correct
2. Considering beam to be narrow, determine the
location of image formed after refraction from curved 1. If the incident beam is parallel the image for the
surface of hemisphere : converging lens will act as an object for the diverging
(a) At 2R from P, real * lens. This point will be :
(b) At 2R from P, virtual
(a) Real image for lens and virtual object for 2"^^
* lens
(c) At R from P, real
(b) Virtual image for1®*^ lens and virtual object for 2"*^
(d) At R from P, virtual lens.
3. Considering beam to be narrow, determine the (c) Real image for 1" lens and realobject for2"^^ lens
location of the image formed after TIR from plane (d) Virtual image for l" lens and real object for 2'^'^
surface of hemisphere : lens
(a) Vertically below O, on the surface of hemisphere, 2. Which of the following can be a value of the effective
virtual
focal length of the zoom lens ?
(b) Vertically below O, at a distances of 2R, virtual
(a) 18cm Cb) 12cm
(c) Vertically below O, on the surface of hemisphere
(c) 28 cm (d) 38 cm
real
(d) Vertically below O, at a distance of 2R real 3. The final image distance $2 will be such that:
(a)*0<s^<|/2| Cb)|/2|<s'2<2|/2l
(c) 21/2! <s^ <-
(d) Any of the above three options
1230 ^ omcs
1. The minimum focal length of the eye lens is :
(a) L2cm (b) 2.'5cm
An object AB is seen by the eye lens. The length A'B' of the' (c) oo (d) 25/11 cm
(image formed on the retina by the eye lens is dependent on' 2. If the eye is kept very close to a converging lens (focal
'0' because radius of the eyeis constant for a person duringj length = 10 cm) and at the optical centre of the lens
:observation of an object i? = 1.25 cm. An object of fixed; and an object is kept at distance' d' then the maximum
height will appear to be of different heights as seen from distance' d' of the object from the lens so that its image
different distances because of different values of ' 0'. Stars can be seen clearly by the defect free eye is :
are of very large size but they appear to be very small ' (a) 10cm (b) 25cm
because they are at very-very large distances '0' is called (c) 50/3 cm (d) 50/7 cm ,
jangle of vision or visual angle. For larger '0', the object will 3. The value of angular magnification of this simple
appear larger, 0 depends on both 'h' and 'd'. If '0' is very microscope (mentioned in the previous question) is :
•small, then the object is not visible to us. To increase '0'the (a) 2.5 (b) 1.25
object should be kept nearest to the eye but then the object (c) 5 (d) 3.5
^cannot be seen clearly because in that case the eye lens
cannot adjust its focal length to make the image on the (} •W . v> V
retina. The objects infront of the eye can have a minimum
'distance 'D' from the eye for their image to be formed on iTlie plane mirrors are placed symmetrically on disks
Iretina 'D' is called LEAST DISTANCE OF CLEAR VISION. [revolving in different directions as shown. 'S' is the source
:D ~ 25 cm for average grown-up person. If an object is kept land the beam from is reflectsrepeatedly from the mirrors.
ifrom distance 'D' to from a defect free eye, then its
image can be formed on retina and it will be clearly visible.
iSometimes an object kept in this range forms a very small
angle and hence cannot be seen like dust particles, bacteria
etc. To increase their '0' microscope (for small object near
.'the eye) or telescope (for large objects but far away) is
j
used. A simple microscope is a converging lens (for
^example lens used by watch repairer). The image formedj 1. If the angular velocity of rotation of the disks is co, then
by this lens acts as the objectfor the eye. Their performance] the angular velocity H of rotation of a beam that is
[isrepresented by a parameter called' angular magnification'-i consecutively reflected n times from the mirrors is :
0 ' (a) 2nco (b) (2n - l)a)
or'magnifying power'. It is represented by 'm' m = ^ •;
®max [ (c) (2n - l)2to (d) none of these
here 0 = angle of vision of the image formed by the optical; 2. If the disks rotate in the same direction with angular
•instrument Qikemicroscope or telescope) ©max = maximumi velocity of rotation co then the angular velocity Q of
angle of vision that an object can subtend on the eye when] rotation of a beam that is consecutively reflected n
no optical instrument is used (for example if the object is times will be :
kept at 'D' then '0' is maximum). (a) 2na)
eve fens ^ v Retina (b) 2nco if n is odd and if n is even
(c) (2n-l)2co
(d) 2co if n is odd and 0 if n is even
GEOMETRICAL OPTICS
231

MATCHING TYPE PROBLEMS 0 ^

Cd) (s)
1.. Column shows O^-object and Os-image. The optical X
. system responsible can be a spherical mirror, plane
mirror or a thin lens. In the case of the lens and
spherical mirror, straight line shows the principal axis.
Match column-I and column-II :
Ct)
\ Column-I
\ \ Column-II

Ca) • O2 CP) Concave mirror between


Oj and O2
3. A small extended object can
Cb) ?i Cq) Diverging lens between
move infront of a fixed
?2 Oj and02
concave mirror considering
(c) Cr) Convex mirror between
pole as origin and sign 2f
Oj and O2 convention, match column-I
and column-II. Column-I
Or shows position of object and its velocity v column-II
Cd) 0/ Cs) converging lens between shows characteristics of image.
Oj and O2
Or
\ Column-I \ \ Column-ll
Ct) Inclined plane mirror
Ca) Object between / and c ap Cp) Velocity of image
somewhere between
proaching towards mirror u > 0
(v > 0)
2. In column-I possible instantaneous velocity vector of Cb) Object between / and p and re Cq) Velocity of image
the image with respect to ground are shown. These are ceding from mirror (u < 0) u<0
corresponding to the situations shown in column-ll.
Match column-I with the situations shown in Cc) Object between c and and re Cr) Size of image increas
ceding from mirror (u < 0) ing
column-II.
Cd) Virtual object and approaching Cs) Size of image de
\ Column-I \ \ Column-ll mirror (v < 0) creasing

%r»"i 4. Column-I shows incident wavefront and


corresponding refracted or reflected wavefront. Arrow
(a) Cp)
C ^ j below incident wavefront shows direction. Refracted
wavefront can be propagating right or left.

Column-I Refracted or Column-ll


0
Incident reflected Optical
(b) Cq)
F j- wave front wave front
Components

Ca)
I Cp)
S
(c) / (r)
'%
#
Cb)
(
r

Cq)
0
232 OPTICS

' Column-n (Image)


it'. 1
(c) (r) (P) Real, Errect, Enlarged
(q) Virtual, Errect, Diminished ' '
* . * ' i

(r) Real Inverted, Enlarged ' *


(s) Virtual, Erect, Enlarged
(d) (s) 7. Match the following graphs (in column-II) between
position of object (u) and position of image (u). Take
positive sign in the direction of ray.
\ Column-! Column-ll
5. A child in a toy car is moving tov^^ards a cubical
container with a velocity of 3cm/s, whereas fish is (a) Concave mirror (P) v

moving towards child with.a velocity of 4cm/s as


shown in the figure. The wall behind ;the. fish is
0
silvered (Refractive index of water is jr = 4/3). Match
the following.

• «

="4/3'4cm/s
3 cm/s (b) Convex mirror (q)

\ Column-I \ \ Column-Il

(a) Velocity ofchild as observed by fish by (P) Zero '


one refraction only
(c) Convex lens (r)
(b) Velocity of fish as observed by child by (q) 6 cm/s
one refraction only

(c) Velocity of image of fish by reflection (r) 8 cnv's


from mirror as observed by child
Cd) Velocity of image of child by reflection (s) 7 cnys
from, mirror as observed by fish.

6. Match the following (d) Concave lens (s)


An object,0 is kept perpendicular to the principal axis
• Of a spherical mirror. Each situation (a, b, c and d)
gives object coordinate u in centimeters with sign, the
type of mirror, and then the distance (centimeters,
without sign) between the focal point and the pole of
the mirror. On the right side information regarding the
image is given. Correctly match the situations on the
left side with the images described on the right side.
Column-I

Situation u Mirror
a -18 Concave, 12
b -12 Concave, 18
c -8 Convex, 10
d -10 Convex, 8
[gEOWETRICAL OPTICS
n ^ I ""

10 In column-I certain combinations of small object and


8, Aveiy small circular object is kept infront of an optical image are marked. Match them with possible
device as shown in figure. The plane of object is reft-acting/reflecting body is column-II. In all cases,
parallel to the optical device. Match the images as seen object is perpendicular to x-axis.. Object cant be at
by the observer tignoring magnification). zero distance from the interface :
Column-ll
Y Columii-I
Column
Coiumh-11
a) Plane mirror Virtual image of [P)
(a)
(The observer is at A) virtual object Converging
C lens
x-axis

a, . • ^

Real image of real Cq): air / water


(b)
object. •x>axis

(b) Concave mirror


(The observer is at A) L
s

Erect real image of (r) Convex mirror


C
(c)
shne size as object

x-axis

(c) Convex mirror


(The observer is at A)

(d) Erect and virtual (s) water


image size than
x-axis
the object

Light isincident from right toleft


(d) Convex leiw
(The observer is at B) (t)
x-axis -

Diverging lens

11, The graphs given apply to aconvex lens offocal len^


/, producing a real image at a distance u from the
9. A metal plate containing an illuminator circular optical centre when self luminous object is atdistance
aperture is placed at one end.of an optical bench and a u from the optical centre. The magnitude of
screen, at the other end. By means of aconvex lens the magnification is m. Identify the foUowing graphs with
image of the aperture is formed on the screen, the the first named quantity being plotted along y-axis.
diameter of the image- being 4cm. If the lens is
displaced 17.5 cm along the optic bench, an image of Column^; ; .\ \ . . Columh-H . '
the aperture again appears on the screen, the diameter (a) Vagainst u (P)
of the image being 2.25 cm. Then match the following:
\ Column-1 \ \ Column-lb /
(P) 122.5
(a) FocalTength of the lens in cm
(b) Diameter of the aperture in cm (q) 3.3 , 0
/ .
(r) 3
(c) Distance between the aperture and
the screen in cm
(s) 30
(d) Power of lens in Dioptre
<234 OPTICS

(b) -1 against -1 (q)


ASSERTION AND REASON
V u

1. Direction: Each question contains statement


1 (assertion) and statement 2 (reason) .
Each question has 4 choice (a),Cb),(c) and (d)'out of
(c) m against v (r)
which only one is correct.
(a) Statement-1 is true, statement-2 is true and
statement-2, is correct explanation for
statement-1.
(b) Statement-1 is true, statement-2 is true and
statement-2 is not the correct explanation for
statement-1.
(c) Statement-1 is true, statement-2 is false. •,
(d) Cm+1) against y (s)
(d) Statement-1 is false, statement-2 is true.
1. Statement-1: The brilliant colours that often
appears on the throat and belly of a humping bird will
vary with the viewing angle.
Statement-2: The indescence of that makes the
brilliant colour is not due to pigment but is due to an
interference effect caused by structures in the feathers.
12. A bird in air is diving vertically over a tank with speed 2. Statement-1: A parallel beam of light travelling in
6cm/s. Base of tank is silvered. A fish in the tank is air can be displaced laterally by a parallel transparent
rising upward along the same hne with speed 4cm/s. slab by distance more than the thickness of the plate.
Water level is falling at rate of 2cm/s. Statement-2: The lateral displacement of light
(take: |i water = 4/3) travelling in air increases with rise in value of
refractive index of slab.
3. Statement-1: A solid glass sphere is placed in air. A
light ray entres into the sphere from outside. The ray
cannot undergo total internal reflection inside the
sphere.
Statement-2: The angle of incidence at surface
: 77777777777^777777777^ cannot be greater than the critical angle for air-glass
system.

\ Column-I \ \ Column-II 4. Statement-1: A beam of white light enters the


curved surface of semicircular
(a) Speed of the image of fish as seen by tp) 12
the bird directly piece of glass along the normal.
The incoming beams is moved
(b) Speed of the image of fish formed after Cq) 4 clockwise (so that the angle 0
reflection from the mirror as seen by increases). The color of the
the bird
refracted beam is red before it
(c) Speed of image of bird relative to the (r) 9 • steps energing from flat surface.
fish looking upwards Statement-2: The index of refraction for light at the
(d) Speed of image of bird relative to the (sj 3
red end of the visible spectrum is lesser than at the
fish looking downwards in the mirror violet end.
5. Statement-1: A convex lens suffers from chromatic
aberration.
Statement-2: All parallel rays of monochromatic
light passing through a convex lens do not come to a
focus at the same point.
GEOMETRICAL OPTICS 235
6. Statement-l: In a glass medium red light travels Statement-2: Laws of reflection are valid only for
faster than blue light. smooth, polished surfaces.
Statement-2: Red light has a wavelength longer 15. Statement-1: A beam of white light is incident on a
^ than blue. transparent glass hemisphere as shown in figure. The
7. Statement-1: The focal length of a lens depends on beam is rotated clockwise so that angle 9 increases, as
±e material of the lens but the focallength of a mirror the refracted beam approaches a direction parallel to
is independent of the material from which the mirror the horizontal, it appears red.
is made.
Statement-2: Law of reflection is independent of
material and surrounding medium where as law of
refraction depends on the mediums. White
light
8. Statement-1: Deviationof a ray of light produced by
reflections from two inclined mirrors is independent of Statement-2: Critical angle for a pair of medium
the angle of incidence. aepends on RIS of the medium and is given by
Statement-2: Deviation of a ray of light produced by -1
—— and R1 in turn depends upon
ic = sin
reflectionfrom one mirror is dependent of the angle of J
incidence.
wave-length of light.
9. Statement-1: Real images form on the side of a 16. Statement-1: The colour of green object seen
refracting surface that is opposite to the object and through red glass appears to be dark.
virtual images form on the same side as the object.
Statement-2: Red glass absorbs all other light except
Statement-2: Virtual images cannot be taken on the red colored light.
screen.
17. Statement-1: Dry sand appears bright.
10. Statement-1: Convex mirrors are used as rear
Statement-2: Dry sand is smooth.
mirrors in cars or scooters
18. Statement-1: If there were no atmosphere, the
Statement-2: A convex mirror gives a wider field of
duration of the day on the earth would decrease.
view than a plane mirror.
Statement-2: Due to refraction in the atmosphere,
11. Statement 1: The sun seems to rise before it
the sun appears to rise earlier and sets later.
actually rises and it appears to set after it actually sets.
19. Statement-1: We can hear around corners, but we
Statement 2: Near the horizon total internal
cannot see around corners.
reflection takes place.
Statement-2: Wavelength of sound in much greater
12. Statement 1: The coating of optical fibers of a light
than wavelength of light.
pipe is of a material having refractive index more than
that of the fiber, to optically insulate from each other. 20. Statement-1: In the shown situation if middle
portion of the lens is painted black, then complete
Statement 2: The optical fiber works on the
image of the object will form but of low intensity.
principal of total internal reflection.
^ Lens
13. Statement-1: A bright metal ball is first blackened Object A
with soot in a candle flame and then immersed in
water, it will again appear bright. 2F F
Statement-2: At water-air surface total internal V
reflection can take plane.
Statement-2:More the reflected or refracted ray
14. Statement-1: An observer on the west facing beach intersect, more would be the intensity of then
of a large lake is observing the beginning of sunset. corresponding image.
The water is very smooth except for some areas with
small ripples. The observer notices that some area of
the water are blue and the remaining is pink.
236 OPTICS

AN8WER9

Leve1-3: Comprehension Based Problems

Passage-1: !• (a) 2. (b) 3. (a) Passage-2: 1. Cb) 2. Cb) 3. Cb)

Passage-3: !• (a) 2. (b) 3. (b) Passage-4: 1. Ca) 2. Cc) 3. Ca)

Passage-5: 1* (b) 2. Cb) Passage-6: 1. Ca) 2. Ca) 3. Ca)= -

Passage-7: !• (d) 2. (a) Passage-8: 1. Ca) 2. Cb) 3. Cc)

Passage-9: 1- (b) 2. (a) 3. (d) Passage-10: 1. Cd) 2. Ca) , 3. Ca),

Passage-11: !• (c) 2. Cb)

— \ .

B Matching Type

I. (a)-r, t; Cb)-r, t; (c)-s, t; Ccl)-s, t 2. Ca)-p, q; (b)-r, t; (c)-p, t; (d)-p, s, t


3. (a)-q, r; (b)-p, r; Cc)-p, s; (d)-p, r 4. (a)-p, s; Cb)-q, r; (c)-q, s; Cd)-p ,r
5. (a)-r; Cb)-q; (c)-p; (d)-p 6. (a)-r; Cb)-s; (c)-q; Cd)-q
7. (a)-p; (b)-s; Cc)-q; (d)-r 8. (a)-p; Cb)-p, q; (c)-p; (d)-q, s
9. Ca)-s; Cb)-r; (c) -p; (d) -q 10. Ca)-q t; (b)-p, q; Cc)-s; Cd)-p, q, t
II. (a)-r; Cb)-s; (c)-q; (d)-p 12. (a)-r; (b)-s; (c)-p; Cd)-q

QAssertion and Reason


1. Cb) 2. Cd)' 3. Cc) 4. Ca) 5. Cb) 6. Ca) 7. Ca) 8. Cb)

9. Cb) 10. Ca) 11. Cc) 12. Cd) 13. Cb) 14. Cb) 15. Cb) 16. Cb)

17. Ca) 18. (a) 19. Ca) 20. Ca)


Solutions

S 22
Required time = — = — = 2.2s
S Level-3: Comprehension Based Problems 2. (b)
u 10

Passage-1 Initial velocity given to man at the time of drop is


2. (b) 10 ni/s upwards
To = 0.6 The height through which he can rise.
Tj = 0.6x 2 (m = 2) 10'
hi = = 5m
= 1.2 2g 2x10
A = IT X (1.2) = 1.447C cm" .*. The greatest distance over which he can see the
3. (a) image of source is G'J = (5m + 12m) = 17 m
ri = 2(0.5 + 0.10 = l + 0.2t S = ut + -gt^
^ =0.2 -12=10t-5t2
dt

Passage-3 5t^-ia-12 = 0
1. (a) 2.84s
The ray diagram will be as shown in figure. 10

3. (b)
t f
1
If the velocity of balloon is 20 m/s, the man can rise
upto a distance

' t ;
10m.
♦ 20m/s

2m

10m

HI = 2m = AB
DS = CD=lm
AH = 5m = lOAD
GH = lOCD = 10x1 = 10m
_ _u^ _ 20^ = 20 m

GJ = GH + HI + IJ = 10m + 2m + 10m = 22m.


^ 2g 2x10
The greatest distance over which the man in the But he can see the image only upto G during upward
balloon can see the image of light source in the mirror motion. Time taken from H to G =
5 is 22m 10 =20ti-lgti'
238 OPTICS

•I- t L-f.
tf -4ti+2 = 0 Object distance Uj = 12-
40d (-28d-480),
4±Vl6-8 (d-40) (d-40)
ti = = 0.586 s
From mirror equation
Then he can see the image from G to J during (d-40) -1
+
downward motion (28d + 480) 18
Time taken from G to H = t2 = = 0.586 s = ^^^-40) (240-46d)
Then taken from H to J = t. V 18 28d + 480 ~ 18(28+ 480)
12=20 t3+-gt| 18(28d + 480)
=

(46d - 240)
-20±V400 + 240
5t| + 20t3 -12 = 0 => fg = = 0.53 s The image is formed at a distance
10
18(28d + 480)
= 12-
(-240 + 46d)
Total time taken
For the second refraction at the convex lens -
= tj "'"^2 ^^3 ~ (0.586)2 + 0.53 = 1.7s
1 1 1 ,
Passage-4 d 18(28d + 480) 40
12-
1. (a) (46d - 240)
Since final image will coincide with the object if the
ray retraces its path after reflection from the concave Solving d = 15 cm
mirror i.e. the ray strike the mirror normally. The [Note: First method is simple for calculation]
normal rays at the mirror after being extended must 2. (c)
pass through the centre of curvature of the mirror for The final image will coincide with the object if the ray
the refraction at the convex lens retraces its path after reflection from the concave
X , ' mirror.

; °t
I d cm
, ' u =-(36-12) cm + 24 cm
V= d

f2=40cm

12 cm
f2=60cm

fi=18cm 12 cm

u = (2/-12) = 24cm
f2=18cm
V= d

2 24 40 A = - 60 cm
1_ 1 ^ 1 _3 + 5
d 40 ~ 24 ~ 120
d = 15cm
Aliter
Let the object be placed at a distance d above the lens
l_l l_d-40 120

V 40 d 40d d = 40cm

40d 3. (a)
V
d-40 Shift produced by the glass plate
A real image is formed due to first refraction at the Ax = l--lt = 1— 16 = 2 cm
lens this image is an object for reflection from the 3
convex mirror
for the mirror, the object is placed at a distance
[12-t-6 + d-Ax] = (16 + d)
239
geometrical q^PTics,
The deviation being 90° theincident rayandemergent
ray are at right angles.
Passage-7
1. (d)
6 cm
Let X% be the boundary separating the two media.
T Total time taken = time taken (ti) from A to 0 in
12 cm medium (1) +timetaken(t2)fromOtoBinmedium.
A^

fi=18cm

Jhe image is coincident with the object itself if


- (16+d) = 36cm d = 20cm
Passage-5
2. (b)

aseci

bsecr

a sect bsecr

PB = -iPA^ +AB^ = 150cm


0 = 90°-i Z90° 2. (a) To optimize time, ^ ^
independent of radius,
disapearing relates to angle. d(aseci ^bsecr^ ^^
r = i+e = 90° ^ drl^ Ui ^2 J
sin 90° 1 asecitani di b seer tan r = 0
+
1^ =
sini COS0 V dr V'

di b cos^i
h =
PA ~ 120 4 —

dr a cos^ r
Passage-6 Passage-8
1. (a); 2. (a); 3. (a)
1. (a)
Let Vbe the angle of refraction at Q. The ray diagram shows the refraction at curved
sinOi . Q h
m = —^,smei=- surface for three rays, one for central ray, one above
sinr ^ central ray at one below the central ray.
sinr = r = 30°
2

Considering ABC as a triangular prism


r + r' = A
30° + r' = 75°=>r' = 45°
Thus, the ray is incident oii BCat 45°. Itis reflected at . -11 1
For glass-air pair, critical angle is, ic - sm
the same angle on to AD.
From the figure BSD = 360°-(45°+135°+60°) =120 -1^1 1=30<=
(consider the quadrilateral) ^ ir ~ sin

Angle of incidence at AD =120°-90° = 30°


01 = 02
For central ray, angle of incidence is 45°, so TIR takes
place. For ray above central ray, Zi >45°, so TIR takes
Total deviation of the ray = Oi - 30°) +(180°-90°) place. For ray below central ray, Zi is decreasing Le.,
-(02 -30°) = 90° <45°, and at one place may become less than critical
(last one subtracted because itis opposite)
'240 ;<)PTics»

angleAfor glass-air-pair and rays may refract at plane 1_1 J 1_


2. (a)
surface.-. . eq V U 2.5 -X
2. (b)
Here; we are draw the ray diagram for narrow parallel
beam: '
•H,
For refraction at surface, using refraction at 2.5cm
spherical surface :
Xjnax ^ /eq = "laX. ^ CyC iS foCUSSCd aS 00
Jl i
/e'^ /lens 2.5 X
/lens = lOcm => x.= 10 cm
3. (a)
e hlF D D
m - - — - — = 2.5cm
0. .h/D F 10
1^2 • _
Passage-11
2co + 2w 4(0 . I
/ -^co

v = 2R i.e., image will form at a distance, of 2R


from P on a straight"line"joining P.and.O-directly
1. (c)
opposite to P. And the image /, would be virtual.-.
3. (c)
As the beam is narrow, all the rays get TIR from'plane
DO
surface and form a real image at 12i. e., on the surface
of hemisphere and vertically below 0.
Passage-9
2. (c)
/1I/2I 12x18 216 2. (b)
^ Ifil-fi+d 18-12-hd 6+d
24 < / < 36cm Q O:
3. (d)
So =
I/2I
Matching Type Probjems
r 1/2I -1
Ji-d
Passage-10 2. (Vi/M)jir - ~l — (^O/M )x
1. (d)
1 1 _1
V u f
V = 2.5cm in all cases So, if(uo)x
u = ~x
i^ backward direction
1 1_1
and if (uo):if « then,
2.5 X f
(ui will be in forward direction
fmax X IS max. => X = 00 dm d f-v
In case of (u j) y = Xo = Jo
fmax ~ 2.5 cm [ dt dt I u
/min => * is inin If object is approaching focal point (P) then, 'm'
1 1^ increases and [yi| will decrease.
2.5 25 ~ /
. 25
/min
.GfOMETRICAL OPTICS
). < ^2 • ths frame of miiror

virtue

c
("l)) Real

(i^i)

(^i)x = - - C^o)

(^l)v =
virtual
In:the frame of ground :
If (Ui);, » Virtual

(t) Uj > U2 : In the frame of mirror


virtual ""1
I
("l)x
0 y ,•
^ ,("|)y T ' V, . ;
IReal C

Uj > U2: In the frame of mirror : • In the frame of ground


if < ^2
0 • i" ^" "ug.

C - F

In .th'e.:ground frame >yn

v^r-'.Real

In the franie.of mirror Vi <V2'- In the frame of mirror


;(ui -uo)

vrrtua <

In the ground frame

in .the ground frame


242
optics!
?>T'

f-v
(c)\m\ = 1-^ = ^-1
f /

3' Vjjyf - -m f o/M


9. Diameter of the aperture
0= = V4x 2.25 = 3cm
(d)lml +l =y
•i*. • '.'i ~4
Magnincation, mi = — = — Vi =
iii 3'

1__J. 3 ^ -7 ^ ^_-4ui
/ 4ui 4ui 12. (a) Velocity of fish in air = 4X—= 3t
1 I 4ui - 4ui
ui =-v^lui|=—,/ = — Velocity of fish w.ht. bird= 3+'6 = 9T
(b)' Velocity of image of fish after reflection from
\viHu,\=^-u,=^u,=17.5cmAjI 1

mirror in.air. = ,4x —= si .


4
Uj =^5Z5cm; Ui = 70cm ' - .
Distance between the aperture' and screen w.r.t. bird = -3 + 6 = 31
=|ui| + |ui|= 122.5 cm (c) Velocity of bird in water = 6x —= 84-
• = ^ ; -'i' i .'-r "
. v u 'f • y ' • w.f.t.'fish'='84-4=124-
1 1 _ 1 1 _ 122.5 - 30 cm (d) Velb'city of bird in water after; reflection from
70 -52.5 / / 70x52.5 niirTor = 8t ,
Focal length of the lens ,= 30 cm . . . w.r.t. fish =,8,-4 =.4?, ,
Power = - = — = 3.3D
/ 30
Level-1: Assertion and Reason
11. (a)
v/u >(i?) 2. (d)t sini 1--\<t

8. Cb)
Due to reflection at single mirror,
deviation produced 5=-180°-2i.
0
Due to freflection's from two inclined / !
1 I plane mirrors, the deviation
Cb) ^= produced
(-U) / \\\\\\\\\ (

] = 20
Negative slope,
positive intercept

77T77777T7T7
Geometrical OPTICS
243
10. (a) 14. (b)
Convex mirror gives a wider field of view than plane Here statement 2 is a standard concept and is true.
mirror
Statement 1 is also true but not explained by
Statement 2. The correct explanation for Statement I
is: the different colours ofwater arise due to specular
and diffuse reflections. The smooth areas of the water
Field of view 5 Field of View
of a plane mirror ; of a convex mirror
will reflect the lightspecularly from the sun, which is
somewhat pink in colour at the beginning of sunset
and hence water appears pink. While the light is
reflected diffusively from areas where ripples are
present, as the beginning of sunset-sky is almost blue
andhence reflected light from allparts ofthesky make
some part of the water to be appeared as blue.
H. (c)
15. (b)
Due to refraction in the atmosphere the rays from the
rising sun. are bent and reach the eye of observer As 0 increases, incident angle approaches the critical
before it actually reaches. Horizon and exactly in the angle, >pjj and as = sin"^ for violet
'same wayeven afterit has gone down the horizon rays
from the setting sun continue to reach the observer. colouris least and for red colour maximum. So, violet
colour gets TIRfirst; followed,by other coloursand red
would ^e Ae last one to get TIR-. ^
12. (d) ,: . . , . ^ . ... 17. (a) " .•
The coating of optical fibers of a light pipe is of a When light rays fall on dry sand, due -to smooth
material having refractive index less, than that of the surface, they are reflected back and hence dry sand •
^ j fiber, to optically irisulate from, each other. It work on appears bright.. ,
the principle of total internal refraction.' "• 19. (a).^- -•, ..V-
13. (b) Diffraction (bending of waves) occurs when obstacle
Due to blackening of ball, somewater droplets (in the ' size is comparable'to waveleiigth; • ' •
form of thin film) vvould be there on ball's surface, as. a 20. (a)
result, light getSi.TIR from, water-air interface 'ahd Both statements are correct and statement 2 IS
hence the ball appears bright.'Statemeht-2 is also true correctly explains the- Statement 1
but doesn't explain 1. i

.lie.'.-)
1 . V ... I

1. )• I ' ' I• *- •
Venlcal
scale

.•Vtiul:

My^jfceye

Object fit

WAVE OPTICS

INTERFERENCE OF LIGHT WAVES At such point the permanent, destructive


Consider a, ripple tank in which there are "two needles Interference is indicated by open circles.on. the diagram.
oscillating in phase, each producing expanding circular Similarly, where two crests or two troughs overlap, the
wave- fronts of equal magnitude. (Fig. 2.1). Recall that a line oscillations produced have twice the aniplitude. These
or a surface connecting points of constant pHase on a wave is points of permanently- constructive interference are
called a wave front* The solid lines represent crests and indicated by solid circles in the diagram. . ' '
dashed lines represent troughs. We assume that the The principle ofsuperposition saythat at points where a
amplitude of the waves does not decrease wiith distance. crest (trough) ofone ripple system meets a crest (trough) ,of
When a crest from one source meets a trough from the other, .another ripple system the resultant (observed) amplitude of
the surface of water has zero displacement. the water surfacewill be equal to the sum of the individual
! Nodal curve amplitudes. Constructive interference is said to occur
because the ripple systems are in phase with each other at
Anti-nodal! these points. Similarly, destructive interference occurs at
curve.
points where a crest meets a trough, and the resultant
amplitude is now the difference between the individual
amplitudes.
Depicst wave crests' by •solid curves. "R-oughs, lying
, midway.b,etween the .crests, are not shown. The antinodal
cuiyes are .formed, by joining iip points'of constructive
interference ]and the nodal curves by joining up points of
. destructive interference. " ' ' • ' • • ••
- Althouglj the ripples travel outwards, the antinodal and
nodal curves will stay in the same place so long as Ae
'dipping' frequency of the source is the same. Stationary
antinodal and nodal curves can also be obtained if the
needles areattached toseparate bars provide they vibrate at
the same frequency and there.is a constant phase difference
between them.

Fig. 2.1
245
LWAVE OPTICS _ „ ^
Conditions for Obtaining a Stationary Interference Phase
break
Pattern: Coherent Sources
The rules for achieving this condition are:
The frequency of the waves must remain constant.
There must be a constant phase difiference between the
waves leaving the sources.
Interference phenomena can be produced by waves
generated by different kinds of sources: radio, microwave,
acoustic and so on. Sources giving rise to these interference
Fig. 2.3
phenomena are said to be coherent.
Stationary interference patterns may be produced by Monochromatic light has a unique wavelength and is
any number of sources acting together, so long as the above represented by a perfect sine wave. Laser light is an
rules are satisfied. extremely good approximation to monochromatic light,
Interference using Ordinary Light Sources possessing a coherence length of several kilometers.
The antinodal and nodal produced by two distinct Quasimonochromatic light can be thought of as a
ordinary light sources, such as two sodium discharge lamps, mixture of monochromatic waves of slightly different
do not stay in the same place at all times, and a stationary wavelength and can be represented by the wavetrain." Light
interference pattern is not obtained. The light leaving the from a sodium discharge lamp has a, coherence length of
two sodium lamps is not coherent because it is impossible to about 2-3 cm. White light, on the other hand, is a mixture of
correlate the light emission processes in the two sources. monochromatic waves of' widely different wavelengths
which add to produce a light pulse: the Wave are only in
The initial phase (f) of a light wave can be related to the
phase within a very narrow region where constructive
atomic processes occurring in a light source.
interference occurs. ' '. " •• •
However, it is impossible to say exactly when an atom
Light is a vector phenomenon; superposition the electric
will radiate because the emission process is completely
and. magnetic fields are vector fields.
random. Fig. 2.2 pictorially describes the emission of light
by! a single atom in terms of wave packets. Other atoms in In accordance with the principle of superposition, the
the source behave similarly, but with different emission electric field intensity E, at a point in space, arising from the
times.
separate fields Ei,E2,... of various contributing source is
Timet,
given by
£ = Ej-h E2 +•••
^ >

, Fig. 2.2 . The optical disturbancej or light field E, varies in time at


an exceedingly rapid rate," roughly ' .
Adding together the wave packets generated by' all •' '4.3 XlO^.'^Hzj^td. .7.5X10^^Hz "
atoms in the. light source ' produces a succession of
making the actual field •an impractical quantity to
wavetrains (see Fig. 2.3).'Eachwavetrainhas'afmite leng'th.
detect. On the'other hand,'the irradiance I can be measured
In passing from one wavetrain to the'next there is an abrupt
directly with a wide variety .of sensors (e.g., photocells,
change in phase and plane of polarization. Thus, it is not
bolometers, photographic emulsions, or eyes).
possible to relate the phase at a point in wavetrain B to a
point in wavetrain A. Consider two point sources, Sj and S2, emitting
monochromatic waves of the same frequency in a
There are, in fact, about 100 million breaks in phase
homogeneous medium. Let their separation a be much
every second," which means that each wavetrain lasts for
greater than X. Locate the point of observation P far enough
about 10 ns, the so called coherence time' the
longitudinal coherence length is equal to vt^.
away from the sources so that at P the wavefronts will be
planes. For the moment, consider only linearly polarized
waves of the form
246 OPTICS

Ei(r, t) = Eqi cosCKj- r-cot + e^) lAl =lA^i-hlA2l


—> —> —^ ^ We thus conclude that the intensity observed uppp
and E2(r,t) = Eq2 cos(K2- r-cot + £2) the superposition of incoherent waves equals.the,
sum of the intensities produced by each of tlji^j,
waves individually :

For coherent waves, cosS has a time-constant value G^ut


a different one for each point of space), so that
I =I^+l2+2^[lJ^cosb ;.:<2)
At the points of space for which cos8 > 0, the intensity7"
will exceed +12', at the points for which cos5 < 0,it will be
smaller than -i-l2- Thus, the superposition of cohereni:
light waves is attended by redistribution of the light flux' ifi'
''K2/K2/ / / space. As a result, maxima of the intensity will appear at
some spots and minima at others. This phenomenon is'
called the interference of waves. Interference manifests
itself especially clearly when the intensity of both interfering"
waves is the same : = 12- Hence, according to eq. (2), 'at
the maxima 1 = 41^, while at the minima' 7 = 0. "For
incoherent waves in the same condition, we get the same'
intensity / = 21^ everywhere [see eq. (l)]. '- '
It follows from" what has been said above that when a
surface is illuminated by several sources' of light (for
example, by COtwo
. lamps), aii interference pattern ought to'be
observed with a characteristic alternation of maxima and

Wavefronts
(a)

(b) •
. X
, I .

Fig. 2.4,
. . . ..'1 %
Let us assume that two waves of-the sanie frequency,
being superposed on each other, produce'oscillations of the - d/2-
same direction, namely,
' d/2'
Aj cos(cot-t-aj); A2 cos(cot-f-a^) ' " • •
at a certain point in space. The amplitude of the
resultant oscillation at the given point is determined by the
expression
(b)
A"- = AfA| + 2A1A2 cos5 Fig. 2.5
where 5 = a2-ai
If the phase difference 6 of the oscillations set minima of intensity. We know from our everyday experience,
up by the waves remains constant in time, then however, that in this case the illumination of the surface
the waves are called coherent. diminishes monotonouslywith an increasing distance from
the light sources, and no interference pattern is observed.
The phase difference 5 for incoherent waves varies
The explanation is that natural light sources are not
continuously and takes on any values' with an equal coherent.
probability. Hence, the time averaged value of cos8 equals
zero. Therefore
2471
WAVE OPTICS

The incoherence of natural light sources is due to the where 5=^(Ax)


X
fact that the radiation of a luminous body consists of the
wives emitted by many atoms; The individual atoms emit As the maximum and minimum values of cos5 are +1
wave trains with a duration ofabout 10 ®sand a length of and -1respectively. The maximum and minimum values off
about 3m. The phase ofa new train is not related inany way are given by
to that of the preceding one. In the light wave emitted by a ^max =
body, the radiation of one group of atms after about 10 sis ^min =
replaced by the radiation of another group, and the phase of
the resultant wave undergoes random changes. The conditions for maximum intensity are
'.Coherent light waves can be obtained by splitting (by 5 = 2n7i, n = 0,l,2,...
means of reflections or refractions) the wave emitted by a or Ax = nX
single source into two parts. It these waves are made to The conditions for minimum intensity are
cover different optical paths and are then superposed onto
each other, interference is observed. The difference between 6 = (2n-l)n, n = l,2,3,...
the'optical paths covered by the interfering waves must not or Ax = (.n-l/2)X, n = 1,2,3,..
be-very great because the oscillations being added must When Ii =l2 ^^hen
belong to the same resultant wave train. If this difference I = 410 cos^8/2
will beofthe order, ofone metre, oscillations corresponding
to 'different trains will.be superposed, and the phase Clearly if 5=±n, ±3jt,..., the resultant intensity will be
difference between ,them will continuously change in a zero and we wUl have a minima. When
chaotic way. , . , . • . . - - (|) =0,+271, ±4Jt,"..., the intensity will be maximum
When two displacements are in phase, then the
resultant amplitude will be the sum of the two amplitudes;
this is constractiye interference. cos2 5/2

. Similarly if0 = it, Stc, 5ji, ..., then . ^ >


. a = Uj -a2' - '' ^
When superposing waves
are in opposite phase -the
resultant amplitude is the
difference of the - two Fig- 2-7_.
amplitudes; this is • Fig. 2.8(a) shows the waves emitted by sources Sj
destructive interference.
and S2. The waves from the source start in phase and
' The difference in the arrive in phase, leading to constructive interference
distances from Si and S2 to a at the point.
given point is caUed the path Path difference= S2P - S^P
difference. , Fig. 2.6
A path difference of one
wavelength corresponds ,to,.a • • \ / \ .
phase difference of 27t radian because a shift mposition ot ISource 1
one wavelength" along the wave "changes its phase by a Constructive
interference
complete cycle.
For apath difference Ax the phase difference in radians Source
is

6=^ (path difference) == ~ (Ax)


X "A,
'Source 2 • VV'
The factor ^ is number of wavelengths corresponding Fig. 2.8 (a)
to path difference. Each wavelength corresponds to aphase
difference of tc. - c Fig. 2.8 (b) shows two waves starting in phase but
• The intensity of a wave is proportional to square ot arriving in opposite phase.
amplitude, therefore eqn. (8) reduces to ., ;
J=Ij+l2+2^//72COs5
[248
OPTICS]
In Fig. 2.8 (b), the distances travelled by waves differ by light is first allowed to pass through a single small hole in
odd integer number of wavelengths. order to approximate a single point source S. The light
spreads out in spherical waves from the source Saccording
to Huygens principle and is allowed to fall on a plane writh
Source 1 two closely spaced holes, and S2. In amodem version 0/
this experiment, a laser is typically used to illuminate.^e
two holes. In either case, the holes become two coherent
Destructive
sources of light, whose interference can be observed on a
interference screen some distance away.
Source
The geometry of the experiment is simple. Parallel
yv waveffonts of a monochromatic wave are incident on two
Source 2
+
identical narrow slits, each of width a separated by
Source centre-to-centre distance d as shown in the figure. The slit
widths and their separation are of the order of 'the
Fig. 2.8 (b) wavelength of the incident monochromatic light.
In Fig 2.8 (a), the distances travelled by waves differ by Monochromatic light after passing through two slits S-^ and
any integer number ofwavelengths : 52 acts as coherent sources of light waves that interfere
constructively and destructively at different points on'the
^2-^1 = 2X, 3X,...,nX ' screen to produce a pattern of alternating bright and dark
Young's Double Slit Experiment (YDSE) fringes.
The decisive experiment performed by Thomas Young in Fig. 2.10 (a) , (b) and (c) show two top views of the
1802 is shown-schematically in Fig. 2.9. Monochromatic double slit and the screen. Fig. 2.10 (a) shows how abright
fringe arises directly opposite the midpoint between the
slits. At this point the distances ,and X2 to the
1
1
• y
Bright fringe '
(Constructive,
Bright fringe 'interference)
1 (Constructive
interference)
S2 yx
(a) (b)

S 1' 1« fringe.
(Constructive
interference)

P -r- Fig^ 2.10

slits are equal, each containing the same number of


wavelengths.. The waves arrive in same phase. Therefore
constmctive interference takes place, leading to a bright
fringe. Fig. 2.10 (b) shows another bright fringe where Xn is
larger than Xj by exactly one wavelength. The waves arrive
in same phase. Abright fringe also occurs symmetrically on
the other side of the midpoint where x^ exceeds x, by one
wavelength. Fig. 2.10 (c) shows the formation of a dark
fringe. At this position X2 exceeds x^ by exactly half a
wavelength; the waves arrive in opposite phase thereby
(b) interfering destructively.
Fig. 2.9
AVEOPTCS .249;
•'[ "Now we consider ' a • The nth and (n + l)th maxima are
p"6iht P on the distant nXD (n + 1)XD
sbreen,- at a distance D from yn=—r d
>'"+1=—3— d
slits''with D » d. The small Separation between adjacent maxima is defined as
afc'df the circle from P is fringe width ^.
alftiost a straight line. From Arc of circle O XD .
the'"^- Fig. 2.11, path centered at P - ^=yn+l-yn=-^
difference Ax = d sin0. Fig. 2.11 Similarly separation between minima is same. As long
The condition for as d and 0 are small, the separation between interference
constructive interference is To distant fringes is independent of n (order of fringe), i.e., the fringes
' ^ 'A x = S2P - SiP = ±iik, screen ^ are evenly spaced.
\ ^ .where n = 0,1,2,... As vertical distance y is related to 0 by 0 = y/D, so
A0 = AyID referred to as angular fringe width.
j;.!-dsin0 = ±nX
[Condition for maxima] D d
1. , f '

' ' The condition for


Arc of circle,
destructive interference is centered at P \ Concepts: 1. Determination'of Interference at
Fig. 2.12 yPoints Close to the Cental Maximum: Optical path
Idijference corresponding any point P on the screen in a YDSE
Graph of intensity lisd'sinQ (Fig. 2!^14). •. -- ' ;
Bright fringe

,1, Dark fringe


m = 2
Double silt
m = 1

9 > 0 rad Relative


m = 0
i0< 0 rad intensity

m = -2

m =-3 Fig. 2.14


Distant screen-
IfP is close to the central maximum, at a distance y from
Fig. 2.13 it (y of the order of a millimeter).
Then 0 is small and hence,; ' ^ '
Ax = $2?- SjP = ±1" ~ .1
—|X, where n = 1,2,... Q^tanQ-ylD. %
•'HoM'b'er,' 8l —dsinQ >' . -
dsin0 =P±|^n - —
JX, ;.'[Condition for minima] f/ «d0. " [Since sin 0 = 07
• If the, separation between screeii and slits is large, i Optical path difference -^
D » d, we have
sin0='tan0='0 =
y . !• -'(i ...ax

D
i 2. Bright fiihge : For an n^ order bring fringe, the
where y is the vertical distance from the centre of the Ipath differenceblshouldhe nXyCn = 0,1,2,3,...). ' - :
pattern. Position of nth bright fringes and dark' fringes are
respectively •-''/[ ' '*/ - ' y.--nx

. .'^=.nA.., pr.y„-=.xg] ; -; .or


fXD^' '•
:..(2)

and n —
I'Vv-D
A,— Putting n - 0,1, 2, 3, etc., we get
2j d
Jo =0,
Each Value of n corresponds to particular bright or dark Xb 1
fringe. ' ' yj =
d'
The absolute value of n is called the order of
interference.
250

2XD XD
J2 = P= (in air)
d d
3XD X'D
ys = and so on. and (in the liquid)
d
! 3. Darkjringe : For the nth ordered dark fringe, the X'
P' =|^|P =P/|x ••.(6) V
path difference 51 should be (2n~l)~; (n-1,2,3,...).
2
where, |i is the index of refraction of the liquid.
d
Angularfringe width fco) : Itis defined as the angle
XD
subtended at the slits, by one complete fringe.
or y„ =(2n-l) ...(3)
'2d Thus, oo=P (since p « D)
D
Putting = 1, 2, 3, etc., we get

yi
XD =1
~ d
.-.P =^ ...(7) •
2d
3XD
y2 =
2d
5XD
ys =
2d
and so on.
Td C r.-.-.rJP.
P

4. As long as y is too small compared to D (i.e.,tan 0 = 0), •• J.co

,the fringe separation is independent of the order and they are


]Spaced evenly, with a bright fringe alternating the dark one i
,and vice versa. Thefringe width D-

Fig. 2.15

5. The central brightfringe corresponds to the one which (0«:X, hence, even the angular fringe width also]
is a result of the interference of waves with no path length decreases, when the YDSE set up is completely immersed in ai
difference. As we move away from the central bright fringe, liquid.
along the screen, the optical path difference increases, and i.e., (o' = a)/)x ' -
'when it becomes X., the first bright fringe is encountered; 7. Let us call the distance between two adjacent intensity
[similarly, when it becomes2X, 3X etc., the second, third, etc., maxima the distance between interference fringes, >
.bright fnnges are encountered. The maximum path length and the distance between adjacent intensity minima the
'difference between the waves coming from the slits and width of an interference fringe. The distance between
[interferring on the screen can be as large as d (when fringes and the width of afringe have the same value equal to
j » D,0 = 71/2 and sin 0 = 1).
P=^ d .
..(8)'^
d 'd'
central maximum will be where stands for the According to-eq. (8), the distance between'the fringes;
.'X. X'
cj
grows with a decreasing distance d between tbie sources. If d^
greatest integer less than or equal to djX. were comparable with D, the distance between the fringes'
Hence, total number of bright fringes (in case of linear would be of the same order as X, i.e. would be several scores of
slits) will be micrometers. In this case, the separate fringes would be
~d absolutely indistinguishable. For an interference pattern tO\
+ 1 ...(5) become distinct, the above-mentioned condition d«D must^
X
be observed. ;
i 6. The positions and number offringes, for a given YDSE
8. If the intensity of the interfering waves is the same
,set up depends upon wavelength, when a YDSE set up is
(12 =I2 =Io)} according to eg.
'completelysubmerged in a transparent liquid, the wavelength
of light decrease and hence the positions and number of I = Ij +12 +2-Jlff^ cosB the resultant intensity at the
points for which the phase difference is 5 is determined by the
fringes get affected. For fringes close to the central maximum,
expression
khefringe width is given by,
WAVE OPTICS 251

I -2Iq(1 + cosb) = 4IqCos^ — If the displacement x' is much smaller than the width of
2 an interference fringe Ax = DX/d [see Eq. (1)], then the
Since 5 is proportional to A, then in accordance with 5 maxima from differentsections of the source will practically
Igrows proportionally to x. Hence, the intensity varies along be superposed on one another, and thepattern will be likeone
the screen in accordance with the law of cosine square. The produced by a point source.
right-hand part of Fig. 2.16 shows the dependence of I on x When x'~ Ax, the maxima from some sections will
obtained in monochromatic light. coincide with the minima from others, and no interference
Sc
pattern will be observed. Thus, an interferencepattern will be
distinguishable provided that x' < Ax, i.e.,
^ px
-.(9)
2^ d
X
or < ...(10)

We have omitted the factor 2 when passing over from


expression (9) to (10).
Fig. 2.16
Formula (10) determines the angular dimensions of a'
The width of the interference fringes and their spacing source at which interference is observed. We can also use this'
depend on the wavelength X.The maxima of all wavelengths formula to find the greatest distance between the slits at
will coincide only at the centre of a pattern when x = 0. With which interference from a source with the angular dimension .
an increasing distance from the centre of the pattern, the (j) can still he observed. Multiplying inequality (4) by d/^, we
maxima of different colours become displaced from one arrive at the condition
another more and more. The result is blurring of the
interference pattern when it is observed in white light. The d< —

number of distinguishable interference fringes appreciably


grows in monochromatic light. 10. Shape offringe: For the spherical waves emitted
by Si and S2. Such waves can be expressed as
9. Theseparate sections of the light sourceproduce waves
whose phases are in no way related to one another. For this Ei(ri,t) ^ EQiiri) exp [i{kri - cot+81)]
reason, the interference pattern appearing on the screen will
be a superposition of the patterns produced by each section and E2(r2,t) = EQ2ir2) exp [i{kr2 - cot + 82)]
separately.
The terms Tj and r2 are the radii of the spherical
wavefronts overlapping at P; they specify the distances from
the sources to P In this case
Ac|) = kiri -r2) + C5i +83)
0"-..
0 ... 4/2
I ^ 410 cos'^hkOi -r2) +C5i ^,82)] .
0'—-
O'J...
0--.- Maximum irradiance occurs when
01.—
dri -r2) = [27m + (82 -S^)]/ k ...(12). ,
and minimum when

C^i-r2) = [Tm'+CSs-8i)]/k ...(13)

Fig. 2.17
OPTICS

/'/
/o
/
\ \

!
n = 0
0 ;

Z' y/ S2
\\\
\
Mp
Fig. 2.19

11. The surfaces are hyperbolic. Inasmuch as the y-axis,


is an axis of symmetry, the cqrrespondirig'^^'bright fringe
surfaces are generated by rotating the entire.'pattem about
the y-axis. One should then he able to visualize the intercept
of these surfaces with the plane of an observational screen
[placedanywhere in the vicinity. In particular,'d hreen placed
perpendicular to the ox axis, as infigure, intercepts hyperbolic
lares that appear as straight-line fringes 'hear the axis,;;
[whereas a screen placed perpendicular to the oy axis shows'i
Iconcentric circular fringes centeted on the axis. Because thei
Ifringe system extends throughout the space surrounding thej
[two sources, the fringes are said to be nonlocalisied.
. 12. The holes S, Sp and $2 ofYDSE ar usually replaced]
Fig. 2.18
[by parallel, narrow slits to illuminate more fully thei
interference pattern. The effect of the array of point sources^
Either one of these equations-defines a family of surfaces,I along the slits, each set producing its ownfringe system asjustl
leach of which is a hyperboloid of revolution. The vertices of •described, is simply to elongate the pattern parallel to thel
\the,hyperboloid^ are^sepqrqted by distance equal to the right; fringes, without changing their" geotnetrical relationships4
'hajidfides ofEqns. (12) drid_ (T3f Thefoci are located at Sij [This is true even when two points along a source slitare not)
^ahd 82-Ifthe waves are in phase at the emitter, 61-62 =^0,, mutually coherent. , -I
1 andEqns. (12) and (13) can be simplified to, 1 ! 13. The effects offinite coherence length: As P on;
\screen is takenfarther above or below the axis, path difference^
! [maxima] (r^ - r2) = 2Kn/k = mX —(14)
(which is less than orequal to S1S2) increases. Ifthe primaryi
[minima] 0\ - r2) = Kn'/k = —n'X ...(15) 'source has a short coherence length, as the optical patH\
2 idifference increases, identically paired wave groups will no]
i for maximum and minimum irradiance, respectively. Fig. •longer be able to arrive at P exactly together. There will be anj
[2.18 (a) shows a few of the surfaces over which there are! lincreasing amount of overlap in portions of randomly phasedi
\irradiance maxima, the dark and light zones that would hej [wave groups, and the contrast ofthe fringes will decreases. Iti
jseen on a screen placed in the region of interference arej jis possiblefor Al^ to be less than path difference. In that case/'
'known as intei^erence fringes Fig. 2.18 (b). 1 .instead of two same phase portions of the samewave groups
^arriving at P, only segment of different wave groups will
pverlap, and the fringes will vanish.
WAVE OPTICS 253

The wavelength is shorter in the medium than in


vacuum. If the light beam passes through a thickness t of a
medium,

Number of wavelengths in plate = =—


X^ X/n X
which shows that a thickness t of the medium has as
many wavelengths as there are in a length pt of vacuum.
Therefore in terms ofwavelengthSj a thickness tin a medium of
1 ri-r2>A( refractive index n is equivalent to a path length pt in vacuum.
The quantity nt is called equivalent optical path
length.
O In Fig. 2.22 shown two light rays of s - .. ^7 —
identical wavelength and initially in
phase in air travel through two n2
different media of refractive indices
and n2, same thickness t. The
wayelengths of the waves will be
different in the two media; so the
two waves will no longer be in
phase, when they emerge.
Number of wavelengths in medium 1,

H, . . N,-='^
Xi
=X/uj
' Fig. 2.22

Number of wavelengths in medium 2,


xr _ t t not
N 9 —— —
r-i-r2<Al Xo
-2 X In-
'v/;i2

To find a new phase difference we subtract the number


of wavelengths of the waves in the two media; assuming
(b)
Fig. 2.20 n2 > nj, we have

X
Equivalent Optical Path Length Phase difference corresponding to difference of one
When a beam of light travels from one medium to wavelength is 27c; hence phase difference corresponding to
another its speed changes but its frequency, does not. N2 - iVi is .
Designating the , , ^ !
. A.
Medium
, In other words we can. say that the equivalent opticar
paths of wave in media l and 2 are h2t and n^t respectively.
Thus'the path difference. Ax = n2t - n^t.
Displacement of Fringes
When we introduce a thin transparent plate in front of
one of the slits in YDSE, the fringe pattern shifts toward the
side the plate is present.
The path length S-^P is
Fig. 2.21
SiP = (SiP-0,i,+tpi3,e
The time required for the light to reach from Sj to the
wavelength in vacuum by X and the wavelength in the
point P is
material by X.n,/we have
X c- ^
= _ = n or X^=-
Mz£ +i =l[5^P_t +nt]
C V c
X„ V n
= -{S,P + (.n-m
c
1254 OPTICS

4uia^
2Caf + 02)

2#
O'-
Cl+ P)
•Shift
OJ

Ax = d sin e
InASOO"
.00'
tane = 9=-
D
OO'd
Ax = d sin e = d0 A narrow monochromatic beam of light.of intensity I'^is
D
Fig. 2.23 incident on a glass plate as shown in Fig. 2E.2. Another
identical glass plate is kept close to the first one andparalleh
which shows that by introduction of the plate the to it. Each glass plate reflects 25% of the light incident on it]
effective path length increases by (n - l)t. The position of and transmits the remaining. Find the ratio of minimum and\
the central bright fringe which corresponds to equal optical maximum intensities in theinterferencepatternformedbythel
path length is shifted to 0', where two beams obtained after one reflection at each pla.te. ^ I
SiO' + (n - IX = S2O' 91/64 ' ' • i
- -
31/^ -

S20'-SiO' -(n-l)t
-

From ASOO', -
/
\
/
00' \l/4'
tan0-6 =
D <
' 11 y/
COO')d
Also 520'-Si0' ='Ax = dsm0-d0 =
D
(OO')d Fig. 2E.2
Therefore
D
D(n-l)t Solution : At each surface reflection and transmission
and 00' =
takes place. Intensity of reflected and transmitted rays at
first plate is J/4and 3//4 respectively. Incident intensity at
L 1 second plate is 37/4; 25% of, it is reflected' and 75% is
transmitted. Intensity of reflected and transmitted rays at
iTwo'coherent sources of light'of intensity ratio P produce 37 97 ' • • ' ' " '
second plate is respectively..The,reflected rays from
-interference-pattern. Prove that in the interference pattern
second plate reaching first plate are incident" at first plate.
^max ^min 2# • •
The reflected and transmitted intensities are 37/64 and
max ' ' mm P 97/64. ., •
'.where ^min maximum and minimum intensities;
.1^ • 3
|in the resultant wave. As =io — H— = - 7
2 8
Solution : If Oi and 02 are amplitudes of superposing
waves and/1,/2 ^re intensities, ^min ~ ~ -^0 = - 7

P=r =4'" =VP


•4 2 ^2 ^2' Hence
J_
49
^max'= af + ai + 20^02 = (aj + a^)^
imin = + of - 2aia2 = (Ui - 02^
-I. _ (Qi"+.a2)^--(ai -02)^
^max -^min (aj+02)^+ (ai-02)^ Find the maximum intensity in case of interference ofn identi-l
icalwaveseach of intensityI q if the interference,is (a) coherentl
I • • j
.and (b) incoherent. !
-WAVE OPTICS 255

Solution : The resultant intensity is given by


I = Ii +I2+ COSCj)
(a) The sources are said to be coherent if they have
constant phase difference between them. The intensity will In a Young's double slit experiment with wavelength 5890 Ai
be maximum,when ([> = 2n7r; the sources are in same phase. there are 60 fringes in the field of vision. How many fringes'
will be observed in the samefield of vision if wavelength used]
Thus 27777=
is 5460A ? !
Similarly, for n identical waves,
^max ~ +... ) = n Iq Solution: The fringe width is given by
(b) The incoherent sources have phase difference
-that varies randomly with time ^ d
' ' Thus [COS(j)]ay = 0 •If the extent of field of vision is I,
Hence j =i^+j^ UN,Pi
. ( , Hence for n identical waves, where N, is the number of fringes formed with
wavelength X. ^
I = Iq+Iq +...= rIq
As the field of vision is fixed,
UNiP, =N2P2, • ,
X^D^ _ (X2D'
Suppose thai one of the slits in a Young's experiment is wider d J \ d
khan the other;so the amplitude of the light reachingthe cen- or N,X,, = N2X2
itral part of the screenfrom one slit^ acting alone, is twice that
•from the othqr slit, acting alone. Obtain an expressionfor the Thus
60x5890 ^^^^72^,3
light intensity I at the screen as afunction o/0, the angular po- X. 5460

•sition.

Solution : The expression for resultant intensity at


0 ,
any point is , , . , .. " • A beam of light consisting of two wavelengths 6500 A andi
1 = k(:Af + A2+2AiA2Cos^) ' •, ...(l)
'5200 A is used to obtain interference fringes in a Young's
where Kis' cdn^stant'of prppdrtioriality, is the path double slit experiment. What is the least distance from the\
difference between interfering",waves, Aj "and A2 are central maximum, where the bright fringes due to both the.
amplitudes of superposing waves. As A^ = 2A2 = 2A, we wavelengths coincide ? The distance between the slits is 2 mm:-
have, the intensity at the central position, ' ^ * • •and the distance between the plane of slits and thescreen is
Io^MAi+A2)^ ='k(_2A+A)^.= 9kA^ - • 120 cm. \

So, k = Solution: The position of'nth bright fringe on the


9A- screen is • . .
The path difference at any point,!? is " • " " ' nXb' • " U . '•'
~ ' Ax = dsinG
•- - 2tc
The phase difference "= — d sin 6 Let the nth bright fringe of 6500 A and the nth bright
fringe of 5200A coincide; then
Thus eqn. (1) becomes .,
mx6500xD nx5200xD
I =
I 0
(2A) ^ +A^ + 2(2A) (A) cos
2jrd sine^
d d
9A' . J Thus the minimum values of m and n are 4 and 5
5A^ +4A'^C0sj^ 27cd\sin0^J.
* -•

In
respectively.
9A- 4x6500x120
Hence y = = 0.156 cm = 1.56 mm
•In 2''7tdsin0 0.2
1 + 8 cos ,as. cos 20 = cos —-1
2
•In a YDSE perfonned. with wavelength 'k =5890A the] i/n a YDSE for wavelength X=589nm, the intetferencS
'langular fringe width is 0.40°. What is the angular fringel ffiinges have angular separation of 3.50x10^^ radian..FM
^ immersed in water ? Iwhat wavelength would the angular separation be 10.0%';
j [greater ? ' J
Solution; Angular fringe width is given by
Solution: The angular position ofmaxima is given by
f.=i dsine = nX

So, Rair _ ^air d0 - nX [For small 0, sin0?^,0]


Pe - —
The angular separation oftwo adjacent maxima is - >
owater _ ^ water
Pe - — AQ = -
d

Pf - X,, Let angular separation be 10% greater for wavelength


I water X'.
' water
l.lOX V
_ n.
'^air Then
d
^ water 4 or
^ - LlOA, - Cl.lOx 589).= 648nm
Thus
pr"=^pf
4
=:o.40»x^
- -4
=0.30=
EMgSiiBlfcj.'.s- jA YDSE p perfonned with two.\yav.elengths/simultaneously,
iXj -48Onm.and,X2.—.0QdhTh-.,The distahce between the
iFirni the angular separation AQ between; the corresponding' • islits is 5.0,mThahd the;sliis a^^^ thescreen. What is
'points on the 'screen, i/,-0 is small enough-so-that'smQ ~Q:, .-the separation ofthe screeit between third orddr'Cn'-3) bright
'Refer to the iritehsi^ distkbution curve ofFig?2:9. There are]:
•points oh the curve where the intensity ofthefringe is halfthe'
' j > Solution; Condition for.maxima is
dsinB = nX
Solution:, For identical slits'the intensity on the
screen is given by ' ' • or dB-nX '
(j)' nX
I = 4/n COS^ ^ 0 =
~d
The angular separation oftwo maxima, associated with
where
different wavelengths and same order is
where dis slit separation: and'X is-the'wavelength.- • A0 ="^^'2 ~^1)
• d .
The intensity at the centre .of tlie interference pattern is•
47o. We want.the an^laf position for which / =21q. The separation^ on ^a. screen a distance^P^away is •
So 2/0 = 4/0 tos^ Av = DfflnrAft}=rp^ft^ nD{X2~Xi)
d
(as tan0 - 0 for small angles)
(j) = 2 cos ^ ^ J
= — rad. 3(1.0)
.V5 (600-480)x.l'0 -9
.S.Qx 10^-3
7t 27c dB
Hence' [sin0-0]
2 X.- = 7.2xl0~^m

0 =
4d ' ••
Another point of half•• intensity lies at 0=-X/4d, \^W0 pointsourc^ separated by.2 .0 mare radiating in phaiej
symmetrically plaped relative to ihe central maxima. So the jWif/i X- 0.50 rh.A detector movk in a circular path around!\
width of the pattefmaf'half mtSisify'i^ 2X /4d =X/2d. the two sources in a plane containing them. How manyl
deleted ? •
WAVtOPTKS 257

Spjutipni Condition for maxima is


P !
dsinG - nX

• A '0.50^
Sin 0 = — = n = 0.25n
d 2.0

As' sinG lies between -1 and 1, so we wish to find all


values of h for which
i0.25n|<l d sin 6

These 'values are -4, - 3,- 2, -1,0, +1, + 2, + 3 and +4.


For each df these there are 2 different values of 0 except for
-4 and +"4; A.single value of0, -90® and+ 90°, is associated Fig. 2E.13
with n = -4/and n =+4 respectively. Thus there are 16
different angles in all and'therefore 16 maxima. Qi - l)t = nX = dsin0
(ii-l)t (1.17-l)x 1.5x10"^
J .12 : sin0 =
d ,-7
3x10
iJn aYDSE the incidentyUldw light is composed of two wave-j = 0.085

llengths 5890 Aand 5895 A. The distance between the slits is^ Hence the angular position of central maxima is
JO^^mahd thescreen isplacedl maway. Up to whatordercan\
p . - ,
0 = sin-^(0.085) = 4.88®
fringes beseenon thescreen and howfarfrom thecentre of thel For small angles sin 0 = 0 - tan 0
[screen does this occur ? - 1
As tan0 = —
Solution: when the maxima of one wavelength D

coincides with minima of the other, the fringes will y _(p-l)t


so
disappear. D d
n(5895)D _ (n + 1/2)5890D ,
So, Shift of central maxima is Y= —5^. This formula.
d ~ d d
or ' • n = 589 can be used if D is giyen.
Hence fringes will be >visible up to 589 order of
14 I
wavelength 5895 A.
^ Position of this order on the screen is !A doubleslit e^eriment is immersed in a liquid of refractive
nW. 589(5895) xl Hndhc. 1.33. It has slit separation of 1 mm and distance
=
,-3 •
= 0.347 m [between tfie plane of slits dnd screen 1.33 mrn. The slits are
10 dllufninated by^a parallel beam of light whose wavelength im
13, lair is 6830 A. Calculate the fringe 'width. • j
Solution: The experimental'setup is in a' liquid,
\ln a YDSE light of.wavelength A =-5000 A is used,, which', therefore the wavelength of light will change.
lemerges in phase'^from^two slits a distance.d =3xlO~^jji] _ Xatr _ 6300'_ 6300x10"^°
'.apart. A transparent-sheet of thickness t = 1.5x10^^ m, re-l ^ liquid m
11 1.33 1.33
\fractive index \i = l. 17,isplaced over one of the slits. Where!
[does the central maxima of the interference now appear ? ^ Fringe.width p=
d fid
Solution: The path difference introduced, due to 6300x10"^° 1.33
introduction of transparent sheet is giveri by Ax = (jx - l)t. X
1.33 10"
If the central-maxima occupies, position of nth fringe,
then = 6.3xl0"^m
[258

15 ^1-sine") .. -cose
Lim = Lim 0
e-» ic/2 cose J q^k/2 sine
'Two coherent sources emit light of wavelength X, separation Thus at 0 —uj2. we have n = 0, i.e., zeroth' order
\between them, d=4X. (a) A detector Amoves along the' maxima is formed at x = ©o. *- •
ly-axis. What is the maximum number ofmaxima registered if
,the detector moves along they-axis ?(b) Repeat the problem ' Hence along x-axis, beginning from slit B three other
maxima are registered. '' •
•if the detector moves along thex-axis.

>In YDSE the source is red light of wavelength 7x10 -7 m.


When a thin glass plate of refractive index 1.5 at this;
! t
wavelength is put in the path of one ofthe interfering beams, \
rdsine=
'the central bright fringe shifts by IQ-^m to the position]
So: previously occupied by the 5th bright Jringe. Find thel
D »
D
d
thicknesss of the plate. When the source is now changed tol
i - t igreen light ofwavelength 10~^ m, the centralfringe shifts to ai
_ --
position initially occupied by the 6th brightfringe due to redl
light. Find the refractive index ofglassfor the green light Also]
Solution; (a) The difference at any point on the estimate the change in fringe width due to change inl
screen is, \wavelength. ' ^
dsinG = nX
Solution: introduction of a glass plate shifts the
n = — sine central maxima by
X
n is maxinium when sinB is maximum; thus
d • X
maximum n = — = 4 The central maxima shifts by five fringes; therefore
X
Thus apart from central maxima at 6= 0, eight other ...(1)
maxima, 4 on either side of central maxima are registered.
Total maxima registered are 9 and-8 minima lie between • _ 5x7x10"^
Therefore, t = - 7pm
them. ...
•CPrvI) , (1.5.-1)
Ch) 'The path difference at any, ; ~ -— - - , . .Similarlywhengreen light is used,.the central maxima
point Q on the x-^is, is shifts by 6 fringes; therefore
, Ax = AQ - BQ
- ! f
'' . " AB ' I 7
A.Q
^" hPG ...(2)
(1 - sin 0)
COS0
•Fig. 2E,15 (b) ; •. Dividing eqn; (l) by'(2); wVgef ''
Condition for maxima is
• d
^ (1- sine) = nX
COS0
_ 4(1-sine) Pg =1-6 ' ^,
n =
cose As given in the example,
^ point B, 0=0, n=4; we get fourth order maxima,' at =10-3, =2x10-^
X- oo, 0 = 7c/2, n is indeterminate at this position.
Also Pg__ (^b-P/d) ^ ^5
„ , ^(l-sin9b°) ' 0 Px ^RD/d) X-R 7
cos 90® 0
'We can solve the expression Tor n.with the help of E or

Pb •7 7,
Hospital rule of limits.
Pg-Pb- 2 •
Pb' ' 7
Thus AP = Pg=-_xp„ =_o.57xlO-^'m
259
WAVE OPTICS
•..I——

The minus sign denotes that fringe width will decrease, Thus Ax = 0.3t = 5X + — = ——
6 6
when green light is used. -7
31X 31x54x10 = 9.3pm
or t =
6x0.3 0.3x6

7n a .Yl>SE the upper slit is covered by a thin glass plate of.


refractive index 1.4 while the lower slit is covered by another^
iglass plate, having the same thickness as the first one butl A' coherent parallel beam of microwaves of wavelength.
[having refractive index 1.7. Interference pattern is observed'. \X = 0.5 mm falls on a Young's double slit apparatus. The
\using light ofwavelength 5400 A. It isfound that the point P; [separation between the slits is 1.0 mm. The intensity of
on the 'screeni. where the central maxima (h = 0) lay before' microwaves is measured on a screen placed parallel to the
the 'glass plates were inserted, now has 3/4 the original [plane of the slits at a distance of 1.0 mfrom it as shown in'
.intensity. It is further observed that what used to be the fifth' [Fig. 2E.18 (a) If the incident beam falls normally on the
maxima earlier, lies below point Pwhile the sixth maxima lies [double slit apparatus, find the y-coordinates of all the
iabove Calculate the thickness ofglass plate (Absorption of 'interference minima on the screen, (b) If the incident beam
lligHthythe glass plate maybe ; •makes an angle of30°with thex-axis (as in. the dotted arrow
.shown in the figureX find the y-coordinates of the first
[minima on either side ofthe central m^mum^
- -6th minima " ~ ' ~"v ..

- - 5th maxima s
y Si d = 1.0 mm
,
"30" •X.

D=1.0m-
Flg.2E.17

Solution: when both the slits are covered by glass Screen


Fig. 2E.18(a)
plates the additional path difference at point P(position of
central maxima in the absence of plates) is
Solution: Position of a point on a screen is. ,
= = a7-1.4)t = 0.3t ,
Path difference af fifth maxima is 5X and at sixth y = DtanO = ItanO
• N. - 1 - •- ~ ' '1 , 'j "•
minima-is —. As thepoint Plies between fifth fnaxiina and
2 ' - , . • ^ •
For first minima, n=1, sihOi = tanBi =-^j= ^ .
^^jtth minima, the path difference at is . ' ' '.L \ 3 3
Ax"== 5X, + An' .
For second minima, n = 2, sin02 = tan02 =
V • ^ -----

The resultant phase,difference,is ='^10;c,+.-^'Ax' So-the positions of minima are

The" resultant'ihtehsity is given-by


y.^^ =tan01 =—^ =0.258m
• -Vis
9 (b 3
Ir = 4JoCOS - and y2 = tan02 = = 1.13m
According to the problem The minima are symmetrically placed on either side of
;Jk=^ =4^oCOs'| , central' maxima; therefore there will, be 4 minima at
positions ±0.258m and ±1.13m on the screen."
2-<i'
-COS- = -
3 • . ••• , (a) When incident rays, "are incident normally, the
waves arriving at slits are in phase, zero path difference
.'before.slits. Pathdifference after'slits>'at-point P, is dsinO.
(|) = 2cos~^ — = — Condition for minima at y-axis is
2 3>: «
Path difference Ax' corresponding to phase difference

3 " k3 27tJ 6
[260
OPTICS
y
[S 19

; s, y . AYDSE is performed in a medium ofrefractive index 4[3.a\


i \light of 600 nm wavelength is falling on the slits having'
10.45 nm separation. The lower slit $2 is covered by a thin
s. d sine }glass plate ofthickness 10.4\im and refractive index l.'s. Thel
Imterference pattern is observed on ascreen placed 2.5 mfrom\
Fig. 2E.18 (b)
[the slits as shown in Fig. 2E. 19. (a). Find the location o/t/zej
^central mcmmum (brightfringe with zero path difference) onl
.they-axis. (b)Find the light intensity at point O relative to>
dsin0 = (2n-l)- maximum fringe intensity, (c) Now, if 600 nm light is
2
\replaced by white light of range 400 to' 700 nm, find the
sinfl = _ C2n -1)(0.5) _ (_2n -1) [wavelength ofthe light thatforms maxima exactly atpoint 0. |
2d 2x1 4
'(All the wavelengths in this problem arefor the given medium
As sine < 1; < 1 or n < 2.5, as n = 1,2,3,... 'of refractive index 4(3. Ignore absorption.)
I ^ ^
Hence only first order and second order minima are
possible.
(b) Path difference before slits = dsin(f) s,.
I ' s O
Path difference after slits =dsin0
As path of rays before slits is longer at5^ and 52? >S^P
after slits, so net path difference for first minima is
(a)
P
t I
Fig.2E.19 ^

Solution; (a) Path difference atpoint? on the screen


d sine
D
At the position of centtal maxima the optical path
Fig. 2E.18 (c), lengths S2? and 5^? are equal,
• • I.e.;
' 'CS2?-t)^ t _ S,P
'dsiri0'-dMh(l) = ± — c/p.2 '
2

sine - sin(b± — ,•• ' ' - - - / where;ni = 4/3,'^2= 3/2


. . ,• 2d or iii(:si?-s2?) =ai2-^ii)t •' •
0.5
= sin 30° ± or
2x1
'3' 1
= - or — y.(P-2-PlfrD
4 4
Pid
Thus tane = ~ or _ [(3/2)-C4/3)]X 10.4X1.5
V7 Vis
So the position of first minima on either side of central (4/3) X0.45x10"^
maxima is = 4.33 mm
' /
(b) At point O net path difference
y =Dtan0 =-^m and —
V7 - 'Vis Ax = |^-1 t
Pr
271 Ax '
Net phase difference Ad) = —
X
MVE OPTICS 261

2k = 8.74xl0^'^m
(10.4x10"^)
6x10 Position of 5th dark fringe.
13
71 n— AD
2) 9XD
3^5 =
d 2d
Thus intensity I = cos^C(p/2)
9x6x10"^ xl
j 21 1371 = 13.1 X10"^ m
= /max COS 2x2.06x10"^
= 1.31mm
! = -/
I ^^max
I ^
21
(c) For maximum intensity at point O
.r ^ I Ax = nX (wheren = 1,2,3,...) ;/n a modified YDSE source S is kept infront of slit Si. Find the]
Path difference at point O ]phase difference at apoint 0 thatis equidistantfrom slits Sj
1.-5 'and§2 and a pointPthat is infrontofslitSj in the following]
Ax = -1 (10.4x10"^) = 1300nm isituations: « !
4/3
Thus maximum intensity will correspond to ^ N-Xq-H
is 1 Pj
-A = 1300nm,
1300
nm,
1300
nm,...
i 'X. SJ
o!
2 3

In Ithe given range required values are 650nm and


433.33hm.

c>:c 20 I i Fig. 2E.21 I j


fa) A liquid of refractive index \l isfilled between the screen]
In a YDSE performed with light of wavelerigth 6000 A, the ,and slits. ' i
screen is placed Im from the slits. Fringes formed on the -,(b) Liquidjs filled between theslit and thesource
screen are observed by a student sitting close to the slits. The
student's eye can distinguish two neighbouringfringes. If they Solution: while calculating path difference it must be
subtend an angle more than 1 minute of arc, calculate the remembered that it is equal to difference of optical path
maximum distance between the slits so that the fringes are lengths from source S to point on the screen.
'dearly-visible. Usingthis information calculate the position of Total path difference is -
[3rd bright and5th. darkfringe from theceritre ofthescreen, j
^^tota]' before sljts, + [Ax] afterslits
Solution: The.angular fringe width, i.e., the angle > = CSS2 - SSi) + CS2P - Si?)
subtended by a fringe at the centre of slits is given by
B (a) When liquid is filled between slits and screen,
•D d ,,. • ;
According to the given condition,
7C
and [Si/'luquid = lpSlP^ak
B = — > At point O: 1132O = pSjO, no path difference is
d 180x60
introduced after slits.
rad.
As r =
180x60 So, =SS2 —SSi =-Jd +^0 ~-^0
6x10'^ x 180x60 Thus, phase difference, -
So d<
7t .
-3.
A^^ =^(^^jd^ -vxq -Xq) ''
d < 2.06xl0"-'m ' A ,

At point P :
Thus d„.,.r = 2.06xi6"^m = 2.06mm
Position of 3rd bright fringe. tA.*:] beforeslits =-yjd^ XXq - Xq
SAD 3 X 6 X10"^ X1 [A x]after slits ~ 1^2/' - ^.(52? SjP)
Js = -3
2.06x10
262

OPTICS

d
= as V = —
2D ^2
T S
Therefore [A x] + 2_ j ^ 0.8 mm
Q I
2D
Thus phase difference,
40 cm
AA
Ad = 27c

X [V^^ +^o-^0] +—
2D ii •2 m-

,(b) When liquid is filled between source and slits. Bn—H


10cm
At point O:
Ftg. 2E.22 (a)
before slits = (^^2 +S5i)]jqyjj
= 01552+p55i),i^ Solution: (a) path difference before slits,
55i -552 = dsin'^
Path difference after slits,
after slits ~520 —5jO = 0 t.

52P- 5fP = dsinG


' - • • ^^•^^total +Xq —^STq) For point P to central maxima, net path difference atP
Thus phase difference at P, ''' must be zero, i.e..
dsin(|) = dsiiie
A<|) =y[gV5^7^-;c„] ' ...(ly
At point P :

(Ax)before slits =(552 +^'^l)liquid


= (p552+p55i)3^,
fA =(SjP - SiP)^ = =il
. ' • D 2D
2 d sin 6
.(^•'^^totai ~ il(.^S+xl~Xo) +~
. • - 2D
Thus phase difference at P, Di = 2m
D2 = 10cm
AA = —
271 Fig, 2E.22 (b)
Ad
X
P-4^^.+-xl~Xo+~
2D
Note that 55i >55^ and 5;iP >SjP, i. e., before slits the
}>&
path up to 5i is longer and after'slits the.path S2P is longer.
22.
For small anglVsine-'tanG, sin(]) tan(j)
Avessel ABCD of10cm width has 'two smallllt^S, and!' So, from eqn. (1), tari^="tanG
sealed with identical glass plates of equal thickness. •
y^^z.
'The distance between the slits is O-STntn. POQ is the line ^1 D2
perpendicular to the plane AB and passing through O, the yD,
•middle point of Sj and S2. Amonochromatic light source is^ or
y=^^ = 2cm .
Di
keptatS,40
.the cmbelowPand2mfrom
shts^ shown thevesselthetoposition
in Fig. 2E.22. (a) Calculate illuminate
of Cb) men liquid is filled up to OQ. the central maxima
will be formed at Qif optical path difference before slits
nn Now^^Shtfringe
OQ. on theinto
aliquid is poured other
thewall
vesselCDandfilled
w.r.t. theupline
to produced due to asymmetry of source S, is nullified bv
OQ. The central brightfringe isfound to be atQ. Calculate the optical path difference, after slits.
refractive index of the liquid. -
263
WAVE.OPTICS
10
P' = 4x10 ^ X— = 4x10
D 1 f\r\
100

The resultant power at the focus after superposition of


two waves is
P=Pa +Pb +2^/pJ^ cosij)
where (j) is phase difference.
The introduction ofmica sheet inthepath ofAcreates a
Fig. 2E.22 (c) path difference (p. - l)t.
CnS2Q-SiQ) = dsin(l) (p - l)t = (1.5 -1) X2000A =1000 A
Cn-l)Di=dx^ Phase difference A(|) =^ Cp -l)t = x1000 =-
^1

As S2Q = SiQ =P2 and sin(t) = tan(fi = —,


y
Therefore. P=lO""^ +4x10"'^ +2Vacr^K^aF) cos|
= 7xl0'-®W
So (^_l)10 = 8xl0
Uoo FRESNEL'S MIRRORS
0

Thus, ' H=1.0016 Fig. 2.24 . shows Fresnel's bimirrors apparatus to


produce interference by division of the wavefront. Light from
23 k> a slit Sis reflected by two place mirrors slightly inclined to
each other. The mirrors produce two virtual images Sj and
In a modified YDSE a monochromatic uniform and paralleh S2of the slit, the interference fringes are observed in the
beam of light of wavelength 6000 Aand intensity (lOjn) I region BC, where the reflected beams overlap. ^
Wlm^ is incident normally on two circular apertures AandB'
of radii 0.001 m and 0.002 m, respectively. Aperfectly^
transparentfilm ofthickness 2000 Aand refractive index 1.5^^
j/br the wavelength of 6000Ais placed infront of aperture A;
'(see Fig. 2E.23). Calculate the power in watts received at the,
focal spot Fof the lens. The lens is symmetricallyplaced w.r.tA
the apertures. Assume that 10% of the power received by each,
aperture goes in the original direction and is brought to thej
focal spot. ,

Fig. 2.24

•Fi If 6is the angle between the planes of the minors, then
Si and S2 subtend angle 2fi at the'point 6f;intersection M.
between the mirrors.
If I is the distance between the slit and the minors
intersection and Lis the distance between the screen and the
Fig. 2E.23
minors intersection, then the separation between the
images Sj and S2 is
Solution: The power transmitted by apertures Aand d ^ 1(20) = 210
B are
and D = l+L
P. = — [7cC0.0pl)^] = 10 Thus, thefringe width is given by
XCl + L)
or p=
[7cx(O.Od2)^] =4xlO^W • 218
=0
Only 10% of transmitted power reaches the focus. or

K=10-^x^ =10-^W 10
264 OPTICS j

Fresnel's Biprism screen is placed in contact with the end of the mirror the
Fig. 2.25 shows the Fresnel's biprism experiment edge O of the reflecting surface comes at the centre of-a dark
schematically. The thin prism P refracts light from the slit fringe instead of a bright fringe. The direct beam does not
source S into two beams AC and BE. When a screen MN is
suffer any phase change. This means that the reflected beam
placed as shown in the figure, the interference fringes are undergoes a phase change of n radian. Hence at a point P on
the screen the conditions for minima and maxima are
observed only in the region EC. If the screen MN is removed,
the two beams tvill overlap over the whole region AE.

mm
d =2iiA(n-1)
mA.(Zi+i2)
Vm ~

mm

Fig. 2.26: Lloyd's mirror.

Fig. 2.25: Interference'with Fresnel's biprism. Coherent sources are the virtual , SiP - SiP = nX, n = 0,1,2,3,... [minima]
images S,and $2 of source S,formed by refraction in the two halves of the prism. "
S2P-SiP =|ll+i|X [maxima]
If A is the angle of refraction of the thin prism and p. is
the refractive index of its medium, then the angle of The Phase Change on Reflection
deviation produced by the prism is A ray of light is incident on air-water interface; let the
,6 = Aai-l) amplitude reflection and transmission coefficients be and
If li is the distance between the source and the prism, respectively. The amplitudes of reflected and transmitted
then the separation between virtual sources is waves are ar^ and atj respectively. From the principle of
reversibility of light, the system retraces its whole previous
' • - d = 26?i = 2A(p-a)Ii •/. . motion. The wave of amplitude ar^ gives a reflected wave of
If" 12 is the distance between the prisni and the screen, amplitude ar^ and the transmitted wave of amplitude ar^t^.
then the distance between virtual sources and the screen is The wave of amplitude ati
given by '
incident Reflected Reflected Incident i
II I I I • iD = sl'i +12'' 'A.'.' 'amplitude amplitude amplitude amplitude {
Thus, by using the results of Young's experiment, the Transmitted
Air amplitude ' '
fringe width is given by • '
Water'

^ -d .^"25/1 /v'"-'
Transmitted Incident
amplitude amplitude;
or P= 1+^
26 . k. Fig. 2.27

gives a reflected wave of amplitude ar2ti and


or [3 =
.Z4(p-1) transmitted wave of amplitude atjt2.
Fringes observed in the Fresnel's biprism experiment are So, ar{ + at-^t2 = a
vertical straight lines. > >• ^ ...(1)
The Lloyd's Mirror Experiment Further, the waves of amplitudes atir2 and ar^ti must
In ,this, arrangement jthe light:, reflected from a long cancel each other. • . - - .
mirror and the light comingdirectlyfrom the source without atir2 + ariti = 0
reflection produce interference on a screen. M important,
feature of this experiment lies in the fact that when, the r2 = -ri ...(2)
WAVE OPTICS
265

Eqn. (2) shows a difference of phase of tz between the where 7n = 0,1,2,...


twoxases; a reversal of sign means a displacement in the
opposite sense. If there is no change of phase on reflection (3) One of the reflected waves experiences a phase
from 'above, there must be a phase change of tc from below change of n radian upon reflection and the other wave does
not.
and vice-versa.
When lightgets reflectedfrom a densermedium there is an
It is material which wave suffers a phase change; the
conclusions in the previous case are first reversed.
abrupt phase change of k; no phase change occurs when
reflection takes place from rarer medium. Condition for destructive interference :
2fxt = mX
Thin Film Interference
Condition for constructive interference :
When light passes the boundary between two
transparent media, same light is reflected at the boundary. "2pt =fm +i
As shown in the figure some light is reflected from first
interface and some from second interface. If we consider a 0,1,2,..
monpchromatic incident light the two reflected waves are
also Imonochromatic and coherent because, they arise from Y :
^R2
the same monochromatic incident light wave via amplitude Air
Li / "- < /
divisipn. These waves interfere, since they are superposed \ : Ac
along the same normal line. fr\ I / \
I

The phase difference between two interfering waves is Film


• \d / \
due to :
: Vb a
(1) Optical path difference (due to distances
travelled);
(2) Reflection from a denser medium.

L-
Fig. 2.29

O If the raj^ of light are incident obliquely on the thin


film, the optical path difference is 2p.t cos r, wherer is
angle of refraction.

A = \l(AB + BC)-AN AN = iAD + DOsini =


1 , i A , .
tanr

2t ^ 2tsini. . , ., ,
Fig. 2.28 Vcosr J tanr ,. - - ,
From SnelFs law, sini =|xsinr; on-substituting it and
The second factor is irrelevant for reflection at rarer solying we get , ,, , ,. ,, ., . . , ,
medium. Three situation may arise : c ' • - ) • t - •

A = 2|.it cosr <' ' •'


(1) Neither,wave" experiences' a phase' change upon
reflection. THE INTERFERENCE CONDITIONS BY
(2) Both the waves suffer- a phase change upon DIVISION OF WAVEFRONT AND AMPLITUDE
reflection. The phase difference 5 at a ^point P between waves
In either of these two cases the phase change due to leaving two lightsources is given by
reflection is irrelevant; no difference in phase results due to 5 = fc(X2-Xi)+((|)2-(|)i)
reflection. The resultant amplitude keeps the same value if (1) the
In either of these cases phase change is determined waves have the same frequency, and (2) ((()2 remains
solely from optical path difference. constant. However, the presence of the phase breaks shown
Condition for constructive interference :
in Fig. 2.31 makes it impossible for CC<l)2-<|)i) to stay
constant for times longer than about 10 ns. Away of
'2iJ.t = mX' ' ' ' circumventing this problem .is to divide the light from a
Condition for destructive interference ' single source into two parts. The standard techniques for
doing this are called division of wavefront and division
2|xt =fm+ijX of amplitude.
266" •
OPTCS
In division of wavefront, the expanding wavefront from
aprimary light source S falls on two narrow, closely spaced
apertures, Sj and ^2, cut in an opaque screen, as shown in
Fig. 2.30 (a). If the apertures are equidistant from the
primary source, as in the simplest experimental set-up. the
phase of the wave at will be the same as the phase at52
because they were emitted by 5 at the same instant. Further,
waves leaving 5^ and S2 have the same frequency as the
primary source. Hence, sources 5^ and S2 satisfy the
interference conditions; they act as secondary coherent
sources. The secondary sources are real in Fig. 2.30 (a) but
they may also be virtual for a biprism.

Optical
sx-v- -
axis

Fig. 2.31: Two-wave interference: (a) Young's two-slit arrangement,' -'


(b) Fresnel's biprism, (c) Fresnel's two mirrors, (d) Loyd's mirror
SfV (e) Billet's split lens._
(a) (b) The narrow slits and S2 in Young's double- slit
Fig. 2.30 arrangement [Fig. 2.31 (a)] intercept portions of the
spherical wavefront and act as real mutually coherent point
In division of amplitude, each wavetrain is split into sources, ' diffracting light in the forward direction.
two. Each part has the same initial phase as the primary Interference among the diffracted waves can be observed
wavetrain and change its values instep with it,thus keeping anywhere in the region of overlap (shaded portion) behind
constant, the wavetrains appear to come,from the plane of the Fig. 2.31.
virtual secondary coherent sources. These are. 5, and 5, in
Fig. 2.30 (b). • ' •- ...... / BILLETS LENS
. Before consideririg interference bydivision ofwavefront Consider a lens of focal length f cut into equal, Halves
and aihplitiide in detail,'it is essential to ldiscuss Huygen's along any diameter and separated away bya distance a with
principle of secondary sources. ^ ' '' ' an opaque substance filling the gap as shown in Fig. (2.32) .
Interference by Divjsionpf Wavefront
Let Sbe asingle slit ata'distance ofbfrom the lenses (b> f).
Evidently the two lens halves form two distinct images ofS.
•Coherent waves can be obtained from a 'point source' in'
two way^. In the wavefront division approach, the spherical
wavefront emanating from the point source is split and then"
recombined after introducing an appropriate path optical axls^
fo,\ e
difference. This is what happens in Young's two slit M
arrangement. Alternatively, the amplitude of the incident
wave is split at an interface between two media (division of optical axis^
amplitude) to generate two waves which interfere upon A02
recombination..
Some of the standard arrangements used to observe
two-wave interference by division of the wavefront are i .^ Fig. 2.32
shown in Fig. 2.31.
From- the lens equation we have,
c= JL .
"r.f
WAVi'OPTICS
267
-From similar ASj SS2 and AOj SO2 V2

we "have,
$282 _
C C
SoP = D
3D
b + c b
d a = D 1 + +...
or
b + c 3D
b
[Expanding by binomial theorem, and ignoring higher
d = p powers of —1
0 D

= 1+ ^ S2^-SiP = X/3.
[ (b-f)
.*. Phase difference = —
ab 3
b-f Similarly,'S3P = [(SiP)^ + (2dff^ = (D^ +4^2)^2
For suitable values of a, b and f it is possible to obtain Using d = ^2kD/3, S3P - SjP = 4k/3
the value of d~ 1mm comparable to the order of the slit
separation adopted in Young's double slit experiment, so (effectively)
that interference fringes can be observed on a screen placed Corresponding phase difference = 27c/3. .
at a distance of 1 to 2m from the images. -• -Applying vector diagram method, [See Fig. 2E.24 (b)] -
Two lens pieces': If a convex lens be cut in such a the resultant amplitude =A . i ^
way that its central portion of a width 5C^ mm) is cut out
and the two halves are tightly fitted against each other, then
corresponding to a single slit placed symmetrically in front
of the combination yields two image sources. Usually, the • A,>/3
[(resultant)
distance /o' ofthe slit S is less than the focal length f of. the
lens pieces.
i ^
24. ' ; Fig. 2j.24Jbl
.'. Resultant intenrity I «: 3Ag
jPig. 2E.24 (a) shows three equidistant slits 82,82 and sj i.e., ' '
^each emitting light of intensity Iq, and separated- by a» I = 3Io.
^^distance d—2k D/3/where k is^^the wavelength of thel 25 1
• - I

^monochromatic light illuminating the slits. Find the intensity^ ( ' '

,J
r

.

N .•
r

I
jat point Pdirec^ infibnt of the riiVSj(Here k'^<,D). ^ I
dn Young's double slit'^eriment the screen is di a.distahce^DI
' •.] " T •
i^om the plane of the slit and slits are iltuminated by planel
. V f ..

T.St
• I I . j«.' fnonochromatic light ofwavelength-k ". P'is a^point'dn ijiel '
d
p'i ' r. t'l *I • screen at a distance y'from the tehtral hicddmum.'if-by sotnei-
--S2 special-arrangement/the sUts be moved symmetrically apart
withra ,relative velocity v, estimate: the, number of fringes
crossing.thepoint,P per unit time. • ^^

Screen Solution: Let l be the distance between the two slits,


at any instant, then
Fig. 2E.24 (a)
it is given that . v = —. •Ui)
dt •
j Solution: First let us find the optical path difference, The path difference reaching the pointP from the' slits is
between the waves'interfering at P. .
V2 evidently'A =y^ ! . '' • ' ' '•
! S^P =[CSjP)^ +d2]V2 ^ ^ ^^ 1 +
•D\ Differentiating both sides with respect totime t,we get
Since, d = ^2XD/3 dA _ y dl yv
d^ = 2XD/3 dt d 'dt D
[Fromeqn. (i)]
268 ~ ' "
OPTICS
Since, a change in optical path difference of X, I ni = -,-,-.
3 1 1
corresponds to onefringe, so the number of fringe crossing or |cose|
point P, per unit time is
At D - D
^dA ^1 _ yv But COS0 = —

9 4 1
cos0|^ -
26 +y^ 16^ 16' 16
D^+y^ 16 16
Fig. 2E.26 (a) shows an arrangement to observe interference's or = —,—,16
D' 9 4
pattern, by two coherent sources Sj and S2 separated by a
',distance 4X. Ascreen is placed at a large distance Dfrom thel 2_7D^ _2 ..^2
y = -,3DM5D"
-sources normal to the linejoining them.
Determine the distance y(y •;tO or such that the intensity,
at point P is same as that at point O. y- ±y,±V3D and ±Vl5D.
In caseof linear slits, the positive and negative valuesof
7 : Y
y respectively stand for points lying above and below. 0. In
case of point sources, the three values of y denote the radii
of the circular fringes.
if*—4X—H
oi :
(Not to scale) Screen TVo parallel beams of light P and Q {separation dlj
'Containing radiation of wavelength 4000 A and 5000 A'
! , Fig. 2E.26 (a) {which are mutually coherent in each wavelength'
•separately) are incident normally on a prism as shown in
Solution; since 'Fig. 2E.27. The refractive index of prism as a function of
D :>> X,, so D can be taken to wavelength is given by the relation
be the distance of the screen
from . the mid-point of the
p{X) =L20 +-^ i
X^
sources Sj and 82-^
obviously, ' 'the_ _path sin0 - 0.8

difference between the waves


interfering-bt point 0 on the
screen is 4X ; hence; point p is
point, of maximum-intensity. C: ^ ••!
Fig; 2E.26 (b)
[Fig. 2E.26(b)] \ . ! I ' Fig.2E.27(a) ' ' ' | .,
Let 0 be the angular position of a maximum (point P)
with respect to the central line S1S2O. The path difference jwhere is in Aand bis apositive constant. The value ofb is
between the interfering waves at P will, evidently be \such that the condition for total reflection at the face AC is\
Al = 4X COS0. •just satisfiedfor one wave and is not satisfied for the other. ,
(a) Find the value ofb. (b) Find the deviations of the beaml
'transmitted through the face AC.
(.•.•Al = S^P-S2P = CSiM+MP-S2P) = S^M) '(c)A convergent lensis usedto bring these transmitted beams',
Since |cos0|<l,
into focus. If the intensities of the upper and lower beams
|A/|<4X.. •immediately, after transmissionfrom theface AC are 41 and f
Also, for an intensity maximum at pointP .respectively, find the resultant intensity at focus. I
Al = nX; n eI Solution: (a) At face AB rays,pass without deviation.
Al = 3X,2X,X. Both the rays are incident at angle 0 at the face AC. Critical
(so that n < 4 and n ^ 4 for points other than O) angles for the two wavelengths are
. 4XIcos0|=3X,2X,,X
"26^1
sWAVEiOPTICS
+72 +2^^2 C0S(1)
••

\d = 7+ 47 + 2V(47)(7)
•'r
= 97
' 0)

\ln a modified YDSE the region between screen and slits isj
Wmersed in a liquid whose refractive index varies with time]
Fig. 2E.27 (b) 'as p.; =(5(2) - (7(4) until it reaches a steady state value]
I5/4. Agloss plate of thickness 36\im and refractive index 3(2)
sin0c =
M-i
) sinSc = —
1^2
jis introduced infront ofone ofthe slits, (a) Find the position ofi
,central maxima as afunction oftime and the time when it is\
where |ii =1.2 + 'at point O, located symmetrically on the x-oxis. (b) What isj
(4000)^ 'the speed ofthe central maxima, when -it is- atI0?
b -j
|i2 = 1.2 +
(5000)2
Since b is a constant, > 1I2; hence
T I
a '^
sinOc < sin0c or Oc <
iSjtpS
If angle of incidence 0=Qq, the ray of wavelength .' kv ^gMg :
will graze the surface AC and the ray of wavelength X2will
refract atangle less than 90°, because angle ofincidence 0is D = 1m
less tlian critical angle Qq. d = 2mm

1 1 '
Therefore, lii =
Fig. 2E.28 (a)
sin0^c
Solution: We consider a point P on the screen.
or —=1.2
0.8
+(4000)2
^ or b = 8xlO^(A)2
Optical path length, , ,, ^
(d) Let r be the ^angle of refraction of ray Q. From •• , [SiPliiquid =[FiSiP]air-
Snell's law, and [SaP- t]liquid .+ tgiass =M-i[S2P-tl'air +[^gf]air "
sinr = sine =0.8^1.r. 2o x-^
8x10^ '
Hence opticalpaA difference at P,.
= Qx 1S2P + (ft g ~It1) t]air
I' i . I' I' ' >
" ,
^'

..='0.9856 Ax =l7i,S2P +(Pg^M'j)ti-M-(SiP-,-, . , ,


= ^i(S2P-SiP) + (p^.-T|X'i)t.
Hence, • - -r-~80°20'..
Angle of deviation = r - 0 = 27° 10' For p point P.at the screen iri the' absence of liquid;
yd • • /
As 0 = sin~^(0.8) - 53°10'
(c)' The transrhitted beams from P •and; Q coritain
wavelength X2 only'as wavelength X^ is totally interrially
reflected. Thus we have two parcel beams emanating from
points P and Q, that are focussed by a convex lens.
The optical path difference between two waves,
Ax = (I'G)air - (^)glass
. = D£sinr-lidtan0 , . ,
'• Fig. 2E.28 (b)
COS0

If a liquid is filled,
COS0 , . IX/vd
= 0
•• [S2P-SiP]liquid =^ • ••
Therefore, the waves are in same phase.
or
dy ^ d/i =_ 0
Thus
D, D

(a) For central maxima. yf^2


Ji =
Ax = 0 D,

(0.5sin7tt)(2)
y =
yj =- = - (sin Ttt j mm
d

_ (4-ntD (b) The path difference at point P,


~ (lo-r)d Ax = (SS2-SSi) + CS2P-SiP)
At point 0, y = 0
^^^dcd/2)
D,
'1
Dr
^2
Thus T = 4sec. . • V

(b) Speed of central maxima, For constructive interference, _


dy 6r>t dy d^ , ' '
V =
AX = — + = nX
dt (lO-T)^d Di 2D2

6Dt _ 1x36x10
_ -6 ,(10-^)(0.5sin7Et)xlQ:^ (10"^)^!^"^'^^
Thus V = - 3 x 10 ^ m/sec 1 - >. 2x2. '
36d 6x2x10"^
(0.5sin;tt)xl0-®+ 0.25x10"® '
J 29 \-6.
= (5000xl0"^®)n = 0.5xl0'°n -
O.Sn - 0.25 . .
sin Kt =
l/n a modified YDSE the source S of wavelength 5000 A oscil- 0.5
'lates about axis of setup according to .the equation For the minimum value of t, n = 1.
\y = 0.5sin7rt, wherey is in millimetre and tin second. (a)Find
sin Tit = —
theposition of maxima as afunction of time, (b)At what time t 2
\will theintensity at P, apointexactlyinfrontofslitSj, be max- *
Kt = —
limumfor the first time ? • 6
~

or t = —sec= 0.167 sec.


6

d=1mm
30

1m •D = 2m- ^ Aglassplate ofrefractive index1,5 is coatedwith a thin layerl


of thickness t and refractive index1.8. Light of wavelength X.;
travellifig ih air is incident normally on the layer. It ^'partly-
I ' , , Fig.2E.29{a) ,' reflected at the upper and lowersurfaces, of the layer and the1
!two reflected rays interfere. Write the fcondition for, their^
Solution: (a) Let the source.be at a displacement Y •constructive interference.. If X = 648nm, obtain the leastl
and Q be the position of central maxima. At point Q the value of t for which rays interfere constructively. \
waves from and S2 arrive in same phase, thus
Solution: and
-
R2 are the two rays
considered for n = 1.0

interference. is the
result' of ' reflection' "at
Coating - \ •/'\ "=•'•8 i
denser medium; hence it n = 1.5
; Glass plate-. - r
suffers ah additional path
difference X /2and phase
Fig. 2E.2g (b
difference it. - Ray - R2 . Fig. 2E.30
originates after reflection
(SS2 + S2Q) - (SSi + SiQ) = 0 at lower surface; this reflection takes place at rarer medium.
Net path difference.
or (SS2 - SSi) + (S2Q-SiQ) = 0
271
#WAVE OPTICS

2ut + —= nX [for constructive interference] 32


2
(2n-m
2|xt = \ln Fig. 2E.32 shown, a screen is placed normal to the line
[joining the two point coherent sources Sj and 82- The
t will be minimum for n = 1. [interference pattern consists ofconcentric circles.
648
t min nm = 90nm
4\i 4x1.8

31

In a YDSE using monochromatic visible light the distance,


\hetween the plane of slits and the screen is 1.7 metre. At a
Ipoint (P) on the screen which is directly in front of the upper^
\slit, path maximum is observed. Now the screen is moved 50; Fig.2E.32

\crn closer to the plane ofslits. Point Pnow lies between third^ fa) Find the radius of the nth bright ring. ^ •
land fourth minima above the central mcmma and the]
\intensity at Pis one-fourth of the maximum intensity on thei '(b) Jf d= 0.5mm, X=5000A and D=100cm, find the;
[screen. Find the : i
\radius of the closest second bright ring. y;
^.^a) value ofn, ! i^c; Also, find the value of nfor this ring.
wavelength of light if the separation ofslits is 2 mm.
Solution: The optical path difference at P is
Solution: (a] For point Pinfront ofupper slit, y = djl. Ax = SiP-S2p = dcos0
Initially, path difference, = nX ...(1) 0^ • •
Since cos 0 = 1 for small 0,
2
Intensity at any point on the screen is
((]) being the phase difference) Therefore, Ax = d 1-5!
i -~ ^max
T
I 2 2

When the screen is displaced, y where D + d~ D


= d 1-
2D-
J — max
4
For nth maxima, Ax = n^
Hence, A= 1
cosb
.2 I - •

•y
, 2 or '1- = nX
2D-
Also as P lies between third and fourth mjriima, ,.
.11 5tc'< (j).< 77t ' ' • • --'-'-'...(3) (a) y =radius of th'e'nth bright ring
From eqns. (2) ^nd (3), we'get'
•17k 19k '
= or —
3 3 (b) At the central maxima, 0 = 0.
C4/2)d 2:i _ (nXD) 2;t ^ 2n7tD Ax = d = nX,
But ([) =
D' T" D' X D' d - 0.5
n = — = = 1000
[using eqn. (1)] or
X' " 0.5x10"
^-3

_17nfi (putting the values ofD and DO .-. for the closest second bright ring, n = 99a
6
(c) Putting values, r = 6.32 cm.
n = 2 or (38/17). . (but n is an integer)
n = 2 is the only valid answer.- • '' .1
(b) Putting .the value ofn and d in' eqn. CD, we 'get '
X, = 5.9X10"^ m • :: ' ' _ .
OPTICS

•A narrow slit S allows monochromatic light ofwavelength irwo radio transmitter each transmitting a radio offrequencyl
X= 4000A tofall on a piism ofvery small angle as shown in >12 MHz in phase with each other are placed at a distance]
fhefigure. A. screen is placed atadistance I=150cmfrom the jZ =100 mon either side of the runways on the towers os^
^ource to obtain an interference pattern. To determine the .shown in the figure. An aircraft with a ground speed ofv is!
idistance between the virtual images formed by the prism an^ ^flying towards the airport such that its velocity makes an!
[experiment is done. The prism and screen are keptfixed and al jang/e 0with the runway. The aircraft is heading directlyfor]
'convei'ging lens is moved between the prism and the screen.' \the mid-point ofthe towers. ;\
•For two positions of the lens (between the prism and thel At the aircraft's currentposition, the intensity ofthe signai
Iscreen; we get two sharp point imciges on the screen in each] 'from each tower separately would be I^. Find the intensity of]
\case. The images are-separated from each other by distance'- |t/ie combined signalfrom both towers received by the aircraft\
[4.5 mm in one case and 2 mm in the other.'Now the lens is! for heading of0 = 0 and B~n/2. \
femoved dnd inteifererice pattern is obtainedpn the screen.
Calculate: • • -
i=100m

eV-'
s o!
CO ' yf
N-
' /=150cm. , ^
I " • Fig.'2E.33 (a) i
j Fig. 2E;34(a) . . | j
l(ii) For what heading would the aircraft receive no sighaO
(0 The fringe width of thepattern on the screen. [Assuming distance ofplane much greater than I. ' !
[(iO_^ocaZ ^^J}g^_^the lens. •(Hi) If the. aircraft is heading,from- large distance directly
\towards on of the transmitter in the direction perpendicular
Solution: =4.5 mm [to T, find the position of the aircra)ifrom the nearest tower]
12 ~ ^mm '^_"^Znimum_/irst time. |
^ = 3mm = d
XD 4x10-^x1.5 Solution:
P= (i) For 0 = 0, Ac = 0
d 3x10"^ • IsinO
= 2x10-'^ = 0.2 mm' For 0 = 71/2
Ax = 100m = 4X .
• -
{X = 25m ;v = Xf}
S

. 1 = 410
(ii) Assuming • . aircraft
' l- =,150cm.
from poles.
Jll- Fig-2E.34 (b)
Fig.2E.3£(b) |
For minima
(ii)
\ /sin0 = (2n + l)-
2
V = 225-1.51;
225
=> sin8= n = 0, 1, 2, 3 W(D2+x2)
y = 90 cm o
2.5
Ciii) Ax = -Jd^ + - jc
u = 60 cm -
For minima first time Fig. 2E.34 (c)

/ 90 60/. 180 ^ 5 => ^D^ +x^ -x =%/2


X = 393.75m
WAVE OPTICS

So utlon!

V -25 50
Jn thefigure shown A, B and C are three slits each of them
dndividually producing the same intensity I^ at Pq when the 50cm
50cm

{are illuminated byparallel beam of lightofwavelength 'X\ It


lis given that BPg -AP^ = Also given that d« D. Find:
c

2d 25cm
A
01 f = 50cm
H^d-
1m
Fig. 2E.35
Fig. 2E.36 (b)
(i) X is terms ofd and^D
(ii) Resultant intensity alPp; 11
- =>v = -50
u 50

Solution: (i) 2d sine = ~ 6x10"^ xl'


2 P=
10"^
d X
2dx — = ^ - 6x 10"^ - 0.6 mm
• D 2

X =
D

(ii)' Since BPq - APq. = X (®rw(3 /Zat mirrors^/dj?m-an d of


dtghh S is^pZgced areq^dZ^distaric^'' d'from tJie^ nfirrork Find
Waves from A ahd.B'cancel nhe fiihge -^dtU ohqscreen placed at a distance bfrotn thej
Resultant intensity =>/q
Ipoihi of intersection of the mirrors.'A shield Cdoes not allow]
the light tb^pass directly on to the screen:-,The ivavelength of
lightwdve is Xandais the acuieangte between the mirrors, j

}A conv^ Zens offocal length 50eni is cut along the diameierl


jmto two identical halves AaitdB and in theprocess wlafer 0
^ofth'e lehs ihickness f mm is lost. TTiehthe'two hdNes A-ahd A
idrh'put together tofdrm a-composite lens^ Nbw in,front dfihisL , Fig. 2E.37 (a)
^composite lens - a-, source of light' erniiting wqveleh^hi —.f

jX =6000A is placed at a distance Of25 cm as shoivh in thel (bj Two pqini coherent sources are qn-a straight line d= nXj
dpaif. The distance ofdscreen perpertdiculaf to. tke line. ofthe
lfi^re.''Behihdthe kns there is^d screen at a distance 50 cm] Sources 'is pX>>. dffrom •the, nearesf sourde: Calculate' the
from it. 'Find^ the fringe, width of the inteiferehce '^pattern! ofthecen^traLnfaddma:-^ - '
obtained on the scteeh. , - - ' -I
Solution;: .(a) Tlie fringe,width is given by p =
• d
Screen Here" D = a + br
Source d = 2aa
1mm (The reflfected ray rotates by 2a; wheri'the mirror rotates
by a. ) • ^
X.Ca4--'b)'v"''^
25cm 50cm P=
2a a'
Fig. 2E:36 (a) (b) Consider any point' P .on the screen at a
distance'/ from O. Then Df='D^ +y ^'
274 OPTICS^

38
Di=D 1+4
In the Young's double slit experiment light of wavelength
= D 1 +
y
= D + Z_ 400 nm falls normal to the plane of the slits. Interference
2D- 2D pattern is observed on a large screen placed at a distance D'
[parallel to the plane of the slits. When a transparent strip of
y [thickness 4pm and refractive indexpi is placed infront ofone
Similarly, D2 = (.D + d) +
2(D + d) \of the slits then central maxima occupies the position of12th
d-y^ ^maxima prior to the introduction 'of the strip. When another
D2 "D^ = d.~ ^strip of same thickness and refractive index 2 ^ placed in
2D[D + d\
— 1 [front of other slit the central maxima comes to the position bf
P ' \5thmaxima on the same side ofthe12th maximaprior to the
introductionofthestrips. Now the lightis replaced byanother
[light of wavelength X. It is found that central maxima is,
Ijbrmed at a distance 2cm (which isfour times the newfringe
\width) from the centre of the screen. It is given that
/ D1
[separation of the slits is 1mm. Find the value of ^tid
[separation between the plate of the slits andxhe screen D. ;

•st O
Solution: when the first strip is placed in front of the
one of the slits no. of fringes that will shift^
1 Flg.2E.37(b) , ! =^4 =12 (given)' - ,
A. .•

For the point O,D2 - D^ =d = nX (given) , 12X 12x400x10^


Thus there is brightness at O oforder. Since the path V 4X10-^ :
difference decreases, the adjacent dark fringe or minima will
=> ' Hi = 2.2
be at path difference of (n -1/2) X ' >
When the second strip is placed infront of the another
^2 ~ ^1 - n- — slit, number of fringes that will shift
2
=4^1^ =5(given)
d-
2[D(D + d)] I 2 •^ ' 5 X 400 X10*
,-9 1'.
= 0.5 • .
nk 1 -
y
= nX
• '4x10-^
2D(D + d) 2n => . >^ = 1-7
y
Now the prejeht^light is replaced by another light of
2DiD + d) 2n
wavelength X &bn the position of central maximum from
- - ... j I.' I I < I II the centre ofscreten will b^ -* -, ^ -
D(D + d) = 2xl0"fm (given) ,,
•y T d I . \ .'V

Width of central maxima


2x10"^ xlxlb"^
D = = Im
-6

' = 2y = 2
•D(D +d)j2. .ir, 0.5x4x10
Since, it is given that four time of the ne,w fringe width is
equal to 2 cm
4A.2D _ ^ ^
= 2x10"
''d
2x10"^ xTxlO"^
X2 —
4x1

= 5x10 ^m = 500nm
WAVE OPTICS 275

3k>» ^ I Now, — -^d = nX


D
nXD ' _
The figure shows an Youngs double slit experiment with] or
ya =—r-^D
parallel wavefronts inclined at an arigle to theplane ofthe^
slits.^ ,' } (a) Position of the central maxima is given by
* "i
(a) Find the position of cen&al maxima. Jo (n = 0)
(b) Locate the re^on in which fiinge pattern is observed [ (b) In case of Fraunhoffere diffraction, the halfangular
(c)Firid the number offringe widths. • i width of the central maxima is given bya = —wherea = slit
(d) Ifthe central maximum is to be shifted at the origin byl width
a
'
introducing a thin film of refractive index infront of any
one of the tyo slits. What should be its thickness and where
should it be placed.. , . , \

f-.'
screen

I -i

plane wave Fig^2E.39.(c)

Fringe patteni is blDserved in the region of width, as


shown in figure.
!
Fig.'2E.39(a).-,; , 2XD '
b = = d

Solution: Let h the maxima occurs at the point P such" The region is symmetrically located about the central
that optical path difference - d$in0-.dsin(j)'= nX maxima. The ordinates of the lower and upper fringes are
Since 0 and (b are smalb-therefore" ~ ^ ; ^ b
;yi=yo/-:;.
^ ' /
b ' '•
>r.t

* - h ' ^
(c) Number of fringe widths are N = — = —
. V j « w XD
dsin4' j (d) the film should be place'd'in front bf the lower slit.
o.p.d. = dsin0 + Cp -l)t —
d
Since, central m^ima shift^ fo the origin, therefore,
dsin9
ai-T)t^ =yo'
d

or
Fig. 2E.39 (b) (p-l)D
y
sin0 ~ tan0 = — and sin 6 = (b
, - .-D
me
OPTICS

Xa) Athin glass plate ofthickness t and refractive index p is Amonochromatic light 0/X= 5000Ais incident on the slits\
,inserted between one of the slits in a Young's experiment. If •separated by a distance of SxlO-^m. The interference'^
:the intensity at the centre of the screen is I, what was the 'pattern is seen on a screen placed at a distance 1 meterfroml
intensity at the same point prior to the introduction ofthe the slit. A thin glass plate of thickness 3xl0-^m and of
sheet.
refractive index 2.5 is placed between one of the slits and',
.(b) One slit ofa Young's experiment is covered by aglass plate, screen. Find intensityat the centre ofthe screen ifthe intensity^
=1.40) and the other by another glass plane Qi 2= 1-7) of [there is Iq in absence ofplate. J
'the same thickness. The point of central maxima on the^
'screen, before the plates were introduced is now occupied by' Solution: path difference due to introduction ofthin
the third brightfringe. Find the thickness of the plates, when] glass plate
:the wavelength of the light used is 4000A. l (it-l)f

Solution: (a) Let the original intensity prior to the (l) =-^xl.5xl0^
X
=6ju
introduction of the glass plate = Iq.
The path difference introduced by the glass plate - -Jh +7^ +2^/1/2 cosOtt =41 =Iq, here
•^1 = -^2 = 1
^0 = 4/(given)
(5) phase difference = — (jx - i)f
X. • - ^' = 4/=/o
5 7C[p.-l]t I.e., the intensity remains 7q.
I = 41n COS' = 4/0 cos
2

t =~sec^
1 2 7t[p.-l]-t
Iq
4 X
In the figure shown S is a narrow slit illuminated by al
Cb) monochromatic light with wavelength \
-X =589nm. A and B are identical tubes with air lengths
,1 = 10cm each. Dis a diaphragm with two slits. After the am
in tube A was replaced with another gas, the interference
pattern on the screen was displaced by N = 20 fringes. The-,
•refractive index ofair is equal ton = 1.00002777. Determine^
the refractive index of the other gas. j

Fig. 2E.40

Optical path difference


[C52P-t) +^2f]-[C5iP-;t);fp.it] -
- ='52P-SiP + (|i2
_y-d
•^ ^,
4 •B

Fig. 2E.42
- J
D
Let Pbethepoint where optical pathdifference is zero Solution: shift in figure pattern =~t (p.^ -p^)
. y • d'
'3 = fringe width

d NX
Buty = 3(0 (where co is the fringe width) Pi4 - Pa -Pb

Given p^ = 1.00002777, iV = 20, X= 589x lO'^m,


Z= 10x10"^
3X 3 X 4000 X 10-^0
=> t = = 4jjjn
(1.7-1.4)
Pa = 1.00002777 + 0.0001178 = 1.00014557
WAVE OPTICS 277

Hence at C there will be maxima


I 43
For, minima = (2m -1) 7c/2
'Light of wavelength X = SOOOnm falls on two narrow slits,
'placed a distance d = 50x 10"'^ cm apart, at an angle (|) = 30°' or A(l) = (2m -1)
relative to the slits shown in figure. On the lower slit a, for m = - 50 and -49 we get = -lOlTt and -997i:
transparent slab of thickness 0.1mm and refractive index 3/2 respectively
is placed. The interference pattern observed on screen at a Hence the minima closest to 0 = 0 are of order m = - 49
•distance D = 2m from the slits. Then ' and -50 on both side
The phase difference at C when we remove the slab
Acj) = 10071 i.e. m = 50
Hence number of fringes passed over = 150

&•

Fig. 2E.43 (a)


•A double slit is illuminated normally with coherent light. The
[calculate interference pattern is observed on a screen. The apparatus is
'shown. '
\(a) Position of the central maxima?
\Cb) The order of minima closest to centre Cof screen? not to scale •
1(c) How many jringes will pass over C, if we remove thei
transparentshbjrom the lower slit?
coherent light
Solution: Phase difference (t)/cds in
' (|) +fcdsinS - A:i)(l - m)
, , 2k •
where

Screen

Fig.2E.44 (a)

|T7ie width of both slits in the double slit arrangement is,


'increased without altering the separation s.
Fig. 2E.43(b)
[Describe and explain the effect, if any, of this change on
Central maxima is obtained.when A(t)i= 0 '(a)The fringe width. ' ^
kdsin^-i- kdsiuQ^ kbli^ n) = 0/ ](b)fhe intensity'of the fringes. • •
. bin - i) , . . fc) A Young's double slit experimentfor red light is set up as
sin 0 = —:—r'- sin (j)
'shown.

'An interference pattern of light and dark fringes is observed'


= — => therefore 0 = 30°
2 •on the screen. . '
. J ' i

Jtrl
'i 2d, CO The red filter is now replaced by a blue filter. State and
At c,0 = 0 A(t) = A:dsin0-AJ)(n-l) = — 1)
2 •explain the change in appearance, other than change of
'Colour, of the fringes on the screen.
27ud
d
I(ii) The filter in (i) is removed. Describe.the pattern on screen.

Substituting these values, we get,


= -IOOtc = 2mjt => m = - 50

I cos^
fA^'
As
2
278 OPTICS

double slit
Y

red filter

p .
' (D.O)
; J
0 Po(D.O)

02-

S3

source of k
white light * Screen

Fig. 2E.45 (a)


Screen

Solution: The path difference S2P -OP = —sinG,


2i

S.P-OP = --sine, S3P-OP = —.sine -

single silt The relative phases of the waves from S^, S2, S3 when
^ r \ K

\
they interfere at P are • • m ; 'j.-r'A
. Flg. 2E.44,(b), . > "TcdsihG-'v' "TtdsiriG " . ^'MTtdsiriG
•Pi =• . • :? '>.v.
• M'l' f.
Solution: (a). There isno change in the'fringe width. Since we calpilate relative,phases oply we can add ,
• ' G^)'Increases -but overlapping "^increases and
interference pattern darity decreases.' P(D.y) ^ ,
' ' ' 'T ' XD\ Po(D.O)
(c) (i) Fringes width decreases B=— and no.of
.A -
fringes T.
(ii) Only central maxima: is. of white colour. Other
parts are mixture of different colours maximum and
minimas. Fig. 2E.45 (b)

. TtdsifiG X ^
^ = +——to .(})i, (1)2,(1)3 to get
rt 'J 27CsinG '"47rsin9
\Three sources Si,S2 and S3 emitting light of the same = 0, ^2 F —^ , <1>3 = +-
amplitude and wavelength X and in the same-phase are kept
In order t;o. add the vibrations we use the phasor
jos shown; the source S^^atVQ,— ,S'2 dt-^Oj-^jand S3 at method, net am'plinide at P, _
(. V 2^
1^0,--^dj.Alarge screen is placed at x=D,perpendicular to Ap = A + 2Acos ^2jtdsin0jI=A(1 +cos <1))
[the x-axis. Assuming d«D,find the intensity at a pointl
'P(.D, y) on the screen, if y f:< D., . • , . 1 ; r'i ->-1' \ > •.'•iO.i

V L y.' ' 'I''- "v

r ri-> .i

. r'.T'r. .
- O" " •. -
WAVE OPTICS 279

If J« and Iq « a^, then


I = 7o[l+2cos(fcdsin0)]^
For maxima cos (iCdsin 0) = 1
or kd sin 0 = Inn
2nn _ nX
sin0 -
lid'Y
y
Since tan0 ~ sin0 ~ , therefore,
D
nkD
Jn =
•A3 a

FJg,2E.45(c)
Fringe width co = —
27udsin0 d
where (|) =
^ 3x10' = 3m

J =(1 + 2cos(1))2/o 10'

47

'Three radio sources of equal power, and same phase^are' Tn the Young's experiment using monochromatic light, the
^positioned collinearly as shown inthefigure. Thefrequency of 'fringe pattern shifts by a certain distance on the screen when'
leac/i source is 10®H2 and the distance between two adjacent', la micasheet of \l = 1.6 and thickness t = 1.964 micron is'
'source is d = 1km. The distance between two adjacent 'introduced in the path of one of the slits. The mica sheet is
•now removed and the distance between the slits and screen is
•maxima formed on a screen hear 0 at a distance D is'lOkm..
doubled. It is found that the distance between successive-
\(a) What is the radiation wavelength? maxima (or minima) is the same as the observed fringe shiftl
What is the distance D?_ upon the introduction of mica. Calculate X.
\(c) The central-source stops radiating^ and the two other
•sources are moved (symmetrically) to new positions in such a Solution: shift produced due to introduction of mica
Iway that the maximum points are unchanged. What is the sheet
{new distance between the two sources ?
< Screen 1 - ! d
Fringe width when screen is displaced
p_X(2D) ,
, (p - l-Xtp _ ;ic (2D)
. ,d, ; , d ...
or- ^ ' 2
=(1.6-l)xl.964xl0-^ ^
• 2

Fig. 2E.46

Solution: (a) Equation of the three waves arriving at A cylindrical shell of radius I'm has two slits and S2
P are
'separated by a distance of 1mm. Light having a wavelength
- asinCfcc- (j)) X = 4000A is incident on the double slit and produces a'
y2 = asinkx •fringe pattern within the shelL Assume that the intensity does
•not very substantially as one moves from 0 to P.
y^ = dsin(kx + ^) where (j) = kdsinQ
The resultant amplitude of the three waves is (a) Find the fringe width of the pattern at the point 0.
I I
(b) How does the fringe width vary as one moves to an,
y=yi +72+^3
.angular position P, such that ZPCO = 60® ?
A = a[l + 2cos(/cdsin0)]
280 OPTICS?

'(c) Find the total number offringes in the angular range of ..©d 49
60® (region OP). '

'A Young's double slit experimental setup shown in thefiguret


'Consists of two slits Si and S2 separated by a small distance;
I'd' and a screen of mass 'm' at distance D from the plane]
[contairiing the slits. A thin glass plate ofthickness 'K' and}
^refractive index' n' isplaced infront of one of the slits. }
fa) If light of intensity I is emitted by the slits and the glass
plate allows only 64%of the energy to pass through, find the\
<ratio of the intensity of the central maxima and the first
Fig. 2E.48 (a)
minima on the screen. J .
\(b) If a force F = MgtjtQ is applied on the screen in the
Solution: The distance S1S2 = d = lmm [direction as shown in the figure, draw the graph of the^
The path difference, [acceleration of central maxima with respect to time.
SjP - Si? - dsinO, 20 - ^
1
For the n^ majdmum at P, . • 't

dsin0„=nX or r dsin-—-= nX ...(1) D M


I' i
• ' !
i
t

' • - - 2-

At the point P, the angular fringe width 6(j) is given by ! S, ^


V j . .j
•!

, dsin^^-dsin^-X,
2 • -2
. ^ Rough Surface
Fig. 2E.49 (a)
2X '
or - cos—5(!) = X, 6(|) = ...(2)
2 '2
dcosi—
. - >
Solution: (a) =1,12 = 0.641 -
2-
^max=-fl+^,2+W^COS0°
At the pointP, the angular fringe width 5(|) is given by.: = 1 + 0.641 +2?Vo^
dsin^ii+1 -dsin—= X or -cos—5(|) =X = 1.641 + 161 = 3.24/
2 -

or 6({) = ...(3) = 1 + 0.64/ - 2rV^ = 0.04/


dcosI — ^max _ '^-^4 _ g-j^ '•
/min • 0.04 ' -;u
(b) Height of centtal maxima is given by . ,.

' ' dt • d' I • ~ dt - 'dt^' ' ' * ' d - df I


' " f - I I t I f p r . • I t I
• ' '

Now —^ = 0, for t < to


.Fig. 2E.48 (b)

2XR
At the point 0,'the linear width Wq

2XR
and at P, IVp ...(4)
J<t>
dcos

j'.i,f_! I • . '
Putting the values, Wq = 0.8mm, W„ - 0.92mm
I" t r ' . •'
Number of fringes = 1250 [from eqn. (1)]
Fig. 2E.49 b)
IWAVElOPTICS 28f

d\D) t Now resultant wave coming out of S3 has intensity4fo


= g , fort >>to
dt^ :ro-r and wa:ves coming out of.S4 have intensityIq.
(a) Phase difference at S3 = 27C
d^y
= Q for t < to Phase difference at S4 = 2k/3.
dt^
These phase differences are relative to the light incident
on slits Si and S2.
dt^ d
Now S3 and S4 are secondary sources of light.
_ iCOi-1) g 47C
tanG = Phase difference at 0' = —, equal to initial phase

difference between the light reaching at O' = 271 - — = —.


ma 50 3 ,3
Let intensity at O'be
In the arrangemefit shown in Fig. 2E.50, light of wavelength
6000A is incident on slits Sj and S2• Slits S3 and have
r =Jo+4/0 + cos^
CO
CO
been opened such that S3 is the position offirst maximum
above the central maximum and S4 is the closest position, =5/0 +4/0 C0S^7t +^j.= 3/0
where intensity is same as that of the light used, below the (b) Forbrightestfringe, phase difference = 2'n7t, n = 0,
central maximum. The point O. is equidistantfrom Sj and S^
andO'isequidistantjromS3 andS4.The intensity qfincidentl ±i,±2,... ^ , , .
light is Iq. - - • i Let I" be the'intensity of brightest fringe'.
f" =Io +4/0 +2-^/7^74^0 "(where cosi^ =1)
' . ®i I So - =9Jo-,
\ ' '
3 mm
!o' .51
o

1m' . 1m
Thfee-mdnochromatic,'coherent (same phase) sources SjfSz l
and S3 emitting liglit of the same amplitude A ar^,
Fig. 2E.50
wavelength Xare kept as shown: the source Sj Ssj
iFind: .
^ d\ . f 3^ i
\(a) the intensity at O' (on the screen) and at
^3 ^screen is placed at
j(b) the intensityjifth^ brightestfringe. ^x = D, perpendicular to the X:axis.'Assuming d« Dyfind :
Solution: From the ^ven condition, -
v<: (i) the intensity at a 'poihtP{D,y) on thescreen^ ify «D, ^4
DX . -1X 6 X10"^ - 'l •-'. -4 •. \(ii) thecondition that the intensity at the pointRis a maxi-]
OS,
'3 = — = ^ = 2x10 ^ m [mum/minimum,^' _ 1 j
1')' d. ^ , 3x10,-^
- I f
Letlight reaching from S^ and" S2 'td^S:^' has phase Screen

difference ({»and intensity of incident light iSrf 0* y


P .

P(D.y)
Resultant intensity at S4,/ =4Iq cos^ ^ k Si: b Po(D. 0)
2
d 52 d « D
' S2. 53 y « D
As ^=-^0. ; d, k
I r\ 0^ (b • Screen .'
Hence = COS — or cos— = ^;= cos60^
2 2/ (a)
4/0 2
Flg.2E.51 •
A -2^1
or J

. 27C DX Solution: (i) The path difference,


For
.
I'j.,
S2P-0P=
•.2..
"sine,.
• •K2 -
Therefore, S3S4 = OS3 +054 = = - x 10^ m
3d 3 SiP-OP = --sine,
^ 2
mi OPTICS
Q J

S3P-0P = ~sine Solution: (a) At a pointP(0,y), the path difference is


^ 2 Ax = ABsinG = 3X sin0
The relative phases of the waves from Sj, 52,^3 when For minima, the permissible values are (as -1 < sin0 < 1)
they interfere at P are X 3X 5X
jidsinG , TidsinS , 3jtdsin0 3X sin0 = —,—,—
<^3=^^ 2 2 2

or sin0 = —, —
As we are concerned with relative phases only, we can 6 2 6
add (|)o =+ ^to to simplify the calculations From geometry of figure,
^ 27cdsin0 4'rtdsin0 y
smG =
(l>i =0. f2 = ^ >fs =+ Vy'+^
We use the phasor method to get resultant amplitude. Hence, y =+^,±-^,±
= ±
Resultant amplitude at P, V5'"V3'"Vn
( 27tdsin0^ The number of minima on y-axis is six.
Ap = A + 2AC0S
I X (b) For point Q (x, 0) on x-axis.
= A (1 + 2 COS (j)) Path difference. Ax = AB cos0 = 3X cos0
27tdsin0
where (1)= . . For maxima, 3X cos0 = 0, X, 2X, 3X

COS0 - 0, —, 1
Intensity at 3 3
P,lp = Cl + 2cos(j))^7o -W Acceptable values of 0 are
(ii) For a maximum/minimum at 0 = cos"^ —, cos"^ —, 0
P, -3 3 ,

-^=0 From geometry of the figure, we ,


d(t
d(t) have A . 0:
(D + x) ; (x,0)
1 + 2cos(l) = 0 (minimum) tan0 =
3X
cos p = — Fig. 2E.51 c)
Fig. 2E.52 (c)
2
x=0,±3^[2X, +"^^X
, 2tc 27idsin0 _ , 27c
p = 2n7c ± — or 2n7i ± —
3 X 3
53
or d sin 0 =
lY
n ± — X for, a minimum
3}
A lens offocal length f = 40 cmis cut along the diameter into'
and dsinG = nX n + —'iX-for a maximum two identical halves.^ In' this process, a layer of the lensl
2 't =Ifnni in thickness is lost, then tKe'kalves are put together'^
to form a composite lens. In between the focal place and the\
52
•composite lens, a narrow slit'is-placed, near thefocal plane.\
The slit is emitting monochromatic light with wavelength]
Two coherent sources A and B emitting light of wavelength X X= 0.6pm. Behind the lens a screen is located at a distance'^
are placed at positions (-D, 0) and (-D, 3X) respectively, L = 0.5 mfrom it. Find the fringe width and number ofvisible-
D » X. Find: maxima.

(a) the number of maxima on y-axis and their positions, ,


(b) the number of maxima on x-axis and their positions.
I Ay VA
iB(-D.3X) [ B{-D.3X) ^P(O.y)
:a(-d, 0) o
A(-D,0)O
1 (a) (b)
Fig. 2E.52 Fig. 2E.53 (a)
IWAVEOPTICS 283

Solution: From lens equation, we have From triangle QRB, we have


V - (v lines on the left side) a + 9O°+0" + 9O°-0' = 180°
f-u
=> 0" = 0'-a.

--1 = t
u
Since angle a is small, 0',0",8'" are also small.
_u
L/-"J na = n'0' and n'Q"=e'".
Uf
D —L + V —L "h Solving 0', 0" and 0'" in terms of ex, 0' = —a,
f-u ti'
Fringe width, Q// n n-n' ,
0 =—a-a = a and
n n'
uf
L +
XD f~u 0"' - n'~—= (n - n')a.
0) = n'

f-u 5=^a-a +(n-n')a-( —a-a | =(n-n'Oa


n W J -
Uf-u) Fig. 2E.53 b)
or CO = — / + If the source is at a distance a, then
t
d (distance between virtual sources)
as u-> f, ¥
CO = = 2a5 = 2a(n - n')ct.
D = a + b,where b = distance of screen from the system.
From figure, ^ =1!2l ^ y =[l2Ln- f) Xja + b) _Xa +b/'2)'
L-f f
2a(n-n')a ' • 2(n-n')a' '
Distance of the point upto which interference occurs,
For plane wave a ' • >' • •
' yo=
! —(I-/)
ill,. t XL '
+- = — X 6000x10"^°
/ 2 2/ P=
2(n n')a 2(1.52-1.50) x3x^
tLt If
Number of \asible maxima = 180
2fxXf 2Xf'' = 0.2864mm.

54 55 L

^Aplane lightwave ofwavelength X- = 6000 Afalls normally on ^^4 radio telescope sited on the edge of a cliffoverlooking the'
sea operates on a wavelength of 100 m. A radio star rises]
fhe base of a biprism made of glass(n = 1.52) and refracting
pbove the horizon and is tracked by the telescope. The first'
'angle a = 3°. Behind the biprism (see Fig. 2E.54) there is a
minimum of the received signal occurs, when the star is 30°'-
'plane-parallefplate with the space filled up with J:}enzene above the horizontalsExplain why.the minimum occurs and'
•Cn' =.V1.50) . Find fthe width pf a fringe on.a screen placed be-
i-i ^ V- 'V -• •• I ,determine the height of the cliff, assuming that radio waves-
'hind this system.. , . ,, , ., , . , . . ^ ' 'suffer a phase change ofK on reflection at the wptersurface'
(see Fig. 2E.55). '• .t . .
I ' •
i Solution: The radio telescope
is receiving signals direct from the
star (ray XBC) and also by reflection
Fig.2E.54(a) i
' I '-n.*. <m 'm. ^ from the water surface, ,(FAC)^w;hich,
A • • O
is acting like a plane mirror. . In this
Fig.2E.55.
Solution: We will first determine r" case the source of radiation is far' - - -
the deviation produced by the system enough away to be considered at infinity and. the rays are
p^
From the figure, we see that descending on the earth in a'parallel Ijeam. When the path
R
nsina = n'sinS', n \p difference between rays YAC ahdlXBG./see-figure) is such
n'sine" = l-sin0'".
that they reach the telescope exactly out of phase,
Fig. 2E.54 (b) destructive interference occurs and the signal at C will be a
Now, minimum.
deviation of ray 8 = (0' - a) + (6'" - 0")
284 OPTICS

Consider the diagram. If AB is drawn perpendicular to


XC, it will also be perpendicular to YA, and ylB is a plane
wavefront of the incoming beam. Thus the disturbances
A and B have the same phase at all times. A phase change of
Koccurs at A and this is equivalent to an increase in path of
X/2. Thus the path difference between the two rays is Beam splitter
5 = AC + (X./2)-BC. But by the laws of reflection,
ZYAZ = ZCAO = 30°, and therefore ZBAC = 180°-90°-30°
Source (o)(r)
-30°= 30°.

5 = AC + --AC sin30°
2
Film
= - + AC (1 - sin 30°)
2 (a) (b)
Fig. 2.33
Since sin 30° = iL
AC
Wedge Interference
y
AC =
sin 30°
An optical wedge consists of two surfaces inclined at a
very small angle to each other, such as would be produced
and 5 = -"+ a-siri3Q°) = - + y. by placing a piece of paper between two microscope slides
' 2 • sin30° •' ' 2
(see Fig. 2.34); angles of about 1 mradian (0.05°) are
In order for minima to occur, the path difference 5 must tjrpical. ' "
equal an odd number of half wavelengths, Paper'
Wedge
or 5 = (2n + l) —, n = 0,l,2,...
, , . . 2

Then (2n +i)- =- +y. ' ' " Microscope


2 2
< slides
If n = 0, 6 = X /2, y = 0, and this corresponds to the
centre of the interference pattern being a minimum. Since Fig. 2.34. ^ ,
this is the first minimum the telescope receives, C must be at
the first minimum above 0 for the situation shown in the The wedge surfaces may be.considered to be practically
diagram. GWere it a minimum other than the first, a previous parallel to one another because the wedge angle is very
minimum must have occurred for an angle lower than 30°. small. With an air wedge, one phase change of n radians
But we are told that this doesn't ,happen.) Therefore, n = 1 occurs on reflection. This means that, the interference
and ,0 'must be 3X/2. . ... conditions are identical to those for the parallel-sided glass
' OT, 'V " ' " ' block in air. However light is. incident normafiy on ,the
• ' Hence,' • • —=—hy. and'-'.y -X. = lOOm.' ' • wedge. This reduces the interference conditions to
2d = (m + l/2)X for a bright fringe .
... FRINGES OF EQUAL THICKNESS
2d=mX •>for a-dark fringe
Soap bubbles and oil films do not have' uniform
thickness. The thickness of the film at any given point because a'= O,'0= 0 and'COS0 = 1.' ...
determines whether the reflected light has a maximum or a Fringe Separation with ah Air Wedge
minimum intensity. When white light is used, each Fig. 2.35 shows two neighbouring dark fringes of order
wavelength has its own fringe pattern. At a given point on m and m + 1 formed at distances and from 00'. Our
the film, one wavelen^h "may be enhanced and/or another objective is to express (x^^j- —x„,) in terms of d^, and
wavelength suppressed. This is the source of the colours in the wedge angle a.
soap bubbles and oil films-on the road.
For small angles, we can •write,
A wedge-shaped film of air may be produced by placing
a sheet of paper or a hair between the ends of two glass = dja
plates, as in Fig. 2.33. With flat plates, one sees a series of m+1 ^m+l
X

bright and dark bands, each characteristic of a particular Then (^m+l - = (.drn^l "
thickness.
= XI2a
WAVtOPTICS 285

Top '
surface
I(a) White light is incident normally on a lens (n = l .52) that'
is coatedwith a film ofMgF2(.n = L38) .For'what minimum',
thickness of thefilm willyellow light of wavelength 550 nm.
(in air) be missing in the reflected light ? ^
^(b) At what minimum thickness (other than zero) is there^
Bottom [constructive interference in the reflected light ? '
surface
(it)

MgFj
Glass:
Fig. 2.35
Fig.2E.57
This result also applies to bright fringes: it can be
obtained directly. The fringe separation when the wedge Solution: (a) since the index for glass is higher than
medium-is not air,X/2n^a. that of the film, there is a phase change for the reflection at
the film-glass interface, as shown in Fig. 2E.57. There is no
net phase difference introduced by the reflections. Thus, in
this case the condition for destructive interference in the
^Awedge-shapedfilm ofair isproduced byplacing afinewire of, reflected light is ,...
'diameter D between the ends of twoflat glass plates of length' X
2t =•
|L =20cm, asinj^ig. 2E.56. When the airfilm is illuminated' m +

]with lightofwavelength X = 550 nm,therearel2 darkfringes, Notice that this differs from the expression given in the
Iper centimetre. FindD. ' previous section. The minimum thickness occurs when
m = 0; thus
-7
5.5x10
= 99.6nm
4n C4)C1.38)

Fig. 2E.56 Both reflected rays suffer a phase inversion. In order to


produce destructive interference in the reflected light, the
minimum thickness of the film is /4 (so that the path
'Solution: As indicated in Fig. 2E.56, onlyone of the difference is one-half wavelength in the film).
reflected rays suffers a phase inversion. At the thin end of
Although the condition for destructive interference
the wedge, where the thickness is less than X/4, the two rays
interfere destru'ctively. This region is dark in the reflected applies" only to one wavelength, the reflection for bther
wavelengths is also reduced. The combination of the red and
light. The condition for destructive interference in the
reflected light lis ' i, ' •-
violet reflected light gives such a coated lens a purple hue.
(b) 'In "this .'case . the.'.condition for 'constructive
• '.2t,=^'mX. m = 0,1,2,... - ,•
interference-is ^ ^
The change in thickness between-adjacent dark.fringes
is At = X/2. The. horizontal spacing between fringes -

d = l/12cm =8.3xl0~^m. From figure we see that n

D/L = At/d, so The minimum thickness is t = X/2n = 199nm.


XL (5.5x10"^) (0.2) NEWTON'S RINGS
-D =
2d 16.6x10"^ when a lens with a large radius of curvature is placed on
a flat plate as in [Fig. 2.36 (a)]; a thin film of air is formed.
Thus D = 6.6xlO~^m
When the -film is illuminated , vyith monochromatic light.
286 OPTICS^

circular fringes, called Newton's rings, can be seen [Fig. BxampleJ 58


2.36 (b)]. An important feature of Newton's rings is the dark

•In an experiment on Newton's rings the light has a]


^[Wavelength of 600 nm. The tens has a refractive index of i.5j.
land a radius of curvature of2.5 m. Find the radius of the 5th\
Ibrightfringe. . j;
Solution: if R is the radius of
Fig. 2.36 curvature of the lens, then from Fig. r/ R-t •
2E.58, we see that =R^'~(R-t)^, ( r
central-Spot. Jt implied that the light wave suffers a phase where r is the radius of a fringe and t is 1
inversion on reflection at a medium with a higher refractive the thickness of the film. Since t is very
index. If we place ,oil of sassafras between a lens of crown small, wemay drop terms int ^to obtain Fig.2E.58 ' J
glass and a plate of flint glass, the refractive,index of the oil r^^2Rt .,...(1) .
In order to find r, we must first find f. The condition for a;
bright fringe is • '
1
microscope ,
2t-= m + — ...(2)
2J
We note that n = 1 for the air film (the index for.the
glass is irrelevant) and that m = 4 for the fifth bright fringe.
Beam Thus, from eqn. (2), ' '' •
splitter
Collimating
(4.5) (6x10-0
Light > ^ t = = 1.35xl0-®m
lens
source .

Substituting this into egn.'(l), we,find •


7 -Lens r = V2Rt = 2.6x IQ-^m.
.Optical flat' ;

. • 59

A thin lens of long focal length is supported horizontally


short distance above the flat polished end ofasteel cylinder.i
The cylinder is 5 cm high and its lower end is rigidly held.
' .1 ^^Newton's rings are produced between the lens and the upper:,
\end^ of the '<^lihder, using normally incident liglit^of
\wavelength 6000 A , and viewed from above, by means of a
I : \ ^-.1 R-. Imicrpscope. When the temperature of the cylinder is rafss^25'
• . ' R _ t ' '
,1,1 I i l,\-. i:'1"i • ) T; r..-;:; / IC deg, 50'rings niove past thecfoss-wires.Qf.the microscope.,
^WTiat is the coejficient of'Unecir expansion of steel:
Newtons ross Hairs
Rings
Lens

Cylinder Air wedge


(d)
Fig.2.36„
— Cylinder

is between the values for these "two' glasses. Since both Microscope
reflections occurat ali^edium;^_ith. a higher refractive index, Cross Hairs
they should both suffer a phas&inversion and therefore be in TopView Side View
phase. The ^cenfral spQt became bright—and undoubtedly
gave Young much safisfactioh! '
Fig. 2E.59
OPTICS 287

Solution: initially, the cylinder has a length d and a


gap of length to exists between the top of the cylinder and
the bottomsurface of the lensin the portion of the air wedge
viewed in the microscope at the same position as the
cross-wires. (Technically, to is the distance of the lens from
the cylinder as measured along axis AA' in Fig. 2E.59 (b).
However, since the lens is thin, to is the distance of any point
on the bottom surface of the lens from the top of the
cylinder). Let us now see how interference fringes are
produced. Fig. 2E.60 a

Light is incident upon the lens from above. Light is


transmitted through both lens surfaces with no phase <The lens receives monochromatic light (X = 5000A)from a
change. However, the light reflected from the bottom lens- point source at a distance of a = 5cm from it At what
surface and the top of the cylinder are 180®(orX/2) out of 'distance should a screen befixed on the opposite side of the
phase. This phase change is due to the reflection from the [lens to observe three interference bands on it ?
cylinder. In addition, the wave reflected from the cylinder What is the maximum possible number of interference bands
travels a distance 2to greater than the ray reflected from the that can be observed in this installation ? _
lens in traversing the air gap. If this distance is equal to an
odd-number of half-wavelengths (odd because the waves are Solution: The path of the rays in the system is
already X/2 out of phase due to reflection), constructive illustrated in Fig. 2E.60 (b). Here and 52 are the images
interference will occur. Hence of source 5 in the halves of the lens. Obviously, ' '
Constructive interference
.b =
fa '•
2tQ —(2n +1)—r (a-f)
(n = 0,l,2,...)
- . 2

or 2tn =. n + — (n = 0,1,2,...)
2

Hence, for a bright fringe to appear at the cross-wires,


2to = (n -i- 1/2)X, where n is an unknown integer.
After one heats the cylinder through a temperature
difference T, the length of the cylinder is d(l -t- aT), where a
is the.'coefficierit of liriear-expansion of steel," and tlie'gap
will haVe been reduced to t,'where tg-t = daT: ' Fig. 2E.60 (b)

'If'a bright fringe is again seen at ffie" position'of the The distance I between 5^ and 52 can be found from the
cross-wires, then, similarly, 2t_= (m-l-l/2)X, where,rri is ah similarity of triangles SAB and 55^52 :
integer (m ^ n)' ' ' •••••• > ' '
l_ ad
• ' 2(tQ^^tj^'2daT=in^'m)X-
•During .the heatiilg ^roces^V brighVfringes niusf' ' ~

The distance between adjacent interference bands on


have fiassedWer the cross-'W-ires'. Thtis- ' ''' ' ''^ '
-s
the screen is X-','
(n-m)X 50x6x10
a =
2dT 2x5x25 Ah = I
^ [oalDf -af)
ad
= 1.2x10"^ per °C. ' The number of bands on the screen is

Ah

;A central portion with a width of d -.0.5 mmis cut out of aJ where L = — is the dimension of the portion of the
b ' • . ' . •
[convergent lens having afocal lerigtH off - 10cm, asshown^ screen on which'interference bands'are observed. Therefore,
\in Fig. 2ES0(a). Both halves are tightly fitted against'each
NabfX
lother. . D = = 15cm
(adl + abNX ~ bfNX)
The maximum possible number of bands can be
determined from the condition DIFFRACTION
The light rays bend out of their straight line paths.-This
adl +NabX - bjm = 0 (hereD ^ eo). bending of light and other waves into the region which
Hence, "'
N max
= = 5
would be the shadow ifrectilinear propagation prevailed, is
ibfX - abX) called diffraction. ' .i
The number of bands is finite, since the distance Diffraction patterns are usually classified into -two
between them increases as the screen is moved away, and categories depending on where the source and screeii are
dimensions of the portion of the screen on which the placed. In Fraimhofer difiEraction parallel rays (i.e.,
interference pattern appears to grow. plane waves) are used. These tj^es- of waves can be
obtained by using lenses or a distant source and a distant,
observation point. When parallel rays are not used,^the
source andobservation point areat finite distance relative to
iFind the distance between the' neighbouring bands of an .slit, this diffraction is-called FresnePs dl^action.
.'Mejference pattern piydutxd by.adem-with'a radius ofR^l Figure shows parallel rays of monochromatic light.that
tcnij describeddn Sample 40, if this dis'tdnci does not depend pass through a narrow slit. The rays-that pass straight
ibn the position of the'screen. . through are all iii phase; so there will be acentral bright spot
'A^ what position of the screen will the number ofinteiference on the sCTeen. In Fig;2.37 (b) rays make an angle 0such that
\bands be maximum ? . ' •
a ray.from' the top of the slit travels exactly one wavelength
The sdurte'serids monochromatic l^hi with awavelength of more than the ray from" the bottom edge and a ray passing
X= 50'OOA' '•

.,Y,(a).e.=,0 (b)sine = ^
Bright Dark
Fig. 2E.61

••Solutipn; The distance^'between -the •interference


bmds wiU not depend'on the, position of the screen drily if"
^the source is'arrange'd,ih the focal-plane of"the lens. This
directly follows from the expression

. wad

• •When a='f,AW^%W^cmyac•ah3^>yallih ofD. jc)sine=.-|^ • -fd) sine =^


'Bnght." . '.'Dark ^
For this case the path of the rays is shown in Fig. 2E 61 Fig. 2:37
Aglance at tiiis iUustration will show that the number of
interference bands;.ydlbije m is in', .through,cenUre travel one-half wavelength iriore than
position AB: The di^c?'^m^&e screeii to'tiife'lais can be" , -the rays-frorii the bottoiri- edge.. Destructive-interference^will
found froni trianglepi^; remVnibering that tiie-angle a =— take place,between;these two rays; Similarly-a ray^slightly
above the bottbiri^edge-will cancel- a ray that is the same
and AB =R: ' \ ^ distance above the central one: Similarly each ray passing
-through- the'-Iower .half of the slit- will cancel -the
corresporidmg ray passing.through the-upper half..All the
rays destructively interfere in^ pairs. So no light will reach
the screen at this angle.
WAVE:OPTICS
289

Hence sin0 = — [first minimum]


a

In Fig. 2.37 (c) the observation angle 0 is chosen so that


the ray from the top ofthe slit (ray C) travels a distance 3X/2 Show that the intensity at any angle 0 is given by
farther than the bottom ray (ray A). Once again the rays at r . I^sinp/2y , _
one-third distance from the bottom will cancel in pairs with 0 p.. , where the angle p is the phase difference^
V )
those in the middle third. But light from the top third ofthe
slit will still reach the screen; so there will be a bright spot between the waves from the top and bottom edges ofthe slit.',
centred near sin0-3A, /2a. Intensity at this maxima is less Hence derive the condition for intensity minima and maxima'
an ^gle 0. ;
as compared to that at 0 = 0°.
In Fig. 2.37 (d) the rays from the region of the slits Solution: A plane wave at any time t and phase 0
between A and B are half wavelength ahead of the can be written as
corresponding rays from the region between B and C. These
y = A sin (cot - (|))
two regions of the slits therefore cancel each other's effect.
Similarly the region between C and D cancels the region We split the slit ofwidth d into a large number ofvery
between Dand E. Hence at this angle, there will again be a narrow differential slits each of the same width dS. The
minimum of zero intensity in the diffraction pattern. In amplitude of wave from such an element would be
general the intensity minima occur at I—JdS; the amplitude will be proportional to the width of
dsin0 = mX, m = l,2,3,... [minima]
the differential slit.
• The intensity pattern consists of a bright central
maxima flanked by secondary minima. The intensity
of each succeedingsecondary maxima decreases with
distance from centre.
d sin 0
• As m = 0 corresponds to the centre of the intensity
pattern, of the" middle of the central maximum, this
means that the width of the central maxima is twice
that of secondary maxima.

1 , 1
I iX-
i /
\

^ /
/
Fig. 2E.62
1 —r~
-sin 0I The wave disturbance from the bottom most portion of
-3X ^2>. -X 0 X 2\
.a a a a a the slit (coming along ray A) will be represented as

B' ~ 1 'C 1 == IA^jsinwt ...(2)

Fig. 2.38 A wave disturbance along the ray B will be a distance


S sin0 behind the wave which comes from A.
• The secondary maxima are approximately midway
between theiradjacent minima. Thus the angles that The phase difference = (S sin 0) ...(3)
locate the secondary maxima are given by
_ mX-f-(m-1-1)X Therefore the wave disturbance along ray B will have
[dsm.bj maxima = T the form

=±1 "1+2 dy =IA^ Isin[mt -


27cdsin0
= A — sm cot - ...(4)
U J
1290
This is an arbitrary ray; so it represents a general ray Solution: (a) The angles 0 which locate secondary
emanating from any point of the slit. maxima are given by
The resultant disturbance from the slit at an angle 0 can dsin0 =±1 m+i
2
be obtained by integrating dy for all portions of the slits. The
resultant disturbance is therefore Values of P that correspond to secondary maxima are
A . 2kS sinB i
...(5)
„ TtdsinB .( 1
y — sin cot 105 7t
•'Jo0 d^ X

After carrying out integration over 5 keeping t and 0 Substituting p in the expression for intensity, we get
constant, 5 varies from 0 to width of slit d. j ^ sin^[±(m -i-1/2) 7t]
AX /d 27cdsin0
...(6) ° [±Cm+ 1/2)71]^
y = cos cot - cos cot-
271 sin 0 X
If we use the approximation that secondary maxima
^ . A+B . A-B
As cosA - cosB = -2sm sm occur for integral values of m, then we have
sin^[±(m+1/2) jt]-1 for all m. Thus
it then becomes
1 = where m = 1,2,.,.
AX /d TtdsinO . 7cdsin0
[Cm + l/2)7t]'
-y = sm—^ sm cot - ...C7)
7tsin0 X
Cb) For m = 1, maxima
We are interested in the intensity of the wave, which is
proportional to the square of amplitude. = 0.0457o
[(l + l/2)jt]2
From eqn. (7) the amplitude of the wave is
For m = 2, maxima
AX /d Ttd sin0
sm
7csin0 h = = 0.0161
[(2+1/2)71] 2
(.X Id) 2( 7cdsin0
sm
For m = 3, maxima
sin0
T

1. = = 0.0083Io
sin(p/2) [(3 + 1/2)7c)]'
or 1 = 1, ...(8)
CP/2)
64
27tdsin0
where ...C9)
X
Lightfrom a sodium lamp X = 590 nmis diffracted bya slit of
According to eqn. (8), intensity minima occurs where 'width d= 0.30 mm. The distance from the slit to the screen is]
sm
7tdsin0^
= 0
D = 0.87m. Find the width ofjhe central maxinmrn.^ [
)
Solution: The angular positions of m = 1 minima are
which implies that given by
7cd sin0
—7t,27t, StC, . •., TtlTZ ...(10)

or dsin0 = mX [condition for minima] ...(11)


• Note that m cannot be zero. Intensity for 0 ^ 0 can be
evaluated by taking limit, as 0 —> 0, p ^ 0 and
II
sin CP/2)
Lim- 1
D » d
MO (P/2) Plane
monochromatic

So J = Iq, the maximum at the centre of the pattern. waves from


distant source

63 Fig. 2E.64 S

sin6i = ± —
(a) Derive an expression for the intensity of the secondary d
maxima in terms of I q. (b) Use your answerfrom part (a) to y
From figure, sin0 =
find the relative intensities for intensity I^,12,13 at
m = l, m = 2 ,m = 3 respectively.
1 WAVE OPTICS
291
. If Ay is the distance between the two m=1minima, the
values ofy that correspond to m=1minima are y =+1Ay Hence I = 41
f sinp/2
cos"
1 (P/2)
-Ay where p= 27ta sin 0
Therefore 2 A
Vci/ZAy)- + D- The factor[sin2p/2]/p2 depends on the slit width a and
Ay =
2DX IS usually called diffraction factor. The factor cos^(j)/2
depends on slit separation and is usually called the
If d » , we can use the approximation interferencefactor. The intensity of the interference peaks is
modulated by the envelope of the single slit diffraction
V?TF-d pattern as shown in the figure.
Thus 2(0.873(590) _ 3.4 mm The first minimum in the diffraction pattern occurs at
d (0.30) sine = X /a
Since d = 6a, dsine = 6a( —I= 6X
Two slits each ofwidth aand separated by distance d=6a, are As shown in Young's double slit experiment, the
illuminated by light ofwavelength 1in aYoung's double slit ex interference maxima occur for
periment. How many interference fringes appear under the
central diffraction peak ? dsin0 = mX, m = 0,l,2,...
Thus the diffraction minimum (dsine = 6X) coincides
Solution; while discussing Young's double slit with m= 6in the interference pattern; hence m= 6 is not
experiment, we had assumed that the intensity distribution visible. Thus the central diffraction peak encloses the central
due to single slit is interference maxima and five other maxima on either side, a
uniform. This total of 11. The 6th order is called the missing order.
assumption is valid O The number of interference fringes in the central
for a«X, which diffraction peak depends on the ratio d/a,
means that central independent of X; however, the angular position on
diffraction peak is the screen depends on X.
spread out over the Relative Intensity'
entire screen. This
situation is not
realised in practice
because
Distant screen
infinitesimally narrow
slits are impossible in Fig. 2E.65 (a) 5° 0 5" 10°
practice. We had " " ~ (a) Diffraction factor (sin2p/2)/(p/2)2 vs. 0
ignored the diffraction pattern due to each slit. For a far
away screen the diffraction patterns due to slits overlap.
Each of the two slits produces a diffraction pattern on a
distant screen when illuminated at a time; the two patterns
are slightly separated from one another. The expression for
intensity in the double slit experiment is 10° 5° 0 50 10°
(b) Interference factor cos2-§. yg
/ =4JoCos2-^, = 2
2 X
Now we can write a more general expression for the
double slit intensity distribution. The intensity due to one -?v

slit acting alone is given by the single slit diffraction


formula. We have to replace Iq by
sin
10° 5° 0 5» 10°

(c) Intensity. 10 vs,0

Fjg.2E.65
292 OPTICS

LIMIT OF RESOLUTION of distinct vision) are just resolved, then


Resolution is the ability to separate and distinguish X^ I _ 1 ^ TT
fine details of an object or to create distinction between two D~^ ~ 60 180
small objects. The resolving power of an optical or 0.00029x25 cm
instrument is a measure of the instrument's ability to = 0.0725 mm =0.1mm
produce separate images of two point objects.
Hence, two objects kept at Dwill be just resolved, by xhe
Optical instruments usuallyhave circular apertures, that unaided eye if they are separated by approximately
produces a diffraction pattern similar to that of a circular one-tenth of a millimetre.
slit. The expression for the angular position of the first dark
ring surrounding a central maxima is Rayleigh's Criterion
Suppose two distant stars are observed with a telescope.
sinGi = 1.22- Each star is a point source (producing same intensity at the
Earth) and its image on the film is a diffraction pattern due
to the circular aperture. Both the stars produce separate
Circular diffraction patterns. No sustained interference pattern is
'Plane /aperture obtained because the sources are incoherent. The resultant
iwaves
'from eTl intensity is the sum of intensities from each diffraction
distant 02 pattern. Fig. 2.40shows the diffraction patterns oftwopoint
point J sources when they are unresolved,just resolved and clearly
source
resolved.

L »• ^ unresolved
L» d Screen'
Barrier

Fig. 2.39 (a)


just resoivedi
Ordinarily the angle 6 is small so that the (b) '
approximations sin0 - tanG="0 are valid, and we can use
= 1.22-
d well resolved
(c) :
From figure, tanG = —
D Fig. 2.40

=1.22^ The accepted criterion for


d
resolution is Rayleigh's
Note that the size of the central maxima is dependent on criterion.
X/d, i.e., the central maximum is more spread out for When the central
longer wavelengths and smaller apertures.
maxima of one ditiraction
• If the aperture of an objective is large, then the limit
pattern is located at the
of resolution Gis reduced, i. e., relatively more closer
position of the first
objects will be resolved.
minimum of the other
• Resolving power is reciprocal of resolving limit, i. e..
diffraction pattern, the
Resolving power two sources are said to be
Resolving limit
just resolved.
For a good optical instrument, resolvinglimit should be Mathematically two point
small and resolving power high. objects separated by an angle
• Resolving limit of normal eye AG are resolved when AG > AGj^
is (1 minute). If two where
60°
objects subtend an angle 1/60° AG^ =1.22-
d
or more, they will be well
resolved.
The angle AG^ is the Fig. 2.41

• If two objects separated by x limiting angle of resolution for


at a distance D (least distance
Fig. 2.39 (b) an optical instrument. For this criterion intensity of light at
the instrument from each object should be the same.
293
WAVE OPTICS
d[sin (j) + sin(8- = mX.
Regardless of criterion the resolving power of an optical
instrument depends on the wavelengths ofthe light and the The principal result is that the pattern becomes
diameter of the instrument's aperture. asymmetrical, with only the position of the zeroth-order
Resolving limit of a microscope is the maximum remaining unchanged. If([) is counted as positive,
mitiitniim separation between two close and when the grating is rotated anticlockwise (and 0is counted
resolved objects. as positive in the same sense), then the density of
interference maxima becomes larger at positive angles,
Reciprocal of resolving limit is resolving power. whilst it decreases for negative ones. If the lines of the
Resolving limit grating are vertical, all the interference maxima lie along a
1.22X _ 0.61X horizontal straight line.
y =
2sinX, a
s 67
where X is the wavelength of light used and a is the
semivertical angle ofthe cone oflight as shown inFig. 2.40.
Jn a certain experiment ofdijfraction through a single slit of
'Width a =40Xj, it is observed that -J2 times the geometric
[mean ofangular width offirst maximafor Xj =5000 Aand
that of second maxima for X2 =6400 A, is equal to the'
Angular position offirst minima in a new set-up having slit]
Fig. 2.42
'widtha' andwavelengthX^ =400oA. Given that maximum,
order of minima in the new set-up is tiand isfound atangle of
If the medium between object and objective has 190°. Determine the values ofa'and r\. ._]
.refractive index p., then
1.22X Solution: For minima,
y A ;—A
2nsina riX a sin 9 = mX where m = order of minima
Resolving power ofa microscope isincreased by using a Angular position 0= sin0 =0for small0
liquid between object and objective.
when slit width a = 40X and when X, = A, ^:
^ j 66
Angular width of first maxima,
=Angular position of 2nd minima - angular position
!aparallel, thin, monochromatic laser beamfalls on adijfrac-, of first minima
\tion grating at normal incidence. How does the interference
pattern itproduces on aviewing screen change ifthe gating is< 2Xj _ Xj
rotated through anangle <90° around anaxis, whichispar-^ a a a

\allel to the lines of the grating? _ . When X = X2:


Angular width of second maxima.
Solutioni when the monochromatic laser beam falls Difference in angular position of third
on the diffraction grating at normal incidence, the positions
9 of maxima of the interference pattern are given by - and second minima
dsin0 = mX, (m = 0, ±1, ± 2, ± 3,...), 3X2 2X 2 _ ^2
where dis the gratingspacing and X
is the wavelength of the laser beam. If XlX;
Their geometric mean = =
the grating is rotated through an angle
^ around an axis parallel to the lines of Now when slit width = a',
the grating, we have to modify the
above equation. It is enough to consider Wavelength = X3
Fig. 2E.66
the interference from only two slits of . - - No.of minima on one side of principle maxima-= ti
the grating, as shown in Fig. 2E.66. Maximum order of minima = r
The optical path difference consists of two parts: For maximum order of minima
Ai +A2 = dsin(l) + dsin(0-(j)) a' sin 90° = 11X3
Thus the modified equation for the interference pattern Angular position of first minima for new set-up
is
294 OPTICS

'^3 _ 1
the same position and time that B is directed toward +z.
a' q
Since j x It = i, the direction of propagation is toward +x. It
V2
is customary to let the polarization direction be defined as
It is given that 42 X1X2
J along E rather than B. In Fig. 2.45, the wave is polarized
V2 [5000x6400]^^ _ 1 parallel to the y-axis because E oscillates along the ±y
a ri
directions. The plane that contains E and the propagation
Ti = (200/8) = 25 direction is called the plane of polarization.
a' = TjA. 3 = 25 X 4000A = 10|im
Plane of polarization
POLARIZATION
Let us examine waves
travelling on a rope. A
Propagation
rope can be set into
direction
oscillation in a vertical
plane as in Fig. 2.43(a), or Fig. 2.45
in a horizontal plane as in
Fig. 2.43(b). In either case, In case of a plane-polarized wave, E oscillates along a
the wave is said to be fixed line at any point. At any point along a circularly
linearly polarized or —>

polarized wave, E maintains a fixed magnitude, but its


plane-polarized —that
direction rotates in space with a constant angular frequency.
is, the oscillations are in a
plane. Fig. 2.46(a) shows the time dependence of E at a particular
If we now place an point due to a circularly polarized wave that is propagating
—>
obstacle containing a along the +x direction (out of the page). The variation of. E
vertical slit in the path of Fig. 2.43
.the wave (Fig. 2.44), a
vertically polarized wave passes through, but a horizontally
polarised wave will not. If a horizontal slit were used, the
vertically polarized wave would be stopped. If both types of
slits were used, both types of waves would be stopped. Note
that polarization can exist only for transverse waves, and not
for longitudinal waves such as sound.
Light is an electromagnetic wave in which the oscillating
Fig. 2.46
quantities are the electric and magnetic fields. In a plane
—^ ^
may be represented by a vector of fixed magnitude that
wave, the directions of E and B are perpendicular to each
rotates about the x-axis with a constant angular frequency
other and perpendicular to the direction of propagation of
the wave. Light is a transverse wave, and consequently, it CO. In this picture, the tip of the E vector traces out a circle,
can be polarized in different ways. and the components of E—Ey and — oscillate with the
same amplitude and have a phase difference of X/2 radian. If
—>

the tip of the E vector in Fig. 2.46(a) rotates in the clockwise


sense, then the light is right-circularly polarized, whereas if
the tip rotates in the counterclockwise sense, then the light
is Ze/t-circularly polarized.
An elliptically polarized wave [Fig 2.46(b)] is similar to
a circularly polarized wave except that, at a particular point,
(a) (b) Ey and E^. have different amplitudes. In this case the tip of
Fig. 2.44 —>

the E vector traces out an ellipse.


The propagation direction of light is given by the Ordinarily polarized light means plane-polarized light.
direction of E x B. In the figure, E is directed toward +y at Henceforth, we shall confine our - attention to
plane-polarized light.
WAVE OPTICS 295

Light emitted from an ordinary source, such as the amplitude, we have thatJo C^o)^ and / oc (Fg cos0)^. The
filament of a light bulb, is unpolarized. Acting proportionality constant is the
independently, the atoms and molecules emit wave trains of same in each case so that
light, and the polarizations of these wave trains are Analyzer Polarizer

unrelated. The resulting light consists of a random mixture / = /n cos^0 axis axis

of polarizations; it is unpolarized. This is Malus's law. When /

Measurement of Polarization
Light can be polarized by passing it through a polarizer.
0 = 0
Malus's
is substituted
law, we
into
have t
EqCOsS
'
A polarizer is an optical device that selectively transmits 1 = Iq cos^ 0 = /q. That is, the
light having its plane of polarization parallel to the intensity transmitted by the
anal3rzer is maximum when
1 <
polarizer's transmission axis. Light having its plane of /

polarization perpendicular to the transmission axis is the transmission axes of the


Fig. 2.49
blocked out by absorbtion or reflection. Fig. 2.47 shows a polarizer and analyzer are
parallel. Further, when 0 = 7t/2
Transmission radian, I = Iq cos^(7i/2) = 0. That is, the intensity
axis
transmitted by the analyzer is minimum, when the
Detector' transmission axes are perpendicular. Because there is no
positive sense associated with a transmission axis, all
Projection possible intensity values, 0 < / < Jq, are realized in the
Polarizer of transmitted interval from 0 = 0 to 0 = 7i/2 radian.
E field
Unpolarized Unpolarized light consists of light with random
light directions of polarization. Each of these polarization
Fig. 2.47 directions can be resolved into components along two
mutually perpendicular directions. On average, an
beam of unpolarized light incident on a polarizer. The
Unpolarized Polarizer
electric field of the transmitted wave at a particular point light
Polarized
light
and time is parallel to the polarizer's transmission axis.
Malus's Law Light —
direction
Suppose we pass the polarized light from a polarizer
through a second polarizer, as shown in Fig. 2.48. The
|lo
Transmission Fig, 2.50
axis
Transmission Detector unpolarized beam can be thought of as two plane-polarized
axis beams of equal magnitude perpendicular to one another.
Projection of
transmitted E When unpolarized light passes through a polarizer, one of
Analyser
the components is eliminated. So the intensity of the light
Projection of passing through is reduced by half since half the light is
transmitted E
eliminated,
Unpolarized Polarizer
light
Fig. 2.48 .4...
second polarizer is often called the analyzer to distinguish it Polarization by Reflection
from the first polarizer. The azimuthal angle 0 is a measure When light strikes a
of the orientation of the polarizer's transmission axis relative
nonmetallic surface at
to the analyzer's transmission axis. Let Iq represent the any angle other than
intensity transmitted by the polarizer and incident on the
perpendicular, the
analyzer. Notice that the detector can measure Jq if the reflected beam is
analyzer.is removed temporarily.
polarized preferentially
In Fig. 2.49. we show the projection of the field in the plane parallel to
components of the light as it emerges from the polarizer the surface (Fig. 2.51).
with amplitude Eq and from the analyzer with amplitude In other words, the Fig. 2.51
Eq COS0. That is, the analyzer transmits only the component component with
of the wave that is parallel to its transmission axis. Since the polarization in the plane perpendicular to the surface is
intensity is proportional to the square of the wave preferentially transmitted or absorbed. You can check this by
296 ORTIC#
rotating polaroid sunglasses while looking through them at light incident on the analjrzer is completely polarized. Then
a flat surface of a lake or road. Polaroid sunglasses are made = 0 and substitution into eqn. (3) gives P = 1. Thus the
wi±, their axes vertical to eliminate the more strongly degree of polarization ranges from a minimum of zero for
reflected horizontal component, and thus reduce glare. unpolarized light to a maximum of 1 for completely
The amount of polarization in the reflected beam polarized light.
depends on the angle, varying from no polarization at
normal incidence to 100 per cent polarization at an angle
known as the polarizing angle, 0^.This angle isrelated to
the index of refraction of the two materials on either side of 'Unpolarized light passes through two polaroids; the axis o/|
the boundary by the equation lone is vertical and that of the other is at 60" to the vertical, i
IWhatis the orientation and intensity ofthe transmitted light? [
tanGp = ...(1)
Solution: The first polaroid eliminates half the light,
where is the index of
so the intensity is reduced by half: =1qI2. The light
refraction of the material in reaching the second polarizer is vertically polarized and so is
which the beam is travelling and reduced in intensity to
rij is that of the medium beyond
the reflecting boundary. If the /o =fi(cos60°)^ =-Ji.
4
beam is travelling in air, = 1,
then eqn. (1) is Thus, I2 = fo/®- transmitted light has an intensity
one-eighth that of the original and is plane-polarized at a
tan0p = n. •••(2)
60° angle to the vertical.
The polarizing angle 0p is
also called Brewster's angle,
Fig. 2.52
and eqn. (2) is called Brewster's
law. At Brewster's angle, the [When unpolaiized lightfalls on two crossed Polaroids (axes at
reflected and transmitted rays make a 90° angle to each 90°), no light passes through. What happens if d third
other; i.e., 0p +0^ = 90° (Fig. 2.52). This can be seen by ^'Polaroid, with axis at 45° to each ofthe other two, is placed]
substituting eqn. (1), 0.2 = tanOp = sin0p/cos0p, into between them ? _
Snell's law, sinOp = n2 sinO^ and get cos0p=sin0r
which can only hold if 0p = 90°-0^. Solution: The first Polaroid changes the unpolarized
Degree of Polarization light to plane-polarized and reduces the intensity from Iq to
In Malus's law, we assumed that both the polarizer and = 10/2. The second polarizer further reduces the intensity
the analj^er were ideal. That is, all light polarized parallel to by (cos45° )^,
the transmission axis was transmitted, and no light 7^ = 7i(cos45°)^
polarized perpendicular to the transmission axis was
transmitted. Real polarizers do not polarize light completely. = =1/0.
2 ^ 4 °
The quantity used to characterize the polarization of light is
the degree of polarization P The light leaving the second polarizer is plane polarized
at 45° relative to the third po^rizer, so the latter reduces iJie
Suppose the polarizer in Fig. 2.52 is not ideal, but the
intensity to
analyzer is effectively ideal. Let Jy represent the intensity
measured by the detector, when the transmission axes are /3=/2(cOS45°)^=|/2
parallel (0 = 0), and let represent the intensity when the
axes are perpendicular (0 = 7i:/2radian). The degree'of or ^3-gJo-
-1/
polarization P of the light transmitted by the polarizer is
defined as Thus 1/8 of the original intensity gets transmitted. But if
P=1LZLl ...(3) we do not put the 45° Polaroid in at all, zero intensity
f|l +^x results. The 45° Polaroid must be inserted between the other
two if transmission is to occur. Placing it before or after the
As an example, suppose the light incident on the other two results in zero intensity.
analyzer isnot polarized at all. Then Jy = Ij_ and substitution
into eqn. (3) gives P = 0. As another example, suppose the
WAVEiOPTICS

Problems

Interference (Elementary)
1« Consider two waves, one of frequency [Ans. n = |m'--|/m]
Vj = 5.455x10^^ Hz, the other with 7. Show that the total number ofmaxima produced by a
V2 = 5.450x10^^ Hz. If these two waves start two-slit apparatus cannot be greater than (2H/X) -F1.
exactly in phase at t = 0, (a) How long is it before Why is counting the number of maxima not a very
they are exactly out of phase ? (b) How far has each precise way to determine X 7
wave travelled in this time ? 8, For a two-slit interference pattern, show that the
[Ans. Ca) IxlO^^^s, (b) 30pm] angular half-width of each maximum—one-half the
2. Consider two waves with frequencies and V2. After angular separation between the points on eitherside
travellingdistance dthe first wave has made one-half of the maximum at which the intensity is one-half
oscillation more than the second wave. Show that that of the maximum—is
the frequencies are related by Av =Vj - Vj =v/ld, :^ =A0 =A.
where v is the speed of the wave. R 4D
3. Show that if two waves differ in frequency by an 9. When light ofwavelength Xpasses through two slits
amount Av that is much smaller than either a distance d apart, what is the maximum possible
frequency, then the waves differ in wavelength by number of interference fringes ?
A^ = - CAv/v)A.. [Ans. There are -^+1bright fringes and -^-1 dark
4. Two identical waves with free-space wavelength Xq ^ X
start in phase and travel parallel routes for a distance fringes]
d The first wave travels distance din air (nf, = 1.00) 10, One slit in a double slit is increased in size until it is
and the second wave travels distance d in a medium 15 per cent larger than the other, (a) By what
of refractive index n. Show that after travelling the fraction is the intensity at the maxima increased?
distance d the waves have a phase difference (b) What is the intensity at the former minima in
A(i) = 2ju|:(n-l)4/Xo]. terms of the intensity at the maxima ?
5. Two slits separated by 0.200mm are to be used in 11. Ifyou double the width ofa single slit, the intensity
Young's double-slit experiment. Immediately behind of the light passing through the slit is doubled,
the slits is a lens of focal length 0.500m used to form (a) Show, however, that the intensity at the centre of
the interference pattern on a screen located in the the screen increases by afactor of4. (b) Explain why
•focal plane of the lens. What is the wavelength of a this does not violate conservation of energy,
monochromatic light source used to illuminate the 12. pvo narrow parallel slits separated by 0.850 mm are
slits if adjacent maxima of the interference pattern illuminated by 600 nm light, and the viewing screen
are separated by 1.00mm ? is 2.80m away from the slits, (a) What is the phase
[ Ans. 400 nm] difference between the two interfering waves on a
6. A Young's double-slit experiment is observed in air. screen at a point 2.50mm from the central bright
When the whole apparatus (source, slits and screen) fringe ? (b) What is the ratio of the intensity at this
is submerged into a fluid, the screen position point to theintensity at thecentre ofa bright fringe ?
originally occupied by the bright fringe becomes [Ans. (a) 7.95 radian, (b) 0.453]
the location of the m'*'" dark fringe. In terms of m 13. Monochromatic coherent light of amplitude Eq and
and m' m'^ find an expression for the index of angular frequency to passes through three parallel
refraction n of the fluid. slits each separated by a distance d from its
neighbour.
^98^ OPTICS

(a) Show that the time-averaged intensity as a 20. Consider N coherent sources described by
function of the angle 0 is £1 = £q sin(0t + ^), £2 = Eq sin(cot + 2(|)),
n 2
T 1 o r 27rdsin0^ £3 =£oSin(cot + 3(t>),..., £^ = £3 sin(cot+ N(1)).
Ke) =^max l+2cos|^ Find the minimum value of ^ for which
Ef^ =Ei +£2 +£3 +... + £^ is zero.
(b) Determine the ratio of the intensities of the
primary and secondary maxima. FA
[Ans. 360 ]T z
N
14. Coherent light rays of wavelength X strike a pair of
21. In a Young's double-slit experiment using light of
slits separated by distance d at an angle of Oj, as
wavelength X, a thin piece of plexiglas having index
shown in Fig. 2.14. If an interference maximum is
of refraction n covers one of the slits. If the centre
formed at an angle of 02 a great distance from the
point on the screen is a dark spot instead of a bright
slits, show that d(sin02 - sinOj) = mX, where m is
spot, what is the minimum thickness of the plexiglas?
an integer. '
[Ans. ^/2(n-l)]
22. Consider the double-slit arrangement shown in Fig.
2.22, where the slit separation is d and the slit to
screen distance is 7,. A sheet of transparent plastic
having an index of refraction n and thickness t is
placed over the upper slit. As a result, the central
maximum of the interference pattern moves upward
Fig. 2.14 a distance y'. Find y'.
L-
15. Slit 1 of a double slit is wider than slit 2, so that the
light from slit 1 has an amplitude 3.00 times that of
the light from slit 2. Show that m = 0 Zero order
I =(47inax/9)(l + 3cos^ (l)/2) for this situation.
Plastic
16. Determine the resultant of the two waves sheet

= 6sin(100jit) and ^2 = 8sin(100jtt + Jt/2).


[Ans. lOsinaoOnt+ 0.927)]
17. Suppose that the slit openings in a Young's Viewing screen
double-slit-experiment have different sizes so that
the electric fields and the intensities from each slit Fig. 2.22
are different. If = £oi sin(o)t) andjE2 = £©2 sin
[Ans.
(tot + ({)), show that the resultant electric field is
E =Eq sinfcot + 0), where 23. Astronomers observed a 60.0 MHz radio source both
Eq - -JEqi +E 02 ^01^02 C0S(1) directly and by reflection from the sea. If the
Eq2 sin^ receiving dish is 20.0 m above sea level, what is the
and sin0 = angle of the radio source above the horizon at first
maximum ?
18. when illuminated, four equally spaced parallel slits [Ans. 3.58°]
act as multiple coherent sources, each differing in 24. In a YDS£, let D = 120 cm and d - 0.250 cm. The slits
phase from the adjacent one by an angle (j). Use a are illuminated with coherent 600 nm light.
phasor diagram to determine the smallest value of (fi Calculate the distance y above the central maximum
for which the resultant of the four waves (assumed to
for which the average intensity on the screen is 75.0%
be of equal amplitude) is zero. of the maximum.
[Ans. 7c/2]
[Ans. 48 pm]
19. Sketch a phasor diagram to illustrate the resultant of 25. An oil film (n = 1.45) floating on water is illuminated
El = Eqi sincot and £2 = ^02 sin(cot + ^), where by white light at normal incidence. The film is
£02 = 1.50£oi and n/6<^< n/3. Use the sketch and 280 nm thick. Find (a) the dominant observed colour
the law of cosines to show that, for two coherent
in the reflected light and (b) the dominant colour in
waves, the resultant intensity can be written in the the transmitted light. Explain your reasoning.
from
[Ans. (a) Green, (b) Violet]
Jjj = 7j+J2*^2^7j72 cos(|).
WAVE OPTICS 299

26. An air wedge is formed give the positions of bright and.dark fringes on the
between two glass plates screen.
separated at one edge by a
very fine wire, as shown in [Ans. d sin 0 = X. and d sin 6 = nX, ]
Fig. 2.26
Fig. 2.26. When the wedge is
32. Consider a beam of light incident on a grating at an
illuminated from above by 600 nm light, 30 dark
fringes are observed. Calculate the radius of the
angle yw.r.t.'the normal, as shown in Fig. 2.32.Show
wire.
that the interference maxima satisfy the relation
[Ans. 4.35|iin] d(sinY +sin0) = ±mX, (m = 0,1,2,..-.)
27. (a) Both sides of a uniform film that has index of
Incident Emerging:
refraction n and thickness d are in contact with air. waves waves
For normal incidence of light, an intensity minimum
is observed in the reflected light at X.2, and an
intensity maximum is observed at where
Fig. 2.32
Xi > X2- If no intensity minima are observed
between and X2, show that the integer m in eqns. 33. In Fig. 2.33, two coherent point sources of waves, Sj
is given by m = Xi/2(Xi - ^,2). (b) Determine the and S2, are separated by a distance d along the
thickness of the film if n = 1.40, Xj = SOOnm and x-axis. Show that the nodal lines are hyperbolas.
A,2 = 370nm. That is, show that the coordinates of nodal lines
28. Use phasor addition to find the resultant amplitude satisfy the equation (x/a)^ - (y/b)^ = 1.
and phase constant when the following three y
harmonic functions are combined:
£1 = sin(cot + 7u/6), E2 = 3.0sin((ot+ 7jt/2),
£3 = 6.0sin(a)t + 47t/3).
[Ans. 7.99sin(cot + 4.44 radian)]
Interference from Thin Films
29. A thin film of a transparent material with n = 1.29 is
to be placed on a glass (n = 1.50) surface. What is the
minimum thickness for the film such that the
Nodal
reflection of normally incident light with X = 600 nm line
is minimized ?
[Ans. 116nm]
30. For non-normal incidence Fig. 2.33: A nodal line due to
two coherent sources of waves
of light on a thin film show ' is a hyperbola
that
X
[Hint; Recall that (-r2| is a constant on a nodal line.]
Tgjf —•
cos{sin"^[(sin0)/n]} 34. Fig. 2.34. - , - -
shows a radio
, where n is the index of
wave
refraction of the film.
transmitter
Fig. 2.30
and a receiver
31. A pattern similar to the y- separated by a
Screen.
double slit pattern can be distance d and
produced using Lloyd's i Incident located
mirror (Fig. 2.31). At distance
grazing angles almost 100 above
per cent of the light that 'light ^t ii2 ground. Transmitter Receiver
falls on the glass is Glass plate
receiver can • ng.2.34 _ j
reflected so that the Fig. 2.31 receive signals
reflected beam has nearly both directly from the transmitter and indirectly
the same amplitude at P as the beam that passes from signals that reflect off the ground. Assume that
directly from slit S to P. SlitS and its virtual image S' the ground is level between the transmitter and
behave as coherent sources. Develop expressions to receiver and that a 180° phase shift occurs upon
1300 OPTICS

reflection. Determine the longest wavelengths that


Partially silvered
interfere (a) constructively and (b) destructively. surfaces
[Ans. {A^2{Ah^ + d^f^-2d, (b) m^ + d^^-d]
35. A piece of transparent To :
detector I
material having an index of
refraction n is cut into the i
shape of a wedge, as shown in '
Fig. 2.35. The angle of the
wedge is small, and , Source
monochromatic light of Fig. 2.39
Fig. 2.35
wavelength X is normally •
incident from above. If the height of the wedge is h 40. In Pohl's interferometer, a thin sheet of mica is put
and the width is /, show that bright fringes occur at behind a shielded
the positions x = Xlim + l/2)l2hn and that dark light source, as in Fig. ^ ~- IT
fringes occur at the positions x = Xlml2hny where 2.40. Each side of the Screen
m = 0,1,2,... and x is measured as shown. mica reflects light '
36. The condition for constructive interference by from the source, so
reflection from a thin film in air assumes nearly that there are two '
normal incidence. Show that if the light is incident coherent virtual #-d

on the film at a nonzero angle (relative to the sources along a line


normal), then the condition for constructive perpendicular to the
interference is 2ntcos02 = (m + l/2)X, where 02 is screen. Show that the
l-
the angle of refraction. equation for the
37. An air wedge is formed between two glass plates in radius R of bright . _ Fig.2.40
contact along one edge and slightly separated at the circular fnnges on
opposite edge. When the plates are illuminated with the screen is
monochromatic light from above, the reflected light •^R^ +(l +2d +2tf -^jR^ +a +2df =Cn-l/2)X
has 85 dark fringes. Calculate the number of dark
fringes that would appear if water (n = 1.33) were to 41. A two-slit interference pattern is produced by a pair
replace the air between the plates. of slits whose widths are equal to one-tenth, their
[Ans. 113 dark fringes] separation, (a) Show that some of the maxima will
be missing from the interference pattern, (b) What is
38. Measurements are made of the intensity distribution
the value of m for the first missing maximum on each
in a Young's interference pattern. At a particular
side of the centre ?
value of y, it is found that I/Imax = 0.810, when
600 nm light is used. What wavelength of light 42. Two optical flats are placed in near
should be used if the relative intensity at the same contact at one edge and are
location is to be reduced to 64.0% ?
separated by a human hair at the
other edge as shown in Fig. 2.42.
[Ans. 421 nm]
(a) At the thin edge of the air Fig. 242
39. A useful interferometer design is the Fabry-Perot wedge the separation of the glass
interferometer. The essential elements of this design
surfaces is much less than the wavelength of visible
are shown in Fig. 2.39. Show that the condition for light. Does this point appear bright or dark when
constructive interference of the waves that reach the
viewed by reflected light ? (b) If filtered light from a
detector is 2c^/cos0 = mX., where m = an integer and mercury vapour source (X = 546 nm) is used, exactly
d is the distance between the parallel mirrors. nine bright fringes are located between the edges in
contact and the position of the hair. How thick is the
hair ?
[Ans. (a) Dark, (b) 2.32|xm]
43. A Fresnel biprism can be used together with a
point-source located distance d behind the centre
line of the prism to create a two-source interference
pattern. As shown in Fig. 2.43 the two images P{ and
P2 formed by refraction at the prism act as coherent
WAVEOPTICS 301

sources, (a) Show that the separation a of the images curvature is sufficiently slight that it is
is given approximately by a = 2da(n -1). not possible to tell by simple
observation whether the surface is
convex or concave, (a) If one pushes
gently on the centre of the lens,
circular fringes move toward the
centre or away from the centre
depending on whether the lens
surface is concave or convex (Fig.
i PlC-'
2.46). Which way do they move in the
(b)
convex case? (b) An alternative
Fig. 2.46
technique for distinguishing concave
from convex is to view the
Fig. 2.43
interference pattern using white light. In that case
[Hint: Assumethat the prism angle is small (a < 1°) and that the the order of the colour, from the centre out is
image is formed in the same plane as the source. TVace a ray opposite for the two cases. What is the ordere of the
striking the prism such that it is deviated by the angle of minimum colours (from the centre out) for the convex case ?
deviation.] [Ans. (a) Convex move out, (b) from blue to red]
(b) Calculate the fringe separation on a screen 47. A plano-convex lens has index — '
located a distance s = 1.750 m from the back face of a of refraction n. The curved
prism of refractive index n = ,1.500and vertex angle side of the lens has radius of
1.00® if the point-source has wavelength . curvature R and rests on a flat
X = 632.8nm and is located a distance d = 0.750nm glass surface of the same
behind the back face of the prism. index of refraction, with a film
[Ans. (b) 8.52mm] of index ngim between them. Fig. 2.47
44. The thin glass wedge shown in Fig. " ] The lens is illuminated from
2.44 is slid slowly infront of one of the , , above by light of wavelength X. Show that the dark
slits of a two-slit interference | I Newton's rings have radii given approximately by
apparatus illuminated by sodium light r = VmXR/nfii^
(X = 589 nm). The lengA of the wedge where m is an integer andr is much less than R.
is much greater than the width of the ' Fig. 2.44 !
48. The Fig. 2.48 shows a glass rod,
slit, so the part of the wedge infront of
having, one cylindrical and one flat
the slit at any moment varies only slightly in
thickness. In the process, the pattern shifts so that surface, in contact with a flat plate
what was originally the tenth minimum (m = 9) now of glass, (a) What is the shape of
the interference fringes ? What are
lies at the midline? What is the thickness of the
the distances from the line of Fig. 2.48
thickest part of the wedge ? Take n = 1.54 for the
contact of (b) the bright fringes ?
glass.
(c) the dark fringes?
45. In Young's double-slit experiment a thin layer of mica
[Ans. (a) The fringes run parallel to the axis of the rod.
is placed directly behind one of the slits. The In the centre there is a wide dark fringe. The fringes get
thickness of the mica is such that the optical path narrower the farther you move away from the centre line
length of the light in the mica is 0.500X. (a) of contact
Assuming no power is absorbed from the light (b) r = V(n +l/2)XR, n=0,l,2,...,
passing through the mica, describe the change in the (c) r = -JnXR, n = 0,1,2,... ]
far-field diffraction pattern that occurs when the
49. Two flat mirrors form an
mica is inserted behind one slit, (b) If the wavelength
of the light used is 632.8nm, what is the actual
angle close to 180° (Fig.
2.49). A source of light S
thickness of the mica ? (Assume the mica is oriented
is placed at equal
so that its index of refraction is 1.552.)
distances b from the
[Ans. (a) Pattern is shifted by one-half fringe: dark to
mirrors. Find the interval
bright, light to dark, (b) 204 nm] Fig. 2.49
between adjacent
46. A lens with a slightly curved surface is placed on an interference bands on
optical flat generating a pattern of circular rings, screen MN at a distance OA = a from the point of
when viewed in reflected monochromatic light. The intersection of the mirrors. The length of the light
302
OPTICS
wave is ki^Qwn and equal to X. Shield C does not
allow the light topass directly from the source to the length of the lightwave X=6000A. (b)'How mafiy
screen. . interference bands can be observed on-'a screen in an
[Ans. Ah =
A,(a+ &)
]
installation with the biprism described in the
26a previous problem ?
50. Lloyd's interference [Ans. (a) Ah = X(a + b)
= 0.15 cm,
experiment consisted in 2aa(n~l)
obtaining on a screen a
pattern from source S and (.a+b)X ^
fromi its virtual image S' in 53. A biprism with an angle p that
mirror AO (Fig. 2.50).
appreciably differs from 180" is
How will the interference
placed into a vessel filled with a
pattern obtained from
sources S and S' differ
Fig. 2.50 liquid having a refraction index Uj
or serves as one of the walls of this
from the pattern obtained in Young's double slit vessel (Fig. 2.53). Calculate the
experiment ?
angle 5 of an equivalent biprism in
[Ans. The entire pattem will be shifted by width ofdark air. The refraction index oftheprism
fringe.]
substance is n2. Fig. 2.53
51. Two point sources with the same phases ofoscillation
n9-n,
are on a straight line perpendicular to a screen. The [Ans. 8=p + 180' "i-l
"2 Tin-I
nearest source is at a distance of D»X from the
screen. What shape will the interference bands have 54. A convergent lens with a focal length of
onthe screen ? (a) What is the distance on the screen / =10 cm is cut into two halves that are,
from the perpendicular to the nearest bright band if then moved apart to a distance of
the distance between the sources is / = nX » X(n is d = 0.5mm (a double lens). Appraise the
an integer)? (b) Find the radius of the bright number of interference bands on a screen
ring if D= I=nX, n » I, and k= n, n~l, n~2 at a distance ofD= 60cmbehind the lens
etc. ' ifa point source of monochromatic light
(X = 5000A) is placed infront of the lens Ftg. 2.54 1
[Ans.- (a) = 2Dx(j+l at a distance of a = 15 cm from it.
[Ans. N = 25]
55. Why are Newton's rings formed
52. Light from source Sis incident on the Fresnel biprism only by the interference of rays
shown in Fig. 2.52. The light beams refracted by the 2 and 3 reflected from the
different faces of the prism partly overlap and boundaries of the air layer
produce an interference pattem on a screen on its between the lens and the glass
section AB. (a) Find the distance between adjacent (Fig. 2.55),while ray4 reflected
from the flat face of the lens Fig. 2.55
does not affect the nature of the
interference pattem?
[^s. The lens is too thick, interference occurs only with
thm films. The air layer between ±e lens and the class is
thin.]
Diffraction
56. Light of wavelen^ 550 nm faUs on a slit that is
3.50x10 mm wide. Estimate how far from the
Fig. 2.52 ^ntral maximum is the first diffraction maximum
•fnnge if the screen is 10.0 m away ?
interference bands if the distance from the source to [Ans. 2.4 m]
the prism is a =1metre and from the prism to the 57. Ifa slit diffracts 550nm light so that the diffraction
screen 6 = 4 metre. The angle of refraction of the
prism is a = 2x10 ^ radian. The glass which the maximum is aOcm wide on a screen 2.50m away
prism ismade ofhas a refraction index ofn = 1.5. The what will be the width ofthe diffraction maximum'
for light with a wavelength of 400nm ?
[Ans. 5.8 cm]
MVEOPTICS t303;
58. How wide is the central diffraction peak on a screen interference fringes for 550 nm light on a screen
3.50m behind a 0.0655 mm wide slit illuminated by 1.0 m away and (b) the distance between the two
400 nm light ? diffraction minima on either side of the central
[Ans. 4.28 cm] maximum of the envelope.
59. A helium-neon laser emits light that has a [Ans. (a) 1.8 cm, (b)llcm]
wavelength of 632.8 nm. The circular aperture 67. Missing orders occur for a diffraction grating when a
through which the beam emerges has a diameter,of diffraction minimum coincides with an interference
0.500cm. Estimate the diameter of the beam 10.0km maximum. Let a be the width of each slit and d the
from the laser. separation of slits and show (a) that if d = 2a, all
[Ans. 3.09 m] even orders (m = 2,4,6,...) are missing, (b) Show
60. A child is standing at the edge of a straight highway that there will be missing orders whenever
watching her grandparents' car driving away at d _ m-i
20.0 m/s. The air is perfectly clear and steady, and a m2
after lO.Omin the car's two taillights appear to merge
where rrii and 7712 are integers, (c) Discuss the case
into one. Assuming the diameter of the child's pupils
d = a, the limit in which the space between slits
is 5.00 mm, estimate the width of the car.
becomes negligible.
[Ans. 1.90 m if the predominant wavelength is 650 nm]
68. A diffraction grating has 16000 rulings in its 2.4cm
61. A diffraction pattern is formed on a screen 120 cm width. Determine (a) its resolving power in first and
away from a 0.400 mm wide slit. Monochromatic second orders, and (b) the minimum wavelength
546.1 nm light is used. resolution (AX.) it can yield for X = 410 nm.
Calculate the fractional intensity Ijl^ at a point on ' [Ans. (a) 1.60x10''; ^20x10^ "
the screen 4.10mm from the centre of the principal
(b) 0.026 mm; 0.013mm]
maximum.
[Ans. 0.0162] 69. Determine a formula for the minimum difference in
frequency. A/, that a diffraction grating can resolve,
62. The angular resolution of a radio telescope is to be
when two frequencies, f^~ /, are incident on it.
0.100° when the incident waves have a wavelength
[Ans. A/ = //mJV]
of 3.00 mm. What minimum diameter is required for
the telescope's receiving dish ? 70. Monochromatic light falls on a transmission
[Ans. 2.10 m]
diffraction grating at an angle 6 to the normal. Show
that eqn. for diffraction maxima must be replaced by
63. (a) Explain why the secondary maxima in the
single-slit diffraction pattern do not occur precisely d(sin 6 ± sinO) - mX, m = 0,1,2,...
at p/2 = (m -I-1/2)jt where m = 1,2,3,... (b) Show Explain the ± sign.
that the secondary maxima' occur when p/2 satisfies 71. Two wavelengths X and X + AX (with AX « X) are
the relation tan (p/2) = p/2. (c) Carefully and incident on a diffraction grating. Show that the
precisely plot the curves y = p/2 and y = tanp/2. angular separation between the spectral lines in the
From their intersections, determine the values of p order spectrum is,
for the first and second secondary maxima. What is AX
A0 =
the per cent difference from p/2 = (m-l-l/2);i ?
64. Determine, approximately, the angular width at half
maximum (where I = Iq/2) of the central diffraction where d is the slit spacing and m is the order number.
peak for a single slit. 72. Two stars, each of which produces about the same
To be concrete, assume X = 550 nm and intensity at the earth, have an angular separation of
a = 2.60 X10"^ mm. 7 p. radian. If the light from the stars has an average
wavelength of 600 nm, what is the minimum
[Hint: Use graphical methods, or trial and error; the problem
aperture diameter needed to resolve them ?
cannot be solved analytically.]
[Ans. 10.70°]
73. (a) How far away can a human eye distinguish two
car headlights 2.0m apart ? Consider only diffraction
65. If a double-slit pattern contains exactly seven fringes effects and assume an eye diameter of 5.0 mm and
in the central diffraction peak, what can you say the wavelength of 500 nm. (b) What is the minimum
about the slit width and separation ? angular separation an eye could resolve when
[Ans. d = 4a] viewing two stars, considering only diffraction
66. Two 0.010mm wide slits are 0.030mm apart (centre effects ? In reality, it is about 1' of arc. Why is it not
to centre). Determine (a) the spacing between equal to your answer in (b) ?
304

[Ans. (3)^16'kmv Cb) 0.42:] maximum distance between the plane of the sources
74. What is the highest spectral order that can be seen if and the slit be if the diffraction patterns are to be
a grating with 6000 lines per cm is illuminated with resolved ? (In this case, the approximation
633 nm laser light ? Assume normal incidence. sin0 = tan0 is not valid because of the relatively
[Ans. Two orders] small value of o/X.)
75. What is the limiting angle of resolution of a telescope [Ans. (a) 41.8°, (b) 0.593, (c) 0.262m]
that has an aperture diameter of 75 mm ? The light 81. Light of wavelength 632.8nm illuminates a single
from the objects being viewed has a wavelength of slit, and a diffraction pattern is formed on a screen
SOOnm. 1.00 m from the slit. Using the data in the table on the
[Ans. 8.1 ji radian] following page, plot relative intensity vs. distance.
76. Show that if the width of a slit is halved, the intensity Choose an appropriate value for the slit width a, and
at the centre of the screen is quartered. From this it on the same graph used for the experimental data,
might seem that energy is not conserved, since half plot the theoretical expression for the relative
as much power is passed by the smaller slit, but a intensity
quarter as much intensity shows up at the centre of I _ sin^(P/2)
the pattern. Show that the power delivered to the •^max (P/2)^ " ,
screen by the light in the central fringe (between the
What value of a gives the best fit of theory and
minima on either side of the central maximum) by
experiment ?
the two slits is proportional to the width of the slit.
[Ans. a = 99.5pm+ 1%]
77. The spacing A0 between two spectral lines of equal
intensity is the same as their half-width A0]y2- Show 82. How much diffraction spreading does a light beam
that the ratio of the intensity at the point midway undergo ? One quantitative answer is the full width
between the maxima and the intensity at the maxima at half maximum of the central maximum of the
is 2(^7c)^ = 0.81. Fraunhofer diffraction pattern of a single slit. Yoii
can evaluate this angle of spreading in this problem
[Hint: sin(n/2N)« njTN for large N] • and in the next, (a) In result of problem 81 define
78. The resolving power of an optical device whose p/2 = (l) and show that, ' at the point where
purpose is to disperse light according to wavelength J = 0.57tnaxj we must have sin^ = ^/V2. (b) Let
is defined as : = sin(t) and ^2 = Plot and ^2
r =A same set of axes over a range from (j) = 1 radian to
AX ^ = 7c/2 radian. Determine (j) from the point of
where AX is the difference in wavelength of two intersection of the two curves.'(c) Then show that, if
spectral lines that can barely be resolved and X is the the fraction X/a is not large, the angular full width at
wavelength of either line (they have nearly the same half maximum of the central diffraction maximum is
wavelength). Show that the resolving power of a A0 = O.88X/a. • '
grating for the order spectrum in Nm. 83. A pinhole camera has a small circular aperture of
79. (a) Show that the Rayleigh •- ' diameter D. Light from distant objects passes through
criterion for the resolution of the aperture into an otherwise dark box, falling upon
the diffraction patterns due to a screen located a distance L away. If D is too large,
the two sources of the same the display on the screen will be fuzzy because a
intensity passing through a bright point in the field of view will send light onto a
single slit is A8jj « X /a, where circle of diameter slighdy larger than D. On the other
a is the slit width (Fig. 2.79). hand, if D is too small, diffraction will blur the
(b) Show that this leads to an display on the screen. The screen shows a reasonably
intensity at the centre of the joint pattern of two sharp image if the diameter of the central disc of the
fringes of 81 per cent of the intensity at the peak on diffraction pattern, is equal to D at the screen, (a)
either side of the centre. Show that for monochromatic light with plane
80. Suppose that the single slit is 6.00cm wide and wavefrpnts L » D, the condition for a sharp view is
infront of a microwave source operating at 7.50GHz. fulfilled if = 2.44XL. (b) Find the optimum
(a) Calculate the angle subtended by the first pinhole diameter if 500 nm light is projected onto a
minimum in the diffraction pattern, (b) What is the screen 15.0 cm away.
relative intensity ///max at 0 = 15.0® ? (c) Consider [Ans. (b) 0.428 mm]
the case when there are two such sources, separated
laterally by 20.0 cm, behind the slit. What must the
Polarization
84L% At what angle should the axes of two polaroids be
placed so as* to reduce the intensity of the incident Incident

unpolarized light by an additional factor (after the


Detector
first Polaroid cuts cuts it in h^f) of (a) 25 per cent,
(b) 10'per cent, (c) 1 per cent ? Fig. 2.91
[Ans. (a) 30^ (b) 18°, (c) 5,7°]
851 Show that if two equally intense sources of light Cl-cos4cot)
produce light that is plane-polarized, but with their
planes of polarization perpendicular to each other, This means that the ihtensity of the emerging beam
then they cannot produce an interference pattern is modulated at a rate that is four times the rate of
even if they are in phase at all moments. rotation of the centre disc.

86i ' Unpolarized light falls on two polarizer sheets whose [Hint: Use the trigonometric identities cos^0 = (1+cos20)/2
and sin^0 =.(1 - cos20)/2, and recall that0 = tot]
axes are at right angles, (a) What fraction of the
incident light interisity is transmitted ? (b) What 92i Ydii want to rotate the plane of polarization of a
fraction is transmitted if a third polarizer is placed polarized'light" beam by 45.0° with a maximum
between the first two so that its axis makes a 60° intensity feduction'of 10.0%. (a) How many sheets
angle with the axis of the first polarizer ? (c) What if of perfect polanzefs do you need to achieve your
the third polarizer is infront of the other two ? goal ? (b) What is the angle" between adjacent
polarizers?
[Ans. (a) 0, (b) 0.094/o. (c) no light gets '
transmitted] [Ariff. (a)'6, (b) 19°]
87. What would Brewster's angle be for reflections off 93, The-per cent polarization P of a partially polarized
the' surface of water for light coming from beneath beahi of light is defined as
. the surface ? Compare to the angle for total ihfenial' p_LnH2linmLxlOO
reflection^ and to Brewster's angle' ffbfn above the' f max fmin
surface.
wKereJ„-and7n,^ are the maximum ahd-mihimum
[Ans. 36.9°, 53.1°] intensitie's'that are obtained, ""vyHen 'the light passes
SS.'.-' Unpolarized light passes 'trough five sucCeSsi^^e ' through, a p'olarizef that is SlOwiyrotated. Such light
Polaroid sheets each of 'w'hose "axes-makes'a'45° c^' he conridefed as the' sum" Of two unequal
angle with the previous one. What is'the intensity of plahe-pol^ized. beams ."p 'intehsiries anid
the transmitted beam ? '" peipehdiculaf to-each" Other Shbw that the light
[Aris. /o/32] transmitted'by'a'polarizer, whose axis makes an
89;- 'For a particular transparerit'ihedium.sUrrOun'dedby - angle'bto the direction in which is obtained, has
air, show that the criticar angle for'total ifiteriial. intensity
reflection and the pOIariring angle'are relatedbyitfie (1+ p.'cos2({))-
expression cotS^ = sin0^. (1 + p)
90.-' How far above-the horizon is the Moon, when its where p = P/100 is the "ifactional polarization".
image reflected in calm water is" completely 94". Suppose we haveN (ideal) polarizers in the beam in
polarized? = 1--33) see Fig. 2.91 instead of three. The beam incident on
[Ans. 37° above the horizon] polarizer 1 is plane polarized, and polarizer 1 has its
91..'In Fig. 2.91, suppose that the transmission axes of transmission axis rotated an angle B/Nto the plane of
the left and right polarizing discs are perpendicular polarization of the incident beam. Each subsequent
to each other. Also, let the centre disc be rotated, on polarizer has its axis-rotated in the saine sense by an
the common axis with an angulat speed co_. Show that angle oi Q/N to the one before it, so that the last
if unpolarized light is incident on the left'disc Witharf polarizer has its axis rotated by an angle 0 to the
intensity the intensity of the beam emer^fig' plane of polarization of the beam incident on
fi-om' the right disc is. polarizer 1. Show that the intensity transmitted by
the last polarizer is
306 OPTICS-

2N
0 96. A partially polarized beam may be regarded as two
1=1, COS —
N
incoherent beams with different intensities and with
perpendicular planes of polarization. Let/j represent
where Iq is the intensity of the incident beam. the intensity of the more intense beam, and let 12
95. Suppose the angle between the transmission axes of represent the intensity of the less intense beam, (a)
polarizers 1 and 3 in see Fig. 2.91 is held fixed at :c/2 Show that when a partially polarized beam is
radian while the angle 6 between the axes of 1 and 2 incident on an ideal polarizer, the transmitted
is allowed to vary. Show that the intensity at the intensity is
detector is given by / = (7j-72)cos^e +72
T-T (l-cos(46))
i — i n 0 ' 16 ^ where 0 is the angle between the polarizer's
transmission axis and the plane of polarization of the
where Iq is the intensity of the unpolarized beam more-intense beam, (b) Show .that the intensity
incident on polarizer 1. • , ' transmitted by the polarizer rfiay be written as
(1 + Pcos(20))
' (i+p)
•where P is the degree of polarization of the beaih.

' I j . . '

1 I - i" 1 '
WAVebPtlCS 307

eve I ^^nly]i@n^ Sltef^ativM is egrrecl

1. In a double slit experiment-the coherent.sources are index 1.5 and the wavelength of light remains 4200A
spaced 2d apart and the screen is placed a distance D the number of fringes seen will be :
from the slits. If bright fringe, is formed on the
(a) 60 Cb) 45
screen exactly opposite to a slit, the value of n must be:
(c) 30 (d) 20
r ru^ 2d^ 5. In a Young's double slit experiment the slits are
^ 2XD XD illuminated by a parallel beam of light from the
tt: . Cd)^^ medium of refractive index ni=1.2A thin
xd 4XD transparent film of thickness 1.2pm and refractive
2. In YDSE experiment shown in the figure, a parallel index n = 1.5 is placed infront of s^ perpendicular to
beam of light of wavelength (A, = 0.3 mm) in medium path of light. Wavelength of light measured in medium
p-i is incident at an angle 0 = 30® as shown is 400 nm. The refractive index of medium between
(5^0 = S2O). If the intensitydiie to each light wave at plane of slits and screen is n.2 = 1.4. If the light coming
point O is / 0 then the resultant intensity at point 0 will from the film and S2 have equal intensities I then
be : intensity at geometrical centre of the screen is :

|i2=10/9
i

1mm"\ r -

S2
•—D=1m-» L.

(a) zero (b) 2fo[l +cos(407c/9)] (a) 0 (b) 21


Cc) 3/0 (d) lo (c) 41 (d) none of these
3. Two glass plates are touching at one end and 6. A beam of monochromatic light of wavelength
separated by a thin wire at the other end. When a 5.82 X10"^ mfalls normally ona glass wedge with the
monochromatic parallel beam of wavelength 4200A wedge angle of 20 second of an arc. If the refractive
incident normally on the glass plates is reflected, an index of glass is 1.5, find the number of dark
interference pattern of 30 fringes are observed. If the interference fringes per cm of the wedge length :
wavelength of light used is taken 7000A instead of (a) 2percm (b) 5 per cm
4200A, the number of fringes observed will be : (c) 7 per cm (d) 6.5 per cm
7. Which one of the following statements is not true ?
(a) 50 Cb)40
The phase difference between two light waves can
(c) 30 (d) 18
change :
4. In the above question 3, if the free space between the (a) by reflection of one of the waves
plates is replaced by a transparent liquid of refractive (b) by the waves travelling along paths of different
lengths
308 OPTICS

(c) by the waves travelling through media that have a seconds. The minimum value of time t for which the
different refractive indices but the same thickness intensity at point P on the screen exactly infront of the
Cd) by being transmitted through a boundary between upper slit becomes minimum is :
two media with different refraction indices (a) Is (b)2s
8. In a Young's double slit experiment the slit separation (c) 3 s (d)1.5s
is 0.5 mm and the screen is 0.5 m from the slit. For a 14. Intensity observed in an interference pattern
monochromatic light of wavelength 500 nm the I = Iq sin^ 0. At 0 = 30° intensity 7 = 5+ 0.002. The
distance of 3'*^ maxima from the 2^^ minima on the W
other side is :
percentage error in angle if/q = 20—r- IS

(a) 2.75 mm (b) 2.5 mm


Cc) 22.5 mm (d) 2.25 mm (a) 4V3 X10"^% (b) -xlO"^ %
n

9. Two sources Sj and $2 separated by 2 metres vibrate


according to the equations = 0.03 sin Tct and
r^
(c) 4%/3 X10"^% (d) VS X10"^%
%
y2 = 0.02sin7ct where y^ and y2 are in metres. They
send out waves of velocity 1.5m/s. The amplitude of 15. A long horizontal slit is place 1 mm above a horizontal
the resultant motion of particle collinear with and plane mirror. The interference between the light
coming directly from the slit and that after reflection is
S2 and at the middle of 5^52 will be :
seen on a screen 1 m away from the slit. If the mirror
(a) 0.05m (b) 0.01m
reflects only 64% of the light falling on it, the ratio of
(c) O.Hm (d) 0.06m
the maximum to the minimum intensity in the
10. In a standard YDSE apparatus a thin film (p = 1.5, interference pattern observed on the screen is :
t = 21pm) is placed in front of upper slit. How far (a) 8:1 (b) 3:1
above and below the centre point of the screen are two (c) 81:1 (d)9:l
nearest maxima located ? Take D = lm, d = lmm
16. Two monochromatic coherent point sources and S2
X = 4500A. (Symbols have usual meaning) 'are separated by a distance L. Each source emits light
(a) 1.5mm (b) 0.6mm of wavelength X, where L»X. The line S1S2 when
(c) 0.15 mm (d) 0.3 mm extended meets a screen perpendicular to it at a point
11. In a' Lloyd's mirror pcperiment if the mirror reflects A.
75% of light incident on it the ratio of intensity at (a) The interference fringes on the screen are circular
. .interference maxima and minima will,be : ^ in shape
(a) 75 (b) 150 , ' • (b) The interference fringes on the screen are straight
(c) 194 Cd) 200-.- lines perpendicular to the line S1S2 A
12. High quality camera lenses are often coated to prevent (c) The point A is an intensity maxima if L = nX/2
reflection. A lens has an optical index of refraction of (d) The point A is always an intensity maxima for any
1.72 and a coating with an optical index of refraction separation L
of 1.31. For near normal incidence the minimum 17. In YDSE, if bichromatic light having wavelength X^
thickness of the coating to prevent reflection for and X2 is used, then the maxima due to both lights will
wavelength of 5.3 x 10"^ m is : overlap at a distance of from the central maxima.
(a) 0.75pm (b) 0.2pm Take separation between slits as d and distance
between screen and slits as D. Most appropriate option
(c) 0.1pm (d) 1.75pm
for the blank space is :
13. In a Young's double slit experiment set up, source S of X. j + X fXi-X:
wavelength 500nm illuminates two slits and S2 (a) (b) X 2d
2D D
which act as two coherent sources. The source S
D
(c) LCM of d
and ^^^(d) HCF of
d ^ ' d
and ^^
d
P ly
18. In YDSE, let A and B be two slits. Films of thicknesses
and tg and refractive indices and pg are placed
|S' 1mm in front of A and B, respectively. If p^
the central maxima will :
(a) not shift
-1m- -2m-
(b) shift towards A
(c) shift towards B
oscillates about its own position according to the
(d) (b) if tg > and (c) if tg <
equation y = O.bsinTrt where y is in mm and t in
WAVEOPTICS 309

19. In YDSE, a laser light is used to illuminate the closely 12d^ 6d^
(a) (b)
spaced slits, as a result, a clear interference pattern is D D
obtained on screen. If we place smoke particles 3d^ 24d^
between the double slit and the screen, then the (c) (d)
D D
effects of interference will be observed :
24. For the situation shovra in the figure below.
(a) only on screen
X.
(b) in between the space, between the screen and slits BP -AP = —and D» d. The slits are of equal widths,
(c) nowhere
having intensity Iq. The intensity at P would be :
(d) information insufficient
20. In YDSE of equal width slits, if intensity at the centre
of screen is Iq, then intensity at a distance of |3/4ffom :
the central maxima is CP is the fringe v\ndth) :
(a) 1,
• ^ : =1
,Parallel beam

(c)
h ' of light Screen
iWavelength,
4
21. The figure shows the r- " (a) 41Q Cb) 2/o
•10cm/s
' initial position' of a point i20cm/s (c) 3/o
•30cm/s
source of light S, a detector
D and a lens I. Now at i ® 25. Consider an YDSE that has different slits width, as a
Focal length=10cm
t = 0, all the three start '• — result, amplitudes of waves from two slits are A and
moving towards right with different velocities as 2A, respectively. If /g be the maximum intensity of the
< shown interference pattern, then intensity of the pattern at a
The time(s) at which the detector receives the point where phase difference between waves is <t), is:
maximum light is : • (a) Iq cos^ (j)'
(a) 0.56s and 8.94s • (b)3.8s- 3 , 2. - >

(c) 8.94s and 19.62s (d) 0.56s (c) ~[5 + 4c6s(t)] (d) — [5 + 8cos(l)]
22. The YDSE apparatus • is as •, .9^ .. .
'Screen!
shown in the figure below. 26. The light' ^source'S is 'over the center of a circular
• The condition for point'P to be opaque plate of radius 1 m at a distance a = 1 m from
a dark' fringe is (X = it. The distance from the plate to the screen is b = 0.8
'wavelength of light waves) : "S-
m. Find the diameter of the shadow of the plate on the
(a) (l^~l2) + (.l2-U^ = nX • • l2 ^2 ^4 screen...
-J - -

(b) (.h-l2) + Q3-U) = nX


C2n -1);^
Cc) ai+l2)-(l2+U) =
2
(2n - 1)X ; ♦
(d) ai + i2)-a2+k)- ' b
+ i
23. Consider the optical system '/77mm77777777777777777777777777777777\
- p
shovra in the figure that d M.
follows. The point source of
light S is having wavelength
ti (a) 1.8 m
(c) 3.6 m
(b) 2.9 m
(d) 5.4 m
equal to X. The light is
reaching screen only after
reflection. For point P to be
2"^ maxima, the value of X would be (D>> d and
d » ?i):
310 OPTICS
AN9WER8
Level-1: Only One Alternative Is Correct^^^^-t^.
1. Cb) 2. Cd] 3. Cd) 4. Cb) 5. Cb) 6. Cb) 7. Cd) 8. Cd)
9. Ca) 10. Cc) 11. Cc) 12. Cc) 13. Ca) 14. Cc) 15. Cc) 16. Ca)
^ 17. (c) 18. Cd) 19. Cb) 20. Cb) 21. Ca) 22. Cc) 23. Ca) 24 Cc)
K T K" ^ = . . -
ccy"
* * " '
25. CcJ ^ 26.
.•J
. • .'c!/., .• 1 > !
- .iuM; • -vr ' . "l
.'I • -•-
•^. -'< I _-
WAVEOPTICS

Solutions

Level-l: OniyiOne Altetfiatjyeijl^


9. (a) yi = 0.03sinjct
2n , . y. IOjc y2 = 0.02sin TCt
2. (d) A(j) = —dsin6 = ——
X. 3
Each wave have the same period = 2 seconds
I = Iq + 2Iq cos[2jc + C47t/3)]
= 1, or frequency, n = —.
2i
Since the velocity is 1.5m/s,
30B 4200 -Q
3. (d) Number of fringes = — = 30 x - la the wavelength = u/n = = 3m
4. (b) Number of fringes = 30x 1.5 = 45
5. (b) Ax^naCsi -1) +1 xl.5-naCsJ The path difference for a point Ratthe centre
=tCl.5 -1.4) =1.2 X10"® X0.1 and $2 = 0
= 1.2x10"^ =120nm
Hence phase difference = 0
X = 400nmxl.2
2jc
The resultant amplitude
A4) — xl20 = -
^ 400x1.2 A=(Ai^ +Ai +2A1A2 cos(t))^^
Alternative = 0.05 m

Path difference at 0:(iXrei -l)t - j^— 10. (c) (p-l)t =^


1 2.1 X10"® X1
phase difference at 0:f -1 jt—where X = —Xr
2 10"^
r

UiXi —n2X2 x = 1.05x10"^ := 1.05mm

= - after substitution of values


2
CBF
6. (b) P=—
^ 2|re
No.of dark fringes per
:(7/3)[5
1x10"^ 2^10 xlO"^ = 5
P
XD -4-m
p/3 !
8. (d) Fringe width W = — = 0.5mm o
2p/3 ;
Distance of third maxima from central fringe M .
= 3vy -10
nx _ 1x4500x10 45x10
-5

Distance of second maxima from central fringe d 10-3


= 3W/2 (On the other side)
= 0.45x10"3 ^ 0.45mm
Required distance = 3W +1.5W = 4.5W
L
11. (c) With a narrow source and slit, the disturbance sin 7Ct = 0 => TCt = 0, 71, 2jt... t = 1s
are coherent but they have slightly different
amplitudes because one beam loses energy on
14. (c) /=/oSin2 0
being reflected. Differentiating I with respect to 0
The intensities of interfering disturbances can be
— - 2fn sin0cos0
written as and (3/4) (a^) de "
aVs dl _ 2Iq sin0cos0d0
The amplitudes are a and - 2cot0d0
f iQsin^Q
Maximum amplitude = a +
aS _ a(2+V3) Percentage error in angle
2 ~ 2
dl
= —xl00 = -xlOO
Minimum amplitude = a -
ajs a(2-V3) 0 .2fcot0. 0

0.002 6x100
X
(2 + V3)^ C2 + V3)2 2 X 5 cot 30® 7t
Therefore,
(.2-Sf
= -V3xlO-Vo
, 71
= (3.7321)''•= 194 •
12, (c) There is no net change'in phase produced by the 15. (c) Intensity of direct ray =/q = ^0
two reflection
Intensity ofreflected ray = = k\
— = 2ltidcos0 ^ 100 ° { 10 j
2 ^
^max _ (Aq +0.8Ao)^ _
•^min (-4q—6.8Ao)^ 0.2 1
ui=1.31
16. (a) If screen is perpendicular to the' line joining the
' ' sources the fringes will be circular and central
For normal incidence cosG = l"' fringe will'be if;SiS2-= nX. • • '
, X 5.3x10"^ '' 17. (c) Let maxima corresponding to Xj is
d -10 m = O.lum
4pi 4x1.31 overlapping tvith maxima corresponding to
X2- Then the required distance,
13. (a) y' = —, at point P exactly infront of Sj
2 , TI2X2D
yd
d " d
. Ax = — +
D 2D' For least value of y

For minimum intensity Ax - (2n -1)—(n = 1) y = LCM.of and


2 d d

Putting the value we get 18. (d) d Ax = (ji^-l)t^-(p^-l)tB

(0.5sin ;it) x10"® +0.25 x10"® =^ x10'^ = -lists


2
= tg —
0.5 sin TTt + 0.25 = If Ax > 0, then fringe pattern will shift upwards.
2
If Ax < 0, then fringe pattern will shift
downwards.

z 19. (b) The interference is taking place everywhere in


between the slits and the screen as well as on
screen. In the absence of smoke particles no
path is seen in region between slits and screen
flVAVEiOPTIG
313
where interference is occurring but in the 23. (a)
presence of smoke these paths are made visible
T ' "'S;
by smoke. k\m\' \\\\\\\!\\\\\\
2d
20. (b) Let the intensity of individual waves be I, then S*
'7T
3d
/o=47 6d
lA?: io :
/=^ Central ^
maximai
4 '2..
=> AxatP = dsine C8d)x3d
AtP,Ax =
D
Ax-dtanQ = —
D For 2""^ maxima,
A P
Ax=—x— 24d'
7? ,4 , Ax = 2X = 2X
D
=^ —
24. (c) BP-AP =-
3"
AA. = —
A(b 2Tt X ~A< = —
JT d ... X
[A^ = kAx] ^ — sin0 = —
X 4 2
2 3

r = / +J + 2>//2 cos- => -tane = -


2 3
= 2/= 12. d d/4
2
2 D
21. (a) The detector will receive the maximum light
when the image of point source of light X
: coincides, with- . the 8D 3
position of detector.. t Path difference between waves coming from A
•Let at any time t, image of i .s , • 1/ d and B-is
S and detector cpincides ;.-<ioq iot»>,20t ! A ^
dieri, ' • ' ' "
for u =-(100-lot) cm A. 27C X '27C
' - . . . j

V = 20t cm

/ = 10cm Path difference between A and C is


The detector receives maximum light. = dsinG' = dtanO'

•From, -
.
A x^ir
, 3d/2 = 3d^
A/C = d X
V u f - D . , 2D •

4X
^ ^ ^ = 3 = 4X
20t ~(100-10t) 10

27^-197 + 10 = 0 2k
^^A/c —^ X4X —Stc
X
t = 0.56s and 8.94s
The three waves arriving at P can be written as
22, (c) For dark fringe
y <0,
Path difference = (2n -1) 27C , „
y < — and y < 8k, so resultant of these three is
O

2y <0

and y < —.
3

^resultant ~ ^0 "^^^0 ^ 2^/q X4/o COSl20®= 3/q


ORTICSi'

25. (c) As amplitudes are A and 2A, so intensities would r =I +4I +2'J^ cosd
be in the ratio 1:4, let us say 1 and 4J.
^max=^o=-f +4/+2V47^ =9/ r =51 +41 cos(t) =^(5 +4cos(|))
1 _ 1
=> 1 =^-^ 26. (c) ^ =
9

Intensity at any point, d = 3.6m


WAVEiOPTICS

Problems

'© V©I ® iHajPsit


1. A thin film of thickness t and index of refraction 1.33
coats a glass with index ofrefraction 1.50. Which ofthe
' following thickness t will not reflect normally incident
lightwith wavelength 640nm in air ?
(a) 120nm (b) 240nm
(c) 360nm (d) 480nm (a) Parallel fringes are observed.
2. Coherent light consisting ^of two wavelengths (b) Ifwater is introduced into the region between the
Xy =4500A and ^2 =6000A is sent through both plates, the fringe separation decreases.
slits of a Young's double slit apparatus : (c) If the angle of the wedge is increased, the fringe
(a) Central maxima for both wavelengths will separation decreases.
coincide (d) When white light is used there will not be a
(b) The third order bright fringe of X-^^ will coincide completely dark fringe.
TATith the fourth order bright fringe of 2 6. Imagine a Young's double slit interference experiment
Cc) The third order bright fringe of X2 will coincide performed with electron waves associated with fast
with fourth order bright fringe of X^ moving electrons coming from the electron gun. The
(d) The fringes ofwavelength will be wider than distance between successive maximum will decrease if
the fringes of wavelength X2 the :
3. In Young's double slit experiment, phase difference (a) accelerating voltage in electron gun is decreased
between the waves at a point on screen having Cb) accelerating voltage is increased
intensity less than the average intensity on screen may (c) distance between screen and slits is increased
be : (d) distance between the slits is increased
(a) Tt/4 Cb) 2k/3 7. The minimum value of d so that there is a dark fringe
(c) 71 (d) 77t/8 at0 is dnun •For the value of ,the distance atwhich
4. Two rays of light A and B with wavelength 5000A the next bright fringe is formed is x. Then :
travel parallel to eachother in air. Ray A encounters a t
imm thick layerof glass with refractive index n = 1.5. X

Then :
(a) ray B. will complete more oscillation than ray A O' 1;
O
(b) both rays will complete same number of
oscillations
(c) ray Awill complete more oscillations than ray B
(d) the actual difference in number of oscillation
made bytwo waves over the 1mm distance is1000 fb^ '^min il n
(a) djnin =
5. When monochromatic light is incident normally on a
wedge-shaped thin air film, see figure, aninterference (c) x=^ (d) X —dJJ^JJ.
pattern may be seen by reflection. Which of the
following is/are correct-?
316 OPTICS

8. Two point monochromatic and coherent sources of 5X


(d) If d = —, then intensity at O would be minimum
light of wavelength X are each placed as shown in the
figure below. The initial phase difference between the 9. A transparent slab of thickness t and refractive index \i
sources is zero 0. (D»d). Select the correct is inserted in front of upper slit of YDSE apparatus.
statement (s) : The wavelength of light used is X. Assume that there is
no absorption of light by the slab. Select the correct
statement (s) :
(a) The intensity of dark fringes will be 0, if slits are
identical
(b) The change in optical path due to insertion of
Screen
plate is jx t
75l
(a) If d = —,0 will be minima (c) The change in optical path due to insertion of
2 plate is (|J. - l)t , •
(b) If d = X, only one maxima can be observed on (d) For marking intensity zero at the centre of screen,
screen 5X
then thickness could be
(c) If d = 4.8X, then a total 10 minimas would be 201-1)
there on screen

ANSWERS

1. Cb, d) 2. (a, c) 3. (b, c, d) 4. (c, d) 5. (a, b, cj 6. (b,d)

7. Cb, d) 8. (a, c, c, d) 9. (a, c)


Solutions

The frequency of the light (oscillation per second)


ilIevel-2: More thah One Alternative is/are Correcg does not change inside the glass.
> t2 => ray Awill make more oscillations
1. (b, d)
= number of oscillations for ray A = ft^
2^•^t - nX
nX 640 X 3 similarly, = number ofoscillations for ray B= ft2
t = = n 240 n
2^: 2x4 An = rii - "2 = /(ti - ^2) =
t = 240nin, 480nm,...
2. (a, c) = X10"^^1= 10^
Central Bright fringe position has zero path difference 5000x10"^® l3
3X2D __ 4X^D 5. (a, b, c)
(c)
d
^ 28
3. Cb, c, d)
Let maximum intensity be Jq 6. (b, d)
The condition for getting maxima is dsin0= mX. The
wavelength ofelectron will be given as X= hc/v.
The distance betweensuccessive maxima willincrease
I = — if 0 becomes smaller. For this either d should be
increased or Xshould be decreased. For the latter we
KI.. must increase the voltage V.
J^2
7. (b, d)
There is a dark fringe at O if the path difference

^ AJ. 37c
5=ABO -AO'O =^
=> — < A(b < —
2 2

4. (c, d) X !

For ray A,ti =time taken by the ray to travel through d

the glass
O O
=A [d =1mm and =velocity of light in glass'
V„
8

= C/n]
^-3
A = ' = 5X10"^^ s
C 3x10®
d 10' = 12x10-1^8
For ray B, t2 = —
C 3x10' 3 ^ '^min
At =t,-t2=|5-^]xl0-i^=|xl0-i^s
318 lOPTICSI

' ' The bright fringe is formed at P if the path difference ^7X ^9X
Ax = ±—,±—,± —,±—,± —
5' = AO'P - ABP X 2 2 2 2 2

= D+^lD^+x^ -^1d^ +ix~df =X 9. (a, c) A X at P :


Ax= (SiP-t)^;, +tniedium -SaPair
_^ ^ _ (x^ +d^~ 2xd) = X = [SiP-S2Pj-[it-t]3i,
~ 2D 2D ' 2D
Given d = d^„
C1• x=d^=
Solving J 1^
^—
82!
8.. (a, b, c, d )
AX at 0 = d [path difference is maximum at 0] = ai-i)t
yx Earlier, A x at P = SjP - S2P = 0
So, if d = —, 0 will be a minima •
So, change in optical path due to insertion ofslab
d = X., O will be maxima = frL-l)t
5X.
d = —,0 will be minima and hence intensity is For intensity to be zero at P, we have
2 ; ' . ^ (2n - 1)X
Ax = ^
minimum.
2
If d = 4.8X',1;hen total 10 minimas can be observed on . X 3X, 5X,
. -screeiii.S above O and 5.below O, which correspond to
WAVE OPTICS

Problems

1 '
eve I

I :
iK monochromaticbeam of light falls on Young's double slit
In a modified YDSE the region between screen and slits is! [experiment apparatus as shown in figure. Athin sheet of
immersed in a liquid whose refractive index varies with! jglass is inserted in front of lower slit S2.
5 T 5 1
time as ft until it reaches a steady state value —. Ai
2 4 4 1 S1I
3 d .
glass plate of thickness 36itm and refractive index — is' ^ o
2 : So
d«D
introduced infront of one of the slits. OR
a

1. The central bright fringe can be obtained :


(a) at 0 only
(b) at O or below 0 only
(c) at O or above O only
Cd) anywhere on the screen
2. If central bright fringe is obtained on screen atO.
(a) (p -l)t = dsin0 (b) (p -l)t = dcosG
(c) (p-l)t+ dsin0 =0 (d)—^ =—^
p-1 sin0
1. Find the time when central maxima is at centre of 3. The phase difference between the waves interferingat
screen 0. fifth minima is :
(a) T = 4sec (b) F = 2sec (a) Stc Cb) 97t
(c) T = 8 sec (d) T = 1 sec (c) lOit (d) lire
2. What is speed of central maxima when it is at 0 ?.
(a) 10"^m/sec (b) 2xl0"^m/sec
(c) 3xl0"^m/sec (d) 4x 10"^ m/sec
opfics|
(a) h (b)~2/o
Cc) 37o Cd)4/o
iThe figure shows a double-slit experiment in which 2. The minimum value ofZfor which the intensity atOis
Icoherent monochromatic light of wavelength X from a zero, is :
(distant source is incident upon the two slits, each ofwidth , . 370)
(a)
|w(w» X) and the interference pattern is viewed on a 2d
jdistant screen. Athin piece of glass of thickness t and
jrefractive index n is placed between one ofthe slit and the
r ^ ^
®M (d)^ d
jscreen; perpendicular to the light path. Int'ehsity at'P for 3. If ahole is made atO' on AO'O and the slit S4 is closed^
t = Ois given is 7q. then the ratio of the maximum to minimufn observed >
on screen at'O if O'Sg is equal to —, is ;
Ad
Ca) 1 (b) infinity
(c)'34 (d)4
Consider the arrangement -shown - in figure. The >
distance D' is large compared to the separation"
between the~slits.
1. The intensify at a point P as a fuhctiqn 'dft is :
(a) /psin'^l-Cn-Dt | (b) IqCos^ 1
\.x
The experimental^ airarigeihent with Fresnel's double
(c) (d)7osin^|^^(h-l)t mirror at ah.ihcHnation of 12' is shown in figure. The
distances from"the iritersectidn ofthe mirrors to theslit-and
2. The intensity at P is mihinium if the value"of t is : the screen are•ascectively.'equal to 10cm andTSOcm. The
(a)X/n Cb)V{2Crt^ Di iwavelen'gth oflight-lised is X~ 0.55\^
ce) X/Cn~iy (.d)2X/(n-^D • S"v"siit
3. Ifthe width w'of one.o'f tHe slits'isiricfeaSdd to 2w, the
mgbamum'intensity at P'forto is :
(a)'C3/2)/o -(b)(9/2)Ia
(c)' C9/4)Jo

1. The width ofthe fringe inthe interference pattefn is:


(a) 1.1mm fb); 2.2 mm.
|In the arrangement shown in the figure, slits S3 and S4 are (c) LSmiri (d) 3mm
!having a variable'separation Z. Point 0 on the screen is at 2. The number of possible-fringes is ;
(a) 8 (h) 6
Cc) 9 (d) 7
S3
3. Shift of the interference pattern when the slit is
displaced by 1mm along the arc of a-circle of radius
A 0' z 0' r = 0.1 m with centre at 0 is :
(a)a2cni (bj 0.13cm,
S4
Cc)'0.3cm (d) 0.15cm
• D »
4. thefringes ohthe screen will be sharp"when the width
of- the slit is kept at a ma^dmum of :
XD
1. When Z = — the intensity measured at Ois Iq. The (a)'0.55ihm (b) 30pm
2d Cc) 2Dpm (d) 15pm
intensity at O when"Z = is :
321
WAVE.OPTiCS

Two similar and coherent point sources Sj and S2 vibrating ;The YDSE apparatus is modified by placing ah isotropic
in-phase, emit light of wavelength X.. The separation^ Itransparent plate of high melting point in front of one of'
'be^een the sources is 2X. Aperson initially at S2 starts ithe slits. The refractive index ofthe plate ispr = 1.5atroom<
walking along a straight line which is passing through S2 [temperature and its thickness is t =2pm. The refractive
and perpendicular to S1S2. During his journey he finds jindex of plate will increase when temperature increases
jvariation in.intensity of light which is obviously because of land temperature coefficient ofrefractive index ofthe platei
[•interference of light wav^ \(i. e., the fractional change in refractive index per unit rise-
•in temperature) is 2x10"^/ °C. The incident light is having]
Iwavdength X= 600A. The separation between the slits is;
\d =0.2 cm, and separation between the slit and the screeni
S.\ [is 2m. Assume that slits are of equal intensity.
2X [Based on the above information, answer the following
[questions.: ——'
1. Before insertion of plate, mid-point of screen is
location of central mmdma, then this point after the
1; The smallest distance firpm S2 where the person finds insertion of slab is :
minimum intensity, is :
(a) a point of central maxima
c..| , (b) a point of ina^ma but. not the central maxima
(c) a point of minima
(c)-—
,w 4 .Cd)2X (d) neither a point of maxima nor of minima
2. If the plate is heated so that it temperature rises by
2. ,At some place during ,his .walk-the person, finds l6°Q,then how many fringes will cross a particular
•intensity oflight isequal-to intensity due to oneofthe point "on the'Screen ? (Neglect the thermal expansion
sources, the. path. differ,ence between the .waves of plate)':
.arriving at that.place,is:
(a) iOOO (b) 10"^
(a) - Cb) - • ,,..10000
, (c);5000 (d) .——
o

(c) — .(d) 2X 3. •.If intensity ofunconverted slit isJq and thatofcovered


4 -
3. A source slit of.adjustable.width is-used to^illuminate slitis—, theiiintensity at mid-point ofthe screen, is ;
' 2' ' ' . • ' * .
two point sources'0.1 mni-.apart.in• an interference
experiment. The wavelength of.ligh't'used'is.bOOOA. (a) I,
1

V2
Cb)fox|
Tnitially the slit.is closed,-and .then it is-gradually
. opened. The width at which,the fringes on•a .screen (c) 41, (d)io 1-
D = 1 m apart will first disappear is V2
.(a) Imrad (b) 2mrad
(c) Smrad (d) 4mrad
322
OPTICS

MATCHING TYPE PROBLEMS 2. Match the column

1. Light from sourceS(|u|<| /|) falls on lens and screenis Column-I Columh-II
placed on the other side. The lens is formed by cutting (Arrangementtosee fringe (Shape of
it along principal axis into two equal parts and are .pattern) fringe).:
joined as indicated in column 11.
(a) (P) Hyperbolic
Column-! Column-ll 51
Plane of image move to (P Screen
ward screen if |/1 is in 52
creased

Screen i
Sj, $2 are two pinholes
Small- portion of each part
near pole is removed. The (b) (q> Circular
remaining parts are joined. S2.

Images formed will be vir (q) Screen


A Si, S2 are two pinholes
tual
—.. • ^'. .n. isk-f^ ^^
iSS (c) Incoming light (r) Elliptical

^ Screen' UlU
The two parts are separated
slightly. The gap is fflled by
opaque material.
glass wedge,
Separation between im
ages increase if |u| de
creases (d) fring pattern (s) Linear
visible -
I' ' 'incoming -"•[•
- rays - •. h /
Screen

The two'pafts are separated


slightly; The-gap is filled by .\ '.Plano-convexlensj . - , j
opaque material.'
id) Interference pattern can be (s)
obtained if screen is suit 3. In Young's'double slit expe'riinenV,the point source Sis
ably positioned. placed slighdy off the centtal' as' shown in figure.
Tf y= SObnm/jhen'matdi the foUowing
Screeni

Small portion of each part Si i


x=10mm
near pole is removed. The
remaining parts are joined.
2m
t1 20mm O
i

1m •2m
WAVE OPTICS
-- : 32^
\ Column-r \ \ Column-ll plane of slits and screen, the central bright fringe is
(a). Nature and order of interference (P) Bright fringe of order shifted in upward direction.
at &e point P, OP = 10 mm 80

(b) Nature and order of interference (q) Bright fringe of order


^
point 0 , 262

(c) If a transparent paper (refractive (r) Bright fringe of order 32


index |i =1.45) of thickness 62
t =0.02nim is pasted on Sji.e. Statement-2: If path difference , at 0 increases
one of the slits, the nature and or >coordinate of centrarbright fringe will change.
der of interference at P
Statement-1: For the situation shown below, two
(d) After inserting the transparent (s) Bright fringe of order identical coherent light sources produce interference
paper in front of slits S^, the 280 pattern on the screen. The intensity of minima-nearest
nature and order of interference tq.Si is not exactly zero
atO
, - ^'Si
-..../If' Screen

-• ^ ^
Statement-2: Miriiihufn''int'ensity is zero,' when
AS^ERirON AND REASON interfering waves have'same intensity at the location
of supefpbsitibh.'
Direction: Each question contains 'statement 1
(assertion) and statement 2 (reason) Statement-1: In YDSE, if separation between slits is
(a) Statement-1 is true", stkement-2 is true and less than wavelength of light, then no interference
. pattern-could be observed.
statement-2 is con^ect <^plahation for
statement-1. ^ . 'Statament-2: • For interference pattern to be
(b) Statement-1 is true, state'ment-2 •is .tme and observed, light sources have to be coherent.
statement-2 is not the correct explanation for 5. Statement 1: In Young's double'slit'-experimefit the
statenient-l. . . t • v- ' ; twoslits are at distance d apart. Interference patternis
(c) Statement-1 is true, statement-2 is false'. ' observed on a screen at distance D from the slits. At a
(d) Statement;l is false,'statemerit-2,is true. • | p;oint on the screenwhen it is direcdy opposite to one
1. Statement-1: Two thirds of possible valued for phaste of rthe slits, a dark fringe is observed. Then the
difference; result in constructive interference and one , wayelen^h of wave is proportional to square of
third results in destructive-interference^ ,'distarice ^between .the tWo slits.
•'Statement-2: Inteffefehce is"calle'd cdhstriictive'if Statement>2:'For a dark fringe intensity is zero.
the amplitude of die superposed,wayes exceeds that of 6. Statement 1: In interference, experiment, the fringe
' V drie individual wave. " ' —.'-t.< > > . systern Will shift, if a glass, plate is.interposed in the
2. Statement-1 LIn YDSE,, as shown in figure^ central path of.the one of iJie interfering beams and the shift
bright fringe "is^formed at6. Ifa liquid is filled between will be pn the side of the glass plate.
Statement 2: The velocityof light in glass is smaller
than in air.
OPTICS
324

ANSWERS

Level-3: Comprehension Based Problems

1. (a) 2. (c) Passage-2: 1- (d) 2. (a) 3. Cd)


Passage-1:
1. Cb) 2. (b) 3. (d) Passage-4: 1- Cb) 2. (d) 3. Cc)
Passage-3:

Passage-5: 1. (a) 2. (d) 3. (b) 4. (a) Passage-6: !• (b) 2. (a) 3. Cc)

Passage-7: 1. (b) "'2.'(d) S.'cd)

Matching T^e Problemsj

1. (a)-p, q; Cb)-p, q. r, s; (c)-r, s; (d)-p


3. (a)-s; Cb)-p; Cc)-q; Cd)-r

Assertion and Reason

1. (a) 2. (d) 3. ' Cc) 4. Cb) 5. Cc) • 6. Ca)


WAVE^OPTICS

' - • „ .f

eve I

B Level-3: Comprehension Based Problems


We know, / «« 4<i cos ((t>/2)
Passage-1
and Iq oc
2. (c) = + -|j,,)t
//7o = cos^C(f/2)
for central maxima
'I' =Io cos^f -^(n-Dt
Ax = 0

tp _ (4-r)+D 2. (b) Intensity at P will be minimum if


y = -
P-e d • (lO-Dd ({) f 2k+ 1
where k = 0, ± 1, ± 2....
y = 0; for T = 4sec
dy 6Dt n, (2fc + l)
ih)v= I. e. — (n -1) = n
dt aO-Tfd A 2

central maxima is 0 at T = 4sec


2(n-l)
6 Dt _•>
V = 3 X10 m/sec
36d ' for k=0,t =
2(n-l)
Passage-2
3. (d) As the width of one of the slits is increased to 2w,
1. (d) Ax = dsinO-(|i-l)t = 0 for C. maxima.
the amplitude due to slit become 2a
2. (a) If C max. is at O dsinO = (|j. - l)t
I oc (.Qi +a| + 20^02 cos<l))
3. (d) For 5*^ minima (|) = IItc
oc + C2q)^ + (2a)(2a) cos(J)
Passage-3
5a^ + 4a^ coscf)
1. (b) A = path difference between two waves arriving 27u(n - l)t
oc a^(5 + 4cos(|)) = 5 + 4 cos
at

P = (n-l)t 2jt(n - l)t


5 + 4cos
(j) = phase difference between two waves arriving
at
for t = 0, the intensity at P is maximum
P=-|^(A) =^(n-l)t
K A
9^
•^max —f 0
I = Intensity at the central point
[326 " '
OPTICS
Passage-4
Passage-5
1. (b) Intensity at Ois proportional to intensity at S, 1. (a) When a ray turns through a reflected ray turns
and at ^4
through 2a.
/ =fccos^6/2, where k constant, 5-phase Hence d = 2ar
difference
D = fe + r
0 27t Z TCZ

d
\ ^
whenZ = XD/2d-
Six ^^0 ^
Sk w
g _ Tcd XD _ k • .
~ ID ~2 ,^ ,
1 = lo =fccos^(n/4)' -• •
A: = 2fn p^^_X(6 +r)
2ar 2a
2XD
when Z' =

Phase difference 5' = — — = x - Ott =0.55 X10-®,Vt? 2 X 12 ' TZ -- •


=1-1'rim
- ... - , P 2,, ^W 2d
2. (d) The spread on- the screen whose interference
Required intensity at 0 is •• '• -•
fringes are formed is ^a ft.
/' =fccos^6/2 =2focos^(7c);=2f(, ' N (No. of fringes formed):.
2. (d) Intensity at O is zero if - = — _ 4 X1.3 X(1271x180x60)2
^ = 8.22

.e., 5,|| =.=.^ =p=^ o.55xio-®fiV^r'• ' '


Since'nmust be anb'dd number lower than 8.22
3. (c) . Intensity at.Sg =(aI +A^'+ 2Ajeos^)
N=7 '
where.(!).= x —^ ' ' 3.- (b)-The , angle Ithrough the ray,,turns is —The
P 4d \' XD Ad ,'2
; * \ f , '

Is, = ^oC2+ 2cosjt/2r=,'2A^ x 2005^ -4 = 2 ' - •;. : through the same angle. The
I- thtougli which the ray moves on
= 2A^ .• ,the area =f-r-JhrfThe central'fringe and hence
SJ SJ ; all'othef fnriges will'shift bythesame distance.
Required'shift ~~.b " ''
t iO. r
I 4d
O' O lxlO'2
x 1.3 = 0.013m = 1.3 cm
0.1

Amplitude ofwave at S3, '= -JiAq 4. (a) The fringes are still,sharp when d<fi
- - • • 2
Maximum intensity at 0' = (Aq +.V2Ao)^ s^max- = -^nim
1.1 = p.SSmm .
Maximum intensity atO= (VMq - Aq)^
Passage-6
'V2 + I 1. (b) Let the person finds minimum intensity at point
= (^/2 + l)'^ = 34
•^min V2-I P at distance x from 82. '
iWAVEiORTtCS 327
Path difference So, intensity at O is
^|i2X)^ +x^ ~x= (2n--1)h- •-...(1) / = /^+_o + /n X //
cos
IOtc
3

3_ J_
=^ +V^oX(-0.5) =Io 2 V2
3. (d) Forsatellite to be geostationary it has to revolve
in an equatorial plane and its sense of rotation
must be same as ^at of earth's rotation about its
own axis.

In addition to this, the satellite must have a time


To have minimum intensity at smallest distance period of 24 h.
from $2-
For the satellite to revolve around the earth, the
X
'Ji2Xf+x^-x. =3- •••(2) plane of the orbit must contain the centre of
2 earth.
7X
X -
12
Matching Type Problem^
2. (a) Maximum.possible path difference = distance
between-the sources = 2X
Let intensity of lightrffom each source is ./,.then 3. The optical path difference between the two waves
I = I+I + 2^cos^ I arriving at P is
6 = (SS2 + S2P) - CSSi + SjP)
cos(t) = -—,=>•(!)•= —path'diff. =—• '•
2 3. 3 = (SS2-SSi) + (S2P-SiP)
3. (c) The required width - — =dsinOo +dsin0 = +^
' 2d•' -

6000x10"^°
^-3
= 3 X10 ^ =f_3 m rad, v
0.2x10-
y= lOmnr
Passage-7
1.'* (b) When'temperature of'the plafte is incr^ease'd, its-
refractive index increases and, as a result, the
fringes, cross' at a paiticular'point. Here the
number of fringes crossing'through" a point is
due to charige'in refractive index of the plate.
Shift in the location of a particular fringe due to d = 20mm yg - 2mm, D2 = 2m,
changes in refractive index is,y = Di - Im, y - 10mm
• '• d
j, 20x2 20x10
o + = 0.14mm
So, the number of crossed fringes, 1000 2000

For a bright fringe, 5 = = 0.04 mm


X ^1
_ 1.5x2x10"^ xl0x2xlQ -6 T n' =^ =: Q-04 = 80
6x10"^. ^ ' 0.5 X10"^
> .-i V.;: Due to transparent paper, the change in optical path
, = (fi - l)t = (1.45-1) (0.02) mm = 0.009 mm
2. (d) AtO, Ax =(po -l)t =10x10"^m. 5" = 0.14mm-0.009mm = 0.131mm
.. 271 . " ^Tt-xlOxlO"^ lOjc 0.131
= 262
A(b = — X Ax = ^ = n =
-3
^ " 6x10"^ - • 3 0.5x10
328 opn®
Due to transparent paper, the path difference at O 4. (b)
Ax = dsinO
5^ = 5^ - Cn - l)t = (0.04- 0.009) mm = 0.0131 m AXnjax < d if d < X,then AXj^ax < ^, so maxima can be
0.131 present and interference pattern cannot be observed.
n - = 62
Statement 2 is true but doesn't explain Statement 1.
0.5x10"^
5. (c)
Assertion and Reason r j2
Here S-,P =Dand SoP =Vd^ +d^ =D 1+—^
1. (a)
^ 2D^
j2 T,
A([) 1
COS • > — .-. Ax = S2P-SiP = —= -(2n-l)
T 2

2jr 6. (a)
3 3 The fringe system will shift on the side of the glass
3. (c) plate. The shift establishes the fact the velocityof light
At the location of minima, two waves have different in glass is smaller than that in air.
intensities and hence minimum, intensity is not exactly
zero.
^PREVIOUS YEAR PROBLEMS

s Year Problems

each other. The angles of reflection and refraction are


Only One Alternative is Correct r and r\ The critical angle is : (IIT 1983; IM)
(a) sin~^(tarir) (b) sin~^(tanO
1. An electric bulb illuminates a plane surface. The (c) sin~^(tanjV) , , (d) tan"^(sini).
intensity of illumination on the surface at a point 2 m
6. Two coherent monochromatic light beams of
• away from the bulb is 5x 10"^ phot (lumen/sq.cm). intensities I and 4J are superposed. The maximum and
The line joining the bulb to the point makes an angle minimum possible intensities in the resulting beam
60°with the normal to the surface.The intensity of the are : (in 1988; IM)
bulb in candela is : (IIT 1980; IM)
(a) 5/ and/ (b) 5/ and 3/
(a) 40^3 (b) 40 (c) 91 and/ (d) 91 and 31
(c) 20 (d) 40x10"*^ 7. A beam of light of wavelength 600 nm, from a distant
2. When a ray of light enters a glass slab from air : source, falls on a singleslit 1 mm wide and a resulting
(IIT 1980; IM) diffraction pattern is observed on a screen 2 m away.
(a) its wavelength decreases The distance between the first dark fringes on either
G?) its wavelength increases side of central bright fringe is: (IIT 1994; IM)
(c) its frequency increases (a) 1.2 cm (b) 1.2 mm
(d) neither its wavelength nor its frequency changes (c) 2.4 cm (d) 2.4 mm
3. In Young's double slit experiment, the separation 8. Sphericalaberration, in a thin lens, can be redued by:
between the slits is halved and the distance between (IIT 1994; IM)
the slits and the screen is doubled. The fringe width is: (a) using a monochromatic light
(IIT 1981; 2M) (b) using a doublet combination
(a) uncharged (b) halved (c) using a circular annular mask over the lens
(c) doubled (d) quadrupled (d) increasing the size of the lens
4. A glass prism of refractive index 1.5 is immersed in 9. Anisosceles prism of angle 120° has
water (refractive index 4/3). A light beam incident a refractive index 1.44.Two parallel •
normally on the face AB is totally monochromatic rays enter the
reflected to reach on the face BC if: prism, parallel to each other in air,
as shown. The rays which emerge :
(a) sinO > —
9 from the opposite faces :
(a) are parallel to each other
(b) — < sin0 < —
3 9 (b) are diverging
(c) sinO < -
(c) make anangle 2[sin"^ (0.72) - 30° ]with each other
3 (d) make an angle 2sin"^ (0.72) with each other
(d) none of these (IIT 1981; 2M) (IIT 1995; 2M)
5. A ray of light from a denser 10. The focal lengths of the objectiveand the eye piece of a
medium strikes a rarer
compound microscope are 2.0 cm and 3.0 cm,
medium at an angle of m/fffm/im/. ///////////////// respectively. The distance between the objective and
incidence i (see fig.). The the eyepiece is 15.0 cm. The final image formed by the
reflected and refracted rays eyepiece is at infinity. The two lenses are thin. The
make an angle of 90° with distance, in cm, of the object and the image produced
[330
by the objective, measured from the objective lens, are plate as shown in figure. The observed interference
respectively: (HT 1995; 2M) fringes from this combination shall be •
(a) 2.4 and 12.0 (b) 2.4 and 15.0
(c) 2.0 and 12.0 (d) 2.0 and 3.0 n 1 11:
11. In an interference
arrangement similar to
Young's double-slit (IIT 1999; 21V0
(a) straight
experiment, the slits Sj and d/i
(b) circular
S2 are illuminated with
coherent microwave sources. (c) equally spaced
each of frequency 10^ Hz. d/2 (d) having fringe spacing which increase as we go
The sources are synchronized , outwards.
to have zero phase _, 17. Apointsource oflightBisplaced at ^
difference. The slits are " " a distance Tin front of the center of '>d
separated by a distance d =150.0m. The intensity/(G) a mirror ofwidth d hung vertically 1
is measured as a function of0, where 8 is defined as on a wall. A man walks in front of
shown. IfIQ is the maximum intensity, then /(0) for the mirror along a line parallel to
0 <0 <90° is given by: (IIT 1995; 2M) the mirror at a distance 2L from it <-
2L

Caj /(0) = 7o/2for0 = 3O° as shown in figure. The greatest distance over which
(b) 7(0) = 7o/4, for 8 = 90° he can see the image ofthe light source in the mirror
(c) 7(0) = /o, for0 = 0°
(nT2000;2M)
(d) 7(0)is constant for all values of 9 (a) d/2 (b) d (c) 2d (d) 3d
12. Consider Fraunhofer diffraction pattern obtained with 18. Ahollow double concave lens is made of very thin
a single slit illuminated at normal incidence. At the
transparent material. It can be filled with air or either
angular position of the first diffraction minimum, the oftwo liquids Li or7,3 having refractive indices and
phase difference (in radian) between the wavelets
fromtheopposite edges ofthe slit is: (IIT 1995; 2M) P2 respectively (p2 >Pi >1). The lens will diverge a
parallel beam oflight ifit is filledwith:
(a) 7c/4 (b)7i/2 (c)271 (d) tu
13. An eye specialist prescribes spectacles having (IIT 2000; 2M)
•combination ofconvex lens offocal length 40 cm in (a) air and placed in air
' contact with a concave lens offocal length 25 cm. The (b) air and immersed in
power ofthis lens combination, indioptre, is : (c) Li and immersed inI2 •
(11X1997; IM) (d) 7,2 and immersed in
(a) -hl.5 (b)-1.5 (c)-t-6.67 (d)-6.67 19. Adiverging beam oflight from apoint source Shaving
14. Yellow light is used in a single slit diffraction divergence angle a, falls symmetrically on a glass slab
experiment with slit width of 0.6 mm. Ifyellow light is as shown. The angles ofincidence ofthe two extreme
replaced by X-rays, then the observed pattern will rays are equal. Ifthe thickness ofthe glass slabist and
reveal, (HT 2^) the refractive indexp, then the divergence angle ofthe
(a) that the central maximum in narrower emergent beam is:
(b) more number of fringes
(c) less number of fringes
(d) no diftfaction pattern
15. Aconcave lens ofglass, refractive index 1.5, has both
surfaces ofsame radius ofcurvature R. On immersion (HT 2000; 2M)
ina medium ofrefractive index 1.75, itwill behave as (a) zero (b) a
^•
(a)
(IIT 1999; 2M)
convergent lens offocal length 3.5i? (c) sin^'^f— (d) 2sin-ifi
VTI. ..
(b) convergent lens offocal length 3.0R 20. Arectangular glass slab ABCD of refractive indexpj is
(c) divergentlensoffocallength3.5i? immerged in water of refractive index p2(pi >p2)• A'
A (d) divergent lens offocal length 3.0J? ray oflight is incident at the surface AB ofthe slab as
16. Athin slice is cut out of aglass cylinder along aplane shown in the figure. The maximum value of the angle
parallel to its axis. The slice is placed on a flat glass
PREVIOUS YEAR PROBLEfVlS 3^
of o^max such that the ray comes out only from the 25. A ray of light passes through four transparent media
other surface CD is given by: with refractive indices andp4 as shown in
the figure. Thesurfaces of all mediaare parallel. If the
emergent ray CD is parallel to the incident ray AB, we
must have : (HT 2001; 2M)
irHEHEEEis

ill

B IRi 1
(IIT 2000; 2M) /
iiiiiil
Ca) sin

(a) Pi =p2 (b)p2 =^3


(b) sin IXi cos (C) P-3=P-4 Cd)p4
26. A given ray of light
(c) sin suffers minimum
deviation in an
equilateral prism P.
Cd) sin
Additional prism Q
and R of identical shape and of the same,material as
21. In a compoundmicroscope, the intermediate image is:
P are now added as shown in the figure. The ray will
(IIT 2000; 2M)
now suffer : (iiT 2001; 2M)
(a) virtual, erect and magnified
(a) greater deviation
(b) real, erect and magnified
(b) no deviation , , ,
' (c)" real, inverted and magnified
(c) same deviation as before .
(d) virtual, erect and reduced
-(d) total internal reflection i •
22. In a double slit experiment instead of taking slits of
equal widths, one slit is made twice as wide as the 27. An observer can see through a
other. Then, in the interference pattern: ' pih-hole the top end of a thin
(IIT 2000; 2M)
rod of -height h, placfed ' as' "
(a) the intensities of both the maxima arid the minima shown in the figure. The beaker ' 3h
increase - • • •. height is 3h and its radius' h.
(b) the interisity of the maxima'increases and the Whenthe beaker is filled with a
' minima has "zerointensity liquid up to a height 2h, he can • -
"(e) the intensity of the maxima decreases' and that of see the lower end of the rod. Then the refractive index
the minima increases oftheliquid is': (IIT 2002; 2M)
(^) the intensity" of the maxima decreases and the
minima has zero intensity
c.)| <» I (c)
'2
(d)|
23. In Young's double slit experiment, 12 fringes are 28. which one of the following spherical lenses does not
observed to be formed in a certain segment of the exhibit dispersion? The radii of curvature of the
screen when light of wavelength 600 nm is used. If the surfaces of the lenses are as given in the diagrams.
wavelength of light is changed to 400 nm, number of (IIT 2002; 2M)
fringes observed in the same segment of the screen is
givenby: (IIT 2001; IM)
(a) (b) «
(a) 12 (b) 18 (c) 24 ' (d) 30
24. Two beams of light having intensities I and 41 interfere
to produce a fringe pattern on a screen. The phase
difference between the beams is 7t/2 at point A and k at
point B . Then the difference between the resultant (c) (d)
intensities at A and Bis : (IIT 2001; 2M)
(a) 2/ (b) 41 (c) 5/ (d) 71
332
OPTICSl
29. In the ideal double-slit experiment, when aglass-plate 34. Abeam ofwhite light isincident on glass'airinterface
(refractive index 1.5) of thickness t is introduced in from glass to airsuch thatgreen light justsuffers total
the path of one of the interferring beams (wavelength internal reflection. The colours ofthe light which will
A,), the intensity at the position where the central come outto airare : (nx 2004; 2M)
maximum occurred previously remains unchanged. (a) violet, indigo, blue
The minimum thickness ofthe glass-plate is :
(b) all colours except green
(IIT 2002; 2M) (c) yellow, orange, red
(a) 2X (b)2V3 (c)V3 (d)A
(d) white light
30. Two plane mirrors A and
B are aligned parallel to
"2/3 m 35. Anequilateral prismis placedon a horizontal surface.
each other, as shown in A ray PQ is incident onto it. For minimum deviation :
the figure. A light ray is 0.2 m 30
incident at an angle 30°
at a point just inside one Mmimuummmimmminmm}
end of A. The plane of imnmiutmimmii (IIT 2004; 2M)
incidence coincides with the plane ofthe figure. The (a) PQ is horizontal
maximum number of times the ray undergoes (b) QR is horizontal
reflections (including the first one) before it emerges
(c) BS is horizontal
(HT 2002; 2IVI) (d) any one will be horizontal
(a) 28 (b) 30 (c) 32 (d) 34
31. Aray oflightis incident at the glass-water interface at 36. Apoint object is placed at the centre of a glass sphere
an angle i. It emerges finally parallel to the surface of of radius 6 cm and refractive index 1.5. The distance
water. Then the value ofp^ would be : ofvirtual image from the surface is: (IIT 2004; 2M)
Air " (a) 6 cm (b)4cm
(c) 12 cm (d) 9 cm '
x\ Ikv" 4/3 37. A source emits sound of frequency 600 Hz inside
Water
water. The frequency heard in air will be equal to
(velocity of sound in water = 1500 m/s, velocity of
sound inair= 300m/s) ^ ,(ipT 2004; 2M)
(11X2003; 2IVI) (a) 3000 Hz (b) 120 Hz
(a) (4/3)sini (b) 1/sini
, (c) 600 Hz . ^(d) 6000Hz
(c) 4/3 (d) 1
38. Monocromatic light of wavelength 400 nm and 560
32. The size of the image of an object, which is at infinity,
as formed by a convex lens offocal length 30 cm is 2 nm are incident simultaneously and normally on
cm. If a concave lens offocal length 20 cm is placed double slitapparatus whose slit separation is0.12 mm
and screen distance is Im. Distance between areas of
between the convex lens and the image ata distance of
26 cmfrom the convex lens, calculate the new size of
total darkness will be: (nx 2004; 2M)
the image. (UX 2003; 2M) (a) 4 mm (b) 5.6 mm
(c) 14 mm (d) 28 mm
(a) 1.25 cm (b) 2.5 cm
(c) 1.05 cm (d) 2 cm 39. A convex lens is in contact with concave lens. The
33. In the given diagram, CP 0 magnitude of the ratio of their focal lengths is 2/3.
represents a wavefront Q
Their equivalent focal length is30 cm. What are their
immmmmR
and AO and BP, the individual focal lengths? (in 2005; 2M)
corresponding two rays. (a)-15,10 (b)-10,15
Find the condition in 0 (c) 75,50 (d)-75,50
for constructive 40. In Young's double slit experiment intensity atapoint is
interference at P (1/4) ofthe maximum intensity. Angular position of
between the ray BP and this point is: (m 2005; 2M)
reflected ray OP: Cirr 2003; 2M) (a) sin ^iXjdi) (b) sin ^(X/2d)
(a) cos9 = 3X/2d (b) COS0 = X/4<i (c) sin ^(X/3d) (d) sin ^(X/4d)
(c) sec8-cos0 = X/d (d) sec0-cos0 = 4X/d
iPREVIOUS YEAR PROBLEMS 333

41, A container is filled with I (b) area of image is nr^ and area is directly
water (|x = 1.33) upto a I proportional to
height of 33.25 cm. A ^ (c) intensity of image increases if / is increased
' . ^ concave mirror is placed (d) if lower half of the lens is covered with black paper
15 cm above the water |' area will become half
level and the image of an
45. A ray of light travelling in water is incident on its
object placed at the :§ rp=133' surface open to air. The angle of incidence is 0, which
bottom is formed 25 cm n
is less than the critical angle. Then there will be :
below the water level, p VMI
(IIT JEE 2007; 3M)
-' ' Focal length of the mirror
:0:i (a) only a reflected ray and no refracted ray
• is: '
(b) only a refracted ray and no reflected ray
(IIT 2005; 2M)
(c) a reflected ray and a refracted ray and the angle
(a) 15 cm (b) 20 cm
between them would be less than 180®- 20,
(c) -18.31cm (d) 10 cm
(d) a reflected ray and a refracted ray and the angle
.42.' Focal length of the plano-convex lens is 15 cm. A small
between them would be greater than 180° - 20
object is placed at A as shown in the figure. The plane
46. In an experiment to determine the focal length (/) of a
surface is silvered. The image will form at: '
concave mirror by the u-v method, a student places the
object pin A on the principal axis at a distance x from
the pole P. The student looks at the pin and its inverted
image from a distance keeping his/her eye in line with
PA. When the student shifts his/her eye towards left,
the image appears to the right of the object pin. Then,
(IIT 2007; 3M)
A (a) X < / (b) / < X < 2/
..,k: (IIT 2006; 3 M) (c)x = 2/ (d)x>2/
-".(a) 60 cm to the left of lens 47. A light beam is travelling from region I to region IV
••'(b) 12 cm to the left of lens (Refer figure). The refractive index in regions I, II, III
' • (c) 60 cm to the right of lens
(d) 30 cm to the left of lens
and IVare no^~^ respectively. The angle
43. The graph shows, rdationship object distance and of incidence 0 for which the beam just misses entering
•' ' image distance for an equiconvex lens. Theh, focal region IVis : , (IIT 2008; 3M)
" length ofthe lens is : (IIT 2006; 3 M)
i.«l "0 Ho .90° no
i::!; cm
6
"30
0.2m. ,0.6m.

(a) sin -1 (b) sin

(c) sin ^ j (d) sin ^


UCm-31 -30 -20 -10 0(-9, +9)
48. Two beanis of red and violet colours are made to pass
(a) 0.50±0.05cm (b) 0.50±0.10cm separately through a prism (angle of the prisni is 60°).
(c) 5.00 ±0.05cm (d) 5.00 ± 0.10cm In the position of minimum deviation, the angle of
44. Rays of light from sun fall on a biconvex lens of focal refraction will be : (IIT 2008; 3M)
length / and the circular image of suii of radius r is (a) 30° for both the colours
formed on the focal plane of the lens. Then : (b) greater for the violet colour
(IIT 2006; 3M) (c) greater for the red colour
O '
(a) area of image is 7cr and area is directly (d) equal but not 30° for both the colours
proportional to / 49. A ball is dropped from a height of 20 m above the
surface of water in a lake. The refractive index of
[334 •
OPTICS
water is 4/3. Afishinside the lake, in the line of fall of 5. Ashort linear object oflength blies along the axis ofa
the ball, islooking at the ball. At aninstant, when the concave mirror of focal length / at a distance u from
ball is 12.8 mabove the water surface, the fish sees the the pole of the mirror. The 'size of the image is
speed ofball as [Takeg =lOm/s^]: (11X2009; 3M) approximately equal to: (IIT1988;2M)
- (a)9m/s (b)12m/s r\V2 \V2
(c) 16m/s Cd) 21.33 m/s (a) (b)b
50. A biconvex lens of focal length 15 cm is in front of a f
plane mirror. The distance between the lens and the
(c) b (d)b
mirror is 10cm. Asmall object is kept at a distance of . /
30cm from thelens. The final image is: 6. A beam of light consisting of
(11X2010; 3M) red, green and blue colours is
(a) virtual and at a distance of 16 cm from the incident on a right angled
• ' mirror ^
prism, as shown in figure.
(b) real and at a distance of 16 cm from the mirror The refractive indices of the
(c) virtual and at a distance of 20 cm from the material of the prism for the
-'•'ihirror " above red, green and blue wavelengths are 1.39, 1.44
(d) real and at a distance of'20 cm from the mirror and 1.47respectively. The prism will:-
(nx 1989; 2IVI)
(a) separate part of the red colour from the green and
B More than One Alternatives is/are Correct blue colours
1. Aconvex lens offocal length 40 cm is incontact with a (b) separate part ofthe blue colour from the red and
concave lens offocal length 25 cm. The power ofthe green colours ' '
' combination is : (11X1982; 31VI) (c) separate all the three colours from one another
' (a) - T.5 dioptre (b) - 6.5 dioptre Cd) not separate even partially any colour from the
, . (c) +6.5 dioptre Cd)+6.67 dioptre other two colours
2. In young's double slif experiment, the interference 7. An astronomical telescope ' has an angular
^ ' pattern is found to have an intensity ratio between the magnification ofmagnitude 5 for distant,objects. The
bright arid dark fringes as'9. This irfiplies that; ,separation between the'objective And the eyepiece is
, , , (irr 1982; 3IVI)
36 cm and the final image is fomie'd at infinity. The
(a) theintensities at thescreen due to the two slits are •focal length f^-of the objective and the focal length /g
, . 5 units arid 4-units respectively ' ' ' ofthe eyepiece are (jrr 1989; 2M)
(b) the intensities at the screen due toThe two slits are (a) /o'= 45cmand/e =-9crii'" '^- '
, . ^ units and 1 units respectively , , Cb)/o = 50'cm and JA =10 cm .
, (c) the amphtude ratio is 3 . . Cc), A,, =,7.2 cm a'nd'/g = 5cni
(d) the amplitude ratio is 2 ^ A (d) /o = 30 cm and = 6 cm
3. White light is used to illuminate the two slits in 8. Athin prism with angle 4? and-,n^ade fromiglass of
Young's double slit experiment. The separation refractive index 1.54 is combined with another thin
betwen the slits is b and the screen is at a distance prism P2 made from glass of refractive index 1.72 to
dC> b) from the slits. At apoint on the screen directly produce dispersion without deviation. The angle of
in front of one of the slits, certain wavelengths are the prismP2 is ; (nr 1990; 2M)
missing. Some ofthese missing wavelengths are ; Ca) 5.33° Cb) 4° (c) 3° (d) 2.6°
(nx 1984; 2M) 9. Aplanet is observed by an astronomical refracting
(a)X = — (h)X=~ telescope having anobjective offocal length 16mand
d d 3d 3d aneyepiece offocal length 2cm. (IIX 1992; 21VI)
4. A converging lens is used to form an image on a (a) The distance between the objective and the
screen. When the upper half of the lens is covered by eyepiece is 16.02 rii
an opaque screen; (UT 1986;2M) (b) The angular magnification ofthe planet is - 800
(a) halfthe image will disappear (c) The imageof the planet is inverted
(b) completeimagewillbe formed (d) The objective is larger than the eyepiece
(c) intensity of the image will increase 10. Two thin convex lenses'of focal lengths and /2 are
(d) intensityof the imagewill decrease seperated by a horizontal distance d (where d<A,
d < /2) and their centres are displaced by a vertical, 15. A parallel monchromatic beam of light is incident
separation A a^ shown in the fig. normally on a narrow slit. A diffraction pattern is
Ay" formed on a screen placed perpendicular to the
4 i direction of the incident beam. At the first minimum of
the diffraction pattern, the phase difference between
the rays coming from the two edges of the slit is:
(IIT 1998; 2M)
(a) 0 (b) jr/2 (c) tc (d) 2k
Taking the origin of coordinates 0, at the centres of the 16. A ray of light travelling in a transparent medium falls
first lens, the x and y coordinates of the focal point of on a surface separating the medium from air at an
this, lens .system, for a parallel beam of rays coming - angle of incidence of 45®. The ray undergoes total
from the left, are given by: (IIT 1993; 21VI) inemal reflection. If p. is the refractive index of the
(a) x=-^~ y = A medium with respect to air, select, the possible
/1.+ /2 value(s) of p. from the following: (IIT 1998; 2M)
(b)
/l(/2+d) (a) 1.3 (b)1.4. (c) 1.5 (d) 1.6
.y =
/l+/2-d" fl +/2 17. In a Young's double slit experiment, the,separation
A(/,-d) between the two slits is d and the wavelength of the
- . /l+/2-d /l+/2-d light is X. The intensity of light falling on slit 1 is four
times the intensity of light falling on slit 2. Choose,the
fd) X=-^1-^2 +^C/i - d) .y = 0 correctchoice(s). (im9'98; 2M)
/l + /2 ~
11. A'diminished image of an object is to be obtained on a (a) If d = X, the • screen will contain only one
maximum
screen 1.0 m from it. This can be achieved by
(b) If X<d<2X, at least one more maximum
, ' " appropriately placing: (IIT1995; 2M)
(besides the central maximum) will be observed
(a) a concave mirror of suitable focal length
(b) a convex mirror of suitable focal length
! on the screen ^j
(c) If the intensity of light falling on slit 1 is reduced
(c) a convex lens of focal length less than 0.25 m
so that it becomes equal to that of slit 2, the
-(d) a concave,lens of suitable.focal length
12. Which of the following form(s) a virtual and erect intensities of the observed dark and bright^fringes
will increase
. , ,image for all positions of the object? (IIT 1996; 2M) r ^ ^ * '

_ . (a) Convex lens (b) Concave lens


(d) If the iritensity of light falling on slit 2 is increased
(c) Convex mirror (d) Concave mirror
, so that it becoihes equal to that of. slit 1, the
13. A real image of a distant object is formed by a plano intensities of the observed dark and bright fringes
will increase ' ' ' • .
convex lens on its principal axis. Spherical aberration:
(IIT 1998; 2M)
18. A spherical surface of radius of curvature R separates
(a) is absent air (refractive index 1.0) from glass (refractive index
'' '(b) is smaller if the curved surface of the lens faces the 1.5). The centre of curvature is in the glass. A point-
object ' object P placed in air is found to have a real image Q in
' (c) is smaller if the plane surface of the lens faces the • the glass. The line PQ cuts the surface at a point O, and
object PO = OQ. The distance PO is equal to :
(d) is the same whichever side of the lens faces the (IIT 1998; 2M)
object (a) 5R (b 3R (c) 2R (d) 1.5R
14. A concave mirror is placed on a horizontal table, with 19. A ray OP of monochromatic light q
its directed veticaUy upwards. Let O be the pole of the is incident on the face AB of prism
mirror and C its centre of curvature. A point object is ABCD nearvertex Bat anincident J
placed at C. It has a real image, also located at C. If the angle of 60° (see figure). If the ;
mirror is now filled with water, the image will be : refractive index of the material of \
(UT 1998; 2M) the prism is Vs, which of the I
following statement(s) is (are)
(a) real, and will remain at C correct? (UT 2010; 3M)
(b) real, and located at a point between C and 00 (a) The ray gets totally internally reflected at face
(c) virtual, and located at a point between C and 0 CD.
(d) real, and located at a point between C and O (b) The ray comes out through face AD.
336 optics

(c) The angle between the incident ray and the 4. In Young's double-slit experiment, the two slits act as
emergent ray is 90°. coherent sources of equal amplitude A and of
Cd) The angle between the incident ray and the wavelength X . In another experiment with the same
emergent ray is 120°. set-up the two slits are sources of equal amplitude A
and wavelength X, but are incoherent. The ratio of the
intensity of light at the midpoint of the screen in the
B True/False
first case to that in the second case is
1. The setting sun appears higher in the sky than it really (UT 1986; 2M)
is. : (11X1980) 5. A thin lens of refractive index 1.5 has a focal length of
2. The intensity of ligth at a distance r from the axis of a 15 cm in air. When the lens is placed in a medium of
long cylindrical source is inversely proportional to r. refractive index 4/3, its focal will become cm.
(IIT 1981; 2M) (IIX 1987; 2M)
3. A convex lens of focal length 1 metre and a concave 6. A point source emits waves equally in all directions in
lens of focal length 0.25 metre are kept 0.75 metre a non-absorbing medium. Two points P and Q are at a
apart. A parallel beam of light first passes through the distance of 9 metre and 25 metre respectively from the-
convex lens, then through the concave lens and comes source. The ratio of amplitudes of the waves at P and Q
to a focus 0.5 m away from the concave lens. is (IIX 1989; 2M)
(11X19.83; 2M) p.i=r.o "A
7. A slab of a
[12—2.0
^ 1^3= V4.'
4. A beam of white ligth passing through a hollow prism material of
0
gives no spectrum. (IIX ,1983; 2M) refractive index

5. The two slits in a Young's double slit, experiment are 2 shown in fig. 15 cm
-•M

illuminated by two different sodium lamps emitting has a curved


D
light of the same wavelength. No interference pattern surface APE of. 20 cm
"H. -

will be observed on the screen. (IIT. 1984; 3M) radius of


curvature 10 crn and a plane surface CD. On the left of
6. In Young's double slit experiment, performed with a
• source'of white light, only black and white fringes are APE is air and on the' right of CD is water with
observed. ' (11X1987; 2M) refractive indices as given in the figure. An object Q is,
7. A'parallel beam of white light fells on a combination of placed at a distance of 15 cm from the pole P as sho.wn.
a "concave and a convex lens,' both of" the same The distance of the final image of 0 from P, as viewed
material. Their focal lengths are 15 crii and 30 cm fromtheleftis (HX 1991; 2M)
respectively, for the mean' wavelength in .white light. 8. A thin rod oflength ~ is placed along the optic axis of a
On the other side of the lens system, one sees coloured 3
patterns with violet colour at the outer edge. concave mirror of focal length / such that its image,
(IIT!i988; 2M) which is real and elongated, just touches the rod. The
magnification is (IIX 1991; IM)

E Fill in the Blanks 9. A ray of light undergoes deviation of 30° when


incident onanequilateral prism ofrefractive index V2.
1. Alightwave offrequency 5x 10^Hz enters a medium The angle made by the ray inside the prism, with the
of refractive index 1.5. In the medium the velocity of base of the prism is (IIX 1992; IM)
the light wave is and its wavelength is
10. The resolving power of electron microscope is higher
(HX 1983; 2M)
than that of an optical microscope because the
2. A convex lens A of focal length 20 cm and a concave wavelength of electrons is than the
lens B of focal length 5 cm are kept along,the same axis • wavelength ofvisible light. (11X1992; IM)
with a distance d between them. If a 'parallel beam of
11. A ray of light is incident normally on one of the faces of
light falling on A leaves B as a parallel beam,"then d is
aprism-of apexangle,30° and refractive index V2. The
equal to cm. (11X4985; 2M)
angle of deviation of the ray is degree.
3. A monochromatic beam of light of wavelength 6000 A (nX1997;2M)
in vacuum enters' a medium of refractive index 1.5. In
12. Two thin lenses, when in contact, produce a
the medium, its wavelength is "its frequency
combination of power -HO dioptre. When'they are
is (IIT 1985; 2M)
0.25 m apart, the power reduces to -1-6 dioptre. The
PREVIOUS YEAR PROBLEMS
337
focal length of the lenses are m and 2. The phases ofthelight wave afc,d, eand'/ are
(nT1997;2M) (})e and (j)f respectively. It is given that ^
13. A slit of width d is placed in front of a lens of focal (a), 0c.cannot be equal to,0^,, •
length 0.5 m andisilluminated normally with light of (b) 0d can be equal to 0^
wavele^gth 5.89x10"^ m. The first diffraction (c) (0d-0^)is equal to (0c - 0e)
minima on either side of the central diffraction (d) (0(j - 0c) is not equal to (0^ - 0g)
maximum are separated by2 x 10"^ m. Thewidth dof 3. Speed of light is :
theshtis ....m. (1171997; IM) (a) the same in medium-1 and medium-2
(b) larger in medium-1 than in medium-2
Assertion and Reason- (c) larger in medium-2 than in medium-1
(d) different at b and d
Instructions: Each question contains statement-1
(assertion) and statement-2 (reason). Ofthese statements, Matching Type Problem^
mark correct choice if:
(a) Statements-1 and 2 are true and statement-2 is a 1. A simple telescope used to view distant objects has
correct explanation for statement-1 eyepiece^afid objective lenses of focal lengths /g and
(b) Statements-1 and 2 are true and statement-2 is not a fo respectively. Then (IIT 2006; 6M)
' co^ect explanation for statement-1 \ ColumnI \ \ Column 11
(a) Intensity , of light (P). Radius.of aperature
(c) Statement-1 is true, statement-2 is false -received by lens '
(d) Statement-1 is false, statement-2 is true. (b) Angular^magnification Cq). Dispersion of lens
1. Statement-1The formula connecting u,u and/for Cc3 LengdTof telescope Cr). Focal length of^bjective
a spherical mirror is valid only for mirrors whose sizes lens and eyepiece lens
are very small compared to their radii of curvature. Cd) Sharpness of image (s)* Spherical aberration
Statement-2 : Laws of reflection are strictly valid
forplanesurfaces, but notforlargespherical surfaces. 2. An optical component and an object Splaced along its
(IIT 2007; 3M) optic axis are given.in Column 1.The distance between
the object and the component can be varied. The
properties of images are given in column II: Match all
Comprehension Based Problems
the,properties of images from Column II with the
The figure shows a appropriate components given in Column I.
surface .XT (IIT 2008; 8M)
separating two
transparent media,
medium-1 X Column I Column n

medium-1 and
• s
'medium-2. The lines (a) Real imaige
medium-2 •Cp)
ab and cd represent
wavefronts of a light
wave' travelling in medium-1 and incident on XT. The
lines ef and gh represent in medium-1 and incident on
(b) Cq) Virtual image
XY. The lines ef and gh represent wavefronts of the
light wave in medium-2 after refraction.
(IIT 2007; 3 x4M)
! s
1. Light travels as a : (c) (r) Magnified image
(a) parallel beam in each medium
Cb) convergent beam in each medium '
(c) divergent beam in each medium (d) (s) Image at infinity
(d) divergent beam in one medium and convergent
beam in the other medium.
1338
OPTICS
3. Column-I shows four situations of standard Young's \ CoIumn-I \ V Column-n.
double slit arrangement with the screen placed far
(a) ril <P2 • "(p) /\ ^ •
away from the slits and 52- In each of these cases
SiPq = 52^03 ~ and P3
h2'
^1^2 ~ ^2^2 ^/3 where Xis the wavelength of the 1
light used. Inthe cases B, Cand D, a transparent sheet 1
V - i ^
ofrefractive index p, and thickness t is pasted on slit Cb) V '
Pl>P2 Cq)
! \ ^ 1
S2. The thicknesses of the sheets are. different in
different cases. The phase difference between the light ^j- Hi|
waves reaching a point P on the screen from the two
slits isdenoted byeCPJ andtheintensity by/(P). Match
f
CO P2 =P3, • Cr)
each situation given in Column-I with the statement
(s) in Column-n vdid for that situation.
1 ^^3 7 I ' !
\ Column-I \ \ mliimn-TT
L J
II (d) P2>ri3: Cs)
(a) S2I Pz '
1 (p) 5(Po) =00

M
r
1 — —
Po'
I r
rH2\
!
.. 1

(b) 1
(t).
(q)" 6CI\) = 0
~Pl"'

Hi j

Cc) (p-l}/: = V2 '

(r) alritieger Answer-TypePrbblems


'TR«Wgr>l«*#an.'»!iamm'itTnnniiV • .
'pI>
p^. ' 1. Thefocal lengthof a thin biconvex lensis20 cm.When
p 5" an object is irioved from a distance of 25 cm' infront of
r . . !v
1, it to 50 cm, the magnification of its image changes
from 77125 to mjo: The ratio is; (HT2010; 4M)
(d) Cp-lX = 3V 4 (s) nPo)>i(.Pi) .. '"so
1^2 I >2^" . - 2. Image of an object approaching a convex mirror of
Pi\' radius oif curvature 20 m along its optical axis is
Pol
observed to move from — m. to — m in 30 seconds.
: 1 3 7
What is the'spepd pf.the object in km perhour ?
(0 (nT2010;4M)
nP2)>nPi) 3. Alarge glass slab (p=5/ 3) ofthickness Scmisplaced
overa point sourcepflighton a planesurface.ltisseen
4. Two transparent media of refractive indices pj and p3 diat light emerge out of the top surface of the slab
have a solid lens shaped transparent material of from a circular area of radiusR cm.What is the value
refractive index p2between them as shown infigures
in Column-ll. A ray traversing these media is also ofP? (in 2010; 4M)
shown in the figure. In .Column-I different
relationships between pi,p2 arid p3 aregiven. Match
them to the ray diagrams shownin Column-II.
(IIT2010;6M)
PREVIOUS YEAR PROBLEMS

ANSWERS

Only One AlternativSiii^^l

I. (b) 2. (a) " 3. (d) 4. Ca) 5. Ca) 6. Cc) 7. Cd) 8. CO 9. CO 10. Ca)
II. (c) 12. (c). 13. Cb) 14. Cd) 15. (a) 16. (a) 17. Cd) 18. Cd) 19. Cb) 20. Ca)
21. (c) 22. (a) 23. Cb) 24: Cb) 25. Cd) 26. CO 27. Cb) 28. CO 29. Ca) 30. Cb)
31. (b) 32. (b) 33. Cb) 34. Cc) 35. Cb) 36. Ca) 37. CO 38. Cd) . 39. Ca) 40. CO
41. (c) 42. (b) 43. Cc) 44. Cb) 45. Cc) 46. Cb) 47. Cb) 48. Ca) 49. CO 50. Cb)

SMw^than One Alternative is/ate Corr^


1. (a) 2. (b, d) 3. (a,c) 4. (b,d) 5. (d)
6. (a) 7. (d) 8. (c) 9. Ca,b, c, d) 10. (c)
11. (a,c) 12. (b,c) 13. Cb) 14. (d) 15. (d)
16. (c,d) 17. (a,b) 18. (a) 19. (a, b, c)

True/False

1. True 2. True 3. False 4. True 5. -True


6. False 7. True :

Fill in the Blante

1. 2x10® m/s, 4.x 2. d = 15. 3. 4000A, 5x10^'^Hz


4.2 • 5. 6p'. 6. 25/9 7. 30 cm to right of P
• 8. 1.5 9. zero , 10. Smaller 11. 15°
12. 0.125, 0.5 13. 2.945 X 10,-4^'

Assertion aihd Reeom

1. (c)

Comprehension Baed Problems

1. Ca) 2. (c) ; 3: Cb)

Matching Type P^blem^


1. a->p;b^r; c^r; d^p,q,s 2. a p, q, r, s; b q; c ^ p, q,r, s; d p, q, r, s
3. a-4 p,s; q; c-> t; d^ r, s, t 4; a-^p, r; b->q, s, t; c-^ p, r,t; d-^q, s

Integer Answer Type Problems

1. 6 2. 3 3. 6
ORTICS

5. (a) r + r'+90°=180°
I ••1M •••••• •
r'=90°-r .... (1)
i = f
i. (b) The intensity ofillumination at a surface is give n ... (2)
by
I cose Apply Snell's law.
E = : •:

E = 5x 10"^ lumen/cm^
= 5 XlO"^- X10'^ lumen/m^ i.—
Er^ 5x(2)' - 40 candela y\ 90° Denser
or I =
cos 9 cos 60° \ }^ Rarer
velocity (u)
2. (a) Wavelength (A,) = A/
frequencyC/) '
When the ray of light travels from air to glass, (f)
remains unchanged while velocity- (u) decreases.
Hence wavelength X should decreases.
Pp sini = pji sinr"
3. (d) In double slit experiment, fringe width p =
a or Pi, sinr = Pr sin(90°-r)
P2 £2 di = (2)(2) = 4or P2 =4Pi or iiK-^tanr
Pi v^2y

Fringe width will become four times. or sinSc - tanr


4. (a) Total internal reflection or Gc = sin~^(tanr)
occurs when the ray of light 6. (c)
travels from denser medium to
rarer medium and the angle of =(V¥-^/7)2=7
incidence is greater than 7:(d) For first dark fringe, in diffraction pattern,
critical angle. dsinG - X
At P, glass is denser and water
is rarer.

_ _
4/3 y :
Pw =
3/2
m^oma

sinC =where C = critical angle.

0 = C or siiiG =8/9 for critical angle.


g
For total internal reflection, sin0 > —
9
or dl^l =X
^PREVIOUS YEAR PROBLEMS

DX
or

2DX
or Distance =

or Distance -
2 X(2)(600x10"^)
m
.-3
1x10"
12 cm
= 2.4 X10"^ m = 2.4 mm
8. (c) Spherical aberration is . caused because the —15.011

marginal rays and the paraxial rays incident on a lens


are brought to focus at' different points. To reduce .*. Uoforobjectivelens = 15-3 = 12cm ... (1)
1 1" - 1 •
spherical aberration, the marginal rays are blocked by
using a circular "annular mask over the lens. /o
or
1 =

,12 Uq 2 Uq 12 2 ~ 12
• or . Uq = -2.4cm
• From objectives lens; Uq" = 2.4 cm(to left)
' Vq = 12 cm(toright)
9. (c) Zi = 30®,by geometry 11. (c) The intensity /(G) is
sini _ sin 30® measured as a function of 0.
Iq denotes the maximum
sinr 'sinr
intensity. 0 varies from 0® to
1 sin 30®
90®.

or
1.44
sinr = 0.72
sinr
V I(e) =/ocos2^|j' ;
6 = Zr - Zi
tArt-
When
^ 27u
0 = — x -^ =
dy 27tdtan0
X D X
T cos i{ T^dtanQ
7(0) = Iq

where A. = = 300m
10'
itxlSOxtariO
or /(0) = /oCOS^
. 300

or/(0)'= Iq cos^f —tanG


5 = sin"^(0.72) - 30® ^ 2

or C25) = 2[sin-^(0.72)-30®] . (a) When0 =30° 7(0) =Iq cos"|^^^^J


•(
or 0 = 2[sin"H0.72)-30®]
Option (a) is not correct.
0 is the angle at which the emergent rays from the
(b) When 0 = 90°, 7(0) = Iq cos^ (oo)
opposite faces meet each other.
10. (a) The eyepiece forms the final image at infinity. Its Option (b) is not correct.
object should therefore lie at its focus. (c) When 0 = 0, 7(0) = 7o cos^(0° ) •
F denotes focus of eyepiece. I denotes image formed by 7(0) = / q. The option (c) is correct.
the objective lens which serves as object for eye piece.
(d) 7(0) varies with value of 0. Option (d) is not
It should be at 3 cm from eyepiece.
correct.

Hence only option (c) is correct.


' r- • •' . .
' 1U

342 oPTiai

12. (c) The figure depicts


Fraunhofer' diffraction
pattern.
For P to be first minimum;
the phase difference
between OA or OB is tc.
Phase difference from
opposite edges = 27c radian
100
13. (b) For convex lens, power = = 2.5D
40
100
For concave lens, power P2 = - -4.0D
Bi is the image of source B formed by reflection at .
Power of combination = /^ + P2 = 2.5 - 4.0
= -1.5D
SimilarlyB2 is the'image of B formed by reflection at
14. (d) X-rays have very small wavelength of the order of M2.
lA. Light rays have wavelength ofthe order of6000 A-\. • NiW2-MiM2 = d
For diffraction pattern to be produced, by inddent •' . (vMiN^andM2iV2arenormals)
waves, the dimension of slit should be comparable to DB = bc -d/2 - '
the wavelength of waves. No diffraction pattern Will AMjCD and are similar'
be observed with X-rays which replace the yelldw • MjP _ CD / '
light. .MjWi B^Ni
15. (a) When glass lens is immersed in a medium, ts MiNi
refractive index is ^ or BiNi=(CD)

=
a
1.50
1.75
or B^Ni =^ '^| =d
•L
Pi
1 SimilarlyB2i^2 = • -
By lens maker's formula —= ('"pg -1) B1B2 = B^Ni + NiN'2 +
Ui R:'
^^2 ~ (d4- £i+,d)"= 3d '
ori =r^-lY-l-i or — - ](- 18. (d) According tip lens maker's formula
/ l7 Jl. R R, Jl rJ _! l_
-1 =
or / =Z^ =3.5R- 7. Rl \ R2
2
where = nefractive index of material of lens
Positive focal length shows that it behaves like
convergent lens of focal length 3.5 R. " "p ^ = refractiveindex of medium in which lens is
Note, when a glass lens is immersed in a denser, medium, ihe . ' dipped.;: • - ' '
nature of the lens is reversed. A convex lens behaves as a divergimt
P2
lens and a concave lens behaves as a convergent lens. Glass l^ns
=1.5) behaves like this pattern when dipped in carton
disulfide (°p.g =1.5)
16. (a) Locusof equal path difference consists in the hpes
running parallel to the axis of the cylinder. Hen c e
fringes obtained are straight.
17. (d) The ray diagram denotes the passage of light ^Z—/Za^
.Divergent,,. •Convergent
JSmRPROBLEMS ~ '343
^ ^ ^ ^ ^ , . . ,™-^_ -

tfjii >;|X;;, , the lens maintains its liature. 21; (c) The intermediate image in a compound
A concave lens bphaves as a Mivergient lens and a microscope is real, inverted and.magnified.
convex lens beiiaves as a convergent lens. /- 22. (a). When slits are of equal width,
IfHi the nature of lens is.reyersed, ' • + = 4i<T(say) ... (1)
Since n'2''> M-i > i the coiicave; lens will maintain its
. divergentna^re if it is filled with'L^ and immersed in (max zero ='zero ... (2)
h- '• ; .•U'":. • ' V. In secphd case; . . •
19. (b) V^en a;ray;is incidenr^bn a glass vslab, after /i ^ _ 1- . t" . -K([r+ fl~^^
refraction ^pngh slab,'the emergent ray is parallel to I2 W2 -2W 2; -^W^l +#2)
incident ray. • " ' " ' '- " <• ' .. . ' ^ , =^(7^ +7^)^=^/1(1 +^^
,, ; • , "=.K/i x(Z414)^ = 5.8K/1 = 5.8/3 ...(3)

: -KI.(-0:414)2 = 0.17/3, • ... (4)


fmM ^ fmax h^om eq. (3) ^d (1)
-./min ^/min from eq. (4) and ,(2),
Intensities of both> the maxima and the minima,
, s increase.' • >

Emergent rayfljCi is parallel tq SAi .,; • • 1'


23 (b) Fringe width p='^
Emergent rayB2^2 Isparallel to SA2^ ^. . 12 fringes of ki, occupy a segment of screen.
Angle between'and SA2 js equal to angle between • Let N fringes of 2 occupy the same segment. •
, BjCi andB^^r^rawnbaclwards.f
, Angle-= a.. pfi-' ' _ A 'V^ 12x600x10"^ .
N h£.Uj2m orN = •
20. (a) I^ys come out onlyfrom tHb surface CD when the d J • I d ^-9
400x10"
- light is incidenrat the surface'^ ofthe slab, pv . or - N= 18 " • ;•
It meains -that rays after refraction from AB'undergo 24. (b) , /($) = /l +I2 +'2^/i/2 C0S(j)
total internal reflection at surface AD of the slab.
ri +r2 = 90°' ' -
• • :/l -/./2=4/, ; ;•
^ AtAwheri^ = = / + 4/ = 5/ .... (1)
V
Againat BiWhen (j) = n, '. '
C'2)niin = critical angle 0c /g - / + 4/ +,2V/ x4/ ,cpsjc or Ig = 51 - 41
on, Ib = / ;• • ; ' •
sin(0cO ' •'
r/fi .= 5/ -/ ^br /^ -1^ = 41
25. (d) Apply Snell's law.-
max ~ ^9°~^,c ,1 , smi.
For refraction at apply sneli'^ law.l . •V2 =-7—
sinrj

p2. . sin(ri)niax-
'V>'i/jnax- ^ .sin(?0.-^ec).
2,,sinri'. >2^ c

H
<^L/2
B
' vo"- _ sin(ania^>;, smr2

• ; , cosfec). , sinra
-1^4
smi 1^2 ih '
or sinfttniax) = " ^
- ^- •' M-2. •' -c • ^2x^113 x^n4 =1
-1
or a. = sin —cosCGcO
L1^2
111 ^2:%3" •' .
dr. 114=411'.,
Putfc fromeq. (i;
26. (c) No refraction will occur when fay travels from P to
or a, -
.-i
sin cos (sin -1 . Q or Q to k because P, Q, i? are made of same
M-2 i .material. They are all identic^ prisms..
[344

31. (b) Apply Snell's law.


At glass- water surface,
g
sini
Pw = water
sinr

At water - air surface


w sinr
Deviation suffered in prism P remains as sue Pa =
sin 90®
27. (b).When liquid is filled in the beaker up to a height
2h, the image of the lower end A of the rod AS is X X°Pg =1
formed at B so that it is visible to the eye. X x° Pg = 1 or sini p„ = 1
DE=EB = 2h sinr sin90® ® ^
^i = 45° ..(1) or
=
sin I
sinr = ^ =i rvAc^-—2
[v AC 2 =(.GCf+(.GAr] 32. (b) Convex lens forms the image at Ii • I2 is at the
AC
second focus of convex lens. Size of 7^ = 2 cm.
sini
<s>
1^ = Iiacts as virtual object for concave lens. Concave lens
sinr
forms the image of at 72.

sin 45"^
or l^ =
i/Vs
IMIII
or

26 cni>
28. (c) In figure option (c), both the curved surfaces have
same R on the same side. Hence no dispersi Dn is 30 cm

exhibited.

For alens, y=(p. -1) Lens formula ; —


V u

/
R2
For concave lens,
'1'
For no dispersion d = 0
1 = -L i = _l = l
v// V 4~ - 20 . "v 20 4 20 ' 5
>

1 . oru = 5cm = Distance of72 from concave lens.


0 = (dp) or Ri = R2
. V siz'eof iniage 5
.*. Magnification = —= =—
29. (a) Path difference due to slab = (p. -1) t' u sizeof object 4
•sizeof image ,
or Cp - l)t - nX or •— ^=1.25
For minimum thickness t of plate, n shou d be
minimum Le. n = 1
or size of image due to concave lens = 2.5 cm.
.-. (p-l)t = X 33. (b)InAO?M,
PM
X X = COS0
or t = f = t = 2X OP
p-1
d
30. (b) Let N = no. of reflections or 0P =
COS0
N-x = 2V3
In ACOP,
2V3 00
or N =
COS 20 =
OP
0.2 m
2V3 d cos 20
or N = or 0C = (OP) cos 20 or 0C' =
0.2 tan COS0

Path difference = CO -1- OP -1- —


' 0.2 2
or W = 30
RRWIOUSYUR PROBLEMS ••••.345
d cos 29
+
d
1—
X (2n+ l)X.2 _ (,2m + X)X'2 2n + l _ X2
or
COS0 COS0 2 2 2m+ 1 Xtj
_ d(2cos^e-l) d^X 2n + l
+ or = — or lOn = 14m + 2
COS0 COS0 2 2m+ 1 400 5

- 2dcos0 + —
By inspection, the two solutions are
.2 (i) if mj = 2, =3
Forconstructive interference atP, betweenBPand OP, (ii) if m2 = 7, n2 = 10
Path difference = nX Distance between areas correspond to these points,
DX^ (2n2 +1) ~ (.2r^ +1)
.*.Distance AS =
or2dcos0H— - nX or 2dcos0.= n--\X
2 d

(.2h-l)X Put n2 = 10 and ni - 3,


or cos0 .= for n = 1, COS0 = ^
. Ad ' . Ad Ix (400x10"^) 21-7
AS =
-3
34. ic)X increases in the order V, I, B, G, Y,0,R. 0.1x10
•Y,0,R have Xgreater than that of G. or AS = 4x7 X10 ^m or AS = 28mm
V> I, B.have X less than that of G. 39. (a) Letfocallength of convexlens = /i =/(suppose)
• 1 , [_^—...
-3f
^ sine ^ ^/2 - focal length of concave lens = ——
1 ' > •
Also^loc— . I Equivalent focal len^ = 30 cm
• X !
/." sinC oc'X ' ' L -.1-Jl or— = _L
C is greater for Y, 0,R " 30 / 3/ "' F~I{'^F2
~ 30 3/
. Y, 0,R will come out to air. or / = 10cm = Focal length of convex lens.
35. (b) In equilateral prism,-the refracted ray QR runs -3
.-. Focal length of concave lens = — x 10 =-:15 cm
parallel to base. Base is horizontal. Hence QR is
horizontal. .-. Focal lengths are - 15 cm (concave lens) and 10
cm (convex lens).
40. (c) In Young's double slit experiment, intensity at any
point P is given by/ = cos^
'777777777777777777777777777777777777777771
where ^ denotes the phase difference at P.

36. (a) A Point object is at the 1=1, cos

centre of a glass sphere of


radius 6 cm. •
- The rays from the object fall
normally, on the surface of I fn
or cos - = - = cos —
the sphere and emerge [2) 2 U
-undeviated. When. drawn
^ n' . 2n 2jt V 27c
. . ba(^ards, they.meet at O. - • or — = — or (b = — or — Ax = :—
2 3 3 , X 3
tlie image will be formed at the centre O itself.
.•. Distance of virtual image from surface = 6 cm. or Ax = X/3
37. (c) Frequency of sound = 600 Hz inside water. Path difference Ax = S2Q = d sin 0
' Frequency of a wave depends on the source and it does or dsin0 = X/3
• not change during refraction when wave travels from 6=sm-MA
or
• one medium to, another. 3d
.% Frequency heard in air = 600 Hz. 41. (c) 0 = Object, I = Image of object.
38. (d) At;.the area of. total darkness, in double slit _ Real depth
I .apparatus, minima will occur for both the
Apparent depth .
wavelengths which are incident simultaneously and
; normally.
Apparent depth = or — - — =—
U
r*' tn / 10 5
1—1

From mirror: | j or / = +5 cm
ucra
Object distance = u " Ic Au = 0.1, from graph,
Av - 0.1, from graph
u = 15 + ££:f£ = 15 + 25 = 40cm 0 (-9, +9)
• 1.33 1 =1-1 |-31 -30 -20 -10 •
f V u u.cm

For maximum error in /,


errors are added.
25 A/ Au Au A/ •0.1 0.1
u = 15 + = 15 + 18.8 = 33.8 cm — 1 or :—~ + •
1.33
f u2 (5)2 (10)^ (10)'
1 +1 =1 or + 25x2x0.1
V u f -33.8 -40 / or Af.= or A/ = 0.05
100
or / =-18.31 cm
Focal length of lens = / ± A/ = 5.00 ± 0.05 cm.
42 (b) Focal length of equivalent mirror when a lens
Note. As the graph between v and u indicates, it is the curve of a
silvered ata surface is 1 =1 convex lens where u = -10cm,i; =10cm,/ = 5 cm. Object is placed
f. ffm at 2/ and real, inyerted and equal size image is formed at 2/, on the
other side of the lens.

44, (b) Rays of light from -sun are incident upon a


biconvex lens. Sun is at infinity and incident rays form
T
1 a parallel beam. • , -
The beam is brought to convergence in the focal plane
hs- •+I
-20-cm. of the lens. AB is image of the sun.
Here the mirror is a plane (surface) mirror.
/m = ~ •
1 2 1 2 15
— = — + ^ = — or/ = — cm
•/, 15 oo 15 2
Positive focal length means a converging mirror.
For mirror : As per, sign convention
u - -20 cm, / cm ..
. 2 '
1 1
1 11
1 +1- Let r = radius of real image of the suhr Sun is above
V u. f V -20 -15/2 principal axis of lens.
1 - 1 2 3 - 8 _ -5 The real' and inyeited image of diminished size is
u ~ 20 15 60 60 12 formed below the principal axis in the focal plane of
or V =-12cm the lens as shown in figure. '
Image is formed at 12 cm to the left of.lens. Area of circular image of sun = nr' CD
Note. There are three stages of formation of final image In the AOAB,0 is inclination of sun and is constant..
(i) Lens forms the image of object at A. tanG =y => r=/tane
(ii) fj acts as virtual object for mirrored surface.
(iii)J2>the image formed by mirror acts as virtual object for lens. Th' or Area of image = Tc (/tan 0)
• lens forms the final image. ' 2' 2
or Area of image = (tt tan 0)/ . ...(2)
43. (c) Using lens formula
From eq. (1) and (2), area of image = Jir and area is
1-1-1 proportional to / . •
f V u
45. (c) The ray is incident on the interface of'a rarer
1 _ 1 1
medium (air) from a denser medium (water), and the
f 10 -10 angle of incidence is less than the critical angle. The ,
way-will be partly refracted and partly reflected. Also,
^PREVIOUS YEAR PROBLEMS 347J
the Snell's law^-i sine^ = ^ 2 sin02tells that the angle 50. (b) _• For first. reflection through the lens
_ I6"crri "*7
of refraction willbe more than the angle of incidence,
on entering the air the ray will bend away from the 15 cm

normal. See the fig.


0 B 1 11 : •
Refracted ray ' ft ~ mm

water
1

I \
30 cm
6cm
10 cm
1 •. i 20 cm

Reflected ray 10 cm 30 cm
Incident rayij.

The anglebetween the reflected and the refractedray -20 cm-

is less than 18O°-20. 1 1 1

46. (b) On shifting the eye towards left, the image will V -30 +15
appear to the right of the object pin if the image V = +30cm (Real image at A)
. distance is more than the object distance and the Image A is the virtual object for plane mirror. Its
image is real. This happens when the object lies image is at B which acts as virtual object for the
between the focus and the centre of curvature lens
f < X <2f. 1 1 _ 1
47. (b) Asthe beam just misses entering the region IV, the V +10 +15
angle of refraction in the region IVmust be 90°. V = + 6: Real image.
Distance from mirror is 16 cm.
i90° "oi

0.2 ,0.6.m
B More than One Alternatives Is/are Correct
Application of Snell's law successively at different 1. (a) Power of convex lens = = 2.5D
interfaces gives 40

Uq sinB =^sin01= •^^sin82 =—sin 90° Power of concave lens =


25
= -4.0D
,2 6 8
Power of combination =J^ + P2 or P = 2.5 - 4.0
=> sinS = i or0 = sin"^— or P =-1.5 dioptre
8 8
48, (a) When the deviation of a ray • max _ 4h +7^
through a prism is, the' 2. (b,d)
' mm
minimum, it deviates equally at
both the refracting surfaces. In -Ih +^|h
this condition 0;^. = 82 and or

01 = 02 see in figure. -Jh _


or ~ =-Jh4^ or =4-4l^
Now 0j + 02 = A, so 01 = 02 or

=^ =^ = 30», Option (b) is correct.


2 2
Since both the beams are set for minimum deviation, Again
A • •'h1 ^ =Y
the angle of refraction will be 30° for both the colours. h
_2
4
49. (c) x' = or [•.•I °<= (amplitude)^] or —= —
, "rel 1 h 1
Option (d) is correct.
V V2x10xC20-128) 4 , ^ ,
V= = -2 2 i X- = 16m/s.
^rel 1 2
348 OPTI«j
3. (a, c) Those wavelengths are missing for which Ipath
difference Ax = odd —
I2J
-rr- S, •1
P is directly in front of slit . ly
b
y = C2n-l)--
b _ C2n - 1)X If angle of incidence is greater than C, the ray will
or
2~ 2b L - - suffer total internal reflection. Thus ifp is greater than
1.414, the colour will suffer total internal reflection.
V y = —,d = b,D = d
2 For red colour, = 1.39. It is less than 1.414.
For green and blue colours, p g = 1.44.
or X = where n = 1, ^ 3....
(2n - l)d Red colour will emerge out of prism. Blue and green
colours undergo total internal reflection at the faceAC
¥orn =l,Xi =^;Vorn
a
=2,X2=^
3d
of the prism.
7. (d) In an astronomical telescope when the object and
For n = 3, Xo = — final image are at infinity, M andL are given as shown:
Angular magnification M = fo Ife
4. (b, d) Image formed Separation between lenses I = /o + /e
will be complete when
upper half of lens is ^ =5 or /o =5/e
Je ,
blocked. Intensity of the f
image will decrease as fo+fe= 36 orS/e +/e = 36
the incident rays from or /e = 6cm
upper half are cut off. /o=5/e or /o=30cm
5. (d) The object is short. It lies along axis of concave Hence fo = 30cm, /g = 6m
mirror. Differentiate for small variations of v and u. 8. (c) Net deviation due to two prisms is zero.
.111
For a mirror, — + — = —
dv _0 S]^ + 62 = 0
V u f 2 2 ~
V u
^10^1 ~ 1) -^2(p 2 ~ 1) ~
dv Ai,Pi and 61 are for prism.
or ...(1)
du w A2,P2 and 82 are for prism P2.
, . V1 1 1 u u u -Ai(pi -1) -4® (1.54-1)
Agaim, - + - = - or - + - = — Ao —
V u f V u f (P2 -1) (1.72-1)
u u- f V f -4°x0.54
or — = or - = —— (2) Ao —
V f u u - / 0.72
From eq. (1) and (2), we get A2 = -3®
\2 The two prisms are
dv f
combined such that the
du u-f
position of one prism is inverted w.r.t. other
or dv = -(du) / where (-du) = b = Object prism.This is the indication of negative sign.
u-f 9. (a,b,c,d) Tube length of telescope = fo + fe
size Angular magnification = fo I fe
f (a) Distance between 0 and E lens = fo + fe
or dv = b
u-f .-. Distance = 16 + 0.02 or Distance = 16.02 m

6. (a) Let C = critical angle. Consider C = 45*^ (b) Angular magnification = j f^


-16
Li = = —-— = 1.414 M = = -800
sinC sin 45® 0.02

Rays fall normally on face AB. Negative sign shows that final image is inverted.
(c) The image formed by astronomical telescope is
inverted w.r.t. planet.
(d) Objective lens is larger than the eyepiece in terms
of aperture and focal length.
iREVibUS YEAR PROBLEMS
349
10. (c) A parallel beam of rays'coming from the left is 11. (a,c) An image obtained on screen is a real image.
incident on the first lens'of focal length fi. The lens Convex mirror and concave lens do not form real
forms theimage (/j) at itsfocus. This image ) serves image of an object. Options fb) and (d) do not hold-
• as virtual object for second lens. The second lens forms good.
the final image I at distance v2 froni the second lens. /" Options (a) and (c) hold good.
is the focal point of the lens system. For concave mirror, the object should be beyond Cto
(i) Letx denote the distance of/from O. form a real diminished image between F and C of
mirror.
Forconvex lens, minimum distance between an object
and its real image should be 4/.
Minimum(d) = 4/ or IM = 4/, or f = 0.25m
Convex lens of focal length less than 0:25 m can be
used when object is placed beyond (2/) and image is
formed-between 2/ and / on other side of the lens.
The image formation isillustrated bythefigures.

For lens L2,


I 1 1 I 1:
or 2F P
^^2 "2 /2 ^^2 (fl-d) /2
111
or — = — +
'^2 f2 fl-d
Options (a) and (c) are correct.
12. (b, c) Concave lens and convex mirror are diverging
in nature. Forall positions of objects, theyform virtual
I and erect image.
13. (b) Spherical aberration is smaller if the curved
surface of the lens faces the object. The total deviation
is shared between the curvedand the plane surfaces,
14. (d) The image formed wiU be real and located at a
point between C and O.
The ray diagram is depicted in the figure.
or uo =
_ f2(.fi-d).
.... (1)
/i + /2 ~ d Nonnal
atP
X = d + vo

or

• fl + /2 ~ d
or
^ _ /1/2 +d(fi - d)
(2) .
./l + /2 ~ ^
(iO j-coordinate ofthe focal point oflenssystem:
For lens L2,
/2(/i -d) 15. (d) The path differenceof rays
fl + f2 —d coming from the top of upper
Magnification = m = ——^
(fl-d) zone i.e. the upper edge of slit
"2
and the bottom of lower zone
/2 '2 _ /2'
or m = or i.e. the lower edge of the slit is
fl +f2-d ^ fl f2 —d 7i.
or ho = 4/^2 X corresponds to a phase
y = A - /12
fi+f2~d difference of 27r.
or * y ='A - 4f2 A(/i -d) . Hence the phase difference between the rays coming
or y = —^ —
fl +f2-d fl+f2-d from the two edges of slit is 27t.
OPTICS

16. (c,(l) For total internal reflection to occur, sin 60'


= V3 r = 30'
i > critical angle C smr

or sini > sinC From geometry, angle ofincidence at Qis 45'


1 X X *
or sin45''>— or-=>— or p. > V2-
P V2 AtQ, V3sinC45°) =-= > 1
V2 , '
or > 1.414
Possible value of |X can be (c) 1.5 or (d) 1.6. Thus total internal reflection takes place.
•s. 1
17. (a,b) If d = X, then maximum path difference path
difference is given by d sin 0) will be less than So
there will be only central maximum on the screen, 60'X^
because in the equation dsin6 = nX, n can take only
.. one value. 1r ^^60°
If X < d < 2X, then the maximum path differenc; will
be less than 2X. So there will be two more maxibium
on screen in addition to the central maximum. At point R,
Intensity of the dark fringes become zero Arhen . angle of incidence is 30°
intensities at the two slits are equal. Initial inteniity at BysymmetryR = 60°
both the slits are unequal so there will some
brightness at dark fringe. Hence when intensi ty of Net deviation is 90° (second diagram)
both slits is made same the. intensity at dark fringe on
screen will decrease to zero. True/False

1. True
Glass iu
It appears higher due to atmospheric refraction.
2. True
18. (a)
It is a line source of power P and length I
P P • . 1
Intensity I = — = or I
S 2nrl r

3. False
When a ray of light travels from p-i to p.2 after Parallel rays incident on convexlens meet at its second
refraction at a single curved surface. focus after refraction from lens. Let7 be the image./l
M-2 _ _ M-2 "jtj >happens to lie at the first focus of concave lens, as per
V u R the distances given in the question.
As per sigh convention, u = -x, v = +x, R is +\ e,
, Pi =1, P2 =1-5

1.5 1.5-1 1.5 1 0.5


or, — + — = —
X -X R X X R
2.5 _ 0.5
or or x= 5R
, X R 0.75 cm , 0.25 cm ,
^^^
.•. Distance PO = 5R
19. (a, b, c) Using Snell's law at point P
Finalimagewillbe at infinity.The statement is false.
4. True
The statement is true. Light travels
P fi:om air to air in case of hollow
prism. No refraction and no
3oy \v dispersion therefore occur.
^90° The glass slabs forming the prism
-
R D are very thin and permit the rays to pass undeyiated.
. Hence a hollow prism gives no spectrum.
iRRWrdUS YEAR PROBLEMS 351
5. True
/'•/ / =M2iie=5xl0^4Hz
For sustained interferenGe pattern, the two sources
6x10"^
should be coherent. Two diff^entlamps cannot actas
Frequencyof light does not change during refraction.
coherent sources. Hence no<ihterference pattern will
be observed on the screen.'
4. When sources are coherent, = 41q
6. False
When sources are incoherent, 7 = /q +Jq = 2/o
With source of white light, Young's double slit Ratio = — - = 2
I 21 0
- experiment gives coloured fringes on either side of the
central white fringe. 5. According to lens maker's formula
7. True _1 1_
= (li-l)
Violet colour is deviated more than red colour when / .^1 ^2
white light passes through a lens or a prism: Fy < , When lens is immersed in a medium, '"ng must be
for a'lens.
taken for p
Equivalent focal length = F -
J_ 'li. = ^•=^
4/3 4
=1.125
F /i /2 15 -30 30

or F = -30cm.
The combination behaves like a i
j •7'R
f=(X-i) _1_
Ja Ri
J_
^2
r

concave lens.
R « V
Hence it is true that one sees Jm .^1 'R2
coloured patterns with violet
or /JVg -l) = /mC'"ll^ -1)
colour at the outer edge. ! R !
or 15(1.5-1) = /^ (1.125-1)
or
^ _ 15x0.5
Jm = 60 cm fm = 60 cm
in the Blanks 0.125
6. In case of point source, atdistances and r2
1. Velocityt; = — • ='
h
3x10® Foramplitudes Ai andAo,
or V = = 2.0x10® m/s v2
1.5
ii. ^ ^1
. velocity 2x10®
^ = "T r:r = 4.0X10"^-7 m ^2
frequency = 5 x lo^'^ Ai To 25 9^
2. The second focus of the convexTens should coincide •• • Ratio of amplitudes = —
A2 h 9 9
with the first focus of concave lens for the given image 7. When light travels from p2medium to Pimedium at a
formation. •• , single spherical surface, the formula used is
Ml _M^2 _lii -1^2
V

\ Hi =1.0
p, =2.0

20 cm

10 cm
d = 20 - 5 = 15 cm.
3. Formedium, ^ T.V
=^^or = 4000A
-or 15 cm
11 1.5
-20.cm
Frequency
X in air Directionof light is in positive direction.
ill
1:0 2.0- "1.0-2:0
or sin 45°= sin — or 45° =
V • -15
—ib —10 2 Ur\. .
V"
F is centre of curvature of APBi •or 5 = 30°\
11 2 3-4-1 .According to questions also 5 = 30°. It means that the
or —= ^ = — or V = -30 cm
u • 10 15 30 • 30 ray undergoes minimum- deviation: This is possible '
The distance of the final image of 0 from P, as when QRis parallel to base BC. Angle between BC and
vie^ved from the left, is 30 cm to right of P. The image QR = zero. . ^ ^ ' ;
formed will be, virtual. 10. The resolving power of-a' microscope is" inv^sely •'
Note. When the light travels from medium m to medium n 2 at a proportional to the wavelength used. - •
singlesphericalsurface,the formulaused is The resolving power of an'electton microscope is
• ILl _ W = higher than diat of ah opticalmicroscope because the "
V U R. ' y'.
de Broglie wavelength associated widi electron is
Distances measured in the direction of light are positive.
much smaller than the wavelength of visible light.
8. The question deals with lateral magnification when 11. Apply Snell's law of refraction at P: •
rod is placed along opticaxis of concave mirror. Image sin 30° 1 I '
ofAisformed at a'. Image ofBisformed atB itself. B smr vi
• should therefore be at centre.of curvature of mirror.
or

sin r =42x— =-^'= sin 45°


2-42 •
or r = 45°
5 = r-30°=45°-30°=15°
Deviation of fay =.15°.
12. When the lerises^^e in contact,
' AB lies between / and 2/ for real and'elongated image. ••-^P 4^ 4Pi- or 'J\+P2-10 (1)
When lenses have d separation.
.Object distance for A = PA. - • • . P\Po •

•u = PB-AB =2/-/=M i^+P2-^P2=P' or I\+P2—^,= 6


•3 3

Formula for mirror :-V +iu =-^f or 10-S^ =6 or i^P2 =16 .... (2)

From eq. (1) and (2), we get


.1 1 _ 1 Qj. i =zi +A =Z? =8D;P2 = 2D .•. /i=- = 0.125m
" v'^ -5f/3 -f V f 5f 5/ 8

or u = --/= -2.5/ PA = 2.5/ /, = —


J2 2"= 0.5m'
2 **
or BA =PA ~PB = 2.5f-2f 2U)
13. Width of central maxima = ——, during diffraction.
or BA'= 0.5/ ' ' a.

minima

.•.Magnificati6n= ^^or M= or M=1.5


BA //3
9. For a prism.
'i-r-0 maxima

minima

: f=D
=

sinCA/21

sin
^ ,,:_3 2x(5.89xl0"^jx0.5
or 2x10 ^
or
- /n, I 2 d .
sinC6072) or d = 2.945 X10"^m
- 1 : (60^+6
or V2x - = sm —-—
y 2 J l-_:B
PREVIOUSYEAR PROBLEMS 353

uf f u , V
Assertion and Reason^ V - —-— - — = and m = —
u-f l-f/u u/f-1 u
1. (c) The formula is derived under paraxial Values of v may be positive, negative or infinity, also it •
approximation. Only those rays are considered which can have values less than or greater than u.
make very sniall angle with the principal axis. This (a) ^ (p), Cq), (r), (s)
requires a small aperture. Statement-1 is true. (b) Focallength/is positive] So Vwill be positive and
Statement-2 is false, the laws of reflection are valid less than u.
whenever the light is reflected. Nature of surface does Cb)-^Cq)
not matter.
(c) V -
/ _ " Hereu < 0, and / > 0
"Sf ^ • 1+f/u u/f+1
a Comprehension Based Problems V may be positive, negative or infinity, v may be
greater than or less than u.
1. (a) The wavefronts, in both the media, are parallel, (c) ^ (p), Cq), Cr), Cs)
the light will be a parallel beam. (d) The lens is a concavo-convex lens, a converging
h
lens, with positive focal length.
This case is same as in case Cc) for the nature of image.
Medium-1
So Cd)-^Cp), Cq), Cr),Cs)
3. (a)/Cfi) =fi+-^2 +2^/V^cos—
Medium-2
=/q +Iq +2Iq •—j= =C2 +-72)/0
V2

2. (c) Points c and d are on the same wavefronts, 7(^2) = +^2 +27o = 2^//^7^- cos —
=4"d-^so(})g = (jj^because points eand/are on
the same wavefronts.
= /n +/n + 27 0 •
Then clearly,
=^c -'I'e- .-. 7C7i)>7CP2)
3. (b) The ray diagram suggests that on entering
Ca)->Cp),Cs)"
medium-2, the ray • bends towards normal, 27C "K 2tC K
medium-2 is denser than medium-1. (b) 5CPo) = 0^-i>--r = T--r=o
K 4 A. 2.

c^i-l)t-^ .^ = 0
B Matching Type Problems? 8CPi) =
X
2k _ -K
1. Ca)-^(p) 5CP2)=
Greater the radius of aperature, greater will be the T~T
amount of light entering the telescope. 8(Po) ~h "^^2 +2^/7^7^-cos5CPq)
(b)^(r) =7o +7o -I- 27o •cos 2 ~^0
For a telescope, angular magnification —
fe fCPo) = 47o
(c)^(r) 7CP2) =7o +7o +27o cos|^-—^
For a telescope, tube length = —
fe = C2-t-V3)7o
(d) ^ Cr) Cb)^Cg) .
For a telescope, tube length = L + fh
(d) Cp), Cq) and (s)
(c) 5CPo)• =cn -i)t•Y=I.Y=7t:;
Sharpness of image depends on radius- of aperture,
dispersion of lens and spherical aberration. 6C71) = Cp-l)t.f^4 X 4
27t _ TC-
Y~ 2
2. (a) As per the sign convention / is to be taken as
2k _ -K
negative and |X is also negative. 5CP2) = C-1X--
J 6'
354
OPTICS

/(P ) = = — For the first object


^2 3) X ex -^ +1 =^
(C)^(t) + 25/3 u +10.

f c/r* \ Stt 27t


(d)6(Po) =- x - = -Stt => u = - 50 m
For the second object
nPi) = 0 1 ^1_ 1
/(P2)>0, /(Po)>0 • +50/7 u +10

Cd)-^ Cr),(s),(t) u = — 25 m

4. (a)->p, r; (b)^q, s, t;(c)^ p, r,t; (d)^q, s; Speed = —m/ s


30
(P) P- 2 = M- 3i (Since no deviation in medium 2 and 3)
Pi < P2> (Since ray bends towards the normal) = .^x—km/h =3 km/h
30 10
(q) Pi > P2> (Since ray goes away from the normal) 3. (6) Since the rays are going out at angle greater than
P2 > Psj for similar reasons the critical angle total internal reflection will
(r) .Similar to (p) result and therefore light will appear only from a
(s) Similar to (q) . ; , circular region of radius R.
(t) Pi >P2^ (Since ray goes away from the normal) 13
sm0r = — = —
P2 = P35 (Since no deviation in Madiun 2 & 3) ' p 5
-V. From figure
Integer Answer Type Peoblems
1. (6) Magnification,
/ . 20
m -
^25 - •=-4 cm
/ r- 20-25
20' -2\ ^25 _ g
^50 -
20- 50 .S. 3 ''. . m5o
rnso 2
sin0c •=
2.(3) Virtual image of object .behind the mirror is
obtained by the convex mirror. , ,
Equating

10 cm. Solving
R = 6cm
Using Mirror formula
11-1
V u~ f

You might also like